Combo with "Patient Management: Dental Boards Mastery II" and 6 others

Ace your homework & exams now with Quizwiz!

* Each of the following is indicated by a high C-factor EXCEPT one. Which is the EXCEPTION? A. A higher Calset temperature B. A higher potential for bond disruption C. More bonded to unbonded restoration surface areas D. More stress on the walls of the preparation

* A. A higher Calset temperature 1) C-factor a. ratio of bonded to unbounded or free surfaces in a tooth preparation b. the greater the C-factor, the greater the potential for bond disruption from polymerization shrinkage. c. Class IV restoration has the lowest c-factor d. Class I restoration has the highest c-factor 2) Calset composite temperatures a. warmer temperature of 130oF of 155oF causes the composite to flow and adapt to the internal morphology of the cavity preparation resulting in decreased micro leakage and compensates the effects of polymerization shrinkage 3) Composites a. polymerization shrinkage is one of the biggest concerns regarding the composite resins

* The most rapid destruction of alveolar bone is caused by which of the following periodontal diseases? A. Necrotizing ulcerative gingivitis B. Periodontal abscess C. Pregnancy related gingivitis D. Chronic periodontitis

* B. Periodontal abscess 1) The most rapid destruction of alveolar bone occurs in the presence of a periodontal abscess 2) Periodontal abscesses a. occurs when pus accumulates, causing pressure build up within the bone, resulting in bone destruction b. as the periodontal abscess increases in size, the area affected by the abscess is resorbed

Which of the following bone defects typically demonstrates the BEST outcome with regenerative therapy? A. Class II Furcation defect B. Crater C. Four-Walled D. One-Walled E. Three-Walled F. Two-Walled

* C. Four-Walled 1) Prognosis a. BEST prognosis for regenerative periodontal therapies or osseous grafting is associated with defects possessing more walls 2) Craters a. 2-walled defects where the buccal and lingual walls remain while the proximal wall is destroyed 3) Furcations a. have the worst prognosis

* Following a non-surgical extraction, alveolopasties are usually performed with a... A. #1 Woodson B. Bone file C. Highspeed handpiece D. Rongeur E. Finger

* C. Highspeed handpiece 1) Non surgical extractions with sharp bone margins and spiny interseptal bone present indicate cortical alveoloplasty should be performed. 2) Bone is recontoured and smoothened into a more desirable contour to enable a better wound healing, using a surgical hand piece.

Which of the following conditions is associated with localized aggressive periodontitis? A. Cyclic eosinophilia B. Defects in neutrophil chemotaxis C. Elevated phagocytosis D. Lysis of neutrophils

* D. Lysis of neutrophils 1) Defects in neutrophil chemotaxis a. prevents the neutrophils from reaching the infected site of the periodontium b. directs neutrophils to a specific chemotactic gradient where the concentration of bacterial invasion is severe 2) Lysis of neutrophils a. helps to recruit more neutrophils along the infection site 3) Elevated phagocytosis a. active host defense mechanism

Which of the following diagnoses CANNOT be ruled out for a lesion posterior to mandibular 3rd molars? A. Cementoblastomas B. Fibroma C. Salivary gland inclusion defect (Stafne defect) D. Osteoma

*B. Fibroma 1) Fibroma a. tumors that are composed of fibrous tissues that are usually benign b. grow anywhere in the body, but usually grow in tissues that arise from the mesenchymal tissues 2) Stafne defects a. occur in the mandible along the lingual area near the submandibular salivary gland 3) Cementoblastomas a. occur within the cementum of affected tooth 4) Osteomas a. usually occur in the paranasal sinus and nose

Anterior disc displacement

1) Anterior Disc Displacement with Reduction a. articular disc is attached anteriorly to the superior head of the lateral pterygoid muscle and posteriorly to the retrodiscal tissue b. moves anteriorly out from between the condyle and the fossa, so that the mandible and temporal bone contact is made on something other than the articular disc and the disc slides back into place c. sliding of the disc back into place creates the characteristic popping or clicking noise 2) Anterior Disc Displacement without Reduction a. the disc stays anterior to the condylar head upon opening, so mouth opening is limited and there is no "pop" or "click" sound on opening b. no sliding of the disc back into place

Cast gold vs. amalgam

1) Cast gold material provides superior marginal sealing and physical properties which makes it an ideal material for recreating anatomic contours. 2) Cast gold material can be re-adapted and burnished at the finish lines of the cavity preparation to ensure a tiger fit and lesser marginal leakage.

Internal derangement

1) Internal derangement a. condition within the TMJ b. involves displacement or slipping of the articular disc 2) Internal derangement with reduction a. articular disc is moving and sliding in and out of its normal limiting structure as the condyle head functions during the jaw opening and closing movements. b. clicking sound due to articular disc that slides back and forth.

Mucous retetion cysts

1) Mucous retention cysts a. dome-shaped radiopaque entities seen on the floor of the maxillary sinus b. represent focal accumulation of inflammatory exudate that lifts the epithelial lining of the sinus and the periosteum away from the underlying bone c. are benign inflammatory reactions, so there is NO need for surgical treatment

Nasopalatine duct cyst

1) Nasopalatine duct cyst a. heart-shaped radiolucency often found in the median aspect of the hard palate anterior to the 1st molars 2) considered the most prominent odontogneic cyst within the oral cavity

Which of the following does a 0.02 taper indicate for a K-file? A. 0.02mm increase in diameter per 1mm of file length B. 0.02mm increase in diameter per 2mm of file length C. 0.2mm increase in diameter per 1mm of file length D. 0.2mm increase in diameter per 2mm of file length

A. 0.02mm increase in diameter per 1mm of file length 1) Taper a. 0.02mm taper indicates an increase of 0.02mm in diameter per 1mm of file length. b. describes the amount of file diameter increases with each millimeter from tip of the file toward the handle.

The apical foramen is most commonly... A. 0.5-1.mm from the anatomic apex B. less than 0.5mm from the anatomic apex C. more than 1mm from the anatomic apex D. at the anatomic apex

A. 0.5-1.mm from the anatomic apex 1) The apical foramen is usually situated approximately 0.5-1.0mm short of the root apex. 2) Apical constrition a. narrowest portion of the canal located a few mm from the anatomical apex. b. junction between the first layer of dentin and the first cementum layer that covers the exiting portion of the tooth apex.

Retention grooves MUST be deep enough to resist explorer tip displacement an thus should be how many mm deep? A. 0.5mm B. 1.5mm C. 1mm D. 2mm D. 3mm

A. 0.5mm 1) Concept: Retention grooves are the secondary retention features of a preparation and must be deep enough to resist explorer tip displacement. Ideally grooves are 0.5mm deep. 2) The use of retention grooves is recommended in tooth preparations with extensive proximal boxes in order to enhance retention form and counter proximal displacement. 3) The four main characteristics or determinants of proximal grooves are: a. Position b. Translation c. Depth d. Occluso gingival orientation 4) Depth refers to the extent of translation i.e. direction of the movement of axis of the bur. It is about 0.5mm at the gingival floor level. 5) The retention grooves are positioned 0.2mm inside the DEJ maintaining the enamel support.

When treating Actinomycosis of the jaw, which of the following is the most accepted dosage schedule? A. 10 million units IV per day for 10-14 days followed by a long-term oral antibiotic B. 250 mg oral tablets q6H, 7 days C. 500 mg oral tablets q6h, 7 days D. 600,000 units IM Q12H, 14 days followed by an oral antibiotic

A. 10 million units IV per day for 10-14 days followed by a long-term oral antibiotic 1) Concept: Actinomycosis of the jaw is best treated with penicillin antibiotics intravenously for a period of 2-6 weeks 2) Actionmycosis a. requires long term antibiotic treatment in oral doses that may last form 6-12 months depending on the disease severity b. Actinomycosis infection elicits suppuration and inflammation c. Swellings caused by actinomycosis drain pus containing sulfur granules

Which of the following is the minimum number of flutes needed on a carbide bur for it to be considered a "finishing bur" A. 12 B. 18 C. 24 D. 6 E. Finishing carbide burs do not have flutes

A. 12 1) The minimum number of flutes needed on a carbide bur to be considered as a finishing bur is 12. 2) Carbide burs a. used most commonly for excavating and preparing cavities, finishing cavity walls, finishing restoration surfaces, drilling old fillings, finishing crown preparations, contouring bone, removing impacted teeth, and separating crowns and bridges. b. made of tungsten carbide, a metal that is extremely hard and can withstand high temperatures c. carbide has high hardness and can maintain a sharp cutting edge and be used many time without becoming dull d. carbide burs are brittle and have a tendency to fracture under pressure. Because of their brittleness, they are best operated at high speeds with light pressure. 3) Trimming and finishing burs a. have more blades than operative burs and the blades are closer together and shallower, making them ideal for the fine finishing and polishing

A patient undergoing general anesthesia with phenobarbital will pass through which stage of anesthesia? A. 2 B. 3 C. 4 D. 1

A. 2 1) Phenobarbital a. IV drug that is used to control seizures and convulsions and also shortens the time in passing through stage 2 of general anesthesia b. Barbiturate with longest duration c. most widely used anticonvulsant drug in the world d. used in general anesthesia to limit stage 2 anesthesia 2) Stage 2 anesthesia a. patients may exhibit involuntary excitement, delirium, apnea, and exaggerated reflect response leading to possible vomiting

Which of the following furcation classes are most commonly treated by utilization of guided tissue regeneration procedures? A. 2 B. 3 C. 4 D. 1

A. 2 1) Furcation Classes Grade I a. incipient bone loss b. Nabor's probe can feel the depression of the furcation opening Grade II a. partial bone loss b. Nabor's probe enters under the ceiling of the furcation c. have the best prognosis for guided tissue regeneration because they are the least severe of pathos's Grade III a. total bone loss b. through and through opening of the furcation c. entrance to the furcation is not able to be seen clinically Grade IV a. through-and-through b. furcation where entrance is clinically visible

Which of the following describes a case where compound caries may be found? A. 2 surfaces of the same tooth B. 2+ surfaces of the same tooth D. The distal, occlusal, and facial surfaces of a maxillary molar

A. 2 surfaces of the same tooth 1. Compound lesion = lesions that covers 2 surfaces of a tooth 2. Complex lesion = lesion that covers 3 surfaces of a tooth 3. Simple lesion = lesion that covers 1 surface of a tooth 4. Secondary lesion = recurrent lesion; lesion created from a previous lesion that has been restored and is also known as recurrent caries

Which of the following describes the radiant exposure for a curing light producing 1000 mW of power with a focal spot size of 2cm2 and an exposure time of 40 seconds? A. 20,000 mJ/cm2 B. 200,000 mJ/cm2 C. 2000 mJ/cm D. 500 mJ/cm2

A. 20,000 mJ/cm2 1) The radiant exposure for a curing light producing 1000 mW of power with a spot size of 2 cm2 and an exposure time of 40 seconds is 20,000 mJ/cm2. 2) Dental curing light is a piece of dental equipment that is used for polymerization of light cure resin based composites 3) It can be used on several different dental materials that are curable by light. 4) Camphorquinone is the typical agent used as a photo initiator in composite resins.

Tooth calcification begins during which of the following time periods? A. 2nd trimester B. 3rd trimester C. Birth D. 1st trimester

A. 2nd trimester 1) Concept: Tooth formation begins 6-8 weeks in utero 2) During tooth formation, calcification of dental layers occurs within the 14th to 19th week in utero. 3) Calcification of teeth in utero occurs during the 2nd trimester.

How many days will it usually take for a patient to notice post extraction alveolar osteitis (dry socket) if it occurs? A. 3 B. 5 C. 7 D. 9 E. 1

A. 3 1) Concept: Dry socket (Localized Alveolar Osteitis) is a painful condition that occurs 2-3 days following tooth extraction and is self limiting in nature. 2) Dry socket a. have several predisposing conditions but usually occurs due to the dislodgment of the blood clot in the extraction site. b. elicits pain that may last for 2-3 days and may even radiate to other areas of the face.

Which of the following is the average wavelength range that should be emitted by a light curing unit to cure composite resin? A. 400-499nm B. 500-599nm C. 600-699nm D. 700-799nm E. 300-399nm

A. 400-499nm 1) Concept: A wavelength range of 400-499nm provide sufficient energy to activate the camphorquinone molecules and initiate polymerization of composites. 2) The opimum absorption of energy of camphorquinone is at 465nm which is why a narrow spectrum of wavelength is required in the polymerization of composites. 3) When camphorquinone and its amine co-initiator reacts with the blue light spectrum, they create radicals which initiates polymerization. 4) LED curing lights have made composite curing easier and faster because they emit a beam of light up to 1000mW/cm2.

DO amalgam restorations are less likely to fracture between the occlusal and distal portion if... A. A bevels is placed on the pulpoaxial line angle B. All answers are correct C. All unsupported enamel is removed D. No answers are correct E. The preparation includes a dovetail feature

A. A bevels is placed on the pulpoaxial line angle 1) Concept: Pulpo-axial line angle a. The pulpo-axial line angle of a proximal box restoration is an area of stress concentration as occlusal forces interact with the restoration. Placement of a bevel on internal line angle decreases the concentration of forces along the pulpo-axial line angle. b. Rounding of point angles will also help lessen the concentration of forces along the sharp corners/angles of the cavity preparation c. Sufficient depth of the pulpal and cervical floor must be achieved to provide enough bulk of material to resist fracture d. Sufficient width of the isthmus connecting the proximal box to set of the cavity preparation must be achieved to provide enough bulk for the restorative material to resist fracture.

Which of the following describes Metamerism? A. A complication observed when the perceived color of objects is different in differing light sources B. A disease associated with high soda consumption C. A problem caused by over etching D. The closing of dentin tubules E. When color changes due to wetness

A. A complication observed when the perceived color of objects is different in differing light sources 1) Metamerism a. complication observed when the perceived color of object is different in differing light sources b. various light sources produce different perceptions of color c. color of the surrounding environment influences what is in the patient's mouth d. complicates the selection of the appropriate shade of the restorative material

Which of the following statements is TRUE regarding Class IV composite restorations? A. A final veneer layer of microfilled resin will help create a smooth, glossy surface. B. It is recommended to use retentive pins for support on Class IV restorations. C. No answers apply D. Tooth fragments should never be reattached to the remaining tooth structure

A. A final veneer layer of microfilled resin will help create a smooth, glossy surface. 1) A final veneer layer of micro filled resin will help create a smooth and glossy surface due to its small sized particles. 2) Polished surfaces are less receptive to plaque accumulation and extrinsic staining 3) Retentive pins are never recommended for support and retention in Class IV restorations.

A very deep MO cavity was restored with an amalgam restoration one month ago with deep retentive grooves placed into the dentinal line angles of the gingival box. The patient reports pain in the general area of the tooth, what is the most probable reason for this discomfort? A. A pulp exposure that was not detected B. Moisture contamination of the amalgam only after condensation C. The amalgam was over-triturated and therefore, not enough amalgam was expressed during the condensation D. Moisture condensation of the amalgam before condensation

A. A pulp exposure that was not detected 1) Rentetion grooves placed into a deep cavity may potentiate an accidental and undetected pulpal exposure.

Which of the following describes the non-carious level loss of cervical tooth structure that occurs due to cyclic loading of teeth by occlusal forces? A. Abfraction

A. Abfraction 1) Abfraction a. loss of tooth structure caused by flexural forces applied by cyclic loading of the enamel that usually occurs at the cementoenamel junction (CEJ). 2) Attrition a. loss of tooth structure by contact with another tooth 3) Abrasion a. describes the loss of tooth structure by an external force like dentifrice 4) Erosion a. describes the loss of tooth structure through chemical dissolution by acids NOT of bacterial origin

The apical extent of a gingivectomy procedure should be placed... A. Above the mucogingival junction B. At the mucogingival junction C. In the buccal mucosa D. Coronal to the periodontal pocket

A. Above the mucogingival junction 1) Concept: gingivectomy is a periodontal surgery that involves excision of the gingiva. The apical extent of gingivectomy may be equal or apical to th bottom of the periodontal pocket as long as the termination point is above the mucogingival junction. 2) Gingivectomy may be used to: a. Treat suprabony periodontal pockets b. Treat gingival enlargement/hypertrophy c. Treat supra bony periodontal abscesses d. Provide visibility and accessibility for calculus removal and cleaning 3) Extent of the gingiva to be removed is identified with the use of a periodontal probe

Glass ionomers use an acid/base reaction to chemically bond to the tooth surface. What are the acid and bases used to create this reaction? A. Acid-polyalkenoic acid and Base-Alumino-silicate glass B. Acid-Phosphoric acid and Base-Alumino-silicate glass C. Acid-Phosphoric acid and Base-Sodium hypochlorite D. Acid-Polyalenoic acid and Base-Sodium hypochlorite

A. Acid-polyalkenoic acid and Base-Alumino-silicate glass 1) Concept: The setting reaction of glass ionomer cement is an acid-based reaction between the polyalkenoic acid and fluoroaluminosilicate glass powder resulting in a chemical bonding to the tooth surface. 2) When the powder and liquid are mixed, the fluoroaluminosilicate glass powder is attacked by hydrogen ions from the polyalkenoic acid liberating Al+3, Ca+2, Na+1, and Fl-1 ions. 3) After saturation the free calcium and aluminum ions diffuse into the liquid and cross link with 2 or 3 ionized carboxyl groups of the polyacid to form a gel.

Which of the following esthetic properties dose dentin provide? A. All answers apply B. Chroma C. Fluorescence D. No answers apply E. Opaqueness

A. All answers apply 1) Dentin provides a. Chroma: intensity of any particular hue or color. b. Fluorescence: emission of light by a substance that has absorbed light or other electromagnetic radiation c. Opaqueness: absorption and scattering of radiation in a medium. It is neither transparent allow all light to pass through nor translucent allowing some light to pass through.

Which of the following lesions is known to possess the greatest malignant potential? A. Actinic chelitis B. Hyperkaratosis C. Lichen planus D. White sponge nevus

A. Actinic chelitis 1) Concept: Actinic cheilitis a. Actinic cheilitis is documented to possess the greatest malignant potential among the lesions listed in the question presented. It is a counterpart of the actinic keratosis of the skin and is prone to progress into squamous cell carcinoma. It is characterized by a thickening whitish discoloration of the lips at the border of the lip and skin and loss of vermillion border. 2) Hyperkeratosis a. associated with the thickening of the outermost layer of the epidermis and presence of an abnormal quantity of keratin 3) Lichen Planus a. autoimmune disease of skin and mucous membranes 4) White sponge nevus a. involves a defect in the normal process of keratinization of the mucosa.

Which of the following can be diagnosed by the presence of "sulfur granules"? A. Actinomycosis B. Candidiasis C. Histoplasmosis D. Sarcoidosis E Tinea versicolor

A. Actinomycosis 1) Actinomycosis a. Actionmycosis Isreali is the etiologic bacteria behind actinomycosis infection b. presence of yellow sulfur granules confirms the presence of actinomycosis colonies c. characteristics 1) non-painful hard lump in the jaw 2) painful skin abscesses which initiates a red bruise 3) muscle spasms in the jaw leading to locked jaw

Ultrasonic instruments are indicated for each of the following types of patients EXCEPT one. Which is the EXCEPTION? A. Active infectious disease B. Controlled diabetic C. Deep periodontal pockets D. Edematous tissue E. Mildly uncontrolled diabetic

A. Active infectious disease 1) Concept: Ultrasonic instruments should NOT be used for patients with active infectious disease because the aerosol created by the instruments serves to spread the infection around the operator. 2) Diabetes and deep periodontal pockets are indications that ultrasonic instrumentation.

The first step in the delivery of a cast gold inlay is... A. Adjust contacts B. Adjust occlusion C. Burnish, adapt margins D. Cement the restoration E. Seat the restoration with a mallet

A. Adjust contacts 1) Proximal contacts must first be assessed and adjusted to properly placed gold inlays. 2) Contacts if an inlay must always be checked because dimensional change might have occurred during casting. 3) After evaluating the contacts, seat the restoration to check the marginal fit and its occlusal relationship. 4) Gold can be burnished against the margins of the preparation to make it more adapted and tightly fit for a good marginal seal.

Which of the following instruments can be used to hold soft tissue that is not to be excised? A. Adson forceps D. Allis forceps

A. Adson forceps 1) Adson forceps a. designed with a small tip used to hold tissue that will not undergo excision b. have a fine tip, they are best used in holding soft tissues that are located along the small spaces inside the mouth c. have small teeth that allows it to hold and grip the soft tissues in a precise manner d. can be used in holding suture during suture cutting and removal

Which of the following tooth conditions presents the most favorable outcome for tooth whitening? A. Aged-yellow teeth B. Blue-gray teeth C. Patients with amelogenesis imperfect D. Patients with old composite-resin restorations

A. Aged-yellow teeth 1) Teeth Whitening a. yellowing of teeth due to age, coffee, and tea staining will demonstrate the most favorable tooth whitening results due to the stains being extrinsic in nature. b.

Which of the following describes the BEST clinical reason to place a restoration> A. Aid plaque control B. Esthetic contouring C. No answers apply D. Staining is evident on occlusal E. Tooth is sensitive to cold & sweet

A. Aid plaque control 1) The MOST IMPORTANT reason to place a restoration is to aid in plaque control. 2) Restorations a. intended to be preventive in regards to future carious activity and has a primary emphasis on restoring a smooth surface which will NOT allow plaque accumulation b. restore structure, function, and esthetics are also of prime importance

If during a large MOD caries removal, your patient loses consciousness. You should first assess... A. Airway and pulse B. Blood pressure and airway C. Pupil size D. Blood pressure and pulse

A. Airway and pulse 1) When a person suddenly loses consciousness, it is important to assess ABCD's 2) Airway a. check for obstruction 3) Breathing a. check for rise and fall of chest 4) Circulation a. check the person's pulse 5) Diabetes a. check blood sugar if patient is diabetic 6) It is important for the doctor to not feel panic when a person loses consciousness, they must think clearly and decisively to properly restore the person to health and consciousness.

Rubber dam clamps must contact the anchor tooth apical to the height of contour and which of the following to be stable? A. All 4 tips must be in contact with the tooth C. The bow must be pointed toward the distal of the tooth

A. All 4 tips must be in contact with the tooth 1) All 4 tips of the rubber dam clamp should be in contact with the surface of the tooth to provide maximum stability. 2) Rubber dam clamps should be placed apical to the height of contour

Which of the following factors demonstrate an effect on photo polymerization? A. All answers apply B. Distance of the curing light from the restoration C. Irradiance D. Time light is applied to restoration

A. All answers apply 1) Concepts: Photo-polymerization in visible light cure composite resins can be affected by the following factors: a. Time of application of curing light is an important factor. It depends upon the type of resin used, amount of tints or opacifiers incorporated, wavelength of light and its intensity b. Distance of light curing tip from the surface of restorative resin can affect the amount of polymerization c. Irradiance and wavelength of the photo-polmerizing sources should be considered

Each of the following is a consequence (positive or negative) of dentinal smear layer removal EXCEPT one. Which is the EXCEPTION? A. All answer choices are consequences of dentinal smear layer removal B. Increased bond strength C. Increased dentin wetness D. Makes micro leakage more significant because barrier of bacteria to pulp is removed

A. All answer choices are consequences of dentinal smear layer removal 1) Smear Layer a. defined as debris, calcific in nature which is produced by reduction or instrumentation of enamel, dentin, and cementum b. interfere with any adhesive bond formed between the tooth strutter and the restorative material c. 2 strategies used to overcome the low attachment strengths of smear layer 1) removal of the smear layer prior to bonding by acid etching 2) use of bonding agents that can penetrate the smear layer and incorporate it into the bonding layer 2) Smear Layer Removal a. increase wetness of dentin b. increase bond strength c. makes micro league more significant because barrier of bacteria to pulp is removed d. an increase of the fluid flow onto the exposed dentin surface

Beveling the enamel on resin restorations does which of the following? A. All answers apply B. Improves esthetics C. Improves retention D. Reduces microleakage

A. All answers apply 1) Concept: An enamel bevel in the facial area is intended to improve esthetics, retention of the restorative material, and minimize micro leakage. 2) An enamel bevel helps in diminishing the line of demarcation at the interface between the restorative material and tooth surface. 3) Bevels increase the resin bond strength and decreases the chance of fracture and dislodgment of the restoration.

Which of the following is/are considered reason(s) to place bevels into enamel? A. All answers apply B. To enhance esthetics if the margin is in a visible location C. To remove fragile enamel D. To smooth the cavorestorative margins

A. All answers apply 1) Concept: Enamel bevels are generally placed to minimize the line of demarcation at the tooth-restoration interface. The bevel is commonly placed at a 45-60 degree angle. 2) Advantages of enamel bevels are: a. Removal of fragile enamel and strengthening the restoration b. To aid in making margins smooth c. Enhancement of esthetics if the margin is in a visible location

Which of the following properties are improved by adding filler particles added to the matrix of resin composites? A. All answers apply B. Make the material harder, denser, and more resistant to wear C. Reduction of the coefficient of thermal expansion D. Reduction polymerization shrinkage of the composite

A. All answers apply 1) Concept: Filler particles provide strength to the composite resins and help to improve the following physical properties: a. Reduces the coefficient of thermal expansion b. Reduces polymerization shrinkage of the composite c. Make the material harder, denser, and more resistant to wear d. Cause reinforcement of the resin matrix e. Decrease polymerization shrinkage f. Improves workability g. Decrease water sorption h. Increases radiopacity

Which of the following statements describe the key objective of the internal bevel incision used in flap procedures? A. All answers apply B. Allow for better adaptation of the gingiva to the junction of the alveolar bone meets the tooth C. Conserve gingival tissue D. Remove the epithelial lining of the periodontal pocket.

A. All answers apply 1) Concept: The internal bevel incision helps reflect the flap to expose the underlying alveolar bone. 2) Internal bevel incision a. Conserves gingival tissues not affected by the pocket b. Remove the epithelial lining of the periodontal pocket c. Allows for better adaptation of the gingiva tot he junction of the alveolar bone meets the tooth

Each of the following statements regarding mercury is true EXCEPT one. Which is the EXCEPTION? A. All answers are TRUE answers of mercury B. Hg is the only metal that remains the liquid state at room temperature C. Hg cannot combine under ordinary conditions with nickel, chromium, molybdenum, cobalt, and iron, to form amalgam D. Hg demonstrates a higher surface tension than water

A. All answers are TRUE answers of mercury 1) Mercury (Hg) a. only metal that remains in the liquid sate at room temperature b. combines with silver-copper-tin alloy to form dental amalgam. c. can't combine with nickel, chromium, molybdenum, cobalt, and iron under ordinary conditions to form dental amalgam d. surface tension of Hg is higher than water e. in low copper amalgam, Hg is liberated as a result of corrosion to react with an unreacted gamma phase to produce gamma-1 and gamma phases.

The American Dental Association (ADA) Statement on posterior resin based composite restorations suggest their use for which of the following? A. All answers are correct B. Areas where esthetic is important C. Conservative tooth preparation D. Small and moderately sized restorations

A. All answers are correct 1) ADA endorses the use of posterior composites in the following conditions: a. small and moderately sized restorations b. conservative tooth preparations c. areas where esthetics is important

Radiographs reveal that a 16 year old patient with excellent oral hygiene and no missing teeth presents with distal decay on #12. A distal slot preparation can be performed if... A. All answers are correct B. No answers are correct C. The caries has not violated the DEJ D. When the adjacent tooth is missing E. Proximal caries on a tooth with severe attrition

A. All answers are correct 1) Concept: Distal Slot Preparations a. proximal root caries in patients with gingival recession b. teeth with distal decay and no adjacent tooth positioned distally c. teeth with distal decay extending up to the DEJ d. small proximal caries that does not include or undermine the occlusal surface e. proximal caries in tooth with attrition

Methanol poisoning can be treated by administration of ethanol because ethanol... A. All answers are correct B. Has a higher affinity for alcohol dehydrogenase C. It prevents injury to cranial nerve I (optic nerve) by methanol metabolites D. Inhibits metabolism of methanol into formaldehyde

A. All answers are correct 1) Concept: Ethanol reverses or prevents methanol poisoning because it competitively inhibits alcohol dehydrogenase, preventing methanol from being turned into formaldehyde. 2) Alcohol Dehydrogenase a. has higher affinity for ethanol than methanol 3) Unmetabolized methanol is excreted immediately by the kidneys after ethanol is administered. 4) Ethanol administration also prevents optic nerve injury caused by metabolic acidosis during methanol poisoning.

Patients who chose not to treat tooth decay may experience... A. All answers are correct B. Dietary deficiency C. Gingivitis D. No answers are correct

A. All answers are correct 1) Untreated caries --> affect the pulp --> irreversible pulp damage --> tooth loss 2) Progression of tooth decay a. loss of esthetic b. change in eating habits c. occlusion issues

Resin Modified Glass Ionomers are _____ compared to Resin Composite materials. A. Better with fluoride recharging B. More color stable C. More wear resistant D. Stronger

A. Better with fluoride recharging 1) Resin modified glass ionomer (RMGI) a. have better fluoride recharging as compared to composites 2) Compared to RMGIC a. stronger b. more color stable c. more wear resistant

Which of the following is the reason zinc is a constituent of amalgam? A. All answers are correct B. To enhance its mechanical properties C. To prolong the service of the restoration D. To reduce marginal fracture

A. All answers are correct 1) Zinc a. is added to amalgam to prevent the oxidation of other metals in the alloy during the manufacturing process, which keeps the alloy from turning dark. b. accomplishes this by combining readily with oxygen to form zinc oxide 2) Zinc Benefits a. enhances the mechanical properties of amalgam b. reduces marginal fracture rate c. prolongs the service of the restoration

Which of the following tooth structures is (are) affected by tooth bleaching? A. All of the enamel only B. Dentin only C. Surface of enamel only D. Both dentin and enamel

A. All of the enamel only 1) Bleaching agents like hydrogen peroxide and carbide peroxide act on the tooth enamel only unless root dentin is exposed during bleaching. 2) Oxidation of enamel surface stains and impurities is carried out by these bleaching agents. 3) Dentin and pulp are typically NOT affected by bleaching procedures. 4) Discoloration of tooth enamel caused by extrinsic stains can be corrected with the help of tooth bleaching or tooth whitening procedures.

Which of the following osseous grafting materials is harvested from a human donor and placed in another human? A. Allograft B. Allolastic C. Autograft D. Xenograft

A. Allograft 1) Concept: Allografts a. describe bone that is taken from another human being and placed in another person 2) Autografts a. describe bone taken away from one patient and placed in that very same patient 3) Xenografts a. describe bone taken from an animal like a horse, pig, or cow, and placed in a human 4) Alloplastic/synthetic grafts a. describe inert biologic fillers such as hydroxyapatite, different polymers like PMMA

Which of the following statements regarding the creep of amalgam is CORRECT? A. Alloys with higher copper content demonstrate lower creep values B. Alloys with lower copper content demonstrate lower creep values C. Creep value is NOT a property of amalgam restorations, only noble ones D. The lower the creep value, the worse the marginal integrity of the restoration

A. Alloys with higher copper content demonstrate lower creep values 1) Concept: Alloys with higher copper content demonstrate lower creep values because they exhibit little of no gamma 2 phase. 2) Creep a. defined as the time dependent plastic permanent deformation of amalgam under static or dynamic loading. b. causes protrusion of amalgam out of the cavity. The protruded edges are unsupported and weak and are further weakened by corrosion leading to fracture, ditching, and micro leakage of the restoration. c. the gamma 2 phase is primarily responsible of higher creep values

Which of the following materials are contraindicated as a sealant? A. Amalgam B. Glass Ionomer C. Nanohybrid resin D. Unfilled resin E. Microhybrid resin

A. Amalgam 1) Sealant a. biocompatible with the oral tissues b. resistant to dissolution by oral fluids c. lasts for a long time in the oral cavity d. application is not technique sensitive 2) Amalgam a. contraindicated for small restoration since it is brittle b. needs a minimum of 1.5mm thickness to obtain flexural resistance 3) Glass Ionomer a. being used as a sealant because of its ability to release fluoride

Which of the following is NOT an advantage of amalgam when compared to other restorative materials? A. Amalgam bonds directly to tooth structure B. Clinicians are less likely to remove additional tooth structure when removing an amalgam restoration due to its color C. Less-time consuming to place D. Lower cost

A. Amalgam bonds directly to tooth structure 1) Concept: Amalgam does NOT bond directly to the tooth structure 2) Mechanical features like convergence of cavity walls, adequate preparation depth, grooves and slots are incorporated in cavity preparations to retain amalgam fillings. 3) The advantages of amalgam restorations as compared to other materials are: a. Less time consuming to place b. Lower cost c. Less technique sensitive d. Less likely to remove additional tooth structure when removing an amalgam restoration due to its color

What design features of a DO alloy preparation would proven the restoration from falling out into an edentulous space? Select all that apply. A. Amalgapin in the proximal box B. Bevel of pulp axial line angle C. Convergent axial walls D. Occlusal dovetailes

A. Amalgapin in the proximal box C. Convergent axial walls D. Occlusal dovetailes 1) Concept: Placing occlusal dovetails provide resistance to dislodging forces for the restoration 2) Convergent axial walls keep the restoration in placed by providing retention and resistance to dislodgment to perpendicular and shear stress 3) An amalgapin in the proximal box provides added resistance and prevents displacement into adjacent edentulous space.

Neoplasia of both epithelial and connective tissue components is exhibited by which of the following? A. Ameloblastic fibroma B. Cementoblastoma C. Odontogenic adenomatoid tumor D. Ameloblastoma

A. Ameloblastic fibroma 1) Ameloblastic fibroma a. benign odontogenic tumor arising from epithelial and ectomesenchymal tissues and produces no mineralized product b. appears radiographically as sharply demarcated radiolucent lesion 2) Ameloblastoma a. benign tumor derived from strictly odontogenic epithelium and not ectomesenchyme tissue.

The most common procedure for treating chronic recurrent TMJ dislocation is... A. An interpositional arthroplasty often combined with a coronoidectomy B. Corrective orthognathic surgery with articular eminectomy usually combined with capsular placation C. Total joint replacement D. Coronoidectomy and release of temporal is muscle from the mandible

A. An interpositional arthroplasty often combined with a coronoidectomy 1) Concept: Interpositional arthroplasty procedures a. separate the non-mobile portion and mobile portion of the TMJ to ensure that the condyle and the temporal bone do not directly contact each other. b. recontours the bones within the temporomandibular joint and uses fascia, cartilage, or any suitable substance in between the two bones to correct the damaging and painful friction between them c. reduces painful movements and re-establishes motion within the TMJ 2) Coronoidectomy a. also performed to manage trismus and pseudoankylosis

A 40 year old patient diagnosed with an acute apical abscess presents to your office for the extraction of tooth #3. Penicillin prophlaxis was administered because the patient had a history of rheumatic fever and valve replacement. Periorbital and labial edema were discovered 15 minutes following administration of penicillin. What is the most likely diagnosis for what this patient is experiencing? A. Angioneurotic edema B. Congestive heart failure C. Hematoma formation D. An acute anaphylactoid reaction

A. Angioneurotic edema 1) Concept: Angioneurotic edema was initiated due to an allergic reaction to penicillin 2) As the mast cells in the body being to react against an allergen, histamine release occurs, resulting in an allergic reaction and angioedema. 3) Symptoms of angioneurotic edema include a. swelling of the periorbital area, lips, and tongue b. urticaria c. shortness in breathing in severe cases

Infection, loss of interocclusal (intermaxilary) space, and vitamin B12 deficiency, are all possible etiologies of which of the following conditions A. Angular chelitis B. Apthous stomatitis C. Atrophic glossitis D. Media rhomboid glossitis E. Xerostomia

A. Angular chelitis 1) Concept: Angular cheilitis a. Angular cheilitis has a common etiology of infection, loss of interocclusal/intermaxilary space, or vitamin B deficiency. b. Characteristics of angular cheilitis include: 1) erythema 2) moist maceration 3) soreness 4) pain 5) burning 6) pruritis 7) ulceration and crusting at the corners of the mouth 8) There may also be inflammation of the vermillion commisures and adjacent mucous membranes. 2) Angular cheilitis is also known as angular cheilosis, commisural cheilitis, angular stomatitis and perleche.

Which of the following terms can be used to describe replacement resorption? A. Ankylosis B. Apical pathosis C. Painful D. Rapidly progressing E. Acute apical absces

A. Ankylosis 1) Replacement Resorption a. process wherein the tooth firmly attached to the bone, leading to ankylosis b. possible outcome observed in a replanted avulsed tooth c. inflammatory response initiated by the damaged PDL results in its replacement with alveolar bone 2) Ankylosis a. describes a tooth wherein an area of root is devoid of an attachment apparatus

Which of the following best describe replacement root resorption? A. Ankylosis B. Apical pathosis C. Chronic pain D. Rapid progression E. Acute inflammation

A. Ankylosis 1) Root Replacement a. occurs in response to dental trauma or severe damage to the periodontium b. bone grows and attaches to the root causing ankylosis c. happens more commonly among patients who suffered severe cases of avulsion and luxation

Which of the following areas is the MOST COMON site where adenomatoid odontogenic tumors occur? A. Anterior Maxilla B. Posterior Maxilla C. Posterior Mandible D. Soft Palate E. Anterior Mandible

A. Anterior Maxilla 1) Concept: Adenomatoid odontogenic tumors are found most commonly in females under 20 years of age in their anterior maxilla and are associated with an unerupted tooth. b. Adenomatoid odontogenic tumors are a benign epithelial tumor of bone that may or may not mineralize. c. They manifest radiographically as a well-defined radiolucent lesion which sometimes contains radiopaque areas.

Each of the following are true regarding the treatment tooth with a sinus tract EXCEPT one. Which is the EXCEPTION? A. Antibiotics are indicated B. Root-end surgery should be performed if the tract persists after the root-canal C. The sinus tract should heal within a month of the root canal D. The tooth should be treated with a root canal

A. Antibiotics are indicated a. Chronic Apical Abscesses present with sinus tracts that eventually drain into the oral cavity although they may drain into the gingival sulcus (which may imitate a periodontal abscess) b. Teeth with sinus tract do NOT require antibiotics because the infection drains c. Apical periodontitis (acute or chronic) should be treated with root canal therapy. d. If the tract does not heal within a month or so, root-end surgery should be performed e. Teeth with chronic apical periodontitis may develop an epithelial lining and form a true periodontal pocket if they are left untreated.

The functional psychiatric disorder most commonly seen in elderly patients is which of the following? A. Antisocial personality disorder C. Depression E. Senility F. Anxiety

A. Antisocial personality disorder 1) Older people that seldom experience good social interactions are more prone to develop antisocial personality disorder or ASPD.

Which of the following locations would a perforation demonstrate the BEST prognosis? A. Apical 1/3 of root B. Coronal 1/3 of root C. Floor of pulp chamber D. Middle 1/3 of root

A. Apical 1/3 of root 1) Apical Perforations a. occur through the apical foramen or the body of the root to form a new canal b. the more apical the perforation the more favorable the prognosis for the tooth (except for coronal perforations)

A vestibuloplasty procedure is performed by... A. Apically repositioning the muscular and mucosal attachments of the mandible C. Reflecting the gingiva and using Rongeurs to remove bone from the alveolar crest

A. Apically repositioning the muscular and mucosal attachments of the mandible 1) Vestibuloplasty a. procedure performed to lengthen the residual ridge and deepen the oral vestibule b. relocating or repositioning the muscle attachments and mucous membranes covering it into a deeper or more apical position c. also accompanied with the use of a mucosal graft or a skin graft

Each of the following conditions is characterized by an ulcerations that are preceded by a vesicular lesion EXCEPT ONE. Which condition is the EXCEPTION? A. Apthous Stomatitis B. Coxackie virus infection (Hand-foot-and-mouth disease) C. Herpangina D. Herpes zoster E. Herpetic stomatitis

A. Apthous Stomatitis 1) Concept: Apthous stomatitis is characterized by the repeated formation of ulcers in the mouth, in otherwise healthy individuals. b. Apthous stomatis ulcers occur periodically and heal completely between attacks with pain symptoms ranging from a minor nuisance to interfering with eating and drinking. c. Ulcerations present in apthous stomatitis cases are NOT preceded by vesicular lesions. d. All other conditions listed are characterized by the presence of ulcers with vesicular lesions.

Cavity liner placement should be limited to which of the following? Select all that apply. A. Areas of prep where there is a possibility of a minute pulpal exposure B. Areas where carious tissue is left in the prep C. The entire axial wall of prep D. Used in conservative sized preps instead of an adhesive agent

A. Areas of prep where there is a possibility of a minute pulpal exposure B. Areas where carious tissue is left in the prep 1) Concept: The use of cavity liners should be limited to those areas of the preparation where there is a possibility of a minute pulpal exposure. 2) The placement of cavity liners protects the endangered pulp from thermal, chemical, electrical, and mechanical trauma. 3) The choice of cavity liner material depends upon the remaining dentin thickness. If the remaining dentin thickness is greater than 1.5mm, glass ionomer cement can be used as cavity liner but for a remaining thickness of 0.5mm or less, calcium hydroxide should be used, then glass ionomer should be placed. 4) Application of cavity liners along the entire wall will reduce the retention or bonding of restorative material.

Which of the following is the least effective method of addressing and diagnosing TMJ disorders? A. Arthrography B. Arthroscopy C. Magnetic resonance Imaging D. Pantomograph E. Cone-beam computed tomography

A. Arthrography 1) Arthography a. considered the least effective tool in the assessment and diagnosis of a temporomandibular joint (TMJ) dysfunctions b. involves multiple radiographic images with contrast medium in order to assess different aspects and angles of the TMJ and are difficult to place and interpret 2) Pantomographs a. Radiographic images used in orthography are 2-dimensional representations of a 3-dimensional object, so accurate diagnoses are difficult to make at best b. 3) Arthroscopy a. less invasive surgical procedure wherein an arthroscope is inserted into a small incision approximating the TMJ to directly examine or treat inferior portion of the joint 4) Cone-Beam Computed Tomography (CBCT) a. provides clear and accurate images of different orientations of the condylar head and the glenoid fossa through their para-sagittal, axial and coronal image reconstruction views 5) Magnetic Resonance Imaging (MRI) a. currently the modality of choice in assessment and diagnosis of TMJ because it can provide direct visualization of the articular disk and intra-articular abnormalities within the TMJ

Which of the following are recognized methods to cure resin sealants? Select all that apply. A. Autocuring B. Chemical polymerization C. Photopolymerization

A. Autocuring B. Chemical polymerization C. Photopolymerization 1) Concept: Pits and sealants are most efficient in sealing the caries susceptible in a tooth 2) Pits and fissure sealants are placed after minimal and no preparation of the tooth surfaces and then cure to make a hermetic seal. 3) The curing of sealants can occur through chemical or auto-curing as is the case in self cure pits and fissure sealants. 4) VLC sealants are cured through the mechanism of photo-polymerization with the help of visible light curing units.

Nociception from the face is transmitted on all of the following pathways EXCEPT one. Which pathway is the one EXCEPTION? A. Autonomic nerves B. Cervical spinal nerves C. Ninth cranial nerve D. Fifth cranial nerve

A. Autonomic nerves 1) Concept: Autonomic nerves are related to muscular or motor reflexive functions including peristalsis, breathing, and heart beat and are not involved in pain perception. 2) Nociception a. process of transmitting and identifying noxious stimuli from the nociceptors of the peripheral nervous system 3) Cranial nerve V and IX are both sensory and motor nerves that transmit nociceptive stimuli from the face.

A 6-year old presents to your office for extraction of L and S. The mandibular incisors have not yet erupted. The indicated method of space maintenance for this patient is... A. Band and loop B. Distal shoe C. Lower lingual holding arch D. Nance appliance E. No treatment needed, only observation

A. Band and loop 1) Concept: Band and Loop a. Band and loop is the best space maintainer for the early loss of a mandibular primary 1st molar when mandibular permanent incisors are not yet present b. fixed appliance made to prevent the medial movement into extraction sites and: 1) are durable since they are cemented to tooth structure 2) remain in the patients mouth until its permanent successor erupts 3) demonstrate high compliance rate since it is a fixed device 2) Lingual holding arch a. are contraindicated in cases where mandibular permanent incisors have not yet erupted 3) Nance appliances a. are cemented on the maxillary permanent 1st molars and have an acrylic button that rests on the palate to resist forward movement 4) Hawley appliances a. are used to actively move teeth, not necessarily maintain position in the dental arch

Which composite resin constituent provides the most radiopacity? A. Barium B. Fumed Silica C. Silicate D. Quartz

A. Barium 1) Composite Fillers a. like quartz, silica, barium, strontium, aluminjm, and zirconium are incorporated in the composite resin for strength, translucency, and radiopacity 2) Barium a. metal particle that is a well-known replacement filler for quartz because of its ability to improve the radiopacity of composites

To extend the life of a tungsten carbide bur, the bur should... A. Be moving fast before contacting tooth structure B. Be moving slow before contacting tooth structure C Contact tooth structure before rotating D. Be sterilized with heat sterilization

A. Be moving fast before contacting tooth structure 1) The lifespan of a tungsten carbide bur can be extended by bringing the bur up to speed before contacting tooth structure, because it decreases the friction between the bur and the tooth structure thereby decreasing damage to the bur and the pulp. 2) Sufficient rotating speed makes the cutting edge of the bur more efficient even with using light pressure. 3) Heat sterilization corrodes the carbon steel and anneals the cutting edge of the bur.

The proper treatment for a tooth with an uncomplicated/incomplete crown fracture (pulp is not exposed) would be... A. Be vitality tested to determine the extent of pulpal damage B. Have a stainless stell crown placed C. Have the chipped enamel recontoured and smoothed D. Not need to be evaluated with thermal and electric pulp tests E. Immediately direct pulp capped and restored

A. Be vitality tested to determine the extent of pulpal damage 1) Concept: It is important to immediately evaluate tooth vitality as well as the extent of the crack, which in this case is suspected of being an incomplete crown fracture 2) The results of pulp vitality tests will help you to know if the pulp is still vital and unaffected by the fracture. It is essential to determine extent of the incomplete fracture and its cause. 3) If the tooth is vital an the incomplete fracture cause some discomfort, the fractured tooth surface must be restored. 4) The patient should regularly visit the dentist for re-evaluation of the tooth structure and the tooth vitality. 5) Crazing is when there is a fracture line that does not extend to dentin.

Upon routine examination, you notice a rubbery, firm, asymptomatic 2mm circumscribed exophytic nodule within the main body of the parotid gland. The patients states that they have noticed the nodule for about "a year or so". Which of the following is the MOST LIKELY diagnosis? A. Bengin mixed tumor B. Encapsulated sialolith C. Lymph node D. Mucocele E. Squamous cell carcinoma

A. Bengin mixed tumor 1) Concept: Benign mixed tumor or pleiomorphic adenoma presents as a rubbery, firm, asymptomatic nodule within the main body of the parotid land. It accounts for 90% of the benign tumors occurring in the gland. It occurs as a preexisting lesion of long duration. 2) Encapsulated Sialolith a. is a calcified mass with laminated layers of inorganic material b. commonly occurs in the subandibular gland and presents with pain 3) Mucoceles a. occur on the inside of lower lips, gums, roof of the mouth and under the tongue b. they can cause more problems with speech, chewing, and swallowing 4) Squamous cell carcinoma a. malignant condition

Each of the following is a component of the chemical reaction for light initiated composite resin polymerization EXCEPT one. Which is the EXCEPTION? A. Benzoyl peroxide B. Camphoroquinone C. Tertiary amine D. ~468 nm blue light

A. Benzoyl peroxide 1) Concept: Benzoyl peroxide is used as an initiator in self cure or auto-curing composite resins 2) Camphorquinone is an initiator and tertiary amines are activators for light activated composite resins. 3) VLC composites need a light of wavelength approximately 468 nm to initiate (start) the polymerization reaction.

The cariostatic effect of fluoride is manifested during which of the following stages of tooth development? A. Calcification B. Histodifferentiation C. Proliferation D. Apposition

A. Calcification

When sharpening instruments, the _____ of the blade should be placed at a _____ angle to the sharpening stone. A. Bevel, 45o B. Bevel, 90o C. Working end, 45o D. Working end, 90o

A. Bevel, 45o 1) When sharpening an instrument, the bevel of the blade is placed at an angle of 45o to the sharpening stone. This ensure maximum efficiency of the blade to sharpen the instrument. 2) Instrument sharpening is an important aspect of operative dentistry. Dull instruments are not recommended for carrying out procedures in the oral cavity. 3) Dull instruments a. produce more heat b. less efficient and can damage tooth structure c. recommended to discard those instruments which cannot be sharpened.

The _____ of the gingival marginal trimmer corresponds to the nib of the _____? A. Blade, Condenser B. Blade, Hatchet C. Blade, Periodontal Probe D. Shaft, Forceps E. Shank, Explorer

A. Blade, Condenser 1) The blade of the gingival marginal trimmer corresponds to the nib of the condenser. 2) Gingival margin trimmers are cutting instruments while condensers are non cutting instruments. 3) In non-cutting instruments, the blade is replaced by a nib or point 4) Nibs a. working ends or nibs of the condenser may be of any shape but they are usually round with flat ends. Triangular, rectangular or diamond shaped nibs are also used.

Each of the following is a clinical sign of gingivitis EXCEPT one. Which one is the EXCEPTION? A. Bleeding on probing B. Erythema C. Gingival stippling D. Swollen tissues

A. Bleeding on probing 1) Bleeding a. probing erythematous inflamed tissue typically results in bleeding 2) Stippling a. numerous small depressions in the surface of healthy gum tissue 3) Gingivitis causes a. edema, which creates a loss of stippling b. erythema

Which of the following describes an alveoloplasty procedure? A. Bone is smoothed to remove ridge undercuts

A. Bone is smoothed to remove ridge undercuts 1) Alveoloplasty a. smoothing and recontouring of alveolar bone to create a typography favorable for proper wound healing and denture support 2) Types 3) Post extraction alveoloplasty a. removes the alveolar ridge to remove knife edge residual ridges and spiny bony projections which are not good denture stress bearing areas b. remove undesirable ridge undercuts which alter the path of denture placement and consequently affect retention 4) Cortical alveoloplasty a. recontours and smooths bone so that no spiny bone projections or sharp bone margins are present to ensure optimal healing 5) Intraseptal alveoloplasty a. removes labial undercuts, reduces maxillary over-ject and recontours cortical bone by removing interradicular bone and collapsing the cortical plates preserving dentures stress bearing area

Which of the following MOST ACCURATELY describes trigeminal neuralgia (tic douloreux)? A. Brief periods of sharp excruciating pain B. Dull pain with pressure applied to the affected area C. Unilateral episodes of prolonged facial pain D. Unilateral paralysis of the face E. Unilateral uncontrolled twitching

A. Brief periods of sharp excruciating pain 1) Tic douloureux a. Characteristics 1) having brief episodes of sharp/excruciating pain 2) experience paroxysmal attacks of pain lasting from a fraction of second to 2 minutes 3) Affecting one of more divisions of the trigeminal nerve 4) Pain may or may not be persistent between paroxysms 5) Pain is usually a sudden and sharp/stabbing feeling after stimulating a trigger zone 2) Trigger zones a. Trigger zones are affected areas within the branches of the trigeminal nerve b. Trigger zones in one branch, and may spread to other branches over time. The most commonly affected branches are V2 and V3, but pain can be in just about any location on the head, and even the index finger.

Which of the following should be performed in regards to calcium hydroxide (Ca(OH)2) placement during indirect or direct pulp capping procedures to ensure optimal thermal and protective insulation of the pulp? A. Ca(OH)2 should be covered by a stronger base before the restoration is completed B. Cavity varnish should be placed before Ca(OH)2 is placed C. ZnPO4 should be placed before Ca(OH)2 D. Ca(OH)2 should be placed at least 2.5mm thickness around the pulp

A. Ca(OH)2 should be covered by a stronger base before the restoration is completed 1) Concept: A stronger base will ensure that the calcium hydroxide and pulp are well protected and sealed against irritants that may cause further harm to the pulp. 2) Calcium hydroxide dissolves easily and exhibits weak mechanical properties as a base. 3) Calcium hydroxide bases should stay undisturbed when approximating the pulp in order to induce dentin bridge formation within the dentin pulp complex.

Following trauma to primary maxillary incisors, which of the following would MOST LIKELY cause yellowish discoloration> A. Calcific metamorphosis B. Internal resorption C. Pulp hyperemia D. Pulp necrosis E. Blood pigments

A. Calcific metamorphosis 1) Calcific metamorphosis a. is a common reason that teeth appear yellow after trauma occurs and is recognized clinically as early as 3 months following injury. b. characterized by hard tissue within the root canal space of a tooth causing yellow discoloration of the clinical crown

Which of the following types of dental anomalies manifests form a disturbance during the initiation and proliferation stages of tooth development? Select all that apply. A. Calcification B. Mineralization C. Number D. Shape E. Size

A. Calcification C. Number 1) Initiation and Proliferation Stages a. hypodontia b. oligodontia c. anodontia d. hyperdontia e. supernumerary teeth 2) Initiation a. Hyperdontia = patient has more than the normal amount of teeth = supernumerary teeth b. cleidocranial dysplasia =

Which of the following medications may cause gingival overgrowth? A. Calcium channel blockers, phenytoin, and cyclosporine

A. Calcium channel blockers, phenytoin, and cyclosporine 1) Gingival overgrowth a. Calcium channel blockers b. Phenytoin c. Dilantin d. Cyclosporine

Composite resin restorations... A. Can be placed and finished at the same appointment B. Chemically bond to dentin C. Have a thermal coefficient of expansion similar to amalgam D. Release fluoride

A. Can be placed and finished at the same appointment 1) Concept: Composite restorations are restorative materials popularly used because of its: a. High esthetic value b. Ease of manipulation c. Durability 2) Glass Ionomer cement is a restorative material that a. releases fluoride into surrounding tooth structure b. chemically bonds to tooth structure c. is less resistant to wear than composite

4 minutes following the administration of an inferior alveolar nerve block. The patient develops paralysis of their forehead muscles, eyelids, and upper and lower lips o nthe same side of their face as the injection. These clinical signs are most associated with the flowing of the anesthetic into which of the following structures. A. Capsule of parotid gland B. Motor branches of mandibular nerve supplying muscles of mastication C. Ophthalmic division of trigeminal nerve (V2) D. Otic ganglion E. Auriculotemporal nerve

A. Capsule of parotid gland 1) Deposition of anesthesia near the parotid gland caused a block of the facial nerve which resulted to the drooping of the face, paralysis of the forehead muscles and eyelid paralysis.

The most serious effects of repeated exposure to X radiation in low doses is characterized by which of the following? A. Carcinogenesis B. Diarrhea C. Pupura D. Radionecrosis E. Alteration of oral microflora

A. Carcinogenesis 1) Concept: Carcinogenesis a. Repeated low doses of ionizing radiate may induce irreparable DNA damage which leads to errors in replication and transcription that result in carcinogenesis b. Persistent low dose radiation can induce a type of genomic instability in cells that may trigger viral interactions leading to pre-mature aging and carcinogenesis c. Ionizing radiation results in the enhancing of the frequency of mutation and other genetic changes in the descendants of the irradiated cell after many generations of replication. d. Gap junction mediated cell-cell communication and activation of the p53 damaged response pathway may result in carcinogenesis.

Which of the following terms describe the line, or angle formed by the junction of a cavity all with unprepared surface of the tooth? A. Cavosurface angles B. Dimensional angles C. Line angles D. Point angles E. wall

A. Cavosurface angles 1) Concept: The line or angle formed by the junction of a cavity wall with the unprepared surface of the tooth, that is a continuous entity enclosing the entire external outline of the prepared cavity, is termed as the cavo-surface angle. 2) Point angle a. angle formed between 2 cavity walls and a floor or between 3 walls 3) Line angle a. is formed between 2 walls or a wall and floor of a cavity

Which of the following describes which cell layer the nerve plexus of raschkow resides in? A. Cell-free layer B. Cell-rich layer C. Central core of the pulp D. Odontoblastic layer E. Only in the apical 1/3 of the tooth

A. Cell-free layer 1) Cells found in the dental pulp a. fibroblasts (principal cell) b. odontoblasts c. defense cells like 1) histiocytes 2) macrophages 3) granulocytes 4) mast cells 5) plasma cells 2) Nerve Plexus of Raschkow a. located central to the cell-free zone b. monitors painful sensations as well as mediation as inflammatory events and subsequent tissue repair 3) A-fibers a. myelinated nerve that conduct rapid and sharp pain sensations b. close association with the odontoblasts and extend fibers to many but not all dentinal tubules 4) C-fibers a. thinner, non-myelinated nerved involved in dull aching pain b. terminate in the pulp tissue proper, either as free nerve endings or as branches around blood vessels

Ameloblastoma in the mandible are similar to which of the following conditions radiographically? A. Central giant cell granuloma B. Osteosarcoma C. Squamous cell carcinoma D. Ameloblastic fibro-odonoma

A. Central giant cell granuloma 1) Central giant cell granuloma a. benign neoplasm that usually occur in the mandible and occur in women twice as often b. characterized by a large radiolucent multilocular lesion that expands the cortical plate and can reabsorb roots and move teeth c. aggressive form of this condition exhibits rapid growth and is much more likely to absorb roots and perforate the cortical plate and has a high rate of recurrence and can be painful or cause paresthesia 2) Ameloblastomas and Central giant cell granulomas a. radiolucent multilocular lesions b. most likely to occur in the mandible of women age 20-40 years old and often cross the midline

Which of the following portion of primary teeth is staining MOST evident? A. Cervical B. Distal C. Facial/Buccal D. Incisal/Occlusal E. Lingual F. Mesial

A. Cervical 1) Cervical bulbs can trap debris underneath 2) Cervical portions are more constricted in primary teeth making plaque, caries, and stain formation easier

A child with a distal step during the primary dentition will most likely develop into which of the following permanent molar relationships? A. Class II B. Class III C. Class I

A. Class II 1) Concept: Distal Step a. Presence of a distal step during the primary dentition usually results in Class II molar relationship 2) Mesial Step a. Mesial step occlusion usually results in either Class I or Class III molar relationship after eruption of permanent teeth 3) Flush Terminal Plane a. Children with a flush terminal plane occlusion however end up in a Class I, Class II, or end to end type of molar relationship

In which order should an airway obstruction be managed in an unconscious patient? A. Clear the pharynx, check breathing, extend the patient's neck, protrude the tongue and mandible B. Extend the patient's neck, Protrude the tongue and mandible, clear the pharynx C. Extend the patient's neck, clear the pharynx, protrude the tongue and mandible D. Protrude the mandible and tongue, clear the pharynx, extend the neck

A. Clear the pharynx, check breathing, extend the patient's neck, protrude the tongue and mandible 1) Concept: The proper management of airway obstruction of an unconscious patient should be done in the following order: a. Visually or tactilely examine any presence of food, vomit or any foreign object that may obstruct the airway. b. Observe and check the rise and fall of the chest as indicator of breathing. Listen to the mouth and nose for signs of breathing. c. Extend the patient's neck by tilting their chin upwards. d. Protrude the tongue and mandible to provide better mouth opening an to pharynx will open further and improve the patients breathing.

Which of the following disorders is characterized by a delayed ossification of midline structures and supernumerary teeth? A. Cleidocranial dysostosis B. Ectodermal dysplasia C. Neurofibromatosis D. Osteogenesis imperfecta E. Amelogenesis imperfecta

A. Cleidocranial dysostosis 1) Concept: Cleidocranial dysostosis (dysplasia) a. autosomal dominant disorder where there is delayed ossification of midline structures (clavicles), supernumerary teeth, and a prognathic mandible. b. general skeletal condition named by the collarbone (cleido-) and cranial deformities which patients with this disorder often exhibit c. Clavicle presence can range from partly missing to completely missing in 10% of cases, which allows hyper mobility of the shoulders including ability to touch the shoulders together in front of the chest d. mandible is typically prognathic due to hypoplasia of maxilla (micrognathism) and other facial bones e. Pantomographs typically demonstrate multiple unerupted supernumerary teeth resembling premolars, missing genial angles and underdeveloped maxillary sinuses f. Supernumerary teeth often appear unless they reabsorb before adolescence, otherwise they will crown the permanent teeth in an already underdeveloped jaw and will need to be extracted to make space for the permanent dentition g. Bossing (bulging) of the forehead is also observed in these patients. h. A soft spot on the top of the head may appear because the fontanelle failed to close, or closed late.

Which of the following area is primary responsible for the vertical growth of the mandible? A. Condylar Head B. Coronoid process C. Mental protuberance D. Sigmoid Notch E. Alveolar Process

A. Condylar Head 1) Concept: Growth of the condylar head is most responsible for the increase in mandibular vertical height 2) Growth of the alveolar process, coronoid process, mental protuberance, and sigmoid notch do not push the mandible away from the cranium, and therefore are not responsible for the increase in vertical facial height

Which of the following is the most likely diagnosis for a patient presenting to you with a unilateral, slowly progressing elongation of their face causing malocclusion and deviation of their chin away from the affect side? A. Condylar Hyperplasia B. Myofascial pain disorder C. Osteoarthritis D. Rheumatoid arthritis E. Tooth ankylosis

A. Condylar Hyperplasia 1) Condylar Hyperplasia a. idiopathic, slowly progressive unilateral enlargement of the head and neck of the condyle b. typically results in crossbite malocclusion, facial asymmetry, and shifting of the midpoint of chin to the unaffected side c. treatment is condylectomy if it occurs during the active growth period d. orthodontics or surgical mandibular repositioning is performed following growth cessation

Which of the following is/are true of convention Glass Ionomers? Select all that apply. A. Contain basic ion-leachable glass B. Contain water soluble polyacids C. Contain water soluble polymerizable resin D. Use free radical polymerization to cure

A. Contain basic ion-leachable glass B. Contain water soluble polyacids 1) Concept: Conventional glass ionomer contain water soluble polyacids and basic ion leachable glass in their composition. 2) Convetional glass ionomer cements a. undergo a chemical self-setting acid base reaction created by mixing an ion leachable fluoroaluminosilicate glass power with an aqueous polyacrylic or polycarboxylic acid. 3) Resin modified glass ionomers a. were formulated by adding a water soluble polymerizable resin such as 2-HEMA in the acidic cement liquid

Which of the following is NOT an indication for posterior maxillary osteotoy? A. Correction of mandibular retrognathism B. Intrusion of the maxillary tuberosity to prepare for denture fabrication C. Skeletal arch width discrepancies that require lateral positioning of posterior segments D. Closure of the anterior open bite

A. Correction of mandibular retrognathism 1) Posterior maxillary osteotomy a. a surgical technique used to move the maxillary posterior bone segment into a more desirable position in order to correct occlusal disharmony. b. reposition the maxillary posterior segment in order to help correct occlusion problems but CANNOT correct mandibular retrognathism

Acid etching teeth for resin composites will accomplish which of the following. Select all that apply. A. Create a clean surface to bond to B. Decrease thermal conductivity C. Increase retention of the restoration D. Prevent staining/color change E. Prevent microleakage

A. Create a clean surface to bond to C. Increase retention of the restoration E. Prevent microleakage 1) Concept: Acid etching teeth creates microporosities that adhesives can infiltrate to create retention for composite resin bonding. 2) Acid etching roughens the exposed enamel rods and dentin, thus creating micro mechanical abrasions that allows the bonding agent to flow and adhere to the tooth surface. 3) Etching also removes surface debris, allowing for a clean bonding site. 4) Acid etching creates higher bond strength between composite restoration and tooth structure while also decreasing micro leakage.

Which of the following conditions is characterized by chronic respiratory infections, functional disturbances in the secretory mechanisms of various glands, and steatorrhea? A. Cystic fibrosis B. Diabetes Type I C. Mercury poisoning D. Acquired immune deficiency syndrome

A. Cystic fibrosis 1) Concept: Cystic fibrosis is a genetic disorder resulting in the inability of the body to bring salt and water in and out of cells. 2) The situation results in the excessive production of sticky and viscous mucus in the lungs and digestive tract. 3) Cystic fibrosis also affects the production of digestive enzymes that break down fats, causing them NOT be absorbed, resulting in a fatty stool called steatorrhea. 4) Cystic fibrosis also affect the reproductive system and alters the function of sweat glands in the body.

Which of the following conditions is Lymphangioma related to MOST CLOSELY? A. Cystic hygroma B. Hemangioma C. Hemangiopericytoma D. Angiosarcoma

A. Cystic hygroma 1) Cystic Hygroma a. lymphatic malformations b. congenital multiloculated lymphatic lesion that can arise anywhere, found mainly in the left posterior triangle of the neck in children c. considered the most common form of lymphangioma d. contain large cyst-like cavities contain lymph and decrease in size as they approach the surface of the skin surface e. benign f. can be disfiguring g. Types 1) Macrosystic lymphatic malformations (large cysts) 2) Microcystic (small cysts)

The suggested initial treatment for a patient that presents to your office with HIV associated necrotizing ulcerative gingivo-periodontitis is which of the following? A. Debridement with prescription of chlorhexidine 0.12% B. Gingivectomy C. Scaling and root planing with curettage D. Debridement

A. Debridement with prescription of chlorhexidine 0.12% 1) Concept: Necrotizing ulcerative gingivo-periodontitis associated with HIV+ patients should be handled with utmost care specially because of the immunosuppressed condition of the patient. 2) Necrotizing ulcerative gingivitis and periodontitis usually occurs because of the predominance of the anaerobic fusobacteria and spirochetes within the oral cavity, specifically underneath the gingiva. 3) Debridement of the affected gingiva with an adjective prescription of chorhexidine 0.12% rise will help stop the progression of necrotizing ulcerative disease. 4) Necrotizing Ulcerative Gingivitis/Periodontitis a. Sloughing Gingiva b. Blunted papillae c. Fetid Odor d. Bone Loss with Perio

The letter "D" in DMFS stands for... A. Decayed D. Demineralized

A. Decayed 1) DMFS D: decayed or carious tooth surfaces of all teeth present within the oral cavity M: missing tooth surfaces of all teeth present within the oral cavity F: filled surfaces of all teeth present within the oral cavity S: surfaces T: teeth

Over-mixing an admix (dispersalloy) amalgam capsule can result in each of the following outcomes EXCEPT one. Which is the EXCEPTION? A. Decrease in strength of admix alloys B. Decreased working time C. Increase in creep D. Increased contraction

A. Decrease in strength of admix alloys 1) Concept: Over mixing admix amalgam alloy capsule does NOT result in a decrease in overall strength 2) The triturated amalgam should be bright and have a plastic consistency. a. Amalgam that appears excessively splashy and wet looking indicates that the trituration time or amalgamator speed should be increased because it is undermined. b. Amalgam that appears excessively dry indicates trituration time or amalgamator sped should be decreased because it is over mixed. 3) When performing adjustments, it is recommended to adjust trituration time in 2 second increments, until optimum mix is obtained. Over mixing admixed alloy decreases the working time, increases contraction, and creep values. 4) Dispersalloy is designed for use in stress bearing restorations e.g. Class 1 and 2 preparations, when other restorative materials or restoration techniques are NOT indicated.

Which of the following statements describe the key objective of surgery as part of a patient's overall periodontal therapy? A. Decrease the depth of the periodontal pocket B. Eliminate all of the bacteria associated with periodontal disease C. Make is so the patient no longer needs scalding and root planing D. Remove calculus

A. Decrease the depth of the periodontal pocket 1) Concept: The main goal of respective or regenerative surgery as part of periodontal therapy is to reduce pocket depths. 2) Osseous surgery a. includes the recontouring of both hard and soft tissues 3) Resective surgeries a. include gingivectomy and flap surgeries 4) Regenerative surgeries a. include grafting and the use of barrier membranes

Adjusting the shade of restorations using complementary colors will result in which of the following? A. Decreased value B. Increased translucency C. Increased value D. Decreased hue

A. Decreased value 1) Concept: Decrease in value occurs when complimentary colors are used to adjust the shade of a restoration. 2) This becomes a compromising situation when relative brightness is sacrificed for the sake of shade matching. 3) Value describes the relative lightness or darkness of a material. 4) Variations in value are used to create a focal point for the design of a restoration. 5) Gradations of value are also used to create the illusion of depth in a restoration. Value can be remembered by thinking that patients VALUE WHITE teeth: 1) Lighter colors are of increased value 2) Darker colors are of decreased value

Zinc is added to lower-copper amalgam for each of the following reasons EXCEPT one. Which is the EXCEPTION? A. Decreases the restoration longevity B. Enhance its mechanical properties C. Produced delayed expansion when contaminated with moisture D. Reduce marginal fracture rates

A. Decreases the restoration longevity 1) Concept: Zinc in low-copper amalgams do NOT decrease the restoration longevity. 2) If a low-copper, zinc-containing alloy is moisture contaminated, it will result in surface blistering, internal corrosion, and a delayed expansion of up to 4% by volume beginning 3 to 5 days after the contamination and continuing for up to six months, which can lead to a reduction in strength of up to 24%. 3) Delayed expansion and corrosion products result in sealing the restoration which reduces marginal voids and chances of fractures.

Which of the following alterations does free gingival graft epithelium undergo? A. Degeneration B. Dysplasia C. Keratohyalin granule formation D. Orthokeratinization E. Proliferation

A. Degeneration 1) Concept: Free gingival grafts may undergo degeneration since they are harvested from a donor site and are suddenly transferred to the recipient site 2) Free gingival graft procedures a. performed to increase the keratinized gingival tissue support of an implant material or to cover up areas of gingival recession b. usually harvested along the palatal area and is later transplanted and sutured into place at the recipient site. c. during the first few days, the free gingival graft, it may undergo slight degeneration due to the lack of direct blood and nutrient supply. The free gingival graft is able to survive through the help of the underlying connective tissues bed that provides nutrition to its surviving epithelial cells.

How far light is able to travel into tooth structure before being reflected back outward is termed... A. Degree of translucency B. Hue C. Metamerism D. Value E. Penetrance

A. Degree of translucency 1) Translucency a. distance travelled by light into tooth structure before it is reflected back outward is known as its degree of translucency 2) Metamerism a. when the angle of lighting of type of light source make colors appear as the same or different 3) Value a. refers to the lightness or darkness of a certain area or color 4) Hue a. is the color itself and its distinctive shades 5) Penetrance a. degree of the ability of light to pass through

Which of the following is not classified as an infra bony defect? A. Dehiscence B. Interental crater defect C. Trough D. Hemiseptum

A. Dehiscence 1) Infrabony defect a. bone level is apical to the base of the periodontal pocket 2) Root dehiscence a. bone defect where the root surface is covered only with soft tissue after bone destruction occurred on the facial or the lingual aspect of the tooth 3) Interdental crater a. bone defect where a concavity is found in the crystal bone 4) Trough a. bone defects where a crater is formed on the proximal surfaces of the tooth 5) Hemiseptum a. vertical or angler bone defects where one surface of the tooth root has bone support remaining

American Dental Association (ADA) officially recognizes which of the following as a dental specialty? A. Dental public health B. Esthetic dentistry C. Geriatric dentistry D. Implantology E. Hospital dentistry

A. Dental public health 1) dental specialization which focuses on epidemiology, analysis of social health policies, and the dental needs of the general population

Which of the following is most frequently noted in patient populations suffering from chronic temporomandibular joint disorder? A. Depression psychosis B. Passive-aggresive behavior C. Schizotypical behavior D. Sociopathy E. Borderline personality disorder

A. Depression psychosis 1) Depression psychosis a. condition which may worsen with the presence of TMJ dysfunction b. presence of depression is a common condition among people experiencing chronic pain, especially TMD c. most pone to TMJ possess a malocclusion d. factors that cause TMJ disease 1) grinding teeth during sleep (bruxism) 2) overbite 3) rheumatoid arthritis

Pain and tenderness upon palpation of the temporomandibular joint are usually associated with... A. Deviation of the jaw to the painful side upon opening of the mouth B. Flaccid paralysis of the non painful side of the face C. Flaccid paralysis of the painful side of the face D. Impacted mandibular third molars E. Excitability of the second division of the Trigeminal Nerve (V2)

A. Deviation of the jaw to the painful side upon opening of the mouth 1) Jaw of the patient deviates to the painful side during mouth opening because the jaw is limiting itself from excessive movements that may cause further pain and damage to the joint 2) In order to lessen the movements within the painful side of the jaw during jaw opening, the side of the jaw that experiences pain limits its movements. 3) The condyle on the non-painful side maintains its function and therefore appears to move toward the painful side of the jaw.

Which of the following describes the putative explanation for why dental plaque adheres to tooth surfaces? A. Dextrans are insoluble and sticky B. Levans are insoluble and sticky

A. Dextrans are insoluble and sticky 1) After dental plaque is formed, it adheres to the surface of tooth through dextrans. 2) Dextrans are insoluble and sticky and serve to increase the tenacity of the attachment of dental plaque to the surface of the tooth 3)

Fluoride ions are most likely to be inactivated by which of the following dentifrice components. A. Dicalcium phosphate B. Monofluorophosphate C. Polyacrylic spheres D. Anionic detergents

A. Dicalcium phosphate 1) Concept: The interaction of fluoride ion and dicalcium phosphate dehydrate results in the formation of insoluble fluoride compounds that reduce the amount of free and available fluoride ions. 2) Fluoride ions that interact with dicalcium phosphate dehydrate form calcium fluoride. 3) IN order to make more fluoride ions interact with teeth, incorporating dicalcium phosphate dehydrate within the fluoride pastes must be limited.

The MOST COMMON cause of intracapsular restraint of mandibular movement is described by which of the following? A. Disc interference disorders B. Infection C. Myofacial pain disorders D. Akylosis

A. Disc interference disorders 1) Disc interference disorders a. most commonly cause intracapcular restraint of mandibular movement in adults b. associated with dysfunctional condyle disc complex and are also termed as internal derangement disorders c. anterior and medial displacement or dislocation of the articular disc d. Signs and symptoms 1) clicking sounds 2) catching or locking during jaw movements 3) may or may NOT be associated with pain

A chronic infection of the mandible of a middle-aged female patient was treated with oral Pen Vk for 3 weeks. The primary infections appears to be resolved, but the patient now has white patches on the dorsum of her tongue and buccal mucosa. Which of the following is the suggested treatment for this patient? A. Discontinue Pen Vk and prescribe nystatin troches. B. Discontinue Pen Vk treatment and prescribe a broad spectrum antibiotics C. Discontinue all antibiotics only D. Discontinue Pen Vk and prescribe a mouthwash containing metronidazole

A. Discontinue Pen Vk and prescribe nystatin troches. 1) Prolonged intake of antibiotics disturbs the balance of the microflora in the mouth by killing the "normal" bacteria, allowing the fungal species like Candida albicans to grow unhindered by competition. 2) The white patches on the dorsal surface of the tongue that appeared after prolonged antibiotic uses are caused by the increased number of Candida albicans.

Which of the following clinical signs is indicative of acute herpetic gingivostomatitis? A. Discrete spherical vesicles B. Necrotic ulceration of interdental papilla C. Solitary, localized, edematous gingival lesion D. Cold-sensitive teeth

A. Discrete spherical vesicles 1) Concept: Gingivostomatitis is an inflammation of the oral mucosa and gingiva caused predominantly by HSV-1. Herpes presents as numerous pin-head vesicles, which rupture rapidly to form painful irregular ulcerations covered by yellow-gray membranes. 2) Primary herpetic gingivostomatitis a. is an infection transmitted through direct contact or through aerosol droplet that clinically apparent as yellow fluid-filled vesicles that leave ulcers when the vesicles rupture. 3) Secondary herpetic gingivostomatitis a. a reactivation of the latent virus due to trauma, stress or immunosuppression. b. The vesicles maybe found intramurally or extra orally on keratinized surfaces. 4) Prodromal symptoms include a. Fever b. Anorexia c. Irritability d. Malaise

Which of the following directions would a mandibular 2nd premolar move if the 1st molar was extracted A. Distally B. Lingually C. Mesially D. None

A. Distally 1) Concept: Mandibular premolars will move distally because of the presence of space distally and the resorption of the interseptal bone that separates i from the mandibular molar extraction site. 2) The alveolar bone between the mandibular 2nd premolar and 1st molar will resorb, causing it to become thinner and weaker there by providing less support to the mandibular 2nd premolar. 3) The 2nd premolar will have a tendency to collapse into the bony area that is thin, weak, and less resistant.

Which of the following describe the advantages of resin based composite (RBC) as a posterior restorative material? A. Elimination of galvanic currents B. High fracture toughness C. Increased wear resistance D. Low degree of elastic deformation E. No secondary caries

A. Elimination of galvanic currents 1) Resin Based Composites (RBC) a. elimination of galvanic currents is an advantage of posterior (RBC) restorations compared to posterior amalgam restorations b. insulators of electric currents and do NOT transmit electric impulses which can endanger the pulp tissues c. lower wear resistance d. low fracture toughness e. high degree of elastic deformation than amalgam fillings

Which of the following dental sequelae would be MOST PROBABLE for a child to experience if they exhibit growth failure "failure to thrive" in the first 6 months of their life? A. Enamel hypoplasia B. Mandibular hypoplasia C. Microdontia D. Midface hypoplasia E. Dentinogenesis imperfecta

A. Enamel hypoplasia 1) Enamel Hypoplasia a. having signs and symptoms suggesting failure to thrive like enamel hypoplasia might also be suffering from ectodermal dysplasia b. Ectodermal hypoplasia 1) may affect all types of tissues that originated from the ectodermal embryonic layer 2) incomplete development of the following a. hair b. nails c. sweat glands d. enamel of teeth e. oil glands

Which weight loss supplement is considered dangerous due to reported case of sudden cardiac death? A. Ephedra B. Garlic C. Ginseng D. Resveratrol E. Echinacea

A. Ephedra 1) Concept: Reports of serious side effects and ephedra-related deaths caused the U.S. Food and Drug administration (FDA) to ban the sale of ephedra-containing supplements on April 12, 2004. 2) The stimulant and thermogenic effects of Ephedra are due to the presence of ephedrine and pseudo ephedrine, which stimulate the brain, increase heart rate, and constrict blood vessels (increasing blood pressure).

Referred periauricular pain is most common with... A. Mandibular molars B. Mandibular premolars C. Maxillary premolars D. Maxillary molars

A. Mandibular molars 1) The pain that originates from mandibular molars is sometimes referred toward the ear because both are innervated from one source - the mandibular nerve (V3). 2) The nerves from the mandibular molars transmit pain stimuli to the mandibular nerve which also provide sensory innervation to some parts of the external ear. 3) The same origin of innervation and the proximity of the external ear to mandibular molars, the pain perceived within the molar area easily travels and radiates into the external ear.

Severe dysplasia or carinoma in situ is MOST OFTEN associated histologically with which of the following clinical lesions? A. Erythroplakia B. Hairy leukoplakia C. Leukoplakia D. Lichen planus E. White sponge nevus

A. Erythroplakia 1) Concept: Erythroplakia: a. red lesions that considered to be one of the most common premalignant lesions in the oral cavity b. Erythroplakias are MOST COMMONLY found: 1) Floor of the mouth 2) Tongue 3) Soft palate 2) Histopathological examination of erythroplakias exhibit severe dysplasias, carcinoma in situ and invasive squamous cell carcinoma 3) Erythroplakia appears red due to the following premalignant changes: a. absence of keratin production b. reduce number of epithelial cells c. increased vascularity due to inflammation d. thin and neoplastic epithelium

Each of the following aids in the diagnosis of evaluation of Sjogren's syndrome EXCEPT ONE. Which is the EXCEPTION? A. Exfoliative cytology B. Labial salivary gland biopsy C. Salivary flow rate D. Serology E. Sialograms

A. Exfoliative cytology 1) Exfoliative cytology a. uses the scraped off or exfoliated tissues to evaluate and examine lesions through cytopathology analysis and CANNOT be used to diagnose Sjugren's syndrome 2) Labial salivary gland biopsy a. important diagnostic tool identifying affected salivary glands and is performed by taking a small sample of the salivary gland for analysis 3) Sialograms a. used to determine any blockage or clogged structure at the salivary ducts and glands. b. helpful tool to diagnose or rule out Sjogrens syndrome

A carious lesion is detected clinically and is confirmed radiographically extending from the distal pit to the central fossa along the central groove of #19. The planned amalgam preparation and restoration should A. Extend from the distal pit to the central fossa along the central groove. It should also include any enamel undermined by the removal of carious tooth structure B. Extend from the distal pit to the medial pit, along the central groove C. Extend from the distal pit to the medial pit, along the central groove. Buccal and lingual extension should be prepared as well, from the central fossa along the buccal and lingual grooves D. Span the entire occlusal surface, along the central groove, including both the medial and distal marginal ridges

A. Extend from the distal pit to the central fossa along the central groove. It should also include any enamel undermined by the removal of carious tooth structure 1) The depth of the preparation should be at least 1.5-2mm in order to provide strength to the restoration. 2) The pulpal floor depending upon the enamel thickness is almost always in dentin.

The classification of an impacted mandibular 3rd molar is usually based on which of the following factors? Select all that apply. A. Extent of soft tissue and/or bone coverage B. No answers are correct C. Position of the tooth in relation to the long axis of the 2nd molar D. Position of the tooth to the opposing 3rd molar

A. Extent of soft tissue and/or bone coverage C. Position of the tooth in relation to the long axis of the 2nd molar 1) Concept: The classification of 3rd molar impaction is based on their spatial positioning in relation with the long axis of the adjacent 2nd molar and the amount of tooth structure covered by bone or soft tissue. 2) Winter's classification of tooth impaction a. based on the position of the long axis of the 3rd molar in relation to the long axis of the 2nd molar 3) Pell and Gregory classification of tooth impaction a. deals with the position of the lower 3rd molar impaction within the ramus in relation with the 2nd molar.

Which of the following anatomical structures is the primary reason why the buccal aspect of posterior teeth are difficult to surgically treat for periodontal defects? A. External oblique ridge B. Genial tubercles C. Mentalis muscle D. Medial pterygoid muscle

A. External oblique ridge 1) External oblique ridge a. Height of external oblique ridge causes difficulty in accessing the buccal aspect of posterior teeth b. limits the extent of cheek retraction and insertion of instruments into the periodontal defect

When would a periodical radiograph be indicated for a pediatric patient? Select all that apply. A. Extraction B. Occlusal caries C. Pulpal therapy D. Space maintenance E. Trauma F. Interproximal caries

A. Extraction C. Pulpal therapy D. Space maintenance E. Trauma 1) Concept: Periapical Radiographs indicated for the following: a. extractions b. pulpal therapy c. trauma d. space maintenance e. pediatric patients may need to have a periodical x-rays taken prior to fabricating space maintainer to determine if space maintenance is necessary or if the successor will erupt in time f. periapical x-rays serves as a guide an diagnostic tool during pulpal therapy g. dental trauma in children necessitates periodical x-rays to help identify possible root fractures as well as the condition of the supporting bone which is involved in the trauma.

Trochlear nerve dysfunction is indicated by... A. Eyes fil to move down and out B. Eyes fail to move laterally C. Proptosis D. Dilated pupil

A. Eyes fil to move down and out 1) Concept: The trochlear nerve (CN IV) is a motor nerve which innervates the superior oblique muscle. 2) The superior oblique muscle allows rotation of the eyes within the vertical plane and the rotational movement of the eyeball towards and away from the medial plane. 3) Trochlear nerve dysfunction will prevent the eyes from moving up and down.

A 14-year old boy presents to your office stating that an area of his left anterior mandible has been slowly and progressively growing over the last two years. Radiographs reveal a radiolucent lesion 2cm in diameter with a ground glass appearance. Which of the following represents the most probable diagnosis? A. Fibrous dysplasia B. Osteosarcoma C. Paget's disease of the bone D. Florid ossesous dysplasia

A. Fibrous dysplasia 1) Fibrous dysplasia a. normal bone is replaced with fibrous bone tissue causing a "ground glass" appearance radiographically b. bones of the skull, thigh, shin, ribs, upper arm and pelvis are most commonly affected c. 70% of cases is monostotic and involved one bone d. occurs 50% more frequently in the mandible than in the maxilla e. Radiographic Features 1) usually unilocular 2) radiopaque (usually) or radiolucent with a ground glass appearance 3) cortex is thinned and may be displaced 4) displaces anatomical structures and may or may not be displace teeth 5) poorly defined with no corticated border 6) loss of lamina dura

Which of the following conditions describes a benign, non-painful appearance of the tongue with multiple grooves and bad breath? A. Fissured tongue B. Hairy tongue C. Lingual laceration D. Oral squamous cell carcinoma E. Benign migrating glossitis

A. Fissured tongue 1) Concept: Fissued Tongue a. also known as Scrotal tongue, lingua plicate, Plicated tongue, and furrowed tongue is a benign condition characterized by deep grooves (fissures) in the dorsum of the tongue b. grooves may appear unsettling, the condition is typically painless although some individuals may complain of an associated burning sensation due to inflammation caused by poor oral hygiene. c. estimated prevalence of 2-5% d. may be seen at any age e. becomes more accentuated with age f. appearance varied in orientation, number, depth, and length of the fissure pattern g. usually multiple grooves/furrows 2-6mm in depth present h. food particles stuck in fissures may cause halitosis and irritation. 2) Hairy Tongue (lingual villosa) a. is a commonly observed condition of defective desquamation of the filiform papillae that causes the tongue to appear "hairy" and can often create a habitat for bacteria to reside and cause bad breath 3) Benign migrating glossitis (geographic tongue) a. non-painful inflammatory condition of the mucous membrane of the tongue, usually on the dorsal surface that affects approximately 2-3% of the general population.

Which of the following sites is the prognosis LEAST favorable for oral squamous cell carcinoma? A. Floor of the mouth B. Hard palate C. Lower lip D. Soft palate E. Bucal mucosa

A. Floor of the mouth 1) Concept: Squamous cell carcinomas occurring in the floor of the mouth demonstrate the LEAST favorable prognosis 2) The squamous cell carcinoma occurs in the floor of the mouth because it contains such a rich blood supply. 3) Cancer cells require an increased level of nutrition compared to normal cells, so a rich blood supply aids in tumor growth.

Each of the following is a criteria when determining the effectiveness of a visible light-curing unit EXCEPT one. Which is the EXCEPTION? A. Fluoride releasing ability of the composite being cured B. Light exposure time C. Shade of composite D. Wavelength

A. Fluoride releasing ability of the composite being cured 1) Composite resins a. do not typically release and therefore is NOT a factor in determining the effectiveness of a visible light-curing unit 2) Criteria a. wavelength of the curing light is an important consideration b. normal range of composite curing light units is approximately 470 nm. c. shade of the composite resins also affects the penetration of light required for curing. d. exposure time should neither be overextended nor kept short

An examination of a 22-year-old healthy patient reveals a solitary, asymptomatic, flat, round 3mm x 3mm brown lesion on the patient's lower lip. The duration of the lesion is unknown. Which of the following describes the most probable condition? A. Focal melanosis B. Malignant melanoma C. Papilloma D. Peutz-Jeghers syndrome E. Compound nevus

A. Focal melanosis 1) Focal melanosis a. flat, asymptomatic brown to black pigmentations of the lip or other oral sites which develop for unknown reason 2) Pigmented Nevi (common skin moles) and malignant melanoma a. occur on the oral mucosa b. melanotic macule is more more common than either conditions 3) Melanotic macules a. oval-shaped brown b. smooth-bordered and flat lesions c. less than 8mm in diameter (3-4) d. mostly on the lower lip near the midline

Each of the following is a possible result of a successful root canal treatment EXCEPT one. Which is the EXCEPTION? A. Formation of reparative dentin B. Regeneration of alveolar bone C. Regeneration of periodontal ligament D. Apical seal

A. Formation of reparative dentin 1) Concept: Formation of reparative dentin only occurs when the odontoblasts in the pulp are still vital. 2) Reparative dentin is laid down by the odontoblasts in the pulp to protect the pulp and keep it away from the harmful stimuli 3) Apical seal is the main objective of the root canal treatment in order to prevent bacterial invasion of pulp canal and periodical tissues.

Which of the following techniques should be implemented for a patient presenting with rampant caries in an area where esthetics are NOT considered important to the patient? A. Glass ionomer without a bevel B. Resin composite with a bevel C. Resin composite without a bevel D. Resin modified glass ionomer with a bevel

A. Glass ionomer without a bevel 1) Concept: Glass ionomer without placement of a bevel should be used in a case of rampant caries in an area where esthetics is NOT a concern. 2) Glass ionomer composite imparts an acceptable esthetics to the restorations. In a case of rampant caries, fluoride release and fluoride uptake qualities of glass ionomer composite have higher significance than esthetic concerns. 3) Beveling of enamel margins is NOT beneficial in this case because there is no need to blend the color of the restoration to the tooth and does not provide a better seal.

The current dents literature suggests that the transport media that offers the greatest PDL longevity following avulsion is...? A. Hank's balanced salt solution B. Milk C. Saliva D. Warm Water E. Buffered saline

A. Hank's balanced salt solution 1) Concept: Hank's Balanced Salt Solution a. most ideal and reliable storage media for an avulsed tooth b. has the ability to keep the cells surrounding the root viable c. has rich amount of metabolites like Ca, K, glucose and phosphate ions sufficient to provide the cells with the energy to sustain their normal cell metabolism until replantation can occur d. has a balanced pH and is known to replenish lost metabolites within the PDL cells of an avulsed tooth e. helps to improve the prognosis of replantation and decrease the chances of having replacement resorption

Which of the following locations is inflammatory papillary hyperplasia most commonly observed? A. Hard palate B. Labial tissue C. Posterior gingival tissue D. Soft palate E. Anterior gingival tissue

A. Hard palate 1) Concept: Inflammatory Papillary Hyperplasia a. appears as an overgrowth of tissue usually associated with poor denture hygiene, denture overuse, and ill-fitting dentures b. the exact cause of inflammatory papillary hyperplasia is unknown, around 20% of patients who wear dentures 24 hours a day develop it. c. It manifests as an asymptomatic erythematous area, on the hard palate, with a pebbly surface in mouth breathing patients. d. often associated with epulis fissuratum.

An anterior interproximal composite (Class III) restoration prep on an anterior tooth... A. Has cavosurface margins with supported enamel rods B. Should always include the contact area C. Should extend subgingivally D. Should have margins on the facial and lingual surfaces to facilitate cleaning E. Should have 60 degree bevel on the lingual and facial walls

A. Has cavosurface margins with supported enamel rods 1) Concept: Cavosurface margins should be free of unsupported enamel rods because they normally fracture under occlusal stress 2) A 45o cavo-surface bevel exposes more surface area of enamel rods that can be etched and bonded to retain and seal composite restorations, a. NO bevels should be placed on occlusal surfaces

Each of the following devices are effective in removing surface plaque EXCEPT one. Which one is the EXCEPTION? A. Home water-irrigatin device (Water PIK) B. Super-soft toothbrush C. Toothpick D. Dental floss

A. Home water-irrigatin device (Water PIK) 1) The use or oral irrigation devices like Water-Pik helps remove plaque in shallow periodontal pockets 2) Irrigation devices propel a pulsating, high pressure steam of water to flush sub gingival plaque and debris out of shallow pockets. 3) They are ineffective if the periodontal pocket is too deep to be reached by toothbrush bristles or a toothpick. 4) Dental floss is the most recommended oral hygiene method for proximal plaque removal, but not capable of reaching deep into a pocket.

Each of the following devices are effective in removing surface plaque EXCEPT one. Which is the EXCEPTION? A. Home water-irrigating device (Water Pik) B. Super-soft toothbrush C. Toothpick D. Dental floss

A. Home water-irrigating device (Water Pik) 1) Concept: Home water-irrigating device (Water Pik) a. The use of oral irrigation device like Water-Pik helps remove plaque in shallow periodontal pockets. b. Irrigation devices propel a pulsating, high pressure stream of water to flush sub gingival plaque and debris out of shallow pockets. c. They are ineffective if the periodontal pocket is too deep to be reached by toothbrush bristles or a toothpick. d. The use of dental floss is the most recommended oral hygiene method for proximal plaque removal, but is not capable of reaching deep into the pocket.

Using a high speed hand piece with the water spray turned off, but still cooling the tooth with air can cause... Select all that apply. A. Hypersensitivity in the prepared tooth B. Irritation of the pulp from being too cold C. Irritation of the pulp from being too dry D. Too much tooth structure can be removed E. The odontoblasts to be forced into the dentinal tubules causing damage

A. Hypersensitivity in the prepared tooth C. Irritation of the pulp from being too dry E. The odontoblasts to be forced into the dentinal tubules causing damage 1) High speed hand pieces a. produce heat that cause damage to odontoblasts and tooth sensitivity b. heat produced by using high speed hand pieces without water makes the odontoblastic process to dry up and damage the odontoblasts c. movement of dentinal fluid causes irritation and pain stimuli to the pulp d. Heat from the handpiece might cause pulpal hypersensitivity/hyperemia and may progress further to irreversible pulpits.

Which of the following antibodies is MOST prevalent? A. IgA B. IgD C. IgE D. IgG E. IgM

A. IgA 1) IgA a. more prevalent in saliva b. prevent adhesion of the bacteria to the oral tissues because it is bound to IgA instead. c. known as the agglutinating antibody because it clumps or "glues" bacteria together 2) IgG a. more associated with sulcular fluid

A 6 year old child presents with a posterior unilateral crossbite with a functional shift. This child's 6 year molars have NOT yet erupted. When should you intervene to correct this? A. Immediately without waiting for the permanent 1st molars to erupt. B. Wait until the 1st permanent molars have fully erupted and are in occlusion C. When the child is approximately 9 years old D. After all of the succedaneous teeth have erupted

A. Immediately without waiting for the permanent 1st molars to erupt. 1) Orthodontic evaluation and monitoring of a pediatric patient may be helpful in preventing possible orthodontic problems. 2) Preventive orthodontics involves the evaluation and monitoring of the developing dentition of patents ages 6 and below. 3) Normal occlusion can be achieved through the use of appliances to guide the growth and the development of the developing dentition, bone, and musculature. 4) Crossbites can be corrected immediately during the primary dentition will help gain and maintain the space for the secondary dentition.

The success rate of the free gingival graft procedures is affected MOST by which of the following factors? A. Immobilization of the graft at the recipient site B. Presence of a periosteal fenestration C. Whether the donor tissue contains periosteum D. The thickness of the blood clot between the recipient and donor tissue margin

A. Immobilization of the graft at the recipient site 1) Immobilization a. enables the formation of blood vessels and better diffusion of nutrients from the recipient site to the graft b. donor tissue can gain attachment to the recipient site as long as the graft is adapted and placed properly at the recipient site c. formation of new capillaries from the recipient site to the do not site will only occur within 2-3 days after the surgical procedure in the process known as capillary inosculation

Which of the following describes the MOST COMMON reason speech problems associated with cleft lip and palate are observed? A. Inability of the soft palate to close airflow into the nasal area; Inability of the tongue to stop airflow from the epiglottis B. Missing teeth that are involved in the articulation of sounds formed by the tongue difficult to make C. Poor lip musculature or heavy scars of the lips that limit vowel sound production D. Poor tongue control and producing lisping

A. Inability of the soft palate to close airflow into the nasal area; Inability of the tongue to stop airflow from the epiglottis 1) Concept: In cleft lip and palate patients, an inability of the soft palate to close the nasal passageways results in speech problems 2) Improper speech sounds are produced if the soft palate fails to prevent the flow of air into the nasal passageways 3) It is important to close the nasal air passageways for producing distinct and understandable sound. 4) The soft palate closes the nasal area for sound production.

Which of the following will provide the MOST ACCURATE diagnosis for a patient with a large cyst-like lesion (14mm) in area of their posterior mandible? A. Incisional biopsy and histologic analysis B. Needle aspiration biopsy C. Radiographic examination D. Visual clinical examination E. Exfoliative cytology

A. Incisional biopsy and histologic analysis 1) Concept: Histology of the tissue and cystic fluid will aid in the definite diagnosis of the lesion. Incisional biopsy is indicated because it is a large lesion (>10mm) 2) Incisional biopsy a. of the large cystic lesion should be carried out and sent to the laboratory for histologic examination 3) Definitive diagnosis a. is necessary to determine which treatment should be provided for the best results

Name the technique where deep caries are excavated from a tooth, and a small amount of affected dentin is left provided that caries-free DEJ is present, and a base is placed along with a permanent restoration? A. Indirect pulp cap B. Semi-permanent caries excavation C. Step-wise caries removal D. Direct pulp cap

A. Indirect pulp cap 1) Indirect Pulp Cap a. avoids the risk of accidental pulp exposure during excavation of a deep cavity b. affected dentin is usually only a few millimeters from the pulp and is relatively unharmed by indirect pulp capping c. affected dentin is covered using a cavity liner like calcium hydroxide or glass ionomer and is then restored d. indicated for teeth with deep caries with minimal pulpal inflammation e. Steps 1) Infected dentin is completely removed 2) Affected dentin close the pulp is left due to the possibility of pulp exposure 3) Calcium hydroxide, then glass ionomer is placed over the affected dentin 4) Final restoration is placed f. Calcium hydroxide 1) placed on the affected dentin to induce its remineralization and promote the reparative dentin formation

Contraction of which of the following muscles can cause the forward displacement of the condylar head if the condyle is fracture off? A. Lateral pterygoid B. Medial pterygoid C. Stylohyoid D. Temporal E. Buccinator

A. Lateral pterygoid 1) Concept: Protrusive and extrusive movements are produced mostly by the contraction of the lateral pterygoid muscles. 2) During right lateral excursive movements when the mandible moves to the right, the left lateral pterygoid contracts pushing the mandible downward and forward. The right lateral pterygoid is relaxed and does not antagonize the force, causing the asymmetrical depression and protrusion of the mandible. 3) When both lateral pterygoids contract at the same time, it causes a protrusive movement. 4) The lateral pterygoid originate in the infra temporal fossa and attaches to the TMJ disk and the neck of the mandible.

Each of the following may be considered as an indication for gingivectomy EXCEPT one. Which is the EXCEPTION? A. Infrabony pocket reduction B. Medication induced gingival hypertrophy C. Pseudopocketing D. Suprabony pocket reduction

A. Infrabony pocket reduction 1) Gingivectomy a. reduce redundant or hyperplastic gingival tissues and are indicated when there is excess gingiva, preventing the patient from cleaning their mouth properly 2) Infrabony defects would NOT be helped by removing gum tissue, they can only be resolved by bone augmentation

Which of the following describes how a Miller Class I recession defect is differentiated from a Miller Class II defect? A. Involvement of the mucogingival junction B. Location of inter proximal alveolar bone D. Width of Keratinized gingiva

A. Involvement of the mucogingival junction Miller Classification 1) Class I a. Recession does NOT extend to the mucogingival junction 2) Class II a. Recession that extends to or beyond the mucogingival junction, but demonstrates NO loss of inter proximal clinical attachment 3) Class III a. Recession that extends to or beyond the mucogingival junction, with either loss of inter proximal clinical attachment or tooth rotation 4) Class IV a. Recession that extends to or beyond the mucogingival junction, with either loss of inter proximal clinical attachment or tooth rotation that is severe

Which of the following is the primary factor in determining the optimal fluoride concentration for community water? A. air temperature B. DMFT score of the community C. Weight of the average community member D. Adult to child ration of the community

A. air temperature 1) high temperatures --> lower fluoride levels since most people will drink copious amounts of water 2) low temperatures --> higher fluoride levels since most people will drink less water in this type of weather

Which of the following describes why the matrix is burnished against the adjacent tooth when placing Class II restorations? A. It helps to establish an inter proximal contact B. It helps to prevent overhanging restorative margins C. It holds the ring in the embrasures D. It prevents the wedge from being displaced during the restoration placement

A. It helps to establish an inter proximal contact 1) Concept: Burnishing of the side of the matrix band against the adjacent tooth during the placement of Class II restorations helps to obtain an accurate inter proximal contact. 2) Establishment of proximal contact is very important for the longevity of the restoration and gingival health around the tooth being restored. 3) Failing to achieve an optimum proximal contact can lead to pocket formation and overhanging restorations which result in the failure of restorations. 4) Matrices also serve to: a. confine the restorative materials for application of adequate condensation forces b. re-establish the contact with adjacent tooth c. restrict formation of an overhang in the proximal gingival margin d. provision of adequate proximal contour

Why is calcium hydroxide the suggested material for vital pulp capping procedures? A. It induces dentin bridge formation B. It seals the cavity best C. It stimulates primary dentin formation D. It is least irritating to the pulp

A. It induces dentin bridge formation 1) Concept: Calcium hydroxide is the most biocompatible material that has been proved to help induce dentin bridge formation. 2) Calcium hydroxide has been used for a long time and is the gold standard for direct and indirect pulp capping. 3) Calcium hydroxide induces the formation of reparative dentin to wall off and protect the pulp against bacterial attacks.

Which of the following lesions has the highest propensity for malignancy? A. Junctional nevus B. Leiomyoma C. Lymphangioma D. White sponge nevus E. Granulocytoma

A. Junctional nevus 1) Concept: Most junctional nevi are non-cancerous in nature but are capable of transforming into malignant melanoma if changes are seen in their appearance b. Junctional nevi or moles are a benign skin growth whose appears flat, round, and mostly dark brown in color, and originate from multiple skin layers. c. Junctional nevi arise from cells at the junction of the epidermis and dermis meet. 2) Early signs of melanoma can be diagnosed by "ABCDE" A: Asymmetry B: Borders (irregular) C: Color (multicolored) D. Diameter (greater than 6mm, about the size of a pencil erase) E: Evolving over time 3) The most dangerous form of melanoma, nodular melanoma can be diagnosed with "EFG" E: Elevated above the skin surface F: Firm to the touch G: Growing

The facial profile of most people change as they age and mature through becoming... A. Less convex B. More convex C. The same level of convexity D. Less concave

A. Less convex 1) Concept: The facial profile may vary as we age due to the loss of facial fat, muscle atrophy, and loss of skin elasticity. 2) The buccal fat pad is gradually lost in the cheeks, causing the skin to adapt itself to the underlying muscles and bone 3) During the aging process, metabolism slows down and the production of skin, collagen, and muscle fibers is reduced. 4) The constant pull of gravity also contributes to the sagging of the face.

Acute abscesses induce... A. Leukocytosis B. Leukopenia C. Lymphocytosis D. Neutropenia E. Leukemia

A. Leukocytosis 1) Leukocytosis a. patient's white blood cell count is above the normal range b. indicates an inflammatory response where infection may be present c. common in patient's suffering from 1) acute illness such as fungal, viral, or parasitic infection, cancer, hemorrhage, and exposure to substances that include steroids d. may manifest after 1) tiring exercise 2) emotional stress 3) pregnancy and labor 4) epileptic convulsions 5) anesthesia 6) epinephrine administration 2) Types of leukocytosis: a. Neutrophilia (most common) b. Monocytosis c. Eosinophilia d. Lymphocytosis e. Basophilia

A patient presents to your office for a routine examination where note a bilateral grey-white lesion of the buccal mucosa that diapers when stretched. Which of the following is the most probable diagnosis? A. Leukoedema B. Leukoplakia C. Lichen planus D. White sponge nevus E. Hairy Leukoplakia

A. Leukoedema 1) Leukedema a. disappears when the mucosa is stretched b. presents as an asymptomatic blue, grey, or white appearance on the mucosa and is seen predominantly in the buccal mucosa c. occurs bilaterally on the buccal mucosa and is less often on the labial mucosa, the palate, or the floor of the mouth d. acquired condition caused by local irritation and is found more commonly in black skinned people and in those who smoke e. surface of the affect area may appear folded, creating a wrinkled, white streaked lesion

Ventricular arrhythmias can be treated by an intravenous injection of which of the following agents? A. Lidocaine B. Lisinopril C. Propranolol D. Quinidine E. Verapamil

A. Lidocaine 1) Lidocaine a. Intravenous injection of lidocaine may be useful to suppress symptomatic ventricular arrhythmias. b. Routine lidocaine use can reduce the occurrence rate of primary ventricular fibrillation to some extent. c. intravenous lidocaine prophylaxis for patients with acute myocardial infarction in a coronary care unit is not recommended

Which of the following tooth surfaces requires the least amount of reduction for a stainless steel crown for primary mandibular 1st molar? A. Lingual B. Mesial C. Occlusal D. Buccal

A. Lingual 1) Concept: Lingual surface a. of a primary mandibular 1st molars are prepared with the least amount of reduction to ensure an adequate cervical bulge to help retain the stainless steel crown b. mesial and distal proximal surfaces are prepared to facilitate crown placement c. cervical bulge located along the buccal and lingual surfaces of primary posterior teeth provides retention for the stainless steel crown d. lingual cervical bulge is smaller than that of the buccal aspect, so the lingual reduction must be minimal.

Which of the following site is the LEAST likely for Class I carious lesions to occur? A. Lingual surfaces of mandibular incisors B. Lingual surfaces of maxillary incisors C. Lingual surfaces of maxillary molars D. Buccal surfaces of mandibular molars

A. Lingual surfaces of mandibular incisors 1) Class I carious lesions a. lingual surfaces of mandibular incisors are the LEAST common sites for the occurrence of Class I carious lesions b. mandibular incisors are typically morphologically stable and do not demonstrate lingual pits or fissures 2) G.V. Black Classification Class I: occlusal areas and buccal or lingual pits Class II: Posterior Interproximal Class III: Anterior Interproximal Class IV: Anterior Interproximal including the incisal corner Class V: Gingival at facial or lingual Class VI: Cusp tip

Which of the following types of gingival periodontal disease is most prevalent in high school aged children? A. Localized acute gingivitis C. Primary herpetic gingivostomatitis D. Acute periodontitis

A. Localized acute gingivitis 1) Adolescence induce a series of metabolic and hormonal changes that may affect periodontal tissues. 2) Hormonal changes can affect the capillary permeability and thereby increase the fluid accumulation in the gingiva. 3) Adolescents demonstrate a tendency to largely disregard good oral hygiene practices.

Each of the following drugs are used for IV conscious sedation in dental offices EXCEPT one due to its unusually long duration. Which drug is the EXCEPTION? A. Lorazepam B. Diazepam C. Hydroxyzine D. Midazolam E. Propofol

A. Lorazepam 1) Lorazepam a. very potent intermediate acting benzodiazepine anxiolytic agent with a very long duration that makes it less useful in the dental setting b. ideal sedative and anti-anxiety drugs used in dental settings are those that are short-acting and allows for rapid recovery of the patient c. sometimes prefer proposal over benzodiazepines because of its unique ability to provide a recovery time of about 5 minutes of less.

Which of the following histologic features are least likely to be found in normal dental pulp? A. Lymphocytes and plasma cells B. Palisade odontoblastic layer C. Undifferentiated mesenchymal cells D. Cell-free zone of Weil

A. Lymphocytes and plasma cells 1) Lymphocytes and plasma cells a. presence of lymphocytes and plasma cells denote that an infection is present 2) Cell free zone a. located within the pulp complex and is located beneath odontoblasts 3) Undifferentiated mesenchymal cells a. normally present within the pulp b. reserved cells that differentiated into another type of cell when it is needed by the pulp

The tooth most commonly involved in the dental anomaly "dens in dente" or is the permanent: A. Maxillary lateral incisor

A. Maxillary lateral incisor 1) Dens in dente a. infolding of the outer surface of the tooth and must be diagnosed radiographically b. "tooth within a tooth" and is also known as Dens invaginatus c. most often in maxillary lateral incisors and bilateral occurrence is not uncommon

The term for when different types of light sources makes color appear different is... A. Metamerism B. Opalescence C. Translucency D. Fluoroscence

A. Metamerism 1) Metamerism a. where colors which appeared similar in a particular light source appear different when observed with another light sources 2) Fluorescence a. light absorbed by a substance is emitted back in a longer wavelength 3) Opalescence a. appears yellowish-red when light is transmitted through it b. material appears blue in the scattered light that is perpendicular to the transmitted light 4) Translucency a. when light is permitted to pass through a material but is diffused within the material, making the image on the opposite side not clearly visible

What is the most accepted treatment for mottled enamel (Colorado brown stains/Hyperfluorosis/Chalky enamel white enamel with brown spots) A. Microabrasoin with HCl/pumice B. Vital tooth bleaching at home C. Walking bleach after root canal treatment D. In-office bleaching with 35% hydrogen peroxide

A. Microabrasoin with HCl/pumice 1) Mottled enamel a. combination of small white, brown and yellow spots are seen all over the suffices of the teeth 2) Microabrasion a. fast, pain-free treatment that is effective in lightening or completely removing the yellow, white and brown stains seen on mottled enamel b. removes a thin layer of stained enamel with the use of pumice and hydrochloric acid. c. Hydrochloric acid whitens the stains in the tooth surface and has an effect that lasts for a few days after its application.

Which of the following describes the primary advantage of placing a microfilmed resin composite? A. Microfilled resin composites are highly polishable C. Microfilled resin composites are stronger than most resin based composite materials

A. Microfilled resin composites are highly polishable 1) Microfill resin composites a. highly polishable because they contain colloidal silica particles having an average diameter of 0.01-0.04 micrometer b. small particle size of micro-filled composite resins results in a smooth, polished surface after finishing c. highly polished restoration surface is less receptive to plaque or extrinsic staining

Which of the following patient conditions is the strongest indication for splinting? A. Mobility with unilateral shift from centric relation to centric occlusion D. Mobility with discomfort

A. Mobility with unilateral shift from centric relation to centric occlusion 1) Unstable teeth that cannot contain normal position during centric occlusion need additional support to maintain position and therefore must be splinted to the adjacent teeth. 2) Discomfort associated with tooth mobility may be due to: a. other periodontal conditions b. to pulpal conditions c. NOT be a direct indication for splinting

Which of the following are the classic signs of bruxism. Select all that apply. A. Muscle soreness B. Occlusal wear C. Thickened lamina dura D. Tooth mobility E. Wear facets

A. Muscle soreness B. Occlusal wear C. Thickened lamina dura D. Tooth mobility E. Wear facets 1) Bruxism a. repetitive behavior of grinding of clenching or a person's teeth b. induced by 1) stress 2) masseter tension 3) premature occlusion 4) medications

Which of the following pulpal conditions are suggested when a patient experiences prolonged, unstimulated pain while trying to sleep? A. Necrosis B. No specific condition C. Normal pulp D. Reversible pulpitis E. Mild irritation

A. Necrosis 1) Necrosis a. prolonged unstimulated pain is associated with cases involving pulpal necrosis or irreversible pulpits b. describes the death of pulpal cells c. pain associated with pulpal necrosis is often periodically derived 2) Reversible Pulpitis a. pain subsided after the removal of the stimulation in cases of reversible pulpits

The prognosis for internal bleaching is best if the etiology of the stain is... A. Necrotic pulp tissue B. Precipitation of metallic salts C. Silver-containing root canal sealers D. Amalgam restorations

A. Necrotic pulp tissue 1) Internal bleaching treatment a. most effective among root canal treated tooth which have internal stains due to blood, necrotic tissue, and fluids that could have leaked in prior to root canal therapy b. placing peroxide directly into the pulp chamber of a root canal treated tooth c. peroxide wil then directly work on the stains within the tooth structure 2) Staining from silver containing root canal sealers and from mercury of amalgam restorations a. appear dark b. seldom treated by bleaching c. restorative treatments provide the most esthetic result because they remove or cover up these stains

Which of the following actions is indicated if an accessory canal in the middle 1/3 of a tooth with a necrotic pulp and an adjacent radiolucent lesion is noted? A radiograph made after obturation suggest that no material entered the accessory canal? A. Observe the patient with a 6 month recall and radiograph schedule for the next year B. Perform an apicoectomy C. Re-obturate the canal using the continuous wave warm lateral to force material into the accessory canal D. Expose the root surgically then curette the lesoin

A. Observe the patient with a 6 month recall and radiograph schedule for the next year 1) Treatment a. best to observe and evaluate the root canal treated tooth prior to re-obturating the canal even if the accessory canal was not included during obturation b. observation is best because the complete sealing of the main root canal might be sufficient to prevent bacterial re-infection. c. only way to know if the treatment was successful is by having recall and radiographic evaluation to assess bone healing and absence of active infection

A 10-year-old patient presents to your office with a 2.5mm diastema and low attached maxillary labial frenum. Which of the following is the best option for this patient? A. Observe until after eruption of permanent canines B. Place a 6x6 edgewise appliance to close the space C. Provide treatment before complete eruption of maxillary incisors D. Perform a frenectomy when first observed

A. Observe until after eruption of permanent canines 1) Concept: Eruption of maxillary canines in children ranging from age 11 to 12 usually will close the diastema between the maxillary central incisors. 2) Diastema a. Diastemas in between maxillary central incisors normally occur during the mixed dentition stage. 3) Frenectomy will be indicated if the maxillary canines had already erupted and were unable to close the diastema due to a lowly attached or fibrous maxillary labial frenum.

Proper irrigation helps decrease the likelihood of incomplete removal of bacteria, pulp tissue, and dentinal shavings. Which of the following is the best method to ensure complete cleaning of the root canal? A. Obtaining straight line access B. Use of Gates-Gliden burs C. Use of a canal lubricant D. Use of broaches E. Use of a chelating agent

A. Obtaining straight line access 1) Straight line access a. enables the dentist to adequately clean the shape the root canal b. having adequate acces and visibility of the root canal allows the dentist to freely perform proper instrumentation, irrigation, and debridement of the root canal

Which of the following describes a practice where dental patients receive care at specified facilities by a limited group of clinician's on a prepayment basis? Select all that apply. A. Open panel B. Closed panel C. Group practice D. Solo practice

A. Open panel B. Closed panel 1) Closed panel practice a. where dental patients receive care at specified facilities by a limited group of clinicians on a prepayment basis 2) Prepayment plan a. group of dentists sharing office facilities that provide stipulated services at a set premium to an eligible group of patients

Each of the following should be used by a dentist to assess the dental fears of a patient EXCEPT ONE. Which is the EXCEPTION? A. Personality B. Physical response C. Verbal statements D. Behavior

A. Personality 1) Personality a. gives the dentist an idea of the patient's character and capacity to cooperate or interact but cannot be used to assess their dental fears 2) Behavior a. provides relevant information on their perception of dental treatment 3) Physical response a. provides significant cues to their dental fears 4) Verbal cues a. may directly or indirectly suggest the patient's fears and concerns before and during dental procedures

A patient in your chair is suffering from xerostomia, tachycardia, and gastrointestinal inactivity, and you are suspecting atropine poisoning. What should you administer to your patient to treat this condition? A. Physostigmine B. Pilocarpine C. Prednisone D. Romazicon E. Epinephrine

A. Physostigmine 1) Concept: Physostigmine inhibits acetylcholinesterase enzyme from breaking down acetylcholine in the neuromuscular junction, allowing acetylcholine to function properly. 2) Atropine a. competitive inhibitor of muscarinic receptors M1-M5 3) Physostigmine a. is an agent that can reverse atropine overdose

Which of the following anatomical spaces would a needle tract infection following an inferior alveolar nerve block infection involve initially? A. Pterygomandibular B. Pterygopalatine C. Sublingual D. Submandibular E. Lateral pharyngeal

A. Pterygomandibular 1) Concept: The pterygomandibular space will most probably be affected by a needle tract infection after an IAN nerve block. 2) The pterygomandibular space is the anatomical space containing several blood vessels and the mandibular branch of the trigeminal nerve. 3) In order to anesthetize the jaw, the inferior alveolar nerve block is performed with the needle passing into the pterygomandibular space, which may serve later as an opening for infection to set in.

Which of the following describes the intended function of a home dental water irrigation systems (Water Pike)? A. Reduce bacteria on the gingiva B. Reduce biofilm on the tooth C. Reduce pocket depth D. Remove supragingival calculus

A. Reduce bacteria on the gingiva 1) Concept: home water irrigation systems are designed to reduce the biofilms present on the gingiva, NOT the tooth surface 2) Gram+ bacteria produce dextrans levs, and glucans which allow them to better adhere to tooth structure 3) Gram- bacteria produce alginates to help them to adhere to adhere 4) Spirochetes cannot produce the same adherence factors as gram negative and gram positive bacteria, so they can be flushed from the pocket.

After etching a Class III preparation for a composite restoration, the preparation is contaminated with saliva. Which of the following actions are indicated are indicated next? A. Rinse away the saliva with water, dry the preparation, then repeat the etching procedure and restore B. Rinse away the saliva with water, then dry the preparation, then proceed C. Wipe away all saliva with cotton pellet, rinse away the saliva with water, dry, and proceed D. Blow dry the saliva with the air/water syringe then proceed

A. Rinse away the saliva with water, dry the preparation, then repeat the etching procedure and restore 1) Concept: Saliva contains minerals that can immediately remineralize the etched tooth and weaken the bonding of the composite to tooth surface. 2) By drying and re-etching the surface to be restored, the primer and bonding agent can establish better adhesive interface between composite and the tooth structure. 3) Re-etching the tooth will ensure roughening of the tooth surface is achieved and will ensure better adhesion of the bonding agent to the etched tooth structure through increased mechanical retention through increased microtention. 4) Salivary contamination also inhibits the proper setting of composite resins.

Which of the following would a cross-sectional occlusal radiograph be useful in locating? A. Sialoliths in Wharton's duct B. The hyoid bone C. The mandibular foramen D. Maxillary sinus polyps

A. Sialoliths in Wharton's duct 1) Concept: Occlusal radiographs are important when attempting to discern the location of submandibular sialoliths. 2) Stones in the Wharton's duct are made visible in cross sectional occlusal radiographs. 3) Diagnosis is usually made by characteristic history and physical examination. 4) Sialoliths or salivary calculi confirmation requires occlusal x-rays. a. 80% of salivary gland calculi are visible on x-rays.

Each of the following statements about posterior Class II resin composite restoration are correct EXCEPT one. Which is the EXCEPTION? A. Superseal needs to be used to seal the dentinal tubules to prevent sensitivity B. The preparation has more round internal line angles C. The preparation tends to be shallower than for a Class II amalgam restoration D. The preparation tends to have a narrower outline form than for a Class II amalgam restoration.

A. Superseal needs to be used to seal the dentinal tubules to prevent sensitivity 1) Concept: Superseal and dentinal sealers are typically used in amalgam preps only. 2) The preparation should have rounded line angles to conserve tooth structure in posterior Class II resin composite restorations. Sharp angles will eventually lead to breakage or fracture of restoration in these areas. 3) The preparation tends to be shallower than for an amalgam restoration. 4) The preparation tends to have a narrower outline form than for an amalgam restoration. 5) Preparation outline should follow the extent of carious lesion only. There is no extension for prevention.

The perceived form of a restoration is affected by its surface characteristics in which of the following ways? A. Surfaces smoother than usual will give the impression of a larger size B. Teeth appear smaller when the value is increased C. The illusion of increased length can be achieved with horizontal highlights D. The illusion of increased width can be achieved with vertical highlights

A. Surfaces smoother than usual will give the impression of a larger size 1) Smooth Surfaces a. give the impression of larger size and vice versa 2) Increased value a. teeth appear larger than their actual size when the value is increased 3) Illusion a. art of changing the perception of an object to appear different from what it is b. illusion of size, shape, length, and color are created to solve or hide an esthetically difficult situation c. increased length: achieved with vertical highlights d. increased width: achieved with horizontal highlights

During the extraction of #3, a large portion of the palatal root is fracture and forced into the antrum and is not able to be seen. The adjacent teeth and alveolus is normal and not damaged. Which of the following describes the most appropriate method to recover the root though? A. The canine fossa above the level of the premolar roots B. The hard palate in the canine area C. The nasoantral wall above the middle conca/turbinate D. The extraction site of #3 after enlarging the oral-antral communication

A. The canine fossa above the level of the premolar roots 1) Caldwell-Luc operation a. incision is made in the area of canine fossa above the canine or premolar roots b. an opening or window is made in the anterior wall of the maxillary sinus. c. root is retrieved through this opening and the opening is sutured d. Indications for 1) retrieval of a broken tooth root from the maxillary sinus 2) removal of abnormal tissue growths 3) treatment of maxillary infections

What would happen to the caries risk of a mouse that was fed by a gut tube so that no carbohydrates would be allowed in the oral cavity? A. The caries risk of the mouse would decrease B. The caries risk of the mouse would increase C. The caries rate would stay the same as if the mouse was fed directly

A. The caries risk of the mouse would decrease 1) Caries risk a. amount b. frequency c. duration of carbohydrate ingestion d. presence of fermentable food substrate like carbohydrates in the mouth is necessary for caries generation 2) Cariogenic bacteria like Streptococcus mutans and Lactobacillus case: a. ferment carbohydrates into acid b. demineralization occurs with a pH<5 c. secrete collagenase that break down demineralized tooth structure

A split thickness or mucosal skin graft used in a vestibuloplasty receive its nourishment and oxygenation primarily from... A. The exposed periosteum that forms the graft bed B. The mucosa surrounding the graft C. The vasculature in the sub epithelial or submucosal layer moved to the site within the graft D. The exposed bone directly beneath the graft

A. The exposed periosteum that forms the graft bed 1) Mucosal skin grafts a. receive nourishment from the vascular supply coming from the expose portion of the periosteum where the graft is adapted and placed b. attached to the recipient site through fibrin layer formation c. nutrients for the graft are taken from the recipient bed through a process known as plasmatic imbibition d. end capillaries of the graft and the recipient bed then form a vascular network which will provide nourishment to the whole graft later.

Using a cervical pull headgear in the case of an 11-year-old prepubertal child should create each of the following conditions EXCEPT one. Which is the EXCEPTION? A. The forward growth of the mandible will be increased B. The forward growth of the maxilla will be decreased C. The permanent maxillary 1st molars will move distally D. The child's bite will open

A. The forward growth of the mandible will be increased 1) Concept: Cervical pull headgear is an appliance used to REDUCE the forward growth of the maxilla and DOES NOT increase mandibular growth. 2) Cervical pull headgear is composed of the neck strap, and inner and outer face bow a. The neck strap acts as an extra-oral anchorage b. The outer bow connects the face bow assembly with the backstrap c. The inner bow is inserted into the tubes found on the molar bands d. inhibits the growth of the maxilla and causes the maxillary molars to extrude lessening the deep bite e. The headgear must be worn regularly for 14 hours per day for optimum results.

Which of the following is a general description of children exhibiting what is classified as defiant behavior? A. They are also regarded as stubborn and spoiled B. They are usually 4 years of age or younger

A. They are also regarded as stubborn and spoiled 1) Defiant behavior a. too much pampering by parents makes their children act spoiled and stubbornly b. want to get things done in their own way in spite of the consequences c. do NOT like to be advised by their elders d. must be death with positive reinforcement

The dentist should do which of the following when finishing the occlusal portion of a posterior composite restoration? A. avoid altering the centric contact on enamel B. eliminate contacts in the fossae C. make sure to include protrusive contacts into the restoration D. develop centric contacts on cavosurface margins

A. avoid altering the centric contact on enamel 1) The centric contact in enamel should be preserved since it established the original and proper occlusal contact between maxillary and mandibular teeth. 2) Decreasing of removing the centric contact in enamel may cause occlusal disharmony and unsta

Each of the following accounts for the increased risk of caries risk of elderly people EXCEPT one. Which is the exception? A. changes in the oral microflora B. decreased salivary flow C. medication side effects D. poor oral hygiene

A. changes in the oral microflora 1) Oral microflora is relatively constant after initial colonization in childhood. Oral microflora may be modified with changes in diet, salivary flow, oral hygiene, and etc. 2) Age (increased) a. increased number of medications used to treat various disease b. as person ages, acidic are replaced by fibrous and fatty tissue leading to decreased salivary flow 3) Oral hygiene (decreased) a. due to hand dexterity due to increased likelihood of stroke and arthritis

Which of the following is considered the best method of determining if a tooth has irreversible pulpits? A. cold/thermal testing B. palpation C. percussion D. Test cavity E. Electric pulp test

A. cold/thermal testing 1) Irreversible Pulpitis a. pain perceived by a tooth with irreversible pulpits during a cold test or a heat test is more intense and lingers even after the stimulus is removed 2) Healthy vital pulp a. pain perceived with healthy pulp during a cold test or heat test is usually short in duration, and it does not linger

To put the patient at east and increase the general comfort of the patient while at the dentist. The dentist should.... A. demonstrate control through a directive interviewing style C. provide an immediate evaluation of the patient's oral health D. tell the patient what to expect during the appointment E. Provide an immediate evaluation of the patient's oral health to the patient doesn't have to wait and wonder "how bad it is"

A. demonstrate control through a directive interviewing style D. tell the patient what to expect during the appointment

An increase in which of the following is evidence of an aging pulp? A. fibrous elements B. number of odontoblasts C. pulp stones D. size of the pulp chamber E. vascularity F. cellular elements

A. fibrous elements 1) Pulp Chamber a. fibroblasts in the pulp chamber continue to produce fibrous connective tissue and ground substance, increase its amount compared to the other substances within the pulp b. size of the pulp decreases as we age due to the continuous formation of secondary dentin and the formation of reactionary dentin related to cares and other dental trauma c. decrease in pulp size also decreases the vascular supply, nerve fibers and the overall cellularity of the pulp.

When restoring MO or DO decay, an only is indicated... A. if cuspal integrity is in question B. If the opposing tooth structure has an all porcelain occlusal surface C. When there is a need for extra retention D. If the patient has a high caries risk

A. if cuspal integrity is in question 1) Onlays a. reinforce fractured, weakened, or unsupported tooth structure b. weakened cusps are reinforced by reducing unsupported structure and replacing it with a durable material capable of bearing heavy occlusal forces c. restore a cusp of cusps 2) Inlays a. do not restore cusps

Skin grafts provide which of the following functions after a vestibuloplasty procedure is completed? A. prevent reattachment of muscle to periosteum C. provide an adequate amount of nonkeratinized tissue

A. prevent reattachment of muscle to periosteum 1) Vestibuloplasty a. surgeon displaces the muscle attachments in the periosteum of the jaw to deepen the vestibule and expose more alveolar bone needed to create a residual ridge that will provide more denture support b. in detaching a portion of these muscles, more residual ridge becomes more exposed and a new vestibular depth is established c. muscles that were detached have a tendency to readapt to the previous location within the periosteum. In order to maintain the new positioning and prevent muscular reattachment as the previous spot, a skin graft is placed that attaches to the periosteum of the exposed alveolar bone.

Adjunctive Endodontic Therapy

Adjunctive Endodontic Therapy Answers

Anterior Disc Displacement with Reduction

Anterior Disc Displacement with Reduction (http://bodymechanics.com.au/jaw-tmj-clicking-popping-part1.html)

Anterior Disc Displacement without Reduction

Anterior disc displacement (without reduction) (http://bodymechanics.com.au/jaw-tmj-clicking-popping-part1.html)

Anxiety and Pain Control

Anxiety and Pain Control Answers

Assessment and Emergency Care

Assessment and Emergency Care Answers

Which of the following ethical principles requires clinicians to inform their patients about treatment risks and benefits and to protect their confidentiality?

Autonomy 1) right of the patient for their confidentiality

The ability of the patient to independently choose a treatment option (including no treatment) represents the ethical concept of

Autonomy 1) Autonomy a. informed consent b. treated in a manner that fits their personal principles, religion, or beliefs

A marked reduction in the size of a periodical radiolucency can be expected approximately how long after a root canal procedure is completed? A. 1 Month B. 1 Year C. 2 Weeks D. 2 Years E. 3 Months F. 1 Week

B. 1 Year 1) Concept: The continuous bone deposition increases the bone density along the periodical area resulting in a diminished periodical lesion size after about a year. 2) The periodical bone regeneration usually occurs at a rate of about 1.2mm2 per month, gradually reducing the size of the periodical lesion. 3) Bone deposition in the periodical area is not very evident radiographically until sufficient thickness of bone is present.

Which of the following antibiotic regimens is indicated for a 20-kg pediatric patient with a congenital heart defect requiring prophylaxis according to American Heart Association standards? A. 1 g oral amoxicillin 1 hour prior to dental treatment and 500mg 4x per day for 2 days following the procedure B. 1 g oral amoxicillin 1 hour prior to dental treatment and 500mg 6 hours after the procedure is the antibiotic regimen recommended for a 20-kg

B. 1 g oral amoxicillin 1 hour prior to dental treatment and 500mg 6 hours after the procedure is the antibiotic regimen recommended for a 20-kg

Which of the following necessitates an increase from the typical amount of curing time when using a tungsten halogen curing light? 1. Microfilled resin 2. Translucent, incisal shade resin 3. Dentin shade resin 4. A larger diameter curing light tip 5. Cold (refrigerated) resin A. 1, 2, and 5 B. 1, 3, 4, and 5 C. 1, 3, and 4 D. 2 and 4 E. 3 and 5

B. 1, 3, 4, and 5 1) Longer light exposures usually are required for the polymerization of dark and opaque shades. 2) An increase from the typical amount of curing light (e.g. 30-40 seconds), when using a tungsten halogen light is necessary in microfilmed resins, dentin shade resin, cold or refrigerated resin and with a larger diameter tip 3) In addition to classic tungsten halogen light, plasma arc curing systems and blue light-emitting diodes (LEDs) are available which are more efficient, more portable, and more durable.

Which of the following is the correct order for listing the occlusal forces applied to the tooth from greatest to least? 1. Molars 2. Premolars 3. Canines 4. Incisors A. 1,2,3,4 B. 1,3,2,4 C. 3,2,4,1 D. 4,3,1,2

B. 1,3,2,4 1) Occlusal force order (greatest --> least) a. Molars > Canines > Premolars > Incisors b. Molars can tolerate maximum amount of masticatory loads while incisors can bear minimal forces of chewing and mastication c. excessive forces can cause wear or fracture of teeth and their cusps. The normal physiologic contact wear of teeth is 20 micrometers/year. d. molars and premolars are important in maintaining the vertical dimensions of face

Which of the following describes how thick the first increment should be for a Class II restoration to ensure proper curing of composite and lowest strain of the tooth structure to a polymerization shrinkage? (Not counting snowplow or sandwich techniques? A. 0.5mm B. 1.0mm C. 2.0mm D. 2.5mm E. <0.5mm

B. 1.0mm 1) The first increment in a Class II restoration must be 1mm thick if snowplow technique is not used. The first increment is placed on the gingival floor of the restoration. 2) The snowplow technique a. employs the use of flowable composites in conjunction with the posterior composite restorations b. an initial thin layer of flowable composite is placed on the gingival or pulpal floor and left uncured at this stage. c. an initial increment of heavily filled composite resin is placed or pushed over the unset flowable composite displacing most of it into the areas where heavily filled composite cannot penetrate. Its advantages are: 1) Decreased voids in the restoration 2) Decreased gingival marginal leakage

The remaining cusps should be evaluated if the faciolingual width of a Class I preparation exceeds a distance of _____ the facial and lingual cusp tips? A. 1/2 B. 1/3 C. 1/4 D. 3/4

B. 1/3 1) If the faciolingual width of a Class I preparation exceeds 1/3 of the distance between facial and lingual cusp tips, the remaining cusps should be evaluated. 2) If extension from a primary groove toward the cusp tip is no more than half the distance, no cusp capping needs to be done. 3) If this extension is 1/2-2/3 of the distance, consider cusp cupping. If the extension is more than 2/3 of the distance, cusp capping is usually recommended.

How many roots are typically possess by the primary maxillary 1st molar A. 2 B. 3 C. 4 D. 1

B. 3 1) Concept: Primary maxillary molars like permanent maxillary molars also present with three roots. 2) The premolars that replace the maxillary molars usually have one of two roots. 3) Primary mandibular molars typically present with two roots.

When restoring an endodontically treated tooth with amalgam, what depth into each root canal should be achieved to obtain satisfactory retention? A. 2mm B. 3mm C. 4mm D. 5mm E. 1mm

B. 3mm 1) Amalgam a. A depth of at least 3mm into the root canals is essential for the retention of an amalgam restoration in an endodontically treated tooth b. depends upon mechanical retention because it does not demonstrate chemical bonding with the tooth structure c. after completing root canal treatment, roughly 3mm of gutta percha should be removed from the root canals using a heated instrument d. space created int he root canal should be filled and condensed with amalgam before carrying out coronal filling (nayyar core technique)

Which of the following describes the pH threshold at which enamel begins to demineralize? A. 4.5 B. 5.5 C. 6.5 D. 6.9 E. 1.5

B. 5.5 1) When the pH at the surface of a tooth drops below 5.5, demineralization proceeds faster than remineralization (there is a net loss of tooth structure on the tooth's surface) 2) Caries process occurs when bacterial acid demineralization excess the remineralization by salivary components.

Which of the following most accurately describes the difference in the thickness of coronal dentin in primary teeth compared with their corresponding permanent tooth? A. 33% B. 50% C. 66% D. 75% E. 25%

B. 50% 1) Coronal dentin of primary teeth is typically 50% of the quantity of the coronal dentin found in permanent teeth because: 2) Primary teeth have pronounced pulp horns which causing less dentin coverage. 3) Permanent teeth are large in size and have smaller pulp spaces in relation to their crown size. 4) Dentin formation occurs throughout life thus increasing the bulk of dentin in permanent teeth

What percentage of liquid oxygen is in an "E" cylinder showing that it is 50% empty A. 25% B. 50% C. 95% D. 100%

B. 50% 1) The pressure regulator measure the gas pressure of the cylinder, and when the pressure shows 50% because oxygen remains as a gas when it is compressed at room temperature and does NOT become a liquid like N2O. 2) Oxygen cylinders come in a variety of sizes and each use and size is designated by a series of numbers and letters. 3) The number on the side of the tank denotes the number of liters of oxygen the cylinder holds. The letters A, B, C, D, and E designate the size of the cylinder. The "A" cylinder is the smallest cylinder with 34 liters of compressed oxygen; and E cylinder is one of the largest (680 liters).

The settings for the nitrous oxide oxygen sedation devices are: 4 liters/min oxygen and 3 liters/min nitrous oxide. At what concentration is the oxygen being administered? A. 43% B. 57% C. 75% D. 98% E. 33%

B. 57% 1) The nitrous oxide and oxygen ratio having 4 liters/min oxygen and 3 liters/min nitrous oxide shows a 57% oxygen concentration. 2) In order to determine the percentage of oxygen use in the nitrous oxide administration, you must divide the liters/min of the gas desired by the total amount of gas being delivered/minute. (4L/min) / (4L + 3L/min) = 4L/min / 7L/min = 0.57*100 = 57%

Approximately what percentage of a permanent tooth's root structure has typically developed when the tooth first emerges into the oral cavity? A. 33% B. 66% C. 80% D. 90% E. 25%

B. 66% 1) Concept: Approximately about 2/3 of the root should already be formed before the tooth erupts into the oral cavity 2) Based on Nolla's stage of tooth eruption, the tooth begins to erupt when the root reaches about 2/3 of its complete form.

Individual Tooth Pathology

Individual Tooth Pathology Answers

Which of the following is the MOST LIKELY explanation for an audible "click" being heard when a patient opens their mouth? A. A flattened articular eminence B. A lack of coordination between the articular disc and condyle C. A spasm of the lateral pterygoid muscle D. A thickened joint capsule

B. A lack of coordination between the articular disc and condyle 1) "click" being heard when a patient opens their mouth a. anterior disc displacement with reduction b. audible click during mouth opening is a sign of a disc displacement disorder within the temporomandibular joint c. popping or clicking sounds in the TMJ indicate that the articular disc is displaced by the condyle during a normal jaw opening d. as the condyle moves back and forth, the articular disc is drawn away or displaced from its normal position and lack of synchronized condylar movement against the articular disc causes the clicking or popping sound 2) Disc displacement with reduction a. usually the type of TMJ disorder that elicits a "click" or "pop" during mouth opening when the disc moves back into place

Which of the following has the LOWEST probability of being a sign or symptom of a metastatic jaw lesion? A. A symptomless radiolucency B. A tooth that has not erupted C. A tooth with no mobility of unknown origin D. Unilateral paresthesia of the patient's lower lip

B. A tooth that has not erupted 1) Non-Metastasis a. un-erupted tooth has the lowest probability of being associated with a metastatic jaw lesion b. to be in conjunction with an odontogenic cyst such as dentigerous cyst 2) Metastasis a. unilateral paresthesia of the lower lips b. symptomless radiolucency c. tooth having mobility of unknown origin

Which of the following is a mandibular plane angle that is higher than average associated with? A. A vertical growth pattern and a Class II division 2 malocclusion B. A vertical growth pattern and a long lower anterior face height C. Deep overbites with a Class I Division II malocclusion D. Deep overbites with a long lower anterior face height E. Deep overbites with a vertical growth patterns

B. A vertical growth pattern and a long lower anterior face height 1) Concept: Because of a vertical growth pattern, the mandibular plane angle becomes steeper and the lower anterior face height also simultaneously increases 2) Patients with deep overbites and a vertical growth pattern may exhibit steep mandibular plane angles but NOT a long lower anterior face height due to the type of dental occlusion that the patient demonstrates. 3) High mandibular plane angles elongate the face because of a lack of distinction between the jaw line and the jaw angle.

Which of the following describes the BEST method to remove carious dentin near pulp in a Class II preparation? A. A bur with high speed hand piece B. A very slowly rotating large bur C. An explorer tip D. Use a spoon excavator by hand

B. A very slowly rotating large bur 1) Concept: It is recommended to remove the carious dentin near the pulp in a Class II preparation with a large very slowly rotating bur. This precaution is necessary to prevent accidental pulp exposures during caries excavation. 2) Spoon excavators are used to remove very soft various dentin. 3) High speed burs are used to prepare cavity walls, remove hard enamel, and carious dentin away from he pulp. 4) Sharp explorer tips are contraindicated to use near the pulp because they might pierce the dentin and create a small pulp exposure.

Which carbohydrate found in human dies is most responsible for caries? B. Glucose E. Sucrose

E. Sucrose 1) Sucrose (glucose + fructose) is used as a sweetener and is fermented by bacteria into acid, which helps form caries. 2) Glucose is cariogenic, but is less cariogenic than fructose since it is a monosaccharide not a disaccharide

Which of the following instruments is MOST useful when polishing an amalgam restoration? A. Abrasive-impregnated rubber points B. All answers apply C. Prophylaxis cup with tin oxide D. Sof-Lex disks

B. All answers apply 1) Concept: Various polishing instruments such as sod-lex disks, abrasive-impregnated rubber points and prophlyaxis cup with tin oxide are used when polishing amalgam restorations. 2) it is advised to carry out finishing and polishing of am algae restorations at least 24 hours AFTER the placement of the amalgam so that complete setting of the amalgam can occur.

According to G.V. Black classification, the outline from of a cavity preparation is in the shape of the cavity preparation B. Along the cavosurface margin C. After caries removal

B. Along the cavosurface margin 1) Outline Form: describes the outline of the tooth surface to be included in the preparation.

According to G.V. Black, the outline form of cavity preparation is the shape of the cavity preparation... A. After retention forms B. Along the cavosurface margin C. Along the pulpal floor D. After caries removal

B. Along the cavosurface margin 1) Concept: Outline form a. describes the outline of the tooth surface to be included in the preparation 2) Internal Finish Line a. dictates the inner perimeter of the cavity floor 3) External Finish Line a. dictates the external perimeter of the outline form b. is defined by 1) Extent of the carious lesion 2) Restorative margins should be extended into cleanable areas 3) Depth of the cavity preparation should provide sufficient bulk for restorative material (2mm for amalgam)

The treatment with the most favorable post treatment prognosis is a tooth with... A. A history of avulsion that was replanted within one hour of avulsion B. An area of limited internal resorption C. External apical root resorption D. External lateral root resorption E. A vertical fracture that extends through the floor of the pulp chamber

B. An area of limited internal resorption 1) Concept: An area with limited internal root resorption provides a more favorable prognosis because there is less irritation incurred to the pulp. 2) Vertical fractures are best manage with tooth extraction since the tooth will always have a fracture line which weakens the tooth and exposes it to the harmful bacteria inside the mouth. 3) External lateral root resorption entails destruction of root surface of the tooth that my weaken the root itself. Treatment external root resorption is also difficult to perform.

Which direction is the articular disc of the TMJ moved if the lateral pterygoid muscle is contracted? A. Anterio-laterally B. Anterio-medially C. Posterio-medialy D. Posterio-laterally

B. Anterio-medially 1) Lateral Pterygoid Muscle a. depresses and protrudes the mandible during its contraction b. articular disc will be directed antero-medially as the TMJ move downward and forward c. main muscle that pulls the condyle head out of the glenoid fossa

A 17-year old female presents to your office with an indurated swelling in her mandibular premolar area that is asymptomatic. Radiographic analysis reveals a 35mm radiolucency associated with the mandibular premolar area. All teeth test vital. What is the next step of our clinical analysis? A. Alkaline phosphatase determination B. Aspiration/needle biopsy C. CT scan D. Excisional biopsy E. Complete blood cell count

B. Aspiration/needle biopsy Steps a. evaluate all the diagnostic data, the next step in diagnosis is to determine the type of cells within the lesion through needle biopsy b. histopathologic examination of cells within a lesion can determine if the lesion is neoplastic or not c. combine the radiographic, visual, physical, and histopathologic assessment of the lesion improves the chances of arriving at correct diagnosis and the proper treatment

Hypertrophy of the mandibular condyle may lead to each of the following scenarios EXCEPT... A. Anterior crossbite B. Anterior open bite C. Asymmetric facial appearance D. Ipsilaeral posterior open bite E. Unilateral Class III malocclusion

B. Anterior open bite 1) Concept: Condylar hypertrophy can affect the occlusal harmony of teeth and could even result in facial asymmetry 2) Condylar hypertrophy may occur with one or both condylar heads. 3) With unilateral condylar hypertrophy the enlarged condyle lowers the jaw angle. The symphysis of the affected jaw will then move towards the non affected side. 4) This can cause unilateral malocclusion, asymmetric facial appearance, possible anterior crossbite, and ipsilateral posterior open bite 5) Condylar hypertrophy can NOT lead to anterior open bite.

Fusion of teeth is an anomaly which normally involves: A. A bacterial etiology at the morphodifferentiation stage of tooth development B. Anterior teeth C. One more dental unit in the arch than is normal D. The splitting of a tooth bud E. The union of two teeth through cementum only

B. Anterior teeth 1) Fusion a. result of an union of two adjacent tooth buds b. usually found in anterior teeth

Which of the following describes the purpose of silane when it is used as a structural component in resin composite? A. A photoinitiator to activate the polymerization of the resin composite B. As a coupling agent to coat filler particles to promote adhesion C. As a matrix, to which other ingredients are added to create the resin composite D. Silane is NOT a structural component in resin composite

B. As a coupling agent to coat filler particles to promote adhesion 1) Silane a. structural component in resin composite to act as a coupling agent to coat filler particles to promote adhesion b. helps to make a bond between the resin composite on one end and the tooth substrate on the other acting as a bi-functional coupling agent. This process is termed as hybridization and the structure formed is called hybrid layer.

When using matrix bands for posterior proximal amalgam restorations, the band should be trimmed to which dimension? A. 0.5mm below marginal ridge height B. At least 1mm higher than the adjacent marginal ridge C. To be equal with the adjacent marginal ridge D. The band should not be trimmed

B. At least 1mm higher than the adjacent marginal ridge 1) Concept: The coronal portion of the matrix band should be located approximately 1mm higher than the adjacent marginal ridge to facilitate proper contouring and adaptation of the restorative material along the proximal wall and marginal ridge of the restoration. 2) Placing the matrix band too high will cause a. Access and visibility problems b. Difficult instrumentation c. Difficult placement of materials 3) Matrix bands should also be placed tightly hugging the tooth to ensure proper marginal adaptation and anatomical form.

Which of the following variables is considered the MOST important for determining the prognosis of a tooth? A. Alveolar bone level B. Attachment level C. Bleeding on Probing (BOP) D. Probing depth E. Tooth sensitivity

B. Attachment level 1) Clinical attachment level (CAL) a. MOST important variable when determining the prognosis of the tooth 2) Bleeding on Probing (BOP) and Pocket Depth a. can be found on with gingivitis and periodontitis b. less important than CAL when determining the prognosis 3) Variable connection between alveolar bone height and clinical attachment level.

Which of the following describes the loss of tooth structure through contact with another tooth? B. Attrition

B. Attrition 1) Attrition a. loss of tooth structure by contact with another tooth b. occurs most frequently in patients who suffer from bruxism or who habitually intake stimulants both prescribed and illegal 2) Abrasion a. describes the loss of tooth structure by an external force like dentifrice 3) Erosion a. loss of tooth structure through chemical dissolution by acids NOT of bacterial origin

When a tooth is completely removed from the alveolus, it is termed... A. Ankylosis B. Avulsion C. Subluxation D. Luxation

B. Avulsion 1) Concept: Avulsion a. happens when a tooth is forcefully dislodged or removed from its socket as a result of an accident b. avulsed teeth should be immediately replanted back to its socket and be splinted to adjacent teeth for increased stability before further procedures are performed on the tooth 2) Ankylosis a. occurs as the alveolus forms reparative bone (callus) to surround the tooth and replace the damaged PDL fibers.

Which of the following accounts of the deleterious effects of calculus? A. Attachment problems due to surface irregularities B. Bacteria attached to the calculus release irritants C. Mechanical irritation D. The calculus releases irritants

B. Bacteria attached to the calculus release irritants 1) Calculus a. dental plaque that has become calcified and is often covered by a layer of uncalcified plaque which releases exotoxins and other chemicals that cause inflammation

Which of the following is not a characteristic of ectodermal dysplasia? A. Atrophic skin B. Blue sclera C. Defective hair D. Hypoplastic sweat glands E. Hypodontia (oligodontia)

B. Blue sclera 1) Concept: Blue scleras are NOT a characteristic of Ectodermal dysplasia, but are a clinical sign that usually manifests in diseases like Osteogenesis imperfect, Ehlers-Danlos syndrome, and Marfan syndrome 2) Ectodermal dysplasia a. group of genetic disorders which occur due to poor development of organs and body structures that are ectodermal in origin b. may cause abnormal development of failure of formation of the structures within the body that originate from the ectoderm which includes the teeth, skin, hair, and sweat glands

Which of the following procedures should be performed after a dentist reflects a flap, debrides 2 and 3-walled defects, and scales and root planes the teeth of a 55-year old male with deep fibrotic pockets and angular bone loss? A. Apically repositioned flap B. Bone grafting C. Crown lengthening D. Gingivectomy E. Modified Widman flap

B. Bone grafting 1) Concept: Bony defects with 2 or 3 walls in the alveolar bone and teeth that exhibit angular bone loss are considered good candidates for bone grafting. 2) Prognosis of bone grafting success increases for bony defects having 2 or more supporting walls 3) Apically repositioned flaps and crown lengthening procedures are NOT indicated for cases having angular bone loss and large bony defects. 4) Gingivectomy is only indicated for patient having excessive gingival exposure and is NOT used for eliminating deep pockets.

Which of the following is LEAST PROBABLE for adolescents and pediatric patients to develop? A. Ameloblastic Fibro-odontoma B. Calcifying epithelial odontogenic tumor C. Compound odontoma D. Adenomatoid odontogenic tumor

B. Calcifying epithelial odontogenic tumor 1) Concept: Calcifying epithelial odontogenic tumor (Pindborg tumor) is a neoplasm that commonly arises son the posterior aspect of the mandible during 4th and 5th decade life 2) It is a benign tumor that is thought to have originated from the epithelial remnants of the stratum intermedium of the enamel organ. 3) It is a slowly progressing painless swelling that is often associated with impacted teeth and occurs most frequently in the molar area but may also appear near premolars. 4) Presence of amyloid-like deposits appear microscopically and a "driven-snow" appearance is observed radiographically.

In order to effectively translate the roots of teeth, orthodontic appliances must be which of the following? A. Capable of exerting a positive intermittent force B. Capable of exerting a torque/moment C. Extremely gentle in action D. Used with an extraoral force

B. Capable of exerting a torque/moment 1) Concept: Orthodontic appliances must be capable of exerting torque/moment of force in order to effectively translate the roots of the teeth 2) If the line of action of force passes through the center of resistance of a tooth, the tooth will respond to pure bodily movement or translation of the roots. 3) All of the PDL is uniformly loaded during the translation movement. Approximately 100-150g of force is needed to produce this movement.

The most serious effects of repeated exposure to X-radiation in low doses is characterized by which of the following? A. Alteration of oral microflora B. Carcinogenesis C. Diarrhea D. Pupura E. Radionecrosis

B. Carcinogenesis 1) Concept: Carcinogenesis a. Repeated low doses of ionizing radiate may induce irreparable DNA damage which leads to errors in replication and transcription that result in carcinogenesis b. Persistent low dose radiation can induce a type of genomic instability in cells that may trigger viral interactions leading to pre-mature aging and carcinogenesis c. Ionizing radiation results in the enhancing of the frequency of mutation and other genetic changes in the descendants of the irradiated cell after many generations of replication. d. Gap junction mediated cell-cell communication and activation of the p53 damaged response pathway may result in carcinogenesis.

What is the greatest risk to a patient with adrenal insufficiency if they are subjected to a highly stressful situation? A. Atrial fibrillation B. Cardiovascular collapse C. Hyperventilation D. Hypoxia E. Hypotension

B. Cardiovascular collapse 1) Concept: Patients suffering from adrenal insufficiency are at risk of cardiovascular collapse when subjected to highly stressful situations 2) Continuous low cortisol levels during stressful events, contribute a lower cardiac output and low blood pressure that could eventually lead to cardiovascular collapse. 3) Adrenal insufficiency occurs when the adrenal glands produce insufficient amount of steroid hormones like cortisol and aldosterone 4) Cortisol a. stimulate gluconeogenesis in the liver. Cortisol also helps maintain normal blood pressure during stressful situations and creates additional energy to cope up with such stress. b. increases blood pressure by increasing the vasculature sensitivity to epinephrine and norepinephrine which stimulate cardiac output

In patients with cusps undermined, by decay and heavy occlusal forces, the restorative material of choice is... A. Amalgam B. Cast gold C. Glass Ionomer D. Composite resin

B. Cast gold 1) Cast Gold a. Patient with undermined cusps and heavy occlusal forces, cast gold is the restorative material of choice b. Gold is the most durable restorative material c. can be easily finished, polished, burnished and adapted to tooth structure to decrease recurrent caries risk d. highest corrosion resistance of any restorative material

How do preparations for proximal amalgam and cast gold inlay differ? A. All answers are correct B. Cast inlay preparations require a bevel C. No answers are correct D. Resistance form design principles E. Use of instruments F. Amalgam preparations require a bevel

B. Cast inlay preparations require a bevel 1) Concept: Cast gold inlay preparations require a bevel to remove unsupported enamel rods and render a smooth flowing form of cavity preparation 2) The bevel on the margin allows for better marginal adaptation through burnishing. 3) Amalgam has a weak edge strength, requires a 90o cavosurface margin and an adequate thickness for strength and durability.

Which of the following describes how the width of keratinized gingiva is measured. It is the distance from the ... A. Cemento-enamel junction (CEJ) to the depth of the sulcus B. Cemento-enamel junction (CEJ) to the mucogingival junction C. Free gingival groove to the mucogingival junction D. Free gingival margin to the depth of the sulcus E. Free gingival margin to the mucogingival junction

B. Cemento-enamel junction (CEJ) to the mucogingival junction 1) Band of keratinized gingiva a. extends from the free gingival margin to the mucogingival junction 2) Attached gingiva a. free gingival groove to the mucogingival junction 3) Free gingival groove (mucogingival line) a. shallow line or depression on the gingival surface at the junction of the free and attached gingiva 4) Free + Attached = Keratinized Gingiva

New attachment is defined as the attachment of the gingival epithelium to the root surface and the embedding of new periodontal ligament fibers into new... A. Bone B. Cementum C. Epithelium D. Dentin

B. Cementum 1) Concept: Formation of new attachment occurs when the newly formed periodontal ligament fibers embed themselves into the cementum of a preciously derided root surface. 2) New attachment is the reunion of connective tissue with a root surface that has been deprived of its periodontal ligament. 3) The formation of periodontal ligament in the new cementum can only be achieved through guided tissue regeneration.

Which of the following does squamous cell carcinoma of the tongue MOST COMMONLY metastasize to? A. Brain B. Cervical nodes C. Liver D. Lungs E. Bone

B. Cervical nodes 1) Concept: Squamous cell carcinoma of the tongue most commonly metastasizes to the cervical lymph nodes because the lingual lymph drainage occurs through the cervical lymph nodes. 2) The spread of carcinoma in the cervical lymph noes typically exults in its poor prognosis. 3) Surgical neck dissection is typically used to treat this type of cancer.

The earliest manifestation of a carious lesion is... A. Cavitation of enamel B. Change in enamel opacity C. Radiolucency D. Rough surface texture E. Sensitivity

B. Change in enamel opacity 1) change in the opacity of enamel becomes more evident along the areas of active caries progression 2) Caries process demonstrates area of demineralization that are suggested by a chalky white appearance in the enamel. 3) Change in enamel opacity will eventually leda to further tooth surface destruction and formation of tooth decay/cavitation is not addressed immediately.

Composite is bonded to adhesive by which of the following means. A. Both mechanically and chemically B. Chemically C. Mechanically D. No answers apply

B. Chemically 1) Concept: Composite resin restorative materials is chemically bonded to adhesive resin through polymerization through MMA bonds. 2) Mechanical bonding occurs between the adhesive and tooth substrate such as dentin/enamel. 3) The zone in which low viscosity resin of the bonding resin or adhesive system micro mechanically interlocks with dentinal collagen is termed as hybrid layer or hybrid zone.

Sinus tract are MOST COMMONLY found in the gingiva of children because of which of the following reasons? A. Acute periapical abscesses B. Chronic periapical abscesses C. Lateral periodontal cysts D. Periapical cysts E. Periodontal abscesses

B. Chronic periapical abscesses 1) Sinus tracts a. commonly found due to the development of chronic periodical abscesses in the gingival of children. b. Dental abscesses can also be associated with trauma. 2) Acute periodical abscess are not abscessed long enough to create a sinus tract 3) Lateral periodontal cysts and periapical cysts DO NOT contain a tract, but are an epithelial lined pocket. 4) Periodontal abscesses have room to expand into the gingiva, but do NOT commonly result in a sinus tract.

Which of the following defects has the WORST prognosis for bone grafting? A. Class III furcation defect B. Class IV furcation defect C. One-walled D. Three-walled E. Two-walled

B. Class IV furcation defect 1) Furcation Defects Class I a. concavity just above the furcation entrance can be felt with the probe tip b.furcation probe (Nabor's) cannot enter the furcation area Class II a. probe is able to partially enter the furcation - extending approximately 1/3 of the width of the tooth = but it is NOT abel to pass completely through the furcation Class III a. in mandibular molars, the probe passes completely through the furcation between the medial and distal roots b. in maxillary molars, the probe passes between the mesiobuccal and distobuccal roots and toughs the palatal root. Class IV a. same as Class III furcation involvement except that the entrance to the furcation is visible clinically due to tissue recession

* Which of the following terms describes the distance from the CEJ to the depth of the gingival sulcus/pocket? A. Alveolar bone loss B. Clinical attachment level C. Clinical attachment loss D. Gingival pocket depth E. Gingival recession

B. Clinical attachment level 1) Clinical attachment level a. calculated as the distance from the CEJ to the depth of the sulcus 2) Clinical attachment loss a. amount of attachment that has been lost and the gingival 3) Gingival pocket depth a. distance from the free gingival margin to the depth of the gingival sulcus

The preferable treatment option for a fractured incisal of a mandibular central incisor that is not pulpally involved in a 10 year old is... A. A porcelain fused to metal crown B. Composite resin placement C. Glass ionomer placement D. An amalgam buildup with pins for retention

B. Composite resin placement 1) Composite resin placement a. ideal treatment option for an incisal fracture b. 10 years old have large pulps, so a crown preparation is not indicated because it might cause harm to the recovering pulp 2) Glass Ionomer a. have short longevity because cannot withstand high occlusal stress load

While diazepam is being administered via I.V. to a patient for sedation, the patients begins to displays ptosis of the upper eye-lid (Verrill's sign). Which of the following should the clinician think? A. Administer flumazenil to the patient B. Consider the patient to be adequately sedated C. Immediately assist the patient with respiration D. Place the patient in the Trendelenburg position E. Administer additional diazepam and begin the procedure

B. Consider the patient to be adequately sedated 1) Verill's sign a. indicates that the effective sedative drug dose has already reached and dose must be kept unchanged to maintain the level of sedation b. during this stage of sedation, the patient can still understand verbal cues and can still respond to them c. administering additional diazepam during the displaying of verill's sign would cause overdose of the drug d. benzodiazepine receptor antagonist like flumazenil must be administered in cases of diazepam drug overdose.

During the extraction of a mandibular molar, he medial root is fractured. The coronal portion, distal root, and half of the medial root were delivered intact. What instrument should be used to remove the remnants of the medial root? A. Crane pick B. Cryer elevator C. Rongeurs D. Straight elevator E. Lower universal forceps

B. Cryer elevator 1) Cryer elevator a. can be used to gain access by easily removing the intraradicular bone and elevating the remaining root fragment 2) Crane picks a. indicated for the delicate removal of smaller root fragments 3) Rongeurs a. indicated for bone removal, not extraction of teeth

If a hygienist in your office leaves an ultrasonic tip on one spot on a tooth for too long which of the following is most likely result? A. Burnish calculus onto tooth B. Damage tooth surface C. Irreversibly damage pulp D. Dull the instrument

B. Damage tooth surface 1) Concept: Vibrations of ultrasonic instruments may cause damaged to the tooth when the ultrasonic tip is left on one spot for prolonged period of time 2) Ultrasonic instruments must be limited 4-6 minutes per tooth; otherwise harm might be incurred to the surrounding tissues. 3) Other probable sources of tissue damage include: a. use of inappropriate ultrasonic tip b. excessive power range c. excessive pressure d. inadequate water coolant e. use of broken instrument tip

Establishing drainage is an important aspect of emergency treatment for an acute periradicular abscess. An equally important aspect is... A. Adjusting occlusion B. Debriding the canal system of necrotic tissues C. Introducing antimicrobial medicaments into the pulp space D. Prescribing analgesics

B. Debriding the canal system of necrotic tissues 1) Debriding the canal of necrotic tissue a. will decrease the amount of bacteria that form abscesses b. bacterial niche will be removed and the root canal system will be cleared of bacterial infection with good debridement c. after debridement in an acute periradicular abscess, it is also a good practice to place an antimicrobial medicament like Ca(OH)2

Which of the following creates the phenomena of dentinal plugs? A. Adhesive B. Debris from the smear layer C. Oxalic acid D. Primer (from micro/macrotags) E. Sclerotic dentin

B. Debris from the smear layer 1) Dentinal or smear plugs are composed of debris from the smear layer 2) Smear layer is documented to be about 0.5-2.0mm thick layer of debris with a mainly granular substructure that entirely covers the dentin. The surface of the smear layer appears quite irregular. 3) The orifices of dentinal tubules are obstructed by debris tags known as smear plugs or dentinal plugs. These dentinal plugs extend into the tubules to a depth of 1-10 micrometers. 4) Smear layer is reported to reduce dentinal permeability by about 86% due to dentinal plugging. 5) Dentinal plugs are removed by acid etching the tooth surface with phosphoric acid.

Radiographs reveal a radiolucency where the right 3rd molar usually resides. The mandibular right 3rd molar is not present. Which of the following should NOT be included in a differential diagnosis? A. Ameloblastoma B. Dentigerous cyst C. Odontogenic keratocyst D. Odontogenic myxoma E. Residual cyst

B. Dentigerous cyst 1) Dentigerous cyst a. also known as follicular cysts and occur along the crown area of a partially erupted or unerupted tooth b. lining of the dentigerous cyst originates from the epithelial cells of the reduced enamel epithelium of the tooth 2) Cyst present along the area of 3rd molar a. 3rd molar was already extracted may be due to the b. formation of a residual cyst c. ameloblstoma d. odontogenic myxoma e. keratocyst

Which of the following conditions exhibit globular dentin, early pulpal obliteration, defective root formation, periodical granuloma/cysts, premature exfoliation, and appear to be normally clinically? A. Amelogenesis imperfecta B. Dentin Dysplasia C. Dentinogenesis imperfecta D. Regional Dontodysplasi E. Shell teeth

B. Dentin Dysplasia 1) Dentin Dysplasia a. disturbance in normal dentin formation causing b. pulpal chamber obliteration c. alteration or absence of root formation d. premature tooth exfoliation 2) Amelogenesis Imperfecta a. affects ONLY the formation of enamel and NOT dentin 3) Dentinogenesis Imperfecta a. form of dentin dysplasia which causes discoloration or some sort of translucency to affected teeth

Which of the following methods CANNOT be used to help patients control their experience of pain? A. Cortical depression B. Depression of the autonomic pathways C. Psychosomatic hypnosis D. Raising their pain threshold E. Blocking nocioceptive pathways

B. Depression of the autonomic pathways 1) inhibiting pain signals from reaching the brain by blocking the nociceptive pathways help to reduce pain 2) raise the pain threshold using medications like analgesics and sedatives 3) Psychosomatic hypnosis helps condition the mind and body of a patient to handle painful stimuli better

Which systemic disease lowers the patients' immune system capacity and decreases wound healing potential? A. Beriberi B. Diabetes Mellitus C. Pelegra D. Polycystic kidney disease E. Systemic lupus erythematosus

B. Diabetes Mellitus 1) Concept: Diabetes Mellitus is a systemic disease that lowers the body's immune response because of an altered cell mediated immune response 2) Effects a. Impaired neutrophil chemotaxis, causing slower reaction against pathogens and faster rate of progression of infection b. Impaired would healing potential since blood flow to tissue is decreased c. Peripheral and autonomic neuropathy d. Chronic kidney disease e. Cardiovascular disease f. Eye related problems

Immediately following a procedure with prolonged high levels of nitrous oxide, a patient immediately removes the breathing apparatus. What is the most likely consequence? A. Carbon dioxide retention B. Diffusion hypoxia C. Respiratory depression of apnea D. Vasovagal syncope E. Hyperventilation syndrome

B. Diffusion hypoxia 1) Diffusion hypoxia a. may occur when a patient is recovering from N2O anesthesia, large quantities of this gas cross from the blood into the alveolus (down its concentration gradient) and so for a short period of time, the O2 and CO2 in the alveolus are diluted by this gas. b. Diluting the O2 and CO2 can cause the partial pressure of oxygen to decrease, temporarily inducing hypoxia. The decrease in CO2 could also potentiate this effect as ventilation would be suppressed, leading to potential hypoxemia.

Which of the following clinical CNS signs suggests that a patient is suffering from oxygen deprivation? A. Dilated pupils with light reflex B. Dilated pupils without light reflex C. Pinpoint pupils without light reflex D. Pinpoint pupils with stronger light reflex

B. Dilated pupils without light reflex 1) Concept: Patients suffering from oxygen deprivation will demonstrate dilated pupils without light reflex because the brainstem that controls pupillary response lacks oxygen, causing the cells to die or be damaged. 2) Routine assessment of the pupillary reflex a. utilized by physicians for checking brain stem function b. Pupils normally dilate equally to allow more light to enter the eyes allowing vision even in the dark c. Pupils constrict when there is too much light needed by the eyes to see. 3) Lack of pupillary reflex or an unusual abnormal pupillary reflex could be a result of brainstem cell damage, optic nerve damage, occulomotor nerve damage or due to depressant drugs like the barbiturates.

Which of the following is the MOST COMMON cause if intracapsular restraint of mandibular movement? A. Ankylosis B. Disc interference disorders C. Infection D. Myofacial pain disorders

B. Disc interference disorders 1) Disc interference disorders a. most commonly cause intracapsular restraint of mandibular movement in adults b. associated with dysfunctional condyle disc complex and are also termed as internal derangement disorders c. commonly associated with the anterior and medial displacement or dislocation of the articular disc d. Signs and Symptoms 1) clicking sounds 2) catching or locking during jaw movements 3) may or may NOT be associated with pain

Following a difficult surgical extraction, a patient was diagnosed with a postoperative infection and has been placed on oral penicillin for three weeks. At her exam today, you notice that the surgical site is well healed, but there are white patches on the dorsum of her tongue that scrapes off. What is the recommended course of treatment? A. Discontinue all antibiotics only B. Discontinue penicillin and prescribe nystatin troches C. Discontinue penicillin treatment and prescribe a broad spectrum antibiotic D. Discontinue penicillin and prescribe a mouthwash containing metronidazole

B. Discontinue penicillin and prescribe nystatin troches 1) Concept: Prolonged intake of antibiotics disturbs the balance of microflora in the mouth by killing the "normal" bacteria, allowing the fungal species like Candida albicans to grow unhindered by competition. 2) The white patches on the coral surface of the tongue that appeared after prolonged antibiotic use are caused by the increased use are caused by the increased number of Candida albicans. 3) Nystatin troches are considered an effective treatment modality for Candida infections.

When instrumenting the medial root of a mandibular 1st molar, which of the following canal surfaces is the most likely to become stripping-perforated? A. Buccal B. Distal C. Lingual D. Mesial

B. Distal 1) Mesial root of mandibular 1st molar a. distal surface of the medial root of the mandibular 1st molar is most likely to undergo strip perforation during root canal instrumentation b. mesial surface of the distal root and the distal surface of the mesial root in the mandibular 1st molar have a root concavity making the dentin walls very thin c. overenthusiastic root canal instrumentation (overcutting) can lead to strip perforations in these areas and should be avoided

Of the following dental anomalies, the one in which there is great potential for the premature loss of the affected tooth, is: A. Consecrence B. Dwarfed root C. Flexion D. Gemination E. Enameloma

B. Dwarfed root 1) Concept: Dwarfed root characteristics: a. normal size crowns have abnormally short roots b. normally found in anterior teeth only (maxillary central incisors is most common) c. teeth may be lost at an early age due to passive eruption or periodontal disease

A 36 year old woman, Betsy, presents with a badly broken down #19 with a necrotic pulp and an acute apical abscess. Which of the following medical histories would create the most complication in her surgical extraction treatment? A. Asthma B. Epilepsy C. Hypertension D. Mitral valve prolapse E. Undergoing chemotherapy F. Chronic Obstructive Pulmonary Disorder (COPD)

B. Epilepsy 1) Concept: Epileptic disorders vary widely in etiology and manifestation, so management of these patients is performed on a case by case basis. Dental treatment may trigger seizure attacks at any time during the visit. Dental treatment should be deferred if a patient has an epileptic episode. 2) Epileptic seizures occur because of excessive or abnormal neuronal activity in the brain. Epilepsy can be treated but not cured with medication. Approximately 30% of medicated epileptic patients cannot control seizure attacks. 3) Anticonvulsant side-effects may differ among individuals. Epileptic seizures may manifest differently with different signs and symptoms, so this should be taken into consideration prior to performing procedures. 4) The suggested method to manage generalized tonic-clonic epileptic seizure is: a. Roll the person on their side to proven fluid ingestion into the lungs which if not prevented could cause irawy blockage and death b. The person should be moved away from sharp edges with their head supported with a soft cushion of pillow if possible. c. Emergency services should be contacted immediately if seizure lasts for more than 5 minutes, or if it occurred more than once without regaining unconsciousness.

In patient suffering from multiple sclerosis A. A single cartridge of anesthetic will most likely not last as long as it would for a normal patient B. Epinephrine is contraindicated in local anesthetic C. The amount of anesthetic needed for a given procedure is more than for a normal patient D. The amount of anesthetic needed for a given procedure is less than for a normal patient

B. Epinephrine is contraindicated in local anesthetic 1) Epinephrine should not be used for patients suffering from multiple sclerosis because epinephrine can aggravate the condition by regulating the activation of blood lymphocytes. 2) Epinephrine is a cetecholamine and neurotransmitter responsible for the body's flight- or flight response. 3) Elevating the levels of epinephrine also increases blood lymphocyte activity which could cause further damage to the myelin sheaths of the neurons thereby aggravating multiple sclerosis condition.

Which of the following is caused by gastric regurgitation? A. Attrition B. Erosion E. Abrasion

B. Erosion 1) Erosion a. caused by gastric regurgitation or reflux, which is characterized by the back flow of gastric juices in the oral cavity b. contain HCl, which damages the tooth structure because it has a pH<5.5 c. loss of tooth structure through chemical dissolution by acids NOT created by bacteria d. by ingestion of acidic beverages in males e. by bulimia in adolescents or females f. occurs when oral pH drops below 5.5

According to Black's steps in cavity preparation, which step occurs first? A. Clean preparation area in attempts to remove any superficial debris and to ensure a clear working field B. Establish outline form based on location and extent of carious lesion C. Establish outline form by removing deepest portion of carious lesions first D. Obtain convenience form by removing carious lesion and tooth structure adjacent to it to allow adequate observation, accessibility and ease of operation

B. Establish outline form based on location and extent of carious lesion 1) Concept: G.V. Black Summary a. Establish outline form based on location and extent of carious lesion b. Establishment of resistance form c. Establishment of retention form d. Establishment of convenience form e. Removal of any remaining residual caries, enamel pits and fissures, infected dentin or old restorative material if indicated f. pulp protection if indicated g. toilet of the cavity h. procedures for finishing and polishing of the restoration

During an initial exam appointment, the dentist should first... B. Establish rapport D. Gather information

B. Establish rapport 1) evaluate patient's concerns 2) build patient trust

Which of the following is necessary to determine the exact form of an osseous defect? A. Examining the gingival architecture in detail B. Expose the area surgically C. Make multiple radiographs from different angulations D. Measure pocket depths from the cemento-enamel junction E. Measure pocket depths from the free gingival margin

B. Expose the area surgically 1) Surgically exposing an intrabony defect allows the examiner a complete access to evaluate all the dimensions of an osseous defect directly. 2) Gingival architecture may appear normal radiographically despite the presence or a large intrabony defect due to the nature of a radiograph being a 2-dimensional representation of a 3-dimensional object.

Which of the following is the MOST LIKELY adverse effect of internally bleaching an endodontically treated tooth with superoxide? A. Demineralization of tooth structure B. External cervical root resorption C. Reinfection of the canal D. Weakening of the remaining tooth structure

B. External cervical root resorption 1) Concept: The most common side effect of internally bleaching a tooth with superoxide is external cervical root resorption. 2) The presence of a glass ionomer barrier over the root filling material may help to reduce the instances of external cervical root resorption if it is at least 4mm in thickness. 3) The walking bleach technique using sodium perborate is NOT associated with external cervical root resorption. 4) Bleaching dentin dose NOT weaken the tooth structure.

Which of the following treatments is indicated for a 5-year-old patient with a sinus tract and a dark brown discoloration associated with their maxillary central incisor that appeared following trauma. A. Direct pulp cap B. Extraction C. Indirect pulp cap D. Pulpectomy E. Pulpotomy

B. Extraction 1) Extraction a. of the primary maxillary central incisor is induced for a 5 year old child when the tooth is associated with a sinus tract and discoloration b. NOT advisable to retain this tooth considering the proximity of the permanent successor and age of the patient. Retention of this diseased tooth can lead to abnormal function of permanent tooth. 2) Turner's Tooth a. describes when the permanent tooth bud is affected via infection or trauma of a primary tooth 3) Infection a. most commonly affects the canine b. trauma most commonly affects the maxillary central incisor

How is a cast gold onlay superior to a cuspal coverage amalgam? Select all that apply. A. Decreased levels thermal conductivity B. Higher compressive strength C. More precise inter proximal contacts D. Decreased elastic modulus

B. Higher compressive strength C. More precise inter proximal contacts 1) Concept: Cast gold inlays are fabricated more precisely because they are outside of the mouth a. It is easier to manipulate material when there is better access. b. There is no interaction with oral fluids to contaminate the materials. 2) Cast gold inlays have better compressive strength and are less likely to fracture.

Design principles for Class II amalgams include which of the following? Select all that apply. A. Extension of the prep to line facial and lingual line angles for better cleansibility B. Facial/Lingual extension of the gingival box to the point where contact is totally removed C. Removal of unsupported enamel and loose enamel rods D. A rounded pulpo-axial line angle

B. Facial/Lingual extension of the gingival box to the point where contact is totally removed C. Removal of unsupported enamel and loose enamel rods D. A rounded pulpo-axial line angle 1) Concept: Rounded pulpo-axial line angle provides resistance to fracture for the amalgam restoration by lessening the occlusal stress concentration. 2) Loose enamel rods and unsupported enamel has to be removed because they are weak areas of tooth that may fracture against occlusal load. 3) The extension of the facial and lingual preparation within the gingival box should be out of contact to its adjacent tooth to provide room for access, instrumentation and proper restoration of contacts.

Which class of bacteria that is implicated in odontogenic infections is able to survive in both high and low oxygen environments? A. Enteric organisms B. Facultative organisms C. Microaerophilic organisms D. Synergistic organisms E. Obligate anaerobes

B. Facultative organisms 1) Facultative organisms a. can live with or without oxygen 2) Facultative anaeorbic bacteria a. create their own energy source or ATP by aerobic respiration through oxidative phosphorylation and anaerobic respiration through fermentation b. require more energy to survive, so they can produce ATP at a faster rate through anaerobic glycolysis

Which of the following describes the proper length at which the tooth should be filed and gutta percha be placed? A. Filing 0.5-1.0mm beyond the radiographic index and obturating to the radiographic index B. Filing 0.5-1.0mm short of the radiographic index and obturating to 0.5-1.0mm short of the radiographic apex C. Filing to the radiographic index and obturating 0.5-1.0mm short of the radiographic index D. Filing to the radiographic apex and obturating to the radiographic apex

B. Filing 0.5-1.0mm short of the radiographic index and obturating to 0.5-1.0mm short of the radiographic apex 1) Filing procedures a. recommended to keep the preparation length 0.5-1.0mm short of the radiographic apex during root canal treatment. b. leaves a pulp stump (apical constriction), which prevents extrusion of irritating filling materials into the periradicular tissues. c. Filing should be carried out followed by gutta percha filling placement 0.5-1.0mm short of the radiographic apex to provide a hermetic apical seal.

Which of the following statements it CORRECT? A. All answers are correct B. Finishing diamond burs impart less surface damage than carbide burs C. Metamerism aides the dentist while selecting a shade D. Rebonding decreases both marginal integrity and color stability E. Retentive pins should be used in Class IV restorations

B. Finish diamond burs impart less surface damage than carbide burs 1) Finishing Diamond Burs a. impart less surface damage as compared to carbide burs 2) Retentive pins a. in Class IV restorations are CONTRAINDICATED because they can show through the tin enamel and give an unaethetic appearance to the solution 3) Rebonding a. completed at the end of restorative procedures is does NOT decrease marginal integrity and color stability 4) Metamerism a. complication in color perception as various light sources produce different perceptions of color. It will create problems in shade selection.

In the United State populace, which of the following describes the MOST COMMON location premalignant lesions of squamous epithelium are observed? A. Dorsum of the tongue B. Floor of the mouth C. Gingiva D. Hard palate E. Soft Palate F. Buccal mucosa

B. Floor of the mouth 1) Concept: The floor of the mouth and lateral border of the tongue are the most common locations for the development of squamous cell carcinoma inside the mouth. 2) An explanation for the high potential for premalignant lesions in this area is due to its rich blood supply because rapidly dividing malignant cells require more nutrition than normal cells. 3) If an ulcer or lesion develops on the floor of the mouth, healing is difficult because of the high degree of mobility of the tongue and floor of the mouth.

Which of the following describes Low viscosity resins? A. Condensable B. Flowable C. No answers apply D. Packable

B. Flowable 1) Concept: Flowable composites a. are low viscosity resins b. are low viscosity based materials that differ in filler load and resin content. Their filler content is 20 to 25% lower than conventional composites and the increase in resin component particularly diluent monomers such as TEDGDMA which gives the material its flow properties. 2) The greater the filler content in resin, the greater the viscosity of resin and more difficult its adaptation to cavity walls and areas resulting in voids and gap formation is.

A mature pulp does all of the following EXCEPT one. What is the EXCEPTION?... A. Forms dentin B. Forms enamel C. Provides the nerve blood supply to the tooth D. Forms cementum

B. Forms enamel 1) Mature pulp a. lay down tertiary dentin throughout life for protection from physical or chemical insult b. contain capillaries that approximate the dentin just below the layers of odontoblasts, and nourish the entire tooth c. innervated by autonomic nerve fibers that control smooth muscles of arterioles and the afferent sensory fibers coming from the trigeminal nerve

Which of the following are considered an adverse side effect of corticosteroid use EXCEPT one. Which is the EXCEPTION? A. Depression B. Gastro-intestinal disturbance C. Moon face D. Striae E. Weight gain F. Buffala Hump

B. Gastro-intestinal disturbance 1) Concept: Cushing's syndrome DOES NOT include gastrointestinal disturbances 2) Cushing's syndrome describes the signs and symptoms associated with prolonged exposure to inappropriately high levels of the hormone cortisol b, can be caused by taking glucocorticoid or diseases that result in excess cortisol, adrenocorticotropic hormone (ACTH), or CRH levels. c. Signs and symptoms include: 1) Extreme weight gain 2) Buffalo hump (fat deposition in the upper central back) 3) Moodiness, irritability, or depression 4) Muscle and bone weakness 5) Memory and attention dysfunction 6) Osteoporosis 7) Diabetes Mellitus 8) Hypertension 9) Immune suppresion 10) Sleep disturbances 11) Menstrual disorders such as amenorrhea in women and decreased fertility in men 12) hirsutism 13) baldness 14) hypercholesterolemia 15) edema Corticosteroid Adverse Side Effects: "CUSHINGS BAD MD": C: Cataracts U: Up all night (sleep disturbances) S: Suppression of HPA axis H: Hypertension I: Infections N: Necrosis (avascular) G: Gain Weight S: Striae B: Buffalo Hymp A: Acne D: Diabetes M: Myopathy, moon faces D: Depression and emotional changes

Which of the following is an example of non-keratinized gingival tissue? A. Attached gingiva B. Gingival col C. Marginal gingiva D. Outer gingival epithelium

B. Gingival col 1) Gingival tissue a. 75% parakeratinized epithelium b. 15% keratinized epithelium c. 10% nonkeratinized epithelium 2) Keratinized epithelium a. attached gingiva b. some portions of the outer gingival epithelium 3) Parakeratinized gingiva a. marginal gingiva b. portions of the outer gingival epithelium 4) Nonkeratinized gingiva a. gingiva col b. junctional epithelium c. sulcular epithelium

Incipient interproximal caries are typically found... A. Facial to the contact B. Gingival to the contact C. Incisal to the contact D. Lingual to the contact E. At the contact

B. Gingival to the contact 1) Incipient interproximal caries a. found gingival to the contact area b. extend up to half of the thickness of enamel and do not extend beyond the enamel

Which of the following does plaque accumulation DIRECTLY affect? A. Accumulation of calculus B. Gingivitis severity C. Herpes outbreak severity D. Materia alba level E. Periodontitis severity

B. Gingivitis severity 1) Plaque a. The severity of gingival inflammation is directly related to the amount of plaque accumulation b. when bacterial toxins are released by plaque, the gingiva become irritated, initiating a defensive reaction through an inflammatory response 2) Material alba a. soft debris on the tooth surface which is easily removed and does NOT cause gingivitis 3) Calculus = mineralized plaque a. a presdisposing factor for plaque build up.

Which of the following materials is used as the base material for the bonded-base (open sandwich technique)? A. Bonding adhesive B. Glass Ionomer C. Hybrid fill D. Micro fill E. Nanofill

B. Glass Ionomer 1) Concept: The bonded base or open sandwich technique liner is made up of glass ionomer cement or resin modified glass ionomer cement. 2) In an open sandwich technique, a restorative glass ionomer cement or resin modified glass ionomer cement is used in the portion of the restoration near the gingival margin. 3) Open sandwich or bonded base technique is recommended in composite restorations with margins in thin enamel near the CEJ or on cementum or dentin. 4) Composite resins are placed over the glass ionomer cement without impending the contact of glass ionomer cement with oral cavity.

Which of the following materials demonstrates the highest rate of fluoride release, but also the least resistant to wear? A. Compomer B. Glass Ionomer C. Resin Composite D. Resin-Modified Glass Ionomer E. Silica Cement

B. Glass Ionomer 1) Glass Ionomer a. glass ionomer cements have the highest rate of fluoride release and recharge yet possess poor wear resistance b. low wear resistance c. NOT recommended as restorative materials in high stress bearing areas d. mostly indicated as liners under more strong restorative materials such as composites and composers e. can be used in Class III and Class IV cavities where low stresses are encountered 2) Fluoride release order (Most --> Least) Glass Ionomer > Resin-Modified Glass Ionomer > Compomer > Resin Composite

Which of the following materials possess the strongest ability to recharge its fluoride? A. Compomer B. Glass Ionomer C. Resin based glass ionomer D. Resin composite

B. Glass Ionomer 1) Glass Ionomers and Resin Modified Glass Ionomers a. demonstrates the ability to recharge its fluoride capacity b. conventional glass ionomers are regarded as having a better ability to recharge its fluoride capacity c. recharge capability may be due to the porosities present in glass ionomers and resin modified glass ionomers

Healthcare professionals who provide emergency treatment at the scene of an accidents are legally protected by which of the following laws? B. Good Samaritan Act D. Patriot Act E. States Human Rights Act

B. Good Samaritan Act 1) Good Samaritan Act a. protects healthcare professionals who provide emergency treatment at the scene of accidents b. intended to reduce bystanders hesitation to help for the fear of being sued or prosecuted for unintentional injury or wrongful/accidental death

Fabrication of an "emergency" occlusal separator device is indicated for patient with... A. Degenerative arthritis B. Hemarthrosis C. Myositis D. Unilateral condylar hyperplasia

B. Hemarthrosis 1) Concept: Further damage and bleeding of the joint is avoided through the use of an occlusal separator to relieve the injured TMJ from contact. 2) Hemarthrosis a. a condition wherein bleeding occurs in the joint or joint space. b. injuries to joints are the most usual cause of hemarthrosis which results in joint pain and inflammation

When utilizing four-handed dentistry, the assistant should be positioned... A. Different positions depending on where the dentist is working B. Higher than the dentist to facilitate visibility C. Lower than the dentist for ergonomic posturing D. At the same height because they are a team

B. Higher than the dentist to facilitate visibility 1) Four-handed dentistry a. assistant should sit close to the back of the patient's chair higher than the dentist to facilitate visibility b. position is suitable for better view of the site, instruments and materials from the mobile cart or instrument tray c. allows you access to these materials without leaning or overextending your arms d. assistant stools should be placed in a position so that their eye level is 4 to 6 inches higher than the dentist's

Which of the following procedures can stone casts NOT be used for regarding maxillofacial surgery? A. Fabrication of occlusal splints B. Identification of skeletal deformity C. Visual aid for patient education D. Practice of surgical procedure

B. Identification of skeletal deformity 1) Concept: Stone casts are accurate positive copies of a patient's oral structures that enable dentists to study and diagnose the oral conditions of the patient even when the patient is not present. 2) Stone casts a. study models for diagnostic purposes and is used to pan and practice the surgical procedure b. good visual aid to explain the oral condition and proposed procedure to the patient c. models for fabrication of occlusal splints d. limited to providing information of the intraoral structures, and can't provide information on skeletal deformities not the final curve of Wilson of a patient

Autogenous tooth transplants most commonly fail by their 3rd year in which of the following ways? A. Development of root caries B. Idiopathic chronic root resorption C. Inflammatory resorption of the periodontal supporting structure D. Ankylosis of the roots

B. Idiopathic chronic root resorption 1) Idiopathic chronic root resorption a. occurs in patients who underwent autogenous tooth transplantation that exhibit resorption with an unknown cause b. procedure if conducted properly have a relatively high success rate c. Causes 1) replacement resorption 2) trauma 3) resorption due to inflammatoin 4) marginal periodontitis 5) dental caries

A 9-year-old patient presents to your office with a unilateral posterior crossbite with a functional shift. What is the appropriate timing for correction of the crossbite? A. After the permanent 1st molars have fully erupted. B. Immediately, when the child is approximately 9 years of age C. Without waiting for the eruption of permanent 1st molars D. After all the primary teeth have been exfoliated.

B. Immediately, when the child is approximately 9 years of age 1) Concept: Unilateral poserior crossbones are best treated immediately after their diagnosis. 2) Treatment is most appropriate for children who are currently in the late deciduous stage or mixed dentition stage. 3) Correction is suggested at this time because arch expansion is most successful at this age and more space is created to correct the malalignment. 4) Early orthodontic intervention and treatment of unilateral posterior crossbite offers a better prognosis and a higher treatment success rate.

G.V. Black Class IV lesions typically occur on which of the following surfaces? B. In proximal surfaces of anteriors including incisal edges D. incisal edges of cusp tips

B. In proximal surfaces of anteriors including incisal edges 1) Class III: proximal surfaces of anterior teeth not including incisal edges 2) Class IV: proximal surfaces of anterior teeth including the incisal edges 3) Class V: smooth facial and lingual surfaces in the gingival 1/3 4) Class VI: occur on the incisal edges or cusp tips

Retention features for Class V restorations are typically placed on... A. Every wall but the pulpal B. Incisal/Occlusal and gingival walls C. Mesial and distal walls D. The pulpal wall

B. Incisal/Occlusal and gingival walls 1) Concept: Retentive undercuts are prepared within the Incisal/Occlusal and Gingival walls of cervical cavities (Class V preparations). 2) Placement of undercuts should be confined within the dentin layer to prevent the undermining of enamel. 3) Added restorative features like points and grooves within the dentin walls allow more retention for the restoration. 4) Retention for Class V restorations can be improved by lengthening the walls and making parallel opposing walls

Which of the following is the best treatment of generalized acute herpetic gingivostomatitis? A. Begin regimen of diluted vericelli zoster vaccine administrations B. Increase liquid intake, gentle debridement of the mouth, and good oral hygiene C. Prescribe 2g penicillin D. Take a culture of the bacteria to confirm or distinguish your diagnosis from acute necrotizing ulcerative gingivitis (ANUG)

B. Increase liquid intake, gentle debridement of the mouth, and good oral hygiene 1) Concept: The best treatment for generalized acute herpetic gingivostomatitis is: a. Increased liquid intake b. Gentle debridement of the mouth c. Sustaining good oral hygiene conditions 2) The herpes simplex virus causes generalized acute herpetic gingivostomatitis. 3) Generalized soreness of the oral cavity results in the interference of eating and drinking 4) The ruptured vesicles are focal sites of pain and are sensitive to touch. 5) The condition is self-limiting and subsides within 1o to 14 days. Pallative treatment includes a. topical viscous xylocaine b. benadryl for pain relief c. kaopectate to help coat the mouth d. balanced diet to compensate for insufficient nutrition e. Acyclovir, an antiviral may be administered in extreme cases to inhibit the virus. "Magic Mouthwash" contains an anesthetic, benadryl, kaopectate and may congaing the antibiotic or antiviral.

Which method is the best to manage loss of apical seat due to over-instrumentation? A. Filling with gutta-percha cone that is larger B. Increasing the file size and decreasing the file length C. Prescribing analgesics and antibiotics D. Using a chelating agent and packing with calcium hydroxide E. Filling with a warm gutta-percha technique

B. Increasing the file size and decreasing the file length 1) Apical seat a. use a shorter length of file to establish a correct working length b. correction of working length will eventually hep re-establish apical seat, then adequate flaring of the apical portion of the canal can be done with the use of larger sized hand files

Loss of the apical seat that is incurred through over-instrumentation is best clinically managed by: A. Filling with a gutta-percha cone that is larger than the apical perforation B. Increasing the file size and decreasing the length of the file C. Prescribing analgesics and antibiotics D. Using a chelating agent and placing Ca(OH)2 E. Filling with a warm gutta percha technique

B. Increasing the file size and decreasing the length of the file 1) A shorter length of file should be used to establish a correct working length. 2) The correction of working length will eventually help re-establish apical seat, then adequate flaring of the apical portion of the canal can be done with the use or larger size of hand files.

A periapical radiograph shows a deep carious lesion and the patient is in intermittent acute pain. Caries removal dose not extend to the pulp? What is the best treatment? A. Direct pulp cap B. Indirect pulp cap C. Zinc-oxide eugenol then placement of a permanent composite restoration D. Root canal therapy

B. Indirect pulp cap 1) Concept: Indirect pulp caps are indicated for teeth with deep caries with minimal pulpal inflammation 2) Steps of an indirect pulp cap: a. Infected dentin is completely removed b. Affected dentin close the pulp is left due to the possibility of pulp exposure c. Calcium hydroxide, then glass ionomer is placed over the affected dentin d. Final restoration is placed. 3) Calcium hydroxide base is placed on the affected dentin to induce its remineralization and promote the reparative dentin formation.

Which of the following orthognathic surgeries is LEAST stable? A. Genioplasty B. Inferior movement of the maxilla C. Mandibular advancement D. Maxillary advancement

B. Inferior movement of the maxilla 1) Concept: Inferior movement of the maxilla is the LEAST stable because the vertical occlusal force of the masticatory muscles often cause it to relapse. 2) Maxillary orthognathic surgeries like LaFort I, II, and III typically have a higher relapse rate than mandibular orthognathic surgeries because the bone of the maxilla is thinner and less sold than the mandible.

A patient presents to your office with pain associated with the right pre-auricular region of the face. The patient has a maximum opening of 45mm, and presents with a "joint click" associated with the right mandibular condyle. What is the most likely diagnosis? A. Coronoid hyperplasia B. Internal derangement with reduction C. Myofascial pain dysfunction syndrome D. Auriculotemporal syndrome

B. Internal derangement with reduction 1) Internal Derangement with reduction a. condition within the temporomandibular joint that involves displacement or slipping of the articular disc b. internal derangement with reduction denotes tha the articular disc is moving and sliding in and out of its normal limiting structure as the condyle head functions during the jaw opening and closing movements c. movement of the condyle against an articular disc that slides back and forth causes the clicking sound which characterizes a temporomandibular joint disorder. In severe conditions, patients may experience altered or limited mouth opening and locked jaw.

Which of the following locations does new plaque principally accumulate after a patient brushes their teeth? A. Buccal B. Interproximal C. Lingual D. Occlusal

B. Interproximal 1) Concept: Plaque usually accumulates along the areas hardly reached by toothbrush like the inter proximal area. 2) Plaque a. Newly formed plaque may form on all surface of teeth but typically stays longer within areas that are rarely disturbed by oral hygiene practices b. may form alone the buccal and lingual aspects of tooth, but with proper brushing these areas are easily cleaned c. occlusal area may demonstrate some plaque accumulation, but most of the time plaque is dislodged during mastication and brushing

In the permanent dentition, which of the following injuries is most likely to cause pulp necrosis? A. Extrusion B. Intrusion C. Luxation D. Subluxation E. Concussion

B. Intrusion 1) Concept: Intrusion of a tooth into its socket may sever the nerve and blood vessels that supply the tooth, resulting in pulp necrosis a. Intrusion of teeth due to trauma may also cause replacement resorption of the roots or ankylosis 2) Concussion, extrusion, luxation, and subluxation are accidents that are less likely to involve the vascular supply of the tooth.

Carcinoma in situ displays each of the following characteristics EXCEPT one. Which characteristic is the EXCEPTION? A. Abnormal mitosis B. Invasion C. Loss of cellular polarity D. Pleomorphism

B. Invasion 1) Concept: Invasion is NOT a feature shown by carcinoma in situ. The basement membrane is NOT breached in the case of carcinoma in situ. 2) Carcinoma in situ which is also known as an "in situ neoplasm" is an abnormal mass of cells 3) These abnormal cells grow in their initial location and do NOT invade the surrounding tissues. 4) Features include: a. Abnormal mitosis b. Loss of cellular polarity c. Pleomorphism

Which of the following is typically prescribed to treat anxiety? A. Haloperidol B. Lorazepam C. Methohexital D. Zaleplon E. Pentazocine

B. Lorazepam 1) Benzodiazepine effects: a. anxiolysis b. anterograde amnesia c. sedation/hypnosis d. anticonvulsion e. antiemesis f. muscle relaxation

The properties of zinc-oxide eugenol can be describe by the each of the following EXCEPT one. Which is the EXCEPTION? A. It can be easily removed from cavity preparations B. It can insulate the pulp better than dentin C. It has a soothing effect on dental pulp D. It provides an excellent marginal seal

B. It can insulate the pulp better than dentin 1) Zinc-oxide eugenol a. pulpal sedaton b. cavity base c.interim dental restoration d. can be easily adapted into the walls of cavity preparation and will stay intact when properly condensed into the cavity e. zinc-oxide eugenol cement as temporary restorations is specifically indicated when tooth has a very deep cavities eliciting pulp sensitivity and requires further observation prior to placement of final restoration

What is the primary reason for administration of prednisolone during an operative procedure? A. All answers are correct B. It decreases the likelihood of pulpal inflammation C. It reverses an already inflamed pulp D. It serves as a hemostatic agent

B. It decreases the likelihood of pulpal inflammation 1) Prednisolone a. decrease pain stimuli by reducing the production of prostaglandins responsible for the mediation of pain and inflammation b. anti-inflammatory steroid drug that functions as an effective inhibitor of phospholipase A2 that is responsible for promoting inflammation c. down-regulate some pro-inflammatory cytokines and reduce the pulpal inflammation during operative procedures

Which of the following does NOT describe nitrous oxide sedation? A. It alters cardiac output minimally B. It is explosive C. It is not significantly metabolized D. It is used frequently by anesthesiologists E. It is heavier than air

B. It is explosive 1) Nitrous oxide gas a. used to provide anesthesia, anxiolysis, and vasodilation (for IV placement) b. stable and combustible, but does not explode c. very safe and effective d. its effects in the body are immediately removed once the nitrous oxide gas is completely expired by the patient, making anesthetic recovery time faster.

Which of the following pulpal changes is MOST PROBABLE to occur if a permanent tooth with a mature apex is intruded due to trauma? A. Internal resorptin B. Necrosis C. Transient inflammation D. Calcific metamorphosis

B. Necrosis 1) Necrosis a. When a permanent tooth is intruded, the most probable result is pulpal necrosis because the blood vessels supplying the pulp are impeded b. intrusion of primary teeth does not typically result in necrotic pulp because the bloodflow is better because of the open apex c. 90% of intruded primary teeth will re-erupt after 2-6 months

A 33-year-old paint presents with swelling buccal to a mandibular 2nd premolar with only mild discomfort and no severe pain. The involved teeth are vital and possess no periodontal pockets. Radiographs reveal a round radiolucency in the middle 1/3 of the root. Which of the following describes the most probable diagnosis? A. Gingival abscess B. Lateral periodontal cyst C. Osteogenic sarcoma D. Periodontal abscess E. External root resorption

B. Lateral periodontal cyst 1) Lateral periodontal cyst a. arise from the epithelial rest of Malassez (a remnant of odontogenesis) that is more common in middle-aged adult males b. not associated with pain, and usually appears as a unilocular radiolucency on the side of canines or premolar roots c. most commonly seen in mandibular bicuspid area d. involved tooth is usually vital and presents no indication for root canal treatment e. typically treated by surgical enucleation

Which of the following surgical procedures is MOST COMMONLY used to correct maxillary retrognathia? A. Inverted L osteotomy B. LeFort I Osteotomy D. Bilateral sagittal-split ostetomy E. Lefort III Osteotomy

B. LeFort I Osteotomy 1) LeFort I Osteotomy a. most commonly used surgical treatment intervention for maxillary repositioning b. complete mobilization of the maxillary segments is carried out and repositioning is done c. bleeding control and vascular preservation are important to avoid macular necrosis during the procedure d. characterized by an abnormal growth of jaws in which the maxillary growth becomes retarded. Maxilla becomes short and mandible looks forward in relation to it.

Which of the following terms describes where 2 walls of a prepared cavity meet? A. Cavosurface angle B. Line angle C. Point angle D. Wall junction

B. Line angle 1) Concept: Line angle a. Line angle is the junction formed between the two walls of a cavity preparation 2) Point angle a. junction formed between 3 walls or 2 walls and 1 floor or 2 floors and 1 wall of a cavity preparation is known as a point angle. 3) Wall Junction a. point where 2 walls or the floor and walls meet.

Which of the following is the most likely cause of an open gingival margin of a proximal restoration? A. Clinician forgot to burnish the matrix band against the adjacent tooth B. Loading too much amalgam into the prep before properly condensing it into the proximal box C. Over-tightening of the matrix band D. Clinician forgot to place a wedge

B. Loading too much amalgam into the prep before properly condensing it into the proximal box 1) Concept: Inadequate condensation of amalgam into a proximal box before it sets will result in an open gingival margin. 2) Placement of excessive amalgam into the cavity prep often allows the amalgam to set before it is adapted and properly condensed into the floor and walls of the cavity resulting in voids in the restoration and an open gingival margin a. Small condensers should be used initially for proximal boxes b. Larger condensers should be used after the adaptation to the margins and cavity floor are deemed adequate. 3) Incremental filling ensures that amalgam is condensed and adapted to the cavity completely and also reduces incidence of voids and open margins in the restoration 4) Well condensed amalgam restorations demonstrate increased compressive strength, dimensional stability, and corrosion resistance because excess mercury moves to the surface and is removed from the amalgam restoration with proper condensation.

Each of the following is an advantage of a resin composite for posterior restorations EXCEPT one. Which is the EXCEPTION? A. Conserves tooth structure B. Low fracture toughness C. Low thermal conductivity D. Resin composite is radiopaque

B. Low fracture toughness 1) Low Fracture toughness a. is NOT an advantage of using composite resins in posterior tooth restorations b. posterior restorations are prone to undergo heavy occlusal forces and para-functional habits 2) Occlusal evaluation a. amalgam or gold restorations are recommended in such conditions 3) Advantages a. low thermal conductivity b. radiopacity of composite resins

What location are mucous retention swellings and mucoceles most commonly found? A. Gingiva B. Lower lip C. Tongue D. Upper lip E. Buccal mucosa

B. Lower lip 1) Concept: Mucoceles (mucus extravasation phenomenon/retention cysts) are a swelling consisting of collected mucous due to a ruptured salivary gland duct usually caused by local trauma b. Mucoceles present a bluish translucent color, and are more commonly found in children and young adults c. The most common mucocele locations are lower lip, the inner side of the cheek, on the anterior ventral tongue, and the floor of the mouth. d. A mucocele on the floor of the mouth is referred to as a ranula because it looks like a frog belly.

Each of the following is a characteristics of conventional glass ionomer EXCEPT one. Which is the EXCEPTION? A. Low viscosity B. Lowest fluoride release C. Sealants on partially erupted molars D. Used as liner, base, or restorative materials

B. Lowest fluoride release 1) Glass Ionomer a. highest degree of fluoride release b. used as liners, bases, and restorative materials c. used as sealant on partially erupted teeth and as caries control restorations in deep cavities

A 32-year old patient presents to your office with a tender swelling in the submandibular triangle. The most likely etiology is... A. Infected thyroglossal duct cyst B. Lymphadenopathy C. Sialolith obstructing salivary flow D. A mucocele

B. Lymphadenopathy 1) Concept: Tender swelling localized within the submandibular triangle is indicative of lymphadenopathy. 2) Lymphadenopathy a. condition where the lymph nodes become swollen, tender and enlarged. b. could be an indication of infection, malignancy, or an auto-immune disease. c. enlarged lymph nodes within the submandibular triangle are palpable during bi-digital jaw palpation

Which of the following describes the MOST common form of leukemia observed in pediatric patients? A. Granulocytic B. Lymphoblastic C. Monocytic D. Myelomonocytic E. Eosinophilic

B. Lymphoblastic 1) Concept: Acute lymphoblastic leukemia (ALL) is also called acute lymphocytic leukemia accounts for 3/4 of the cases of childhood leukemia 2) Almost all cases of childhood leukemia are acute, which means they develop rapidly. A very low number are chronic and develop slowly. 3) Types of childhood leukemia include: a. Acute lymphoblastic leukemia (Acute lyphocytic leukemia) is the most common type of childhood leukemia b. Acute myelogenous leukemia (AML) is the 2nd most common type of childhood leukemia c. Chronic myelogenous leukemia (CML) is rare in children d. Chronic lymphocytic leukemia (CLL) is very rare in children e. Juvenile myelomonocytic leukemia (JMML) is a rare type that is neither chronic nor acute and occurs most often in children under age 4. f. Hybrid or mixed lineage leukemia is a rare leukemia with features of both ALL and AML

Which of the following permanent teeth cause concern about the eruption sequence in serial extraction cases? A. Mandibular 1st and 2nd premolars B. Mandibular canines and 1st premolars C. Maxillary canines and 1st premolars D. Mandibular 1st molars and incisors

B. Mandibular canines and 1st premolars 1) Concept: The canines often erupt before the mandibular premolars and crowding of anterior teeth may occur if the normal eruption sequence is disrupted. 2) Serial extraction procedures are performed to prevent the crowding of teeth. 3) In serial extraction cases, primary canines are extracted when a child reaches age 8 of 9 to facilitate incisor alignment. The primer 1st molars are extracted after one year to speed up eruption of 1st premolars. The erupted 1st premolars are then extracted to facilitate canine eruption and alignment in the arch.

Occult lesions are pathologies that.... B. Manifest no signs or symptoms D. Cause pain

B. Manifest no signs or symptoms

Which of the following tooth varieties are most known for susceptibility for fracture during extraction procedures? A. Mandibular 2nd premolars B. Maxillary 1st premolars C. Maxillary 2nd premolars D. Mandibular 1st premolars

B. Maxillary 1st premolars 1) Concept: Maxillary 1st premolars are known to fracture during extractions due to the presence of a very thin root. b. Maxillary 1st premolars have variable root forms and normally have two root canals and 2 roots. c. Due to the thinness of the tooth root of maxillary premolars, the roots are easily fracture during extraction.

Which of the following variables accounts for the primary retention of fissure sealants A. Chemical bonding of the sealant to enamel B. Mechanical microretention C. Surface tension D. Adhesion of the sealant to enamel

B. Mechanical microretention 1) Micro-mechanical retention a. primary means of retention for pit and fissure sealants b. shape and depth of pits and fissure provide mechanical retention for sealants c. deep pits and fissures are more retentive than shallow pits and fissures

Which muscle is responsible of the displacement of the proximal portion of the mandible following an unfavorable angle fracture. A. Hyoglossus B. Medial pterygoid C. Mylohyoid D. Lateral pterygoid

B. Medial pterygoid 1) Concept: The medial pterygoid muscle inserts into the lower posterior portion of the mandibular ramus and the angle of the mandible and serves to help close the jaw. 2) Medial pterygoid a. pulls on the angle of the mandible and posterior ramus in order to help facilitate jaw closure b. during an unfavorable mandibular fracture, the medial pterygoid that inserts at the mandibular angle pulls in the same direction as the vector of the displacement

A 42-year old asthmatic patient who smokes and is HIV+ presents to your office for a routine evaluation. Upon examination, you notice an asymptomatic red patch on the midline dorsum of their tongue and a marked loss of lingual papillae. Which of the following is the most likely diagnosis? A. Geographic tongue B. Median Rhomboid glossitis C. Schwannoma D. Ulcerative lichen planus E. Carcinoma

B. Median Rhomboid glossitis 1) Concept: Median Rhomboid Glossitis a. is a condition characterized by a midline erythematous patch on the dorsum of the tongue immediately in front of the circumvallate papillae combined with a loss of lingual papillae b. Median rhomboid glossitis is though to be created by a chronic fungal infection, and usually is a type of oral candidiasis. c. Risk factors include: 1) Smoking 2) Denture wearing 3) Corticosteroid sprays or inhalers 4) Human immunodeficiency virus (HIV) infection

The most significant advantages of acid-etching for composite placement is represented by which of the following? A. Curing shrinkage B. Microleakage C. Pulpal irritation D. Coefficient of thermal expansion

B. Microleakage 1) Concept: Microleakage can be reduced by acid etching the tooth when placing composite restorations. 2) Conventional acid etching techniques demonstrate less micro leakage than the newer self-etched primers. 3) Liquid or gel etchant is placed for about 15-30 seconds. 4) Acid etching results in the formation of microscopic retentive areas about 25 microns in depth.

Tetracycline staining occurs in which stage of tooth development? A. Histodifferentiation B. Mineralization C. Morphodifferentiation D. Proliferation E. Apposition

B. Mineralization 1) Tetracycline a. incorporated into the tooth structure during the mineralization of the tooth because it forms a chelate with calcium b. tetracycline staining occurs after the ingestion of tetracycline antibiotics during tooth development and is diagnosed by degrees of horizontal bands of color ranging from gray to gray-brown and even some shades of green. c. tetracycline staining is found in both the enamel and dentin d. when first incorporated into the tooth structure, tetracycline produces a yellow color that changes to brown and gray with the addition of sunlight over time.

All or part of which of the following muscles is detached when a clinician lowers the floor of the mouth of a patient? A. Genioglossus and hyoglossus B. Mylohyoid and genioglossus C. Mylohyoid and geniohyoid D. Mylohyoid and hyoglossus E. Genioglossus and geniohyoid

B. Mylohyoid and genioglossus 1) Lowering the floor of the mouth a. mylohyoid and genioglossus muscle attachments are detached 2) Mylohyoid muscle a. paired muscle running from the mandible to the hyoid bone, forming the oral cavity 3) Genioglossus a. muscle which runs from the chin to the tongue

Which of the following lesions is the LEAST likely to be visible radiographically? A. Lateral periodontal cyst B. Nasolabial cyst C. Nasopalatine duct cyst D. Traumatic bone cyst

B. Nasolabial cyst 1) Concept: Nasolabial cysts are NOT always visible on radiographs because of their soft tissue origin and extra osseous occurrence 2) Nasolabial cysts a. rare, non-odontogenic, soft-tissue, developmental cysts that occur inferiorly to the nasal alar region b. they are also known as nasoalveolar cysts of Klestadt's cysts

A 35-year-old female patient complains of a painful, rapidly developing ulcer on the left side of her hard palate. This lesion appears crater-like and measures 3.5cm. Which of the following is the most likely diagnosis for her current condition? A. Adenoic cystic carcinoma B. Necrotizing sialometaplasia C. Pleomorphic adenoma

B. Necrotizing sialometaplasia 1) Necrotizing sialometaplasia is a rapidly expanding ulcerative lesion that mostly occurs on the posterior area of the hard palate. 2) Necrotizing sialometaplasia is usually benign and is often painless, and is usually self-limiting and resolves in about 6-10 weeks. 3) Trauma to the minor salivary glands located in the hard palate causes this condition.

Administration of a schedule 2 narcotic with an anti-psychotic drug creates... A. Conscious sedation B. Neuroleptic analgesia C. Psychotomimetic analgesia D. Dissociative anesthesia

B. Neuroleptic analgesia 1) Neuroleptic analgesia a. combined effect of using narcotics with antipsychotic agents b. patients under neuroleptic analgesia show some degree of quiescence (quietness), alteration of awareness, and some analgesia

Caries can be diagnosed accurately without radiographs when... A. All answers are correct B. No answers are correct C. When all posterior teeth have proximal restoration D. When the patient is pregnant E. Only primary teeth are present

B. No answers are correct Caries can be diagnosed accurately without radiographs a. visual/tactile sense b. transillumination c. caries detector dyes d. laser fluorescent light

A 33-year-old woman of African American descent is asymptomatic, but presents to your office for a routine check-up. No evidence of decay is noted, but examination of her radiographs reveal periodical radiolucencies associated with the apices of her mandibular central incisors. Pulp testing reveals all teeth respond similarly. What treatment is indicated? A. Initiate root canal treatment on the two mandibular central incisors B. No treatment is necessary at this time C. Perform test cavities on the mandibular central incisors D. Refer for evaluation to rule out possible malignancy E. Identify the dark, potentially necrotic pulp chambers by transillumination

B. No treatment is necessary at this time 1) Periapical cemental dysplasia a. periapical radiolucency found along the apices of mandibular central incisors and is predominantly discovered in African-American females b. teeth show no signs of decay and are symptomatic, indicating that the periodical radiolucency seen under the apices of the mandibular central incisor is not cause by bacterial infection involving the pulp c. no treatment should be performed since all teeth are vital and non carious based on the oral examination

When removing a mandibular lingual torus from the premolar region of a patient, the flap should include which of the following? A. Incised at the junction of the attached gingival tissue and free mucosa B. No vertical components C. Possess only a posterior component D. Anterior and Posterior vertical components

B. No vertical components 1) Vertical incision a. is NOT necessary because the small amount of attached gingiva on the lingual aspect as well as the proximity of the tori to the floor of the mouth b. placed along the lingual aspect are difficult to suture due to the thickness of mucosa and its proximity to the tongue and accessibility of the incision c. contraindicated within areas that have inferior concavities or bony prominences d. full thickness envelope flaps are sufficient to provide adequate access for the removal of the mandibular tori

An 11-year-old girl presents to your office 4 hours after a fall that fractured her maxillary central incisor to the point that the remaining tooth structure is supra gingival. The exposed pulp is diagnosed as vital. Which of the following is considered the treatment of choice? A. Apexogenesis procedure then root canal treatment B. Non-surgical root canal treatment C. Pulpectromy and placement of calcium hydroxide D. Pulpotomy E. Pulp cap

B. Non-surgical root canal treatment 1) Proper restoration of the fracture maxillary central incisor can only be done if the tooth undergoes root canal treatment 2) Tooth can be restored after root canal treatment and exposure of adequate tooth length.

A patient's radiographs display bulbous crowns, obliterated pulps, and shortened roots. Which of the following conditions is described by these findings? A. Gardner's syndrome B. Osteogenesis imperfecta C. Pierre Robin syndrome D. Scleroderma E. Amelogenesis imperfecta

B. Osteogenesis imperfecta 1) Concept: Osteogenesis imperfecta a. is suggested radiographically by a thin bone cortex and bulbous crowns, obliterated pulps, and shortened tooth roots b. most common congenital bone disorder where patients are born with defective connective tissue, or without the ability to make it usually because of a deficiency of Type-I collagen. c. teeth are described as shell-like because enamel may fracture off and increased wear will occur 2) Types 1) Type I a. collagen is of normal quality but is produced in insufficient quantities b. blue sclera c. bones fracture easily 2) Type II a. collagen is not of a sufficient quality of quantity b. most cases die within the first year of life due to respiratory failure or intracerebral hemorrhage c. severe respiratory problems due to underdeveloped lungs d. severe bone deformity and small stature

Which of the following describes the 1st step in cavity preparation according to G.V. Black? A. Convenience form B. Outline form C. Resistance form D. Retention form

B. Outline form 1) Outline form a. initial extension or the outline form of the tooth preparation should be visualized preoperatively by estimating the extent of the defect, the preparation form requirements of the amalgam, and the need for the adequate access to place amalgam into the tooth 2) G.V. Black's various steps of cavity preparation 1) Establish outline form 2) Obtain retention form 3) Obtain resistance form 4) Obtain convenience form

Which of the following can a clinician modify with nitrous oxide administration? A. Pain stimulus B. Pain threshold C. Referred pain D. Pain reaction

B. Pain threshold 1) Pain threshold a. modified by nitrous oxide because it sedates patients, thereby relieving them of the pain from the operation b. reduces patient anxiety by inducing a temporary state of euphoria which makes them less aware of the sensations of dental treatment 2) Pain reaction and referred pain a. response of the body to certain pain stimuli and are not directly modified by nitrous oxide

Which of the following would suggest that a lesions is MORE likely to be periodontal THAN endodontic? A. Acute pain to percussion with no swelling present B. Pain to lateral percussion and a wide sulcus C. Pain to palpation near the apex of that tooth on the buccal gingiva D. Probe extending to the apex of the tooth

B. Pain to lateral percussion and a wide sulcus 1) Wide sulcus a. indicates attachment loss 2) Pain on lateral percussion a. pressure on the inflamed periodontal ligament, not an apical infection

Which of the following reactive lesions of gingival tissue histologically reveals bone formation? A. Peripheral giant cell granuloma B. Peripheral ossifying fibroma C. Pyogenic granuloma D. Traumatic neuroma E. Irritation fibroma

B. Peripheral ossifying fibroma 1) Concept: Peripheral ossifying fibromas a. Peripheral ossifying fibromas appear microscopically as a combination of a mineralized product and fibrous proliferation. b. The mineralized portion may be bone, cementum-like, or dystrophic calcifications. c. an overgrowth of gingival tissue caused by mucosal irritation or trauma and are similar in appearance to pyogenic granulomas and peripheral giant cell granulomas d. Peripheral ossifying fibromas range from red to pink, and are frequently ulcerated. They may appear as sessile or pedunculated with a size of cm or less. e. They occur 66% in females with a prevalence highest around 10-19 years of age.

Which of the following methods should be employed if the tip of the curing light is narrower than the composite restoration being cured? A. Move the curing tip over the surface for the recommended time B. Place the tip stepwise over each area and expose each area for the recommended time C. Position the tip far away enough from the tooth to illuminate the entire restoration D. Center the tip on the surface and cure the entire composite from that spot.

B. Place the tip stepwise over each area and expose each area for the recommended time 1) Concept: Proper exposure time for every portion of a composite resin will ensure that the entire restoration is completely polymerized. 2) Placing the small tip from one area to another while following the recommended curing time will ensure that no unpolymerized composite will be left behind. 3) Adequate curing of composite restoration is important to ensure the restoration has no weak areas, is properly sealed and can perform at its best during mastication.

Which of the following are considered to be organic components of oral bacterial plaque? B. Polysaccharides and protein C. Potassium and phosphorus D. Calcium and sodium

B. Polysaccharides and protein 1) Plaque is made up of organic components and non-organic components (minerals) 2) Organic components include: a. polysaccharides b. glycoproteins c. proteins d. lipids

Which of the following bacteria has been identified as periodontal pathogens? A. Lactobacillus casei B. Porphyromonas gingivalis C. Streptococcus mutans D. Streptococcus gordinii

B. Porphyromonas gingivalis 1) Porphyromonas gingivalis a. etiologic pathogen of periodontitis

A child presents with a noticeably sore ulceration of their lower lip and no history of obvious trauma. The ulceration appeared several hours after the patient received dental treatment. Which of the following is the most probable diagnosis? A. Herpes labialis B. Post-anesthetic lip bite C. Type IV allergic reaction to the latex in the rubber dam D. Aphthous ulcer secondary to stress

B. Post-anesthetic lip bite 1) Due to the feeling of numbness after anesthetic delivery, a child may either intentional or unintentionally bite on his/her lips. 2) Children under anesthesia should be supervised by their parents and instructed not to eat on the anesthetized side of the jaw to prevent accidental lip biting while anesthesia is still present.

Following receiving dental treatment, a very sore ulceration is noticed on a child's lower lip. The child has no history of labial trauma before entering the dental office. Which of the following most likely explains this ulceration? A. Herpes labialis B. Post-anesthetic lip bite C. Type IV allergic reaction to the latex in the rubber dam D. Aphthous ulcer secondary to stress

B. Post-anesthetic lip bite 1) Post-anesthetic lip bite a. due to a feeling of numbness after anesthetic delivery, a child may either intentional or unintentionally bite on his/her lips b. should be supervised by parents and instructed not to eat on the anesthetized side of the jaw to prevent accidental lip biting while anesthesia is still present

Ideal bone grating material should perform each of the following roles EXCEPT one. Which is the EXCEPTION? A. Induce osteogenesis B. Produce an immunologic response C. Withstand mechanical forces D. Be replaced by host bone

B. Produce an immunologic response 1) Bone grafting materials a. biocompatible b. should NOT produce an immunologic response c. strong enough to withstand mechanical actions d. osteoproliferative e. induce osteogenesis

A 6-year0old patient presents to your office with acute primary herpetic gingivostomatitis, which of the following would not help your patient? A. Encourage the child to drink plenty of fluids B. Prescribe penicillin C. Prescribe a topcial anesthetic rince for use before meals D. Suggest bed rest and minimal contact with others E. Encourage a bland diet

B. Prescribe penicillin 1) Gingivostomatitis a. inflammation of the oral mucosa and gingiva b. cause by HSV-1 that affects children and leads to painful oral ulcerations c. often the initial presentation during the primary herpes simplex infection d. numerous pin-head vesicles, which rupture rapidly to form painful irregular ulcerations covered by yellow-grey membranes e. findings 1) sub-mandibular lymphadenitis 2) halitosis 3) refusal to drink f. most common oral infection of the mouth g. Prodromal symptoms 1) fever 2) anorexia 3) irritability 4) malaise 5) headache

For a given population, the number of disease cases are divided by the total number of people. Which of the following rates were calculated. B. Prevalence D. Incidence

B. Prevalence

Each of the following is an indication for the placement of placing a base EXCEPT one. Which is the EXCEPTION? A. All answers apply B. Prevent or decrease marginal leakage C. Prevent other dental materials from irritating the pulp D. Replace missing tooth structure E. Thermal protection for the pulp

B. Prevent or decrease marginal leakage 1) Bases a. replace missing tooth structure and support the final restoration b. insulate the pulp from drastic temperature changes experiences by the restoration c. provide structure to condense against when placing amalgam d. protect the pulp from agents which may irritate it

When preparing for a tooth for a Class I amalgam, the medial and distal walls are supposed to diverge occlusally to... A. Extend the preparation for ease of cleans ability B. Prevent the undermining of the tooth structure of the marginal ridge C. Resist forces of occlusal loading D. Create a convenience form

B. Prevent the undermining of the tooth structure of the marginal ridge 1) Concept: Marginal ridges should be supported by dentin to avoid fracture under heavy occlusal forces and should NOT be undermined 2) Undermined marginal ridges are easily fracture because the enamel in these areas is unsupported 3) Dentin supported marginal ridges provide the best fracture resistance against heavy occlusal forces.

Which of the following is NOT a reason why fluoride prevents tooth decay? A. Interference of bacterial metabolic activity B. Prevention of bacterial aggregation

B. Prevention of bacterial aggregation 1) Fluoride replaces the hydroxyl groups of the hydroxyapatite crystals of enamel. It then forms fluoroapatite crystals that are significantly more resistant than acid.

Which of the following would most likely be the immediate concern of the clinician following the extraction of tooth T of a 3-year-old patient? A. Prevention of localized alveolar osteitis B. Prevention of post-operative lip biting C. Prevention of postoperative swelling D. Space maintenance E. Alleviating postoperative pain

B. Prevention of post-operative lip biting 1) Concept: The prolonged numbness of the mandible after a lower molar extraction in children may cause accidental cheek and lip biting. 2) Children experiencing numbness of the mandible after tooth extraction also have a tendency to experiment and play around their numb lips and cheeks. 3) In order to avoid lip and cheek biting, it is best to instruct the parents to guide their child and teach them to refrain from chewing foods while the lower arch is still numb.

Which of the following types of excursive movements would be interfered with if a mesial marginal ridge of a maxillary left 2nd molar was built up too high during a restoration placement? A. Non-working B. Protrusive C. Retrusive D. Working

B. Protrusive 1) Concept: An overbuilt mesial marginal ridge of a maxillary 2nd molar tooth will interfere with protrusive movement. 2) High spot will hinder the forward movement as it will come in contact with the opposing tooth earlier. 3) It is ESSENTIAL to remove all the high spots after restoring the tooth so that occlusal trauma is not incurred by the tooth and its antagonist. 4) Articulating paper can be used to identify the high spots in order to carry out their removal. 5) High spots in a restoration will affect the normal functioning as well as cause pain in the long run.

The root canal anatomy of a maxillary 1st molar with 4 canals requires which shape of access preparation? A. Ovoid B. Quadrilateral C. Square D. Triangular E. Round

B. Quadrilateral 1) Maxillary permanent 1st molar with 4 canal orifices a. quadrilateral-shaped access preparation b. provides the most ideal preparation since the pulp chamber of this tooth is also of the same shape c. mesiobuccal and distobuccal orifice of this tooth are most commonly locoed near each other

Pulpotomies are CONTRAINDICATED in primary molars with which of the following? A. Radiographic evidence of deep caries approximating the pulp chamber B. Radiographic evidence of internal resorption C. Sensitivity to sweet D. Exposure of the pulp during caries excavation

B. Radiographic evidence of internal resorption 1) Internal resorption Root canal therapy is indicated to treat cases involving internal resorption. 2) Treatment Options a. When a carious lesion approximates or extends into the pulp chamber of a deciduous tooth, removal of coronal pulp is performed to prevent the spread of the infection into the radicular pulp 3) Pulpotomies a. performed on deciduous teeth to prevent premature tooth loss and potential loss of space b. Contraindications 1) internal root resorption (root canal therapy indicated) 2) perforation of pulp chamber floor 3) over 1/2 root resorption 4) cellulitis 5) localized abscess 6) draining sinus 7) inability to isolate the tooth 8) inability to properly restore tooth after the procedure

When removing 4_ adjacent teeth, a dentist removes the remnants of the intraradicular bone and compress the cortical plates. This procedure is an example of a method for treating... B. Severe facial undercut C. Severe lingual exostosis

B. Severe facial undercut 1) Cortical plates can be compressed after the interradicular bone is removed to remove ridge undercuts 2) Interradicular bone provides the thickness of separation between the labial and palatal cortical bone

A 24-year old female patient reports to your office with small red and white oral lesions that are located on alveolar non-keratinized mucosa. The patient reports that the ulcerations are intermittent and painful when present, but usually resolve in 7-10 days. What is the most likely explanation for her symptoms? A. Primary herpes B. Recurrent aphthous stomatitis C. Recurrent herpes D. Squamous cell carcinoma E. Acute necrotizing ulcerative gingivitis

B. Recurrent aphthous stomatitis 1) Concept: Aphthous stomatitis (Canker sores) is a common cause of benign and noncontagious mouth ulcers reported in roughly 20% of the population. 2) Recurrent pathos stomatitis a. characterized by repeated formation of ulcers on the mucous membranes of the oral cavity that usually take 1-2 weeks to typically heal. 3) There is no cure, and treatments are aimed at reducing pain and speeding the healing process. 4) The formation of apthae involves a T cell mediated immune response triggered by a variety of a factors, including nutritional deficiencies, local trauma, stress, hormonal influences, allergies, and a genetic predisposition. 5) Aphthous stomatitis occurs on non-keratinized mucosa 6) Recurrent herpes occurs on keratinized mucosa

When prepping a tooth for a cast gold restoration, the tooth structure that is weakened by the preparation and subjected to masticatory forces must be A. Etched B. Reduced and protected by the restoration C. Reinforced by a core buildup and cement D Safeguarded by a full coverage restoration

B. Reduced and protected by the restoration 1) Concept: Weak tooth structure must be removed or reduced to provide a strong foundation for the restoration 2) Undermined cusps and unsupported tooth structure are prone to fracture, so capping of cusps may be necessary to protect the tooth and provide strength. 3) The tooth preparation must conserve sound tooth structure and have adequate retention and resistance form.

During an automobile accident, a patient experiences a blow to the face and mouth. Radiographs suggest a horizontal mid-root fracture of maxillary central incisor. The tooth presents with NO mobility and NO symptoms. The tooth does not response to pulp testing. No radiographic lesion is present. The best course of action is... A. Initiate root canal treatment to include both segments of the tooth B. Render no treatment at this time and periodically check clinically and radiographically C. Surgically remove the apical segment and reverse fill the coronal segment D. Extract the coronal segment and surgically remove both segments

B. Render no treatment at this time and periodically check clinically and radiographically 1) Tooth with no pulpal symptoms and mobility a. must be reevaluated after 8 weeks to ensure that the tooth is vital and requires no other treatment b. if the tooth tests non-vital due to the horizontal midroot fracture, the prognosis of restoring tooth decreases significantly c. if the tooth remains vital, calcific healing might have occurred at the site of feature. It is a type of healing wherein calcific callus develops along the fracture site.

Which of the following is the correct sequence for the INITIAL TREATMENT in which mature avulsed teeth are replanted? A. Performing root canal debridement, calcium hydroxide therapy, repositioning, splinting, and filling with gutta-percha B. Repositioning and splinting only C. Respositioning, splinting, and performing root canal therapy D. Performing root-canal therapy, repositioning, and splinting

B. Repositioning and splinting only 1) During an initial treatment, the avulsed tooth is gently repositioned into its socket and then splinted to the adjacent teeth to ensure that the arch form is preserved 2) Maintaining the tooth's proper position within the socket is important for the success of the replantation procedure. Tooth should be splinted until it is stable enough until the necessary treatments can be done.

Space for the mandibular 2nd and 3rd molar eruption is created through the bone... A. Apposition of the anterior border of the ramus B. Resorption at the anterior border of the ramus C. Resorption of the posterior border of the ramus D. Apposition of the alveolar process

B. Resorption at the anterior border of the ramus 1) Concept: The anterior border of the ramus undergoes resorption while bone is simultaneously deposited of at the posterior border of the ramus. 2) Resorption of the ramus along its anterior border creates sufficient space for the eruption of the 2nd and 3rd permanent molars. 3) The continuous resorption and deposition of bone along the borders of the ramus during the growth stages helps maintain the ramus width while providing extra alveolar bone space for the erupting molars.

Adjacent proximal cavities can be restored with composite resin at the same appointment because...Select all that apply. A. All answers are correct B. Restoration and finishing can be completed in the same appointment C. The possibility of having proximal contacts is increased. D. They can be restored with one carpule of composite, so materials can be conserved E. It conserves operating time.

B. Restoration and finishing can be completed in the same appointment 1) Concept: Adjacent proximal cavities are best restored in one appointment because 2) The cavity preparations can be made more conservatively, preserving more healthy tooth structure. 3) There will be adequate access and visibility, enabling the dentist to work more efficiently and effectively. 4) The restorations can be placed easier due to adequate space to finish the proximal margins of the restorations properly. 5) Placing the restorations at the same time will decrease the wasted materials like adhesives etc. that must be dispensed. 6) Placing the restorations at the same time DOES NOT increase the chance of having proximal contacts.

A patient of record calls your office complaining of discomfort after a routine class V restoration was placed yesterday on tooth #31. Which of the following is the likely etiology of the patient' discomfort? A. No liner or base placed under the restoration B. Root dentin was exposed during the finishing of the restoration C. The axial depth of the preparation was too aggressive compared to the actual depth of caries D. The tooth was over-etched with phosphoric acid

B. Root dentin was exposed during the finishing of the restoration 1) Tooth may be sensitive after the restoration of #31 is probably caused by inadequate or inaccurate filling of the cavity preparation resulting to root dentin exposure. 2) Exposed root dentin is more sensitive since the cavity is open and prone to acid attacks

24 hours after the placement of a routine Class V composite, the patient reports discomfort with the tooth. Which of the following is most likely etiology of the complaint? A. No liner or base placed under the restoration B. Root dentin was exposed during the finishing of the restoration

B. Root dentin was exposed during the finishing of the restoration 1) The tooth maybe sensitive after the restoration of #31 caused by inadequate or inaccurate filling of the cavity preparation resulting to root dentin exposure. 2) The exposed root dentin is more sensitive since the cavity is open and prone to acid attacks.

Sudden relief of severe pain associated with a dentoalveolar abscess can be explained by which of the following? A. Neutralizing effect of tissue enzymes B. Rupture of the abscess from the periosteum into the soft tissue C. Walling-off the infection by the body D. Neurotoxic effects of bacterial toxins anesthetize the nocioceptors

B. Rupture of the abscess from the periosteum into the soft tissue 1) Dentoalveolar abscesses a. accumulation of pus within the teeth, gums, and supporting alveolar bone b. accumulation of pus within the dentoalveolar abscess creates hydrostatic pressure, resulting in the sensation of severe pain c. treated through root canal treatment and possibly incision and drainage depending upon case severity d. drainage of the lesion releases the pressure caused by the suppuration and results in sudden relief of pain that is experiences by the patient. e. severe, intense, and throbbing pain which worsens with the passage of time f. can be partially diagnosed by noting the presence of periodical radiolucency

A pantomograph of an asymptomatic adult shows a well-defined 35mm radiolucency inferior to the mandibular canal. The lesion is not palpable. Which is the most likely diagnosis? A. Ameloblastoma B. Salivary gland inclusion defect (Stafne defect) C. Traumatic bone cyst D. Focal osteoporotic bone marrow defect E. Odontogenic keratocyst

B. Salivary gland inclusion defect (Stafne defect) 1) Stafne defect a. depression along the lingual surface of the mandible that is near the tongue and submandibular salivary gland b. well demarcated radiolucency that is approximately about 10-35mm in diameter c. made by ectopically formed salivary gland tissue which is near the submandibular gland d. not pathologic and is considered a pseudocyst since it doesn't have fluid and epithelial lining

Why would it be a good clinical decision to replace a silicate cement restoration on the distal of tooth #11? A. Silicate cement restorations on proximal surfaces cause gingival irritation B. Silicate cement restorations will not maintain the mesio-distal width of #6 C. Silicate cement restorations will result in open contacts D. Silicate cements stain easily are considered anesthetic

B. Silicate cement restorations will not maintain the mesio-distal width of #6 1) Silicate cements a. not good for restoring teeth with proximal contacts b. glass particles in silicate cements are easily dislodged from the restorative filling c. is brittle and prone to surface crazing and chipping on the margins d. unsuitable for restoring: 1) proximal contacts 2) stress bearing areas

Which proper is most important for defining a tooth's character and individuality? A. Color B. Texture of enamel C. Translucency D. Shape

B. Texture of enamel 1) Concept: Enamel texture is the most important factor in determining the individual character of teeth. 2) As enamel ages, it becomes: a. thinner b. less translucent c. worn 3) Changes in enamel are due to ear, abrasion, erosion, and attrition 4) Younger teeth exhibit thicker, more translucent, and rougher enamel 5) Adult teeth appear smoother, shorter, and darker

When prescribing fluoride supplements, the most important factor(s) for the dentist to consider is/are... A. Amount of fluoride consumed already in drinking water B. The age of the patient and concentration of fluoride in the water.

B. The age of the patient and concentration of fluoride in the water. 1) Essential to know the fluoride content of the tap water to recognize if supplementary fluoride sources should be used.

Which of the following is the most probable explanation for a freshly condensed amalgam restoration that begins to chip away when it is carved? A. The alloy was improperly mixed and was not fully wetted with mercury B. The amalgam was condensed after its working time had elapsed C. The amalgam was contaminated by moisture D. A low-copper alloy was used

B. The amalgam was condensed after its working time had elapsed 1) Concept: Over condensation of amalgam alloy will result in chipping away of the filling material during carving. 2) Prolonged condensation pressure will result in a decrease in strength of the amalgam restoration. 3) Amalgam alloys typically take roughly 24 hours to set completely, so undue pressure or disturbance during this time will result in the breakage of restoration.

Which of the following determines the correct total flow of nitrous oxide and oxygen? A. A standard of 6 liters/minute flow rate B. The amount necessary to keep the reservoir bag 1/3-2/3 full C. The largest volume that the patient's lungs can exchange in 1 minute D. The metabolic oxygen requirements of the patient

B. The amount necessary to keep the reservoir bag 1/3-2/3 full 1) It is recommended to fill the reservoir bag up to 1/3-2/3 of the total volume. This amount produces the necessary vacuum required for the optimum flow of nitrous oxide and oxygen. 2) Keep some portion of the reservoir bag empty and free from air to produce the current flow of nitrous oxide and oxygen.

The inferior concha is composed of all or part of which of the following bones? A. Maxilla B. Palatine C. Sphenoid D. Turbinate E. Ethmoid

D. Turbinate 1) Turbinate a. narros and sea-shell appearing bone that extends ion the nasal air passage b. have 4 grooved air channels which divides and forces the inspired air to flow steadily into the nasal passages c. responsible for warming, humidifying, and filtering the inhaled air

For patients suffering from type I diabetes, which of the following precautions should be observed. A. No special precautions are necessary B. The appointment should begin after a meal and insulin injection C. The patient should be seen after a meal and postpone the insulin injection until after the procedure D. The patient should have an empty stomach and postpone the insulin injection until after the procedure E. The patient should have an empty stomach and should take insulin before the procedure

B. The appointment should begin after a meal and insulin injection 1) Concept: Diabetic patients must maintain a normal serum glucose level before, during and after the dental treatment to prevent any further complications. 2) The intake of food will prevent the diabetic patient from having hypoglycemia. Severe hypoglycemia if not properly managed will result in patient's unconsciousness, seizures, and even death if not managed properly. 3) Insulin administration maintains a consistent blood glucose level to prevent hyperglycemia.

Each of the following is a reason adjacent Class I I or Class III cavities should be prepared and restored using a composite resin during a single appointment EXCEPT one. Which is the EXCEPTION? A. Matching the color is easier B. The contact area will be better C. Time is conserved for the clinician and patient D. Access to the cavities is simplified

B. The contact area will be better Adjacent proximal cavities are best restored in one appointment because a. time is conserved for the clinician and patient b. colors will match and age similarly since they are likely to be from the same batch. c. there will be adequate access and visibility, enabling the dentist to work more efficiently and effectively. d. restorations can be placed easier due to adequate space to finish the proximal margins of the restorations properly. e. the fact that the caries to be restored are adjacent does not improve the contact of the restoration

Periapical radiolucencies only are able to be visualized after which of the following is (are) annihilated? A. The facial or lingual periosteum B. The junction of the cortical and cancellous bone C. The junction of the facial and cortical plates D. Both the facial and lingual cortical plates

B. The junction of the cortical and cancellous bone 1) Periapical radiolucencies a. can only be visualized after the cortical bone surrounding the root apex is destroyed ad the infection extends into the cancellous bone of periosteum b. lamina dura surrounding the periodical aspect of tooth is considered cortical bone, and as long as the cortical bone is intact, periodical radiolucency will not be evident radiographically c. evidence of bone destruction that extends into the cortical bone and cancellous bone supporting the tooth.

If you place a direct pulp cap that remains asymptomatic after 6 months, what is the most likely understanding of the pulp status? A. The calcium hydroxide has induced reparative dentin formation to completely heal the exposure B. The lack of symptoms may be temporary, and the tooth may become symptomatic later C. The pulp cap was a success D. Tertiary dentin has been laid down to heal the exposure site permanently

B. The lack of symptoms may be temporary, and the tooth may become symptomatic later 1) Histological analysis and not with clinical signs and symptoms that the tooth elicit, so it is impossible to truly know the pulp status with out extracting the tooth or waiting for symptoms to arise 2) Asymptomatic condition of the pulp at present could never determine the status of the pulp

If a patient that is sedated with nitrous oxide suddenly becomes irrationally excited, which of the following is the most likely explanation? A. The patient exhibits drug seeking behavior from nitrous oxide B. The patient is entering Stage 2 anesthesia C. The patient really likes you D. The patient is entering Stage 1 anesthesia

B. The patient is entering Stage 2 anesthesia 1) Stage 2 anesthesia (excitatory stage) a. describes the moment when the patient begins to exhibit uncontrolled movements, excited actions and possibly delirium b. characterized by 1) irregular respirator patterns and hear rate, pupillary dilation, and uncontrolled body movements 2) this stage is very dangerous and potentially life threatening, fast acting drugs are utilized to pass this stage and attain the third stage of anesthesia as quick as possible.

Which of the following is the MOST important variable for the clinical success of posterior resin based composite (RBC)? A. Choosing the best manufacturer of RBC B. The placement technique C. The proper amount of Bis-GMA in the resin D. The shade of RBC selected

B. The placement technique 1) Concept: The most important variable in clinical success of posterior RBC is the placement technique used to carry out restoration. 2) Posterior composites should be placed incrementally in order to facilitate proper light activation and restore correct anatomic contours. The enamel layer of the restoration which is occlusal 1.5-3mm is placed using an anatomic layering technique. 3) An important step to restore Class II preparations with direct composite restorations is the selection and proper placement of matrix band. 4) Class II composites are totally dependent upon the contour and position of matrix and wedge for establishing appropriate proximal contacts.

Which of the following types of TMJ disorders would a clinician expect to respond best to treatment with an occlusal separator? A. Arthrosis B. Chronic dislocation C. Muscle spasm D. Unilateral condylar hyperplasia E. Capsular Fibrosis

C. Muscle spasm 1) Occlusal Separators a. help the TMJ recover from heavy bite forces following muscle spasms b. separate upper and lower teeth from each other and allow the TMJ to rest and recover c. they also serve to help the muscles of mastication relax

Which of the following describes the function of silanes in resin composites? A. To bind fluoride and enhance remineralization B. To bind inorganic and organic materials with hydrogen and covalent bonds C. To make the composite polish smoothly D. To make the dentin rough

B. To bind inorganic and organic materials with hydrogen and covalent bonds 1) Concept: The function of silanes from resin composites is to bind the inorganic and organic materials with hydrogen and covalent bonds. 2) The bi-functional silane coupling agents are incorporated in composites resins to make a strong bond between the resin composite material and the tooth surface. 3) These attach to the dentinal substrate on one side and the composite resins on the other side forming a hybrid layer.

Each of the following is a reason for placing a post in the restoration of an endodontically treated posterior tooth EXCEPT one. Which is the EXCEPTION? A. To enhance the resistance of the restoration to lateral forces B. To strengthen the root C. To enhance the retention of the core

B. To strengthen the root 1) Post a. primary purpose or using a post in an endodontically treated tooth is to retain the core if there is extensive loss of coronal tooth structure b. enhance the resistance of the restoration to lateral forces depending on the amount of remaining tooth structure, the length and rigidity of the post, presence of ferrule and the presence of anti-rotation features c. placement of posts inside the root canal does NOT strengthen the root tooth.

Which of the following choices describes the access cavity preparation on a mandibular molar with 3 canals best? A. Pentagonal B. Trapezoidal C. Triangular D. Oval

B. Trapezoidal 1) Access preparation for the mandibular 1st molar is trapezoidal in shape since it follows the position of the cusp tips.

An apically repositioned flap margin on the palatal aspect of molar teeth is allowed for by which of the following? A. Avoiding an initial incision that contacts bone B. Trimming the flap margin to the proper length during the procedure C. Utilizing only a continuous suture type for closure of the surgical site D. Utilizing only a sulcular incision in the initial incision

B. Trimming the flap margin to the proper length during the procedure 1) Concept: Apically repositioned flap margins on the palatal aspect need to be placed exactly to its predetermined position during the procedure because the palatal mucosa has no unattached mucosa that can be displaced. b. An apically repositioned flap usually requires displacement of the gingiva to a more apical position, requiring vertical releasing incisions except when the flap is made on the palate. 2) Palatal Mucosa a. firmly attached to the palatal bone and due to its thick nature displacement is not possible

Which of the following disorders is associated with a chromosomal abnormality which can occur as the result of either chromosomal nondisjunction or translocation? A. Klinefelter syndrome B. Trisomy 21 (Down syndrome) C. Turner syndrome D. Ehler-Danlos syndrome

B. Trisomy 21 (Down syndrome) 1) Trisomy 21 occurs because of meiotic nondisjunction, which is a situation where a gamete from a parent contains excess chromosomes. 2) Patients with Down's syndrome possess 3 copies of chrosome 21 instead of 2 copies. This condition is considered the most commonly occurring chromosomal abnormality in humans.

Which of the following is predominantly the basic component of most root canal sealers? A. Polyvinyl resin B. Zinc oxide C. Zinc Oxyphosphate D. Zinc stearate E. Polycarboxylate

B. Zinc oxide 1) Zinc oxide a. antibiotic properties b. serves to prevent reinfection of the root canal 2) Eugenol a. sometime used in conjunction and serves to sedate and sooth periradicular nerve tissues to decrease post obturation sensitivity

A 21-year-old male patient complains of a dull pain in their posterior left mandibular region with a radiograph that reveals not only a radiolucency around the 1st molar roots, but also a radiopacity of bone peripheral to this radiolucency. Which of the following is the best diagnosis? A. a manifestations of systemic bone disease B. a reaction to an apical inflammatory disease

B. a reaction to an apical inflammatory disease 1) Radiolucency is from the tissue destruction that occurred within the periodical area and the radiopacity that denotes the boundary set by the inflammatory cells as they try to confine the infection. 2) Once the infection is completely confined by the inflammatory cells, a periodical cyst or a granulation tissue may form and the damaged bone begins to remodel. 3) The continuous remodeling of the bone creates the solid, well-defined and radiopaque border around the periphery of the periodical cyst or of the granulation tissue.

A 21-year-old male patient complains of a dull pain in their posterior left mandibular region with a radiograph that reveals not only a radiolucency around the 1st molar roots, but also a radiopacity of bone peripheral to this radiolucency. Which of the following is the best diagnosis? A. a manifestations of systemic bone disease B. a reaction to an apical inflammatory disease C. An error in radiographic technique (artifact) D. Normal for this patient E. A developmental abnormality

B. a reaction to an apical inflammatory disease 1) Radiolucency is from the tissue destruction that occurred within the periodical area and the radiopacity that denotes the boundary set by the inflammatory cells as they try to confine the infection. 2) Once the infection is completely confined by the inflammatory cells, a periodical cyst or a granulation tissue may form and the damaged bone begins to remodel. 3) The continuous remodeling of the bone creates the solid, well-defined and radiopaque border around the periphery of the periodical cyst or of the granulation tissue.

Which of the following is the BEST reason a dentist may choose to use a resin-based composite to restore posterior teeth? A. bad oral hygiene B. esthetics C. frequent clinching or grinding D. patient demonstrates trismus E. unable to isolate tooth with rubber dam

B. esthetics 1) Some patient do NOT like amalgam fillings in molars as these fillings sometimes show during speech. 2) With para functional habits such as frequent clenching or grinding of teeth indicate that amalgams or metal crowns are placed.

The primary etiology of for the failure of tooth replantation is which of the following: B. external resorption C. infection D. internal resorption E. pulp necrosis F. ankylosis

B. external resorption 1) External Resorption a. most common cause of failure of tooth replantation b. Types 1) External replacement resorption 2) External inflammatory resorption a. caused by infection following a failed root canal treatment b. external resorption also depends upon the degree of PDL damage

Which of the following is dentin primarily composed of? i. Apatite crystal flakes ii. Elastic fibers iii. Collagen iv. Fibroblasts A. i and ii B. i and iii C. i, ii, and iii D. i, ii, and iv

B. i and iii 1) Dentin a. composed of apatite crystal flakes and collagen fibers b. 45-50% inorganic component in the form of hydroxyapatite c. 30% organic mainly collagen d. 25% water 2) Odontoblasts a. generate an extracellular collagen matrix as they being to move away from the adjacent ameloblasts 3) Dentin formation a. begins immediately before the enamel formation

Loops and helices are incorpated into arch wire to... A. facilitate bodily tooth movement B. improve activation range C. improve anchorage D. provide a center of rotation E. decrease the modulus of elasticity

B. improve activation range 1) Loops and helices a. are incorporated to increase the range of its activation b. activation is created by slightly closing the loops and helices with a pair of pliers c. archwires to be activated to achieve the desired results as it becomes loose after moving teeth in the desired direction, decreasing the tension of the arch wire d. closing the loops and helices result in the tightening of the arch wire

A periosteal elevator is NOT normally used to... B. luxate teeth C. protect the tissue flap from instruments

B. luxate teeth 1) Periosteal elevator a. not designed to adapt to the tooth deep in the alveolar socket b. tips of periosteal elevators are thin and wide and are designed to elevate the soft tissue from the bone c. used to elevate the interdental papilla during surgical and non-surgical extractions d. can also be used as retractors and protect tissue flaps from other dental instruments during an operation

To prove clinical effectiveness for American Dental Association (ADA) approval, an antimicrobial agent must demonstrate..... B. reduction in disease

B. reduction in disease

Which of the following structure(s) must be annihilated for a periodical radiolucency to be visualized? A. the facial or lingual periosteum B. the junction of the cortical and cancellous bone C. the junction of the facial and cortical plates D. both the facial and lingual cortical plates

B. the junction of the cortical and cancellous bone 1) Periapical radiolucencies can only be visualized after the cortical bone surrounding the root apex is destroyed and the infection extends into the cancellous bone of periosteum. 2) Lamina dura is considered cortical bone, and as long as the cortical bone is intact, periodical radiolucency will not be evident radiographically.

Basic Endodontic Treatment Procedures

Basic Endodontic Treatment Procedures Answers

Behavior

Behavior Answers

True/False: New junctional epithelium can form on either cementum or dentin after periodontal surgery Junctional epithelium is re-established as early as one week.

Both are true statements 1) Junctional epithelium re-attached back to cementum and dentin after an apically repositioned flap by re-establishing tight junctions 2) Regeneration of junctional epithelium takes about 10 to 14 days to complete ager surgical procedure. 3) Regeneration of junctional epithelium may occur even after its surgical detachment as along as the root surface is completely derided of plaque and calcular deposits.

Improvements in the oral hygiene of patients suffering from nutritional deficiencies affects the incidence of chronic inflammatory periodontal disease. Improvements in oral hygiene also affect the severity in cases of chronic inflammatory periodontal disease.

Both statements are TRUE 1) Chronic inflammatory periodontal disease a. elimination of chronic inflammatory periodontal disease is possible with the help or improvement in oral hygiene conditions b. improved oral hygiene measures = key to solving most of the periodontal problems 2) Incidence a. as well as severity of chronic inflammatory periodontal disease is lessened by taking proper oral hygiene measures regularly

True/False: The duration a prescribed therapy has an effect on patient compliance. Disease severity as perceived by the patient has an effect on patient compliance.

Both statements are TRUE 1) Concept: Patients undergoing a treatment with a long duration have greater tendencies of becoming less compliant and less dedicated to treatment procedures due to the lack of a immediate treatment result 2) Treatments that last for a very long time also make patients tired of routine medication and causes them to be less compliant. 3) Patients who are hopeful and expect good results from medical treatment are more receptive to taking medication and are more compliant. 4) Patients who perceive that their disease severity is hopeless tend to be less receptive and less compliant with their medication.

True/False After an apically re-positioned flap procedure is performed, junctional epithelium can establish itself on both cementum and dentin The junctional epithelium is re-established within 10 days following surgery.

Both statements are TRUE 1) Junctional epithelium a. re-attached back to cementum and dentin after an apically repositioned flap by re-establishing tight junctions b. takes about 10-14 days to complete after surgical procedure 2) Regeneration of junctional epithelium a. may occur after its surgical detachment as long as the root surface is complete derided of plaque and calcular deposits

True/False: One objective of root-canal obturation is to create a fluid tight seal. Another objective is to create an environment of healing to occur.

Both statements are True 1) Concept: Obturation a. is the step in root canal treatment that involves filling the root canal with a biocompatible material to create a hermetic seal to prevent contamination caused by leakage either from the oral cavity or periradicular tissues into the root canal. b. seals the remaining irritants in the root canal system after proper cleaning and shaping of the canal creates a conducive environment for the periodical healing to occur.

True/False: Spherical alloys contain more mercury than admixed alloys. The higher mercury content is why it is easier to achieve a contact with spherical alloy.

Both statements are false 1) Concept: Spherical amalgam alloys do NOT contain more mercury than the admixed alloys. 2) Spherical alloys a. are easier to condense into areas of difficult access such as around pins because they provide less resistance to condensation pressures. They harden rapidly, are smoother for carving, burnishing and polishing. b. are difficult to achieve tight interproximal contacts with because of their pliable nature and admixed alloys are recommended for areas requiring development of proximal contacts.

A patient should control the amount of plaque in their mouth before undergoing any periodontal surgery; BECAUSE plaque-infected teeth demonstrate a higher incidence of disease recurrence after surgery? A.

Both the statement and reason are correct and related 1. Plaque a. aggregation of bacterial species that may infect the surgical wound after a surgical procedure b. plaque accumulation along the tooth surface predisposes the open wound to pathogens and opportunistic bacteria 2. Surgical Procedure a. patients should strictly follow a proper daily oral hygiene regimen to ensure that they are capable of maintaining good dental health after surgery b. surgical wounds are open areas where bacterial species can easily penetrate and infect the body

Trauma to a tooth can cause the pulp chamber to undergo calcific metamorphosis. Calcific metamorphosis occurs because trauma can induce odontoblasts to activate.

Both the statement and the reason are correct and related 1) Calcific Metamorphosis a. common condition observed in teeth that have experienced trauma and is characterized by an abnormal hard tissue formation and deposition in the pulp space that either partially or completely fills it b. trauma induces odontoblasts to rapidly form extensive amount of reparative dentin, causing the pulp to appear smaller and at some point obliterated entirely c. may occur when the fragments of fracture roots are in tight contact with one another if the apical portion of the tooth is still vital. The calcification will close the fracture line and stabilize the connection of the two fragments of the root.

For a class V restoration where should in the rubber dam be punched? A. 3mm facial to the designate tooth B. 3mm lingual to the designated tooth C. 1mm facial to the designated tooth D. 1mm lingual to the designated tooth E. Rubber dams should never be used in Class V restorations

C. 1mm facial to the designated tooth 1) Rubber dam placement a. rubber dam should be placed 1mm facial to the designated tooth for Class V restorations, enabling the rubber dam to completely cover the area to be isolated and making subgingival placement easier if needed b. Class V restorations require planned measure for maintenance of isolation as it is extremely difficult to place rubber dam in these areas c. Location of Class V preparations makes it very difficult to prevent moisture contamination.

Root planing should be avoided during modified Widman flap procedures BECAUSE the margin of the modified Widman flap will be placed apically to the level of altered root surfaces? A.

Both the statement and the reason are incorrect. 1) Concept: Modified Widman flaps are a periodontal flap that is performed to reduce periodontal pockets. 2) The modified Widman flap provides better access and visibility of the root surface, making the root planing procedure easier as well as also increasing the level of success of the procedure. 3) Pocket depth elimination helps patients maintain better plaque control which will reduce the chances of disease recurrence.

Which of the following concentrations describes the range of lethal dose of fluoride? A. 0.5-1.0 mg/kg B. 100-200 mg/kg C. 20-50 mg/kg D. 300-500 mg/kg E. 500-800 mg/kg

C. 20-50 mg/kg 1) 20-50 mg/kg body weight is the lethal dose of fluoride. Above this range, fluoride starts showing its toxic effects in the body.

How many hours after mechanical debridement with a toothbrush is a mature dental plaque found? A. 1-2 Hours B. 12-24 Hours C. 24-48 Hours D. 3-5 Hours E. 6-10 Hours

C. 24-48 Hours 1) Mature dental plaque is usually reformed on the teeth within 24-48 hours after plaque removal 2) 1st bacteria to attach to the pellicle glycoproteins are gram positive aerobic cocci such as Streptococcus sanguinis. 3) After 24 hours, anaerobic gram negative cocci, rods, and filaments begin to colonize the plaque.

How many hours should a cervical pull headgear be worn to be most effective? A. 10 B. 12 C. 14 D. 6 E. 8 F. 4

C. 14 1) Concept: Cervical pull headgear is an appliance used to correct Class II malocclusion with deep bite. 2) Cervical pull headgear is composed of the neck strap, and inner and outer face bow a. The neck strap acts as an extra-oral anchorage b. The outer bow connects the face bow assembly with the backstrap c. The inner bow is inserted into the tubes found on the molar bands d. inhibits the growth of the maxilla and causes the maxillary molars to extrude lessening the deep bite e. The headgear must be worn regularly for 14 hours per day for optimum results.

Which of the following distinguishes periodontitis from gingivitis? A. All answers apply B. Bleeding on Probing (BOP) C. Clinical Attachment Loss D. Presence of purulence E. Probing depth

C. Clinical Attachment Loss 1) Gingivitis a. inflammation of the gingiva 2) Periodontitis a. inflammation of the gingiva associated with attachment loss

Which of the following describes the maximum amount of resin composite that can be properly cured per layer of restoration? A. 0.5mm B. 1mm C. 2mm D. 4mm E. Depth does not matter

C. 2mm 1) Concept: The maximum depth of resin that can be properly cured per layer is approximately 2mm. 2) Light cured composites are currently among the most popular restorative materials due to their ease of handling. 3) The success of composite resin restorations depends upon the access of high intensity curing light to the material. 4) If the composite thickness exceeds 2mm, inadequate curing occurs. This phenomenon is specially seen with darker shade composites. Filler particles and coloring agents tend to scatter or absorb the curing light within 1-2mm of the material.

How many years should the dental records of an employee who just terminated their employment in your office be kept? A. 10 years B. 20 years C. 30 years D. 5 years E. 1 year

C. 30 years 1) All dental records should be kept as long as possible 2) They are patient property. 3) They may be required for forensic identification by the police.

Which of the following would create the MOST esthetically pleasing Class IV resin-based composite restoration? A. A 1.0mm facial bevel should be placed B. A 2.0mm lingual bevel should be placed C. A facial bevel of ~2.0mm should be placed D. No bevel placed, facially or lingually

C. A facial bevel of ~2.0mm should be placed 1) Concept: A facial bevel of approximately 2mm should be placed in order to achieve the most esthetically pleasing Class IV resin-based composite restorations. Bevel is placed at an angle of 45-60 degrees. 2) Advantages of bevel placement are: a. No line of demarcation between the tooth and restorative material interface (enhancing esthetics) b. Conservative tooth preparation c. 2x high bond strength is achieved d. an adherence with acid etched ends of rods crystals is possible e. better resistance to fracture

Which of the following is the term to describe the enamel bevel placed on the facial side of a Class IV preparation? A. A conventional bevel B. A retentive bevel C. An esthetic bevel D. No answers apply

C. An esthetic bevel 1) Concept: Esthetic Bevel a. The enamel bevel placed on the facial side or surface of a Class IV preparation is an esthetic bevel (Infinity bevel)... b. Esthetic bevels are typically placed at 60o c. This bevel is placed to remove or mask the line of demarcation between the restorative material and the tooth structure. d. If NO bevel is placed, a visible line can be seen at the restorative-tooth interface making the restoration unaesthetic for the patient. e. Placement of this esthetic bevel also makes the restoration conservative, increases the resin bond strength and decreases the chances of fracture. 2) Retentive Bevel a. typically found around the margins of a Class V restoration

Malocclusion is classified by using which of the following planes of space? A. Anterio-posterior, coronal, sagittal B. Antero-posterior, sagittal, vertical C. Antero-posterior, transverse, vertical D. Horizontal, sagittal, vertical; Horizontal, transverse, coronal

C. Antero-posterior, transverse, vertical 1) Concept: Malocclusion Planes a. The three planes of space which are used to classify malocclusion are: 1) Antero-posterior 2) Transverse 3) Vertical 2) All these are the orientation planes or the reference planes used to communicate the dimensions of orthodontic problems. 3) Sagittal plane is used to describe the anterior-posterior relationships 4) Vertical plane is used to give the super-inferior relationships 5) Transverse plane describes right to left relationships.

Which of the following compounds is responsible for the photoinitiaion step of light-cured sealants? A. Benzoyl peroxide B. Bisphenol A C. Camphoroquionone D. Fluoride E. Fumed Silica

C. Camphoroquionone 1) Camphoroquionone a. used as a photo initiator and visible light or argon laser is used as the activator in light cure sealants 2) Benzoyl Peroxide a. incorporated as a component in auto-curing/self-curing sealants 3) Sealants a. most important requirements of a pits and fissure sealant is that it should prevent leakage at its periphery and gives an adequate working time b. adherence of pits and fissure sealants and working time is significantly better in light cured sealants as compared to chemical cure sealants

The indicated method for trimming the margins of an amalgam restoration is by... A. Carving a circumferential trough around the margins of the restoration so that amalgam can be burnished into it for a tight seal B. Carving circumferentially with a sharp instrument resting mostly on the restoration C. Carving circumferentially with a sharp instrument that rests mostly on the tooth surface D. Burnishing from the tooth to the restoration

C. Carving circumferentially with a sharp instrument that rests mostly on the tooth surface 1) Concept: A sharp carver should be placed on the tooth surface at the tooth and restoration surface. 2) Carver instrument is moved circumferentially for trimming the margins of the restoration 3) Instrument should be moved in order to restore the normal morphology of the tooth.

Which of the following eruption sequences is usually observed in the primary dentition? A. 1st molar, central incisor, lateral incisor, 2nd molar, and canine B. 1st molar, central incisor, lateral incisor, canine, 2nd molar C. Central incisor, lateral incisor, 1st molar, canine, and 2nd molar D. Central incisor, lateral incisor, canine, 1st molar, and 2nd molar

C. Central incisor, lateral incisor, 1st molar, canine, and 2nd molar 1) Concept: Central incisor, lateral incisor, 1st molar, canine, and 2nd molar 2) Guidelines for eruption of deciduous teeth a. incisors: 6 months b. 1st molars: 12 months c. canines: 18 months d. 2nd molars: 24 months Note: this is an easy way to remember eruption sequences for deciduous teeth (6 month intervals). Remember that mandibular dentition is normally before maxillary eruption.

Which of the following patient health histories would be more prone to postoperative bleeding after multiple extractions? Select all that apply. A. Angina B. Asthma C. Cirrhosis D. Diabetes Type I E. Diabetes Type II F. Renal disease

C. Cirrhosis E. Diabetes Type II 1) Concept: Patients with diabetes type II and liver cirrhosis are more prone to undergo postoperative bleeding in such cases. 2) Diabetes Mellitus Type II a. less platelet aggregation and impaired wound healing results in prolonged bleeding after multiple extractions 3) Cirrhosis a. impairment of vitamin K dependent clotting factors and prolonged prothrombin time occurs in liver cirrhosis b. Vitamin K dependent clotting factors II, VII, IX, and X are formed and stored in the liver.

Which two preoperative tests should be performed before deciding if general anesthesia should be administered to a patient? A. Complete blood count and bleeding time B. Bleeding time and clotting time C. Complete blood count and urinalysis D. Hemoglobin level and prothrombin time E. Total white blood cell count and prothrombin time

C. Complete blood count and urinalysis 1) Concept: Preoperative tests are essential to ensure optimal health conditions of the patients prior to any surveil procedure 2) Complete blood count a. Assessing the overall health of the patient by evaluating their complete blood count will provide sufficient information about the physiologic health of the body b. Urinalysis provides information about the efficiency of the kidneys, liver and other organs within the body 3) Complete blood count and urinalysis a. Complete blood count and urinalysis both help identify the presence of infection within the body and abnormal conditions that may contraindicate surgical procedures.

A patient independently maintaining a patent airway describes which type of anesthesia? A. Deep sedation B. General anesthesia C. Conscious sedation

C. Conscious sedation 1) Conscious sedation a. can independently act and respond to verbal instructions b. allows the patient to relax and remain calm and cooperative by reducing their sensitivity to pain c. patient is awake but feeling lightly drowsy d. patient maintains a patient airway independently 2) Deep sedation a. characterized by a depressed consciousness such that the patient is unable to continuously and independently maintain a patent airway and experiences a partial loss of protective reflexes and ability to respond to verbal commands or physical stimulation. 3) General anesthesia a. medically induced coma leading to a loss of protective reflexes resulting from the administration of one of more general anesthetic agents.

Each of the following are parts of informed consent EXCEPT one. Which is the EXCEPTION? A. Alternative to treatment B. Benefits of treatment C. Cost of treatment D. Risks of treatment

C. Cost of treatment 1) Informed consent requires that the patient has the capacity to act autonomously with understanding, and without outside controlling influences. 2) Informed consent requires that the patient have the knowledge and understanding of the risks and benefits of the proposed treatments.

Advantages of gas sterilization include ... A. Does not corrode instruments B. Drying time is not necessary C. Ecologically friend and safe to use D. Short processing time

C. Ecologically friend and safe to use 1) Ethylene oxide is considered safe with abatement technology 2) Hydrogen peroxide gas is ecologically safe to use.

In patients suffering from multiple sclerosis... A. Anesthetic dosage required for a given procedure is more than for a normal patient B. Anesthetics do not last as long as it would for a normal patient C. Epinephrine in local anesthetics are contraindicated D. Anesthetic dosage required for a given procedure is less than for a normal patient

C. Epinephrine in local anesthetics are contraindicated 1) Concept: Epinephrine should not be used for patients suffering from multiple sclerosis because epinephrine can aggravate the condition by regulating the activation of blood lymphocytes. 2) Epinephrine is a cetecholamine and neurotransmitter responsible for the body's flight- or flight response. 3) Elevating the levels of epinephrine also increases blood lymphocyte activity which could cause further damage to the myelin sheaths of the neurons thereby aggravating multiple sclerosis condition.

Which of the following describes the loss of tooth structure through dissolution by acid NOT created by bacteria? A. Abfraction B. Attrition C. Erosion D. No answers apply E. Abrasion

C. Erosion 1) Erosion a. loss of tooth structure through chemical dissolution by acids NOT created by bacteria b. most commonly caused by ingestion of acidic beverages in males c. most commonly caused by bulimia in adolescent females d. when oral pH drops below 5.5 e.

What should you do first if you happen upon a collapsed or unconscious person? A. Examine the person for signs of trauma B. Assess the person's pulse C. Establish if the person is unconscious D. Open airway with head tilt/chin lift E. inject 10mL 1:1000 epinephrine into the persons myocardium F. Take a blood glucose reading

C. Establish if the person is unconscious 1) Concept: If you see a collapsed or unconscious person, the most important step is to make sure that the patient is truly unconscious. 2) Conscious a. can be determined by the person's reaction to your voice or through the coordination of their movements 3) Unconscious a. cannot be roused and is unaware of his or her surroundings b. will not remember what has happened to them and may not be coherent c. one way to check is by tapping them on their shoulder and asking "Hey, are you okay?" 4) Next Steps a. after establishing unconsciousness, assess the person's airway, breathing, and pulse before providing first aid

Which of the following does the bonding of composite material to dentin depend on? A. Dehydration of dentin B. Difunctional coupling agents C. Etching with dilute phosphoric acid D. Covalent bonding

C. Etching with dilute phosphoric acid 1) Bonding a. dentin bonding mechanisms rely on difunctional coupling agents like silane, which penetrate into the inherently moist dentin surfaces and are able to copolymerize with composite resins b. bond strength is obtained from the penetration and adaptation of the bonding agent to the demineralized inter tubular dentin and exposed collagen fibers c. interdiffusion zone formed by the bonding agent and the dentin is called hybrid layer

If a patient has not brushed for 2 days, the most common bacterial species found in the plaque is... A. Aggrigatibacter B. Gram negative rods C. Gram positive rods and cocci D. Spirochetes E. Fusobacterium

C. Gram positive rods and cocci 1) As plaque matures, more gram positive rods and cocci popular the teeth creating gingival irritation and gingivitis. 2) When tartar or calculus deposits --> gram negative bacteria increase in number

Which of the following is NOT a MAJOR sign/symptom of rheumatic fever based on Jones criteria? A. Carditis B. Chorea/dyskinesia C. Fever D. Polyarthritis E. Erythema marginatum

C. Fever 1) Concept: Rheumatic fever is an inflammatory disease that occurs after a Strep progenies infection or scarlet fever and is caused by antibody cross reactivity from the Strep M protein 2) Fever is considered a MINOR criteria under the Jones classification, not a MAJOR criteria. 3) Major criteria under the Jones classification are: 1) Polyarthritis a. a temporary migrating inflammation of the large joints, usually starting in the legs and migrating upwards 2) Carditis a. inflammation of the heart muscle (myocarditis) which can manifest as congestive heart failure with shortness of breath, pericarditis with a rub, or a new heart murmur 3) Subcutaneous nodules a. painless, firm collections of collagen fibers over bones or tendons. b. They commonly appear on the back of the writs, the outside elbow, and the front of the knees 4) Erythema marginatum a. a long-lasting reddish rash that begins on the trunk or arms as macules, which spread outward and clear in the middle to form rings, which continue to spread and coalesce with other rings, ultimately taking on a snake-like appearance 5) Chorea (St. Vitus' dance) a. characteristic series of rapid movements without purpose of the face and arms that can occur very late in the disease for at least three months from onset of infection

Each of the following statements regarding flowable resin composite as the first increment in a Class II prep are true EXCEPT one. Which statement is the EXCEPTION? A. Flowable demonstrates 3X greater polymerization shrinkage than hybrid composite B. Flowable has NOT demonstrated any difference in post-op sensitivity C. Flowable has superior mechanical properties compared to conventional composite D. Most flowables are NOT as radiopaque as enamel

C. Flowable has superior mechanical properties compared to conventional composite 1) Concept: Flowable resin composites demonstrate inferior mechanical properties compared to tradiational resin composites. Flowable resin composites demonstrate low strength, wear resistance, and all other mechanical properties. 2) Lower viscosity composite resins exhibit high flow rates and penetrate deeper portions of the preparation to fill the voids and irregularities. 3) Flowable composites exhibit 3x greater values of polymerization shrinkage stresses compared to conventional RBC. 4) The flowable composites are commonly used as the first small increment in snowplow technique in proximal preparations. 5) A small uncured increment of flowable composite is placed on the gingival portion, and then a conventional composite is placed over it. The conventional composite is condensed ti displace most of the flowable composite out of the cavity preparation and rest of it flows into deeper portions where conventional composite cannot reach.

In relation to resin composites, which mechanical property measure the material's ability to resist the propagation of a crack? A. Elasticity B. Flexural strength C. Fracture toughness D. Hardness

C. Fracture toughness 1) Fracture toughness a. measures the material's ability to resist the propagation of a crack b. quantitative way of expressing a material's resistance to brittle fracture when a crack is present c. brittle fracture = characteristic of materials with low fracture toughness 2) Elasticity a. tendency of a solid material to return to their original shaped after being deformed.

What type of flap is most commonly indicated for the surgical removal of a mesiodens? A. Facial flap with releasing incision B. Facial flap without relaxing incisions C. Full thickness palatal D. No answers are correct E. Envelope

C. Full thickness palatal 1) Full thickness palatal a. mesiodens are usually found palatal to the area in between the maxillary central incisors and can be extracted with a full thickness palatal flap b. allows for adequate visualization of the bone surrounding the mesiodens c. provide better blood supply for the flap than partial thickness flaps, rendering the procedure less traumatic and reduces bleeding

When examining a patient for the first time, the dentist notices that there is an abnormally wide permanent mandibular incisor, and that there are only three total mandibular incisors, counting the abnormal wide one. The condition most likely is due to the anomaly: A. Consecrence B. Dilaceration C. Fusion D. Gemination E. Flexion

C. Fusion 1) Fusion a. 2 buds fuse b. must have confluent (connected) dentin c. less teeth in the mouth than expected 2) Gemination a. still have a normal number of teeth in the mouth b. is when ONE tooth bud splits so that there are two crowns but only one shared root (Siamese twin like) c. with gemination patients there is an extra tooth 3) Flexion a. a deviation or bend at the root of tooth b. less than 90 degree bend c. results from trauma during development 4) Dilaceration a. Severe angulation or curve in the root or crown of tooth b. Bend is at junction of crown and root c. Usually results from trauma during development. 5) Concrescence a. a condition where the cementum overlying the roots of at least 2 teeth connects to join the teeth together

Each of the following is an indication for using resin composite materials EXCEPT one. Which is the EXCEPTION? A. Areas where esthetic is important B. Conservative tooth preparations C. Gingival margin is not on intact enamel D. Small and moderately sized restorations

C. Gingival margin is not on intact enamel 1) Concept: Composite resin restorations are contraindicated when gingival margins are NOT intact typically due to concerns regarding fluid contamination 2) It is very difficult to maintain isolation and make a satisfactory bond with etched dentin. 3) It is advised to placed an (Resin Modified Glass Ionomer Cement) RMGIC liner before placement of resin material if it necessary to fill it with composite (like in anterior esthetic cases). 4) Indications of composite restorations are a. small and moderately sized restorations b. conservative tooth preparations c. areas where esthetics is important

Which of the following factors is MOST related to caries initiation in elderly patients? A. Bruxism B. Erosion C. Gingival recession D. Attrition

C. Gingival recession 1) Gingival recession results in exposure of the root surfaces 2) Passive movement of gingival tissue occurs apically with the aging process

Which of the following bacteria are associated with supra gingival plaque? C. Gram Positive rods and cocci D. Gram Positive filaments E. Spirochetes

C. Gram Positive rods and cocci 1) Supragingival plaque a. consists of gram positive rods and cocci b. collection of bacteria, proteins, carbohydrates, and glycoproteins 2) Subgingival plaque a. non calcified mass of bacteria, proteins, and carbohydrates situated inferior to the gingival margin

Which of the following does a 2-day-old plaque biofilm consist primarily of? A. Bacteroides species B. Filamentous organisms C. Gram-positive cocci and rods D. Treponema species E. A non-mineralized pellicle without organization

C. Gram-positive cocci and rods 1) During the initial formation of an undisturbed plaque, majority of bacteria present are gram positive cocci and rods like streptococcus and actinomyces species. 2) Within the 2 day period of undisturbed plaque format, the gram positive cocci and rods cling to each other and increase in number 3) Approximately 25% of other bacterial species may include gram negative rods and coci which eventually will increase as plaque remains undisturbed. 4) S. Sanguinis is usually the first bacterial species to adhere to tooth structure.

A plaque that has been developing for 48 hours is primarily composed of... A. Bacteroides species B. Filamentous organisms C. Gram-positive cocci and rods D. Treponema species E. A non-mineralized pellicle without organization

C. Gram-positive cocci and rods 1) Plaque formation a. within the 2 day period of undisturbed plaque formation, the gram positive cocci and rods cling to each other and increase in number 2) S. Sanguinis a. first bacterial species to adhere to tooth structure

Where is the most common site of a intraoral malignant melanoma? A. Buccal mucosa and vestibule B. Floor of mouth and anterior lingual gingival C. Hard palate and maxillary gingiva D. Soft palate and tonsillar pillar complex E. Lateral and ventral tongue

C. Hard palate and maxillary gingiva 1) Concept: Malignant melanomas usually occur along the hard palate, maxillary gingiva, tongue, mandible, and the buccal mucosa. 2) The incidence of melanomas within the oral mucosa is said to be rare with an occurrence of less than 1% 3) The prognosis of treating malignant melanoma is said to be poor with a 5-year survival rate of about 10-25% in the average. 4) Early signs of melanoma can be diagnosed by "ABCDE" A: Asymmetry B: Borders (irregular) C: Color (multicolored) D. Diameter (greater than 6mm, about the size of a pencil erase) E: Evolving over time 5) The most dangerous form of melanoma, nodular melanoma can be diagnosed with "EFG" E: Elevated above the skin surface F: Firm to the touch G: Growing

The graft with the greatest osteogenic potential is which of the following? C. Hemopoietic marrow D. Corticocencellous block

C. Hemopoietic marrow 1) Hemopoietic marrow a. contains cells that possess osteogenic potential and can differentiate into bone or cartilage depending on what is needed by the body 2) Freeze-dried allografts and xenografts a. are both osteoinductive = induce the formation of new bone through the stimulation of the stem cells in the body b. both DO NOT contain cells that can form bone

Diagnosis

Diagnosis Answers

Which of the following is related to periodontal destruction involving primary teeth due to an enzyme deficiency? A. Aggressive periodontitis B. Cyclic neutropenia C. Hypophosphatasia D. Necrotizing ulcerative gingivitis E. Papillon Lefevre syndrome

C. Hypophosphatasia 1) Hypophosphatasia a. hereditary disease where there is a marked deficiency of the tissue non-specific alkaline phosphatase enzyme. b. deficiency in this enzyme affects the development and mineralization of bone and teeth c. premature primary tooth exfoliation d. severe destruction of the periodontium of permanent teeth

Nitrous oxide sedation of a patient should always be concluded with the administration of 100% oxygen to prevent... A. Hypercarbia B. Hyperventilation C. Hypoxia D. Tachycardia E. Hypotension

C. Hypoxia 1) Hypoxia a. Administering oxygen immediately after nitrous oxide sedation increases the concentration of oxygen in the blood thereby diminishing the occurrence of hypoxia. b. Hypoxia may occur with patients sedated with nitrous oxide with the gas mixture administered has a low oxygen concentration. c. After being sedated with nitrous oxide, 100% oxygen is administered to completely clear the nitrous oxide in the system and help the patient recover faster from sedation.

Paraympathetic innerviation is carried by which of the following cranial nerves? A. II, V, IX, X B. III, V, VII, IX C. III, VII, IX, X D. IV, VII, VIII, X E. II, III, VII, IX

C. III, VII, IX, X 1) CN III = occulomotor nerve a. constricts the pupil 2) CN VII = facial nerve a. innervations to the lacrimal gland as well as the sublingual gland and submandibular salivary glands 3) CN IX = glossopharyngeal nerve a. innervate the parotid salivary gland 4) CN X = vagus nerve a. innervation to the digestive organs and the heart

Bone marrow grafts for jaw defects are typically harvested from which of the following sites? A. A cadeaver B. A rib C. Illiac crest D. Mandibular parasymphysis E. Maxillary tuberosity

C. Illiac crest 1) Iliac crest a. most suitable location for harvesting grafts for jaw defects because of its large size as well as the fact that it contains both cortical as wells the cancellous bone 2) Cortical bone a. present on the outside while cancellous bone is located inside b. best for load bearing areas 3) Cancellous bone a. has a rich blood supple b. high surface area

Which of the following terms describes the rate at which new disease occurs? B. Extent C. Incidence E. Prevalence

C. Incidence

A patient presents with an odontogenic infection evidence by a temperature of 102oF and fluctuant mass in their submandibular space. Which of the following is indicated course of treatment? A. Application of heat and cold to the area to improve circulation B. Aspirin should be administered to decrease the temperature C. Incision and drainage and an antibiotic sensitivity test D. Administration of penicillin Vk to reduce the swelling only

C. Incision and drainage and an antibiotic sensitivity test 1) Concept: The increased number of bacteria built-up by the infection results to its spread into the submandibular space causing the fever of the patient. 2) Incision and drainage: a. allows the accumulated and pressure that build up within the submandibular space to be removed 3) Antibiotic sensitivity testing a. helps to determine the putative etiologic bacteria and guide the clinician to the appropriate antibiotic treatment

Incremental placement for a Class II prep leads to all of the following EXCEPT one. Which is the EXCEPTION? A. Decreased marginal deterioration B. Enhanced marginal adaptation C. Increased post-op sensitivity D. Reduced marginal leakage

C. Increased post-op sensitivity 1) Concept: Increased post-operative sensitivity is NOT among the recognized effects following incremental placement for a Class II preparation because incremental placement of composite resins decrease the polymerization shrinkage of composites and increases the percentage of curing resulting in decreased postoperative sensitivity. 2) Incremental placement also enhances marginal adaptation, decreased marginal deterioration, and reduces marginal leakage of the restoration.

Each of the following is consider a disadvantage of Microfilled Resin Based Composite compared to Hybrid-filled Composite EXCEPT one. Which is the EXCEPTION? A. Decreased tensile strength B. Increased polymerization shrinkage C. Increased stiffness D. Increased thermal expansion E. Increased water absorption

C. Increased stiffness 1) MicroFilled Composites Disadvantages a. Increased thermal expansion b. Decreased tensile strength c. Increased water absorption d. Increased polymerization shrinkage e. allow the restoration to bend with tooth flexure, better protecting the bonding interface f. has greater flexibility and low modulus of elasticity may perform better in certain Class V restorations than a more rigid hybrid composite

A longer gray scale of contrast can be achieved in a radiograph by: A. Decreasing the filtration B. Increasing the focal spot-skin distance C. Increasing the kilovoltage (kV) D. Increasing the milliamperage (mA)

C. Increasing the kilovoltage (kV) 1) Increasing the kV causes the radiographic image to have low contrast and longer gray scales 2) Low kV creates low energy x-rays responsible for producing high contrast and shorter gray scales.

Which of the following describes the proper progression of stages of the pathogenesis of periodontal lesions? B. Incipient, early, established, and advanced C. Initial, early, established, and advanced

C. Initial, early, established, and advanced 1) Initial lesion a. presence of acute inflammatory reaction as the normal healthy gingiva reacts to plaque accumulation 2) Early lesion a. occur when an infiltrate of lymphoid cells, particularly T lymphocytes appear on the site of injury of accumulation 3) Establish lesion a. B lymphocytes and plasma cells suddenly predominates the site of inflammation 4) Advance lesion a. manifests itself a periodontitis and physiologically irreversible loss of bone

Tooth fusion or gemination occurs during which of the following stages of tooth development? A. Calcification B. Eruption C. Initiation/Proliferation D. Maturation E. Apposition

C. Initiation/Proliferation 1) Fusion a. a dental abnormality that occurs during the initiation stage of tooth development where two different tooth buds unite to form one tooth b. involves the mandibular anterior teeth, particularly the lateral incisors and canines

Pulpal necrosis is the MOST COMMON outcome of which primary tooth trauma? A. Concussion B. Mid-root horizontal root fracture C. Intrusive luxation D. Avulsion

C. Intrusive luxation 1) Concept: Intrusive Luxation a. Intrusive luxation of primary teeth among children always result to pulpal necrosis b. occurs when the tooth is pushed into the socket, directly damaging and severing the vascular supply of the tooth, immediately results to pulpal necrosis 2) Avulsion a. avulsed primary tooth will have a better chance of revascularization when implanted immediately and will not always result in pulp necrosis b. If the tooth is replanted after a long period of time, the chances of ankylosis will increase.

How does etching tooth structure with acid improve retention of composite restorations? Select all that apply. A. It displaces the water in dentinal tubules and allows composite to flow deeper into dentin B. It increases the chemical bonding strength C. It increases the mechanic retention D. It removes the surface debris from the cavity preparation.

C. It increases the mechanic retention D. It removes the surface debris from the cavity preparation. 1) Concept: Acid etching removes debris and other impurities from the cavity that may affect the overall result of the restorative procedure. 2) Acid etching a. provides surface roughness to provide mechanical retention without too much loss of healthy tooth structure during cavity preparation b. microporosities created by acid etching allows even small cavity preparations to have rendition despite insufficient retention form c. Acid etching improves MECHANICAL retention NOT CHEMICAL bonding.

Which of the following describes Verrill's sign in regards to IV conscious sedation using diazepam? A. It is not recommended since few patients are adequately sedates at that level B. It is not recommended since it can indicate a too-deeply sedated patient C. It is recommended as an end-point D. It is usually not attainable with diazepam alone

C. It is recommended as an end-point 1) Verill's sign a. during IV sedation helps the anesthesiologist determine the depth and adequacy of sedation b. helps in determining a safe limit in administering the IV conscious sedation drug such as diazepam c. is similar to Ptosis, a condition where there is drooping of the upper or lower eyelid d. the level of sedation can be evaluated by checking several parameters including the Verill's sign or the dropping of the eyelid which signified the effect of anesthesia to the muscles that controls the eyelid e. presence of Verill's sign indicates that the anesthesia needed for the procedure is already adequate and the safely limit has already been reached.

Of the following pathologies, which disease may modify the inflammatory process? A. Hemophilia B B. Hypoparathyroidism C. Leukemia D. Sickle cell anemia E. Hemophilia A

C. Leukemia 1) Concept: Leukemia a. type of cancer of the blood or bone marrow characterized by an abnormal increase of immature white blood cells b. white blood cells or leukocytes are involved in the immune response of the body to infection and inflammation c. Due to the central role of leukocytes in the development and propagation of inflammation, defects in leukocyte function often result in a decreased capacity for inflammatory defense with subsequent vulnerability to infection. 2) Hemophilia a. describes an abnormality in the clotting pathway, which does not interfere with the inflammatory process 3) Sickle cell anemia a. is a defect or red blood cells where they are unable to carry oxygen as efficiently, and has no effect on the inflammatory process

Each of the following would be visible on a periodical radiograph EXCEPT one. Which is the EXCEPTION? A. Calcification of a canal B. Canals of a maxillary 1st premolar C. Lingual curvature of the palatal root D. Open apex of a tooth E. Perforation of the furcation

C. Lingual curvature of the palatal root 1) Root Radiographs a. illustrate a 2-dimensional mesiodistal view of the tooth and surrounding structures, therefore the buccolingual aspects are not able to be portrayed b. canal that suddenly disappears midroot, or appears off center is often the result of the tooth possessing 2 canals c. open apices and furcation perforations can also be seen radiographically

Dental indications for systemic antibiotics include each of the following EXCEPT one. Which is NOT an indication for systemic antibiotics? A. Extraction of a mandibular 3rd molar in a patient with acute pericoronitis B. Full mouth extractions for a patient with severe periodontal disease C. Localized alveolar osteitis following mandibular 3rd molar extractions D. Extraction of a tooth for a patient with an acute periodical abscess

C. Localized alveolar osteitis following mandibular 3rd molar extractions 1) Concept: Localized alveolar osteitis (dry socket) does not necessitate the use of systemic antibiotics 2) Dry socket is a painful condition that usually occurs after a traumatic extraction and accidental early dislodgment of the blood clot inside the extraction site. 3) Alveolar osteitis is treated by rinsing the affected tooth socket with warm saline solution and placement of a sponge with an antiseptic dressing.

An amalgam preparation outline for a primar mandibular 2nd molar is MOST similar to which of the following permanent teeth? A. Mandibular 1st premolar B. Maxillary 2nd molar C. Mandibular 1st molar D. Mandibular 2nd premolar

C. Mandibular 1st molar 1) Concept: The outline for an amalgam preparation for a primary mandibular 2nd molar will be almost identical to the permanent mandibular 1st molar tooth except slightly smaller. 2) The primary 2nd molar and permanent 1st molar are nearly identical in tooth morphology. 3) The primary 2nd molar typically demonstrates 5 cusps like the permanent 1st molar. 2) Mandibular Molars: Primary 2nd = Permanent 1st

Which of the following muscles is usually involved in myositis? A. Digastric B. Lateral pterygoid C. Masseter D. Mylohyoid

C. Masseter 1) Concept: Mositis is a general term for the inflammation of the muscles 2) The masseter muscle is the most commonly inflamed facial uncle. 3) Myositis is also a documented side effect of lipid-lowering drugs like statins and vibrates.

The cusp of Carabellis is most commonly found on which primary tooth? A. Mandibular 1st molar B. Mandibular 2nd molar C. Maxillary 2nd molar D. Maxillary 1st molar

C. Maxillary 2nd molar 1) Concept: The cusp of Carabellis is a feature of the primary maxillary 2nd molar. 2) It is usually positioned at the mesiolingual cusp of the primary 2nd molar which could appear as a small cusp or a triangular protrusion. 3) The cusp of Carabelli is most common among Europeans. 4) In PERMANENT dentition, the cusp of Carabelli is associated with the maxillary 1st molar.

At which of the following sites is an apically repositioned flap procedure generally impossible to perform? A. Mandibular lingual B. Maxillary facial C. Maxillary Palatal D. Mandibular facial

C. Maxillary Palatal 1) Thickness of the gingival epithelium and its supporting attached gingiva make apically repositioned flap procedures impossible for the maxillary palatal area. 2) Attached gingiva in the palatal area is difficult to differentiate with the palatal mucosa because there is an unclear demarcation between the two.

Which tooth is known for having the most consistent root canal morphology out of the following choices? C. Maxillary canines E. Maxillary incisors

C. Maxillary canines 1) Maxillary canine a. longest root in the oral cavity b. root is relatively straight with only some slight palatal or labial tipping along its apex. c. presents with only one root canal most of the time d. diameter is relatively large and oval in shape when viewed in cross-section

The pulpal anatomy of which of the following teeth means that the endodontic access should be triangular in shape? A. Mandibular 2nd premolar B. Maxillary canine C. Maxillary central incisor D. Mandibular canine

C. Maxillary central incisor 1) Maxillary Central Incisor Access a. access preparation is triangular, following the shape of its pulp chamber b. sides of the inverted triangle are guided by the presence of a marginal ridge that border the access opening c. access preparation is designed to provide a straight line view of the canal up to the apex if possible

Alec is hyperactive child that has trouble paying attention, which medication might be prescribed to help him? A. Alpraxolam B. Clonadine C. Methylphenidate D. Doxapram

C. Methylphenidate 1) Mehylphenidate a. psychostimulant approved in treating people with attention-deficit hyperactivity (ADHD) b. reduces the dopamine and norepinephrine reuptake, increasing their levels thereby improving their function as neurotransmitters in the brain. c. similar pharmacological effect to cocaine but lower potency and the duration is longer

The most widely used method of removing or disrupting bacterial plaques in the mouth is which of the following? A. Antibiotics (penicillin, tetracyclines) B. Chemical (mouthwashes) C. Mechanical (Brush and floss) D. Using a surfactant to prevent bacteria from adhering to teeth

C. Mechanical (Brush and floss) 1) Concept: Brushing and flossing a. Mechanical disruption through brushing and flossing remains the best method of stopping the growth of plaque and other bacterial pathogens in the mouth b. the tooth pellicle is where initial bacteria colonizes. By disrupting the newly formed plaque, calculus, and dental caries can be prevented. 2) Antibiotics a. wide spectrum medications are intended to kill bacteria b. however, it is not intended for plaque control c. excessive use of antibiotics will NOT prevent plaque formation but may lead to bacterial death in the oral cavity. This will then cause imbalance in the normal flora of our mouth and allow fungi like candida to take over.

Which of the following directions would a mandibular 2nd molar move if the 1st molar was extracted? A. Distally B. Lingually C. Mesially D. None E. Buccally

C. Mesially 1) Concept: Teeth may tend to move into an edentulous area recently occupied by an adjacent tooth. 2) Edentulous spaces exhibit thinner bone support, so the adjacent 2nd molar may tilt or move medially into the space.

Which of the following directions would a mandibular 2nd molar move if the 1st molar was extracted? A. Distally B. Lingually C. Mesially D. Buccally

C. Mesially 1) Concept: Teeth may tend to move into an edentulous area recently occupied by an adjacent tooth. 2) Edentulous spaces exhibit thinner bone support, so the adjacent 2nd molar may tilt or move medially into the space. 3) Teeth tend to move medially towards the midline, especially when the adjacent tooth is missing.

Which of the following materials will provide the MOST ESTHETIC restoration for a Class V preparation on an anterior tooth? A. Glass ionomer B. Hybrid resin composite C. Microfilled resin composite D. Resin based glass ionomer

C. Microfilled resin composite 1) A Class V preparation should be restored with micro filled resin composite material whenever esthetics is the main concern. 2) Due to their small particle size, microfilmed resin composites have a smooth, highly polishable surface which requires minimal finishing. 3) The low elastic modulus of these materials allows them to flex during tooth flexure, better protecting the bonding interface in Class V restorations. 4) Microfilled resins are NOT typically used for restorations that are going to be in contact with other teeth because they have a low resistance to wear.

Which of the following describes the primary method for retention to enamel by adhesives? A. Dentin tubules B. Macrotags C. Microtags D. Primer application E. Smear layer

C. Microtags 1) Concept: Microtags are believed to be the major contributors to retention to enamel with regards to adhesives. 2) The acid treated enamel surface has a high surface energy allowing the flow, intimate adherence and polymerization of the resin monomer within the pores, creating retentive resin micro tags up to 20 micrometers deep. 3) At the same time, the individual cores of all the exposed individual crystals are solubilized to form a multitude of micro porosities. 4) These countless numbers of mini tags or micro tags formed within the individual crystal cores are major contributors to enamel bond strength.

Which of the following is the MOST significant disadvantage of glass ionomer cements? A. Difficult preparation B. Low bond strength to dentin C. Moisture sensitivity during initial curing D. Pulpal irritation

C. Moisture sensitivity during initial curing 1) Glass-ionomer cements a. set through acid-base reaction b. water loss and moisture contamination must be prevented in order to obtain satisfactory physical properties c. desiccation prevents formation of the polycarboxylate matrix, resulting in a rough chalky surface and decreased surface hardness d. during the latter part of the reaction, water is essential in hydrating the cross link of the polymer matrix to strengthen the cement

Replacement of defective amalgam with a cast gold restoration might be indicated to achieve... C. More ideal contours D. Better marginal seal

C. More ideal contours 1) Cast gold material provides superior marginal sealing and physical properties which makes it an ideal material for recreating anatomic contours. 2) Cast gold material can be re-adapted and burnished at the finish lines of the cavity preparation to ensure a tiger fit and lesser marginal leakage.

A clinical examination discloses a soft, fluctuant, tender swelling in the midline of the hard palate. All maxillary teeth test vital. Radiographs shows a radiolucency between the roots of the maxillary central incisors, which of the following cysts is the most likely diagnosis? A. Incisive papilla B. Median palatal C. Nasopalatine duct D. Periapical E. Nasolabial

C. Nasopalatine duct 1) Nasopalatine duct cyst a. hard shaped radiolucency often found in the median aspect of the hard palate anterior to the 1st molars b. palatal swellings that are asymptomatic c. considered the most prominent nonodontogenic cyst within the oral cavity

For which of the following conditions is the mucosa of the hard palate the most common location of occurrence? A. Lichen planus B. Mucocele C. Necrotizing sialometaplasia D. Sialothiasis E. Apthous ulcer

C. Necrotizing sialometaplasia 1) Concept: Necrotizing sialometaplasia a. rapidly expanding ulcerative lesion that mostly occurs on the posterior area of the hard palate b. usually benign and is often painless, and is usually self-limiting and resolves in about six to ten weeks. c. trauma to the minor salivary glands located in the hard palate causes this condition

A 41 year old female from your practice resorts, developing a symptomatic, rapidly expanding ulcer on her right lateral hard palate that appears crater-like and measures 35mm in diameter. What is the most likely diagnosis? A. Actiomycycosis B. Adenoid cystic carcinoma C. Necrotizing sialometaplasia D. Pleomorphic adenoma E. Squamous cell carcinoma

C. Necrotizing sialometaplasia 1) Concept: Necrotizing sialometaplasia is a rapidly expanding ulcerative lesion that mostly occurs on the posterior area of the hard palate. 2) Necrotizing sialometaplasia a. usually benign and is often painless, and is usually self-limiting an resolves in about six to ten weeks. b. trauma to the minor salivary glands located in the hard palate causes this condition

Which of the following conditions would be best treated by antibiotics? A. Angina pectoris B. Herpangina C. Necrotizing ulcerative gingivitis D. Recurrent aphthous stomatitis E. Verruca Vulgaris

C. Necrotizing ulcerative gingivitis 1) Concept: According to the microbial basis of ANUG, antibiotics seem to be a logical treatment option. 2) Penicillin and metronidazole have been used for treating ANUG. 3) Latest treatment of ANUG involves periodic scaling and root planing accompanied by topical chlorhexidine gluconate.

A 19 year old patient presents with oral pain, erythematous gingival tissues, blunted papillae, spontaneous gingival bleeding, and no attachment loss. There is also a pseudomembrane covering the gingiva. Which of the following is the diagnosis for this patient? C. Necrotizing ulcerative gingivitis D. Necrotizing ulcerative periodontitis E. Plaque associated gingivitis

C. Necrotizing ulcerative gingivitis 1) Necrotizing Ulcerative Gingivitis a. predominance of the anaerobic fusobacteria and spirochetes within the oral cavity, specifically underneath the gingiva. 2) Treatment a. debridement of the affected gingiva b. chlorhexidine 0.12% 3) Acute Necrotizing Gingivitis a. pain b. interproximal necrosis of the papilla (blunting of papilla) c. bleeding of the gingiva d. fetid odor e. low-grade fever f. pseudomembrane

The most common postoperative concern following bilateral sagittal split osteotomy procedures is... A. Devitalization of mandibular dentition B. Infection C. Neurosensory disturbances D. TMJ pain E. Periodontal defects

C. Neurosensory disturbances 1) Bilateral sagittal split osteotomy (BSSO) a. orthognathic surgical procedure performed by splitting the lower jaw bilaterally to reposition it in a more anterior or posterior position and achieve better function and esthetics b. high risk of having neurosensory disturbances like prolonged numbness of the chin and lips while others experience either extreme sensitivity or reduce sensitivity to touch

Which cell is least associated with the chronic periodontitis? A. All answers apply B. B lymphocytes C. Neutrophils D. Plasma cells E. T lymphocytes

C. Neutrophils 1) Neutrophils (PMNs) are phagocytic cells that are the first white blood cells to arrive at infect and injured tissues. 2) Chronic infections involve a. macrophages b. lymphocytes c. plasma cells 3) B lymphocytes become activated and mature into plasma cells which produce antibodies 4) Purulence (pus) is composed of dead and dying neutrophils.

A 50 year-old man presents to your office for a periodic exam smelling of tobacco pipe smoke. Upon examination, you notice a raised white lesion covering most of his hard palate and many red papules within the white lesion. Which of the following is the most likely diagnosis? A. Hairy Leukoplakia B. Lichen planus C. Nicotonic stomatitis D. Papillary hyperplasia E. Leukoplakia

C. Nicotonic stomatitis 1) Concept: Nicotinic stomatitis a. condition that manifests on the hard palate as a white surface lesion associated with pipe and cigar smoking and is not considered to be premalignant b. is induced by the palate being exposed to very hot conditions, so it is also associated with chronic ingestion of hot beverages c. found most commonly in men over 45 years old d. should completely resolve on its own after 1-2 weeks upon smoking cessation. If the lesions persists, a biopsy may be necessary to confirm the diagnosis. e. Immediate cessation of smoking may be advised in order to monitor lesion and see if differential diagnoses are required f. there is usually no treatment necessary since cancer risk is not elevated 2) The palate may appear gray or white and contain many papules that are slightly elevated with red in their center.

A 35 year old patient presents to your office with unrestored teeth #1-#32 with staining of the occlusal pits and grooves without a detectable catch with the explorer. What is the best treatment option... A. Bis-GMA sealant B. Conservative class I amalgam fillings C. No treatment D. Preventive resin restoration E. Topical fluoride placement F. Cyanoacrylate sealant

C. No treatment 1) Presence of stains within the occlusal pits and grooves of a sound tooth 2) Arrested Caries a. dark stains observed in pits and fissures of teeth without any detectable catch is arrested caries, which occur when a formerly caries affected area of tooth remineralized, stopping the caries process. 3) Incipient Caries a. is distinguished by a chalky white appearance on the smooth surfaces of teeth 4) Active Caries a. distinguished during an oral examination as an explorer catch with soft, easy to flake off tooth structure

Which of the following best describes adjunctive orthodontic treatment? A. Early intervention using orthodontic therapy to prevent a more serious future malocclusion B. Orthodontic therapies completed with only simple removable appliances C. Orthodontic treatment to enhance the outcome of periodontal, prosthodontic, or operative therapies D. Limited orthodontic treatment of only anterior teeth for esthetics only

C. Orthodontic treatment to enhance the outcome of periodontal, prosthodontic, or operative therapies 1) Concept: Adjunctive orthodontic treatment integrates favorable orthodontic tooth movement into a treatment plan together with other dental procedures. 2) Orthodontic treatment is completely simultaneously with other dental procedures to improve the result of the overall treatment procedure. 3) Adjunctive orthodontics also eliminates unnecessary tooth attractions, tooth preparations, and other periodontal, pre-prosthetic, or pre-restorative treatment to achieve more optimal outcomes.

Which of the following types of parenting styles usually leads to children that are shy and submissive? A. Empathetic B. Overindulgent C. Overprotective D. Under-affectionate E. Under-attentive

C. Overprotective 1) Overprotective parents a. overprotective nature of parents leads to shyness in children b. show submissive behavior in society and their social circle c. personality development of these children remains incomplete d. produce a lack of confidence in their children which results a permanent psychological trauma in them which they carry with them forever

A 43 year old male fractures at the distoincisal point angle of #9. The facial of the tooth has previously been restored with composite resin, from the site of the fracture, you can see that the layer of composite is very thin and you assume that the remaining strength is questionable. The patient presents a restoration that would provide the best longevity. B. MID composite resin with composipin retention C. PFM crown

C. PFM crown 1) Porcelain fused to metal (PFM) crowns a. are indicated for teeth where the remaining tooth structure is compromised in strength 2) Pin-retained amalgam a. not a good option since it is anesthetically pleasing and pin placement may harm the pulp 3) 3/4 cast gold crown a. good treatment for damaged tooth structure but technique sensitive, and too anesthetic for this case 4) Composite resin restoration with a composipin a. would have good retention b. strength of remaining structure makes this option contraindicated

Which of the following is the MOST COMMON location that Kaposi's sarcoma lesions occur in AIDS patients? A. Floor of the mouth B. Lower Lip C. Palate D. Tongue E. Buccal mucosa

C. Palate 1) Concept: The hard palate is most frequently affected, followed by the gums. 2) Kaposi Sarcoma (KS) a. tumor caused by the human herpesvirus 8 that often appears in patients suffering from AIDS b. The mouth is involved in about 30% of cases, and is the initial state in 15% of AIDS-related Kaposi Sarcoma. c. Lesions in the mouth may be easily damaged by chewing and bleed or suffer secondary infection, and even interfere with eating or speaking. 1) Lesions manifest as macular, patches, or plaques, that are nodular, and exophytic 2) The cutaneous lesions can be solitary, localized, or disseminated 3) Kaposi Sarcoma can involve the oral cavity, lymph nodes and viscera 4) Kaposi Sarcoma diagnoses can be made through tissue biopsy

Which of the following describes the BEST approach a dentist should use to elevate a flap to prevent exposure of a dehiscence or fenestration on a prominent root? A. Apically repositioned flap B. Full thickness flap C. Partial or split thickness flap D. Stripping procedure

C. Partial or split thickness flap 1) Concept: Using a partial thickness flap, a dentist may utilize use the elevate flap to cover up the dehiscence or fenestration of a root. 2) Partial thickness flaps a. enable the repositioning of the elevated flap without exposing the bone. b. With partial thickness flaps, the periosteum is NOT included in the elevated flap and serves to cover the bone and tooth root. 3) Apically repositioned flap a. in this case will only expose more bone and tooth structure and is NOT a good option for teeth with dehiscence and fenestrations 4) Full thickness flaps are NOT a good choice since they will expose the whole bone surface including the dehiscence and fenestration of tooth.

A21-year-old hemophiliac male presents to your office with a necrotic pulp and periodical radiolucency associated with tooth #14. Which of the following is the best treatment for this patient? A. Debride the canals and place calcium hydroxide in the canals and place a permanent restoration. B. Instrument and obturate the canals to mid root level in order to avoid vascular trauma C. Perform a non-surgical root canal treatment D. Perform a non-surgical root canal treatment with systemic vitamin K administration E. Extract #14

C. Perform a non-surgical root canal treatment 1) Hemophiliac Patient need Endodontic Procedure a. root canal treatment is documented to have a low risk intervention for hemophiliac patients. b. completely safe to carry out non-surgical endodontic treatment in this patient, which is indicated for treatment of necrotic pulp with apical periodontitis c. Vitamin K does not affect hemophilia A and B. d. can be treated by addition of the missing clotting factors

Each of the following is present in a healthy pulp EXCEPT one. Which is the EXCEPTION? A. Cell-free zone B. Odontoblastic layer C. Plasma cells D. Undifferentiated mesenchymal cells

C. Plasma cells 1) Concept: A healthy pulp does NOT contain plasma cells because they are stimulated to migrate to the pulp with inflammation and infection. 2) The main components of the pulp are a. Neurovasculature cells b. Undifferentiated mesenchymal cells c. Odontoblastic layer d. Cell free zone

Which of the following should be performed to test the occlusion of an amalgam restoration? A. Instruct the patient to bite down and report how it feels B. Instruct the patient to bite on articulating paper C. Perform the tap test with articulating paper by manually moving the patient's jaw D. The restoration will be correct if it was carved to typical occlusal anatomy

C. Perform the tap test with articulating paper by manually moving the patient's jaw 1) Concept: The tap test is usually performed to test the occlusion of an amalgam restoration. It is performed with the articulating paper by moving the patient's jaw side to side and up and down. 2) It is advised that the occlusion of the restoration should be evaluated. A piece of articulating paper is placed over the restoration and the patient is instructed to close gently into occlusion and manual movement of jaws is carried out. 3) Any high area of contact can be evaluated by the depth of color imparted by the pear and especially if it has a silvery centre. These deep colored or shiny centered areas are reduced and adjusted. 4) Instructing the patient to bite hard on the paper can cause fracture of the restoration.

Which of the following teeth typically drift mesially? A. Mandibular 2nd premolars B. Permanent mandibular central incisors C. Permanent maxillary 2nd molars D. Primary mandibular canines E. Primary maxillary 2nd molars

C. Permanent maxillary 2nd molars 1) Concept: Permanent maxillary 2nd molars typically drift medially following the loss of the permanent maxillary 1st molar 2) The permanent maxillary 1st molars are among the first erupting teeth in the oral cavity. Permanent maxillary 1st molars often decay and become unrestorable, or their extraction is planned by an orthodontist to relieve crowding. 3) In such cases, permanent maxillary 2nd molars have a tendency to drift medially and take the space of the permanent maxillary 1st molars. These teen quickly establish good contact with 2nd premolars. 4) Mesial drift principles a. when a tooth is extracted, the neighboring teeth will typically drift to the extraction space b. mesial drift movement of the unerupted teeth (within the bone) is similar to the bodily movement experienced during orthodontics, especially in the upper arch c. due to the long axis of the teeth, the spontaneous closure of the extraction space in the upper arch is better than the lower arch d. timing of extraction is important

Gingival hypertrophy, coarse facial features, increased hair growth are potential side effects of which drug? A. Carbamazipine B. Hydrochlorothiazide C. Phenytoin D. Venlafaxine E. Valproic acid

C. Phenytoin 1) Phenytoin a. antiepileptic agent that may induce gingival hypertrophy, increased hair growth, and coarse facial features b. used to treat trigeminal neuralgia if carbamazipine is contraindicated 2) Valproic acid a. anticonvulsant b. mood-stabilizing drug c. major depression 3) Venlafaxine (Effexor) a. serotonin-norepinephrine reuptake inhibitor (SNRI) used to treat depression

Each of the following describe the modes of action for cavitrons EXCEPT one. Which is the EXCEPTION? A. Cavitation B. Lavage C. Piezo movement D. Vibration

C. Piezo movement 1) Concept: Ultrasonic instruments (cavitrons) move in a rapid pulsating manner that creates vibration, cavitation, and rinsing water used to cool and lavage tissues. b. Ultrasonic dental scalers use vibration and pulsation in order to break down the mineralized plaque. c. Ultrasonic cleaners produce cavitation, where rapid movement creates tiny vacuum bubbles in the liquid which burst once they comes in contact with contaminants. This reaction is able to scrub particles of bacteria and calculus off of tooth surfaces. d. Ultrasonic lavage action is used for hard to reach areas like endodontic canals. 2) Piezo movement uses different mechanism of action (forward and backward linear motion instead of ovoid movement). b. Piezo instruments can create ultrasonic movements.

You break a size of 35 K-file 1mm short of the apex in the MB1 canal of tooth #15 that was diagnosed with irreversible pulpits. The remaining file can neither be retrieved nor by-passed. It is indicated that the dentist finishes the root canal treatment and does which of the following? A. Extract the tooth B. Perform an apicoectomy of the mesiobuccal root C. Place the patient on a recall schedule for further evaluation D. Amputate the mesiobuccal root

C. Place the patient on a recall schedule for further evaluation 1) Broken file a. broken file inside poses some risks of reinfection due to the incomplete sealing of the canal b. process of instrumentation, cleaning and shaping is compromised because of the broken file, which may result in a failure of the root canal treatment later. c. unretrieved broken file may stay inside the mouth if the tooth is asymptomatic and if a radiographic lesion shows evidence of healing

When performing incisional biopsies of soft tissue, the clinician should do each of the following EXCEPT one. Which is the EXCEPTION? A. Obtain some adjacent normal tissue when possible B. Place a suture through the specimen when removing it. C. Place the specimen in saline if 10% formalin is not available D. Infiltrate local anesthetic around the intended site

C. Place the specimen in saline if 10% formalin is not available 1) Concept: Saline cannot be used as a fixative for the incisional biopsy specimen 2) Incisional biopsy is a procedure that removes a portion of a lesion for laboratory examinations in cases when complete excision is contraindicated. a. are performed on lesions > 10mm in size 3) Placing a suture in the specimen is done to localize the areas to be included in the incision and prevent contamination of the specimen

Which of the following describes the MOST EFFECTIVE method for managing post surgical root sensitivity after periodontal surgery is performed? A. Application of calcium hydroxide B. Application of mineral trioxide aggregate (MTA) C. Plaque control D. Using a desensitizing dentifrice

C. Plaque control 1) Concept: The best way to manage post surgical root sensitivity is to address the central etiology (dental plaque). The acidic metabolites from plaque may cause sensitivity to the exposed root surface after periodontal surgery. Desensitizing dentifrice may provide a temporary relief from root sensitivity but takes up to 2 weeks to take effect. Adequate removal of plaque along the tooth surface will provide a MORE permanent solution to sensitivity. Mineral trioxide aggregates is NOT indicated for addressing root sensitivity but is rather used as a filling material for endodontic treatment.

Which of the following is the MOST PROBABLE diagnosis for a 45-year old patient who complains of a unilateral facial burning and aching pain, intermittent paresthesia and itching, and small white scars in his pre-auricular area. A. Bell's palsy B. Burning mouth syndrome C. Post-herpetic neuralgia D. Psychoneurosis E. Auriculo-temporal syndrome

C. Post-herpetic neuralgia 1) Concept: The most probable diagnosis for these conditions is post-herpetic neuralgia b. It is characterized by nerve pain due to the damage caused by Varicella Zoster virus. c. It is commonly restricted to a dermatome of the skins and follows a Varicella Zoster infection in the same dermatome. d. Post-herpetic neuralgia presents with 1) Possible paresthesia and itching 2) Discomfort to severe pain which can be described as burning or stabbing 3) Persistence of pain for 3+ months 4) Cutaneous scarring may be present

Each of the following statements about post preparations is CORRECT EXCEPT one. Which is the EXCEPTION? A. It is recommended that at least 5mm of gutta percha should remain in the root canal after the post space is prepared B. Posts are indicated when there is inadequate coronal tooth structure for fixed restorations C. Posts reinforce the tooth structure and prevent vertical root fractures D. Posts retain the core for teeth with extensive coronal tooth structure loss

C. Posts reinforce the tooth structure and prevent vertical root fractures 1) Posts a. function of posts are to retain the core, they DO NOT reinforce the tooth b. typically weaken the tooth because the removal of dentin causes stress and further thins the walls of the root, predisposing the root to fracture c. Rule of thumb for post preparation is that you prepared the canals so that 1/2 of the root has gutta percha remaining (or about 5mm+)

A patient calls your office complaining of thermal and masticatory sensitivity following placement of a large MOD restoration yesterday on #20. What most likely explains the etiology of your patient's symptoms? A. Gingival recession B. Microleakage C. Premature occlusion D. Recurrent caries

C. Premature occlusion 1) Masticatory sensitivity a. apical portion of the periodontal ligament experiences excessive pressure or apical periodontal tissue destruction b. vital tooth that recently experienced an excessively high occlusal restoration will exhibit masticatory sensitivity due to being the first tooth to receive the heavy occlusal force during mastication c. premature contact is an area of stress concentration that is transmitted to the periodontal ligament which can cause dental trauma and masticatory sensitivity.

The most significant factor in IATROGENIC pulpal damage is... A. Bacterial invasion B. Heat C. Proximity of preparation to the pulp D. Desiccation

C. Proximity of preparation to the pulp 1) Pulpal damage a. avoided by maintaining dentin thickness is 2mm or more b. number and diameter of dentinal tubules increases with closer proximity toward the pulp, so the dentin becomes more permeable and easily damaged. c. dentin has average thickness of 3mm d. a minimum remaining dentin thickness of 2mm can help prevent pulpal damage e. dentin thickness of 0.75mm, pulpal reaction to increased bacterial load is observed f. dentin thickness of 0.25mm, loss of the odontoblasts is observed g. Other sources of pulpal injury 1) bacteria 2) heat 3) chemical irritants

The diagnosis of which of the following conditions is an oral cytology smear MOST indicated? A. Mucous membrane pemphigoid B. Papilloma C. Pseudomembraneous candidiosis D. Squamous cell carcinoma E. Lichen planus

C. Pseudomembraneous candidiosis 1) Concept: Definitive diagnosis of candidosis requires specialized tests such as a cytologic smear. b. The membranous slough of pseudomembranous candidosis can often times be wiped off with gauze to reveal an erythematous surface underneath. c. The clinical appearance of pseudomembranous candidosis alone is sufficient for the diagnosis of candidosis in most cases. d. Smears are collected by gentle scraping of the lesion with a spatula or tongue blade with the resulting debris being directly applied to a glass slide for a cytopathological exam.

For a dentist to perform a block of the maxillary division of the trigeminal nerve (V2), what anatomical space is the anesthetic injected? A. Buccal space B. Masseteric space C. Pterygopalatine space D. Retropharyngeal space E. Pterygomandibular space

C. Pterygopalatine space 1) From the foramen rotundum, the maxillary division of the trigeminal nerve (V2) passes through the pterygopalatine fossa 2) During the maxillary nerve (V2) block, the needle is bent at a 45 degree angle to facilitate insertion into the pterygopalatine space located medially and posterior to the maxillary 2nd molar. 3) The needle is inserted medially into the pterygopalatine fossa and about 0.9 to 1.8cc of anesthesia is deposited to ensure efficacy of the maxillary nerve block technique.

A 5-year-old presents to your office complaining of spontaneous pain associated with their mandibular 2nd molar. Clinical testing suggest that the pulp is necrotic, and radiographs suggest NO periodical pathosis. What is the best treatment option of this child? A. Direct pulp cap B. Indirect pulp cap C. Pulpectomy D. Pulpotomy E. Extraction

C. Pulpectomy 1) Pulpectomy a. The spontaneous pain from necrotic pulp signifies the presence of infection inside the primary tooth which can only be treated with a pulpectomy procedure b. involves the complete removal of necrotic or partially necrotic pulp and placement of medicaments inside the cavity in order to avoid premature extraction of teeth c. indicated for primary teeth with infection and inflammation that has already reach beyond the coronal aspect of the pulp d. teeth indicated for pulpectomy must also exhibit no pathology resorption along the alveolar bone and its roots

A 6 year-old boy presents with a carious lesion on tooth T and no mobility, a small draining sinus tract is found adjacent to tooth T. Radiographs indicate no furcal bone loss. Which of the following is the suggested treatment? A. Extraction C. Pulpectomy

C. Pulpectomy 1) Pulpectomy a. indicated for cases where there is no tooth mobility but the infection might already have reached the pulp 2) Pulpotomy a. indicated for primary teeth having accidental pulp exposure or a bacterial attack that only affects the coronal portion of the pulp

A 5 year-old presents to your office with spontaneous pain in their maxillary 1st molar and is responsive to cold. The radiograph shows an intact furcation and caries extending to the pulp chamber. What is the treatment of choice? A. Direct Pulp cap with Zinc-Oxide Eugenol B. Extraction C. Pulpotomy D. Pulpectomy

C. Pulpotomy 1) Pulpotomy a. indicated for a primary tooth where infection has reached the coronal pulp but has not yet reached the radicular pulp b. removal of coronal portion of infected pulp to prevent bacterial invasion of the radicular pulp c. coronal pulp is usually excavated with a large spoon excavator or slow speed hand piece with a large round bur in reverse. The pulp stumps are then dabbed with a cotton pellet soaked in 2.6% NaOCl and the tooth is then restored. 2) Pulpectomy a. are performed when pulp completely infected by bacteria and involve the complete removal of pulp tissue

A clinical examination of a 7-year-old patient reveals a fractured right central incisor with a 2-mm exposure of a "bleeding pulp" that occured 3 hours ago. The suggested treatment is A. One-appointment root canal treatment B. Pulpectomy and apexificatin C. Pulpotomy with calcium hydroxide D. Direct pulp cap with calcium hydroxide

C. Pulpotomy with calcium hydroxide 1) Pulpotomy procedures prevent unnecessary extraction of central incisors by removing only the portion of the pulp that was exposed during the trauma. 2) A partial pulpotomy for traumatic exposures is also called a Cvek pulpotomy. 3) The root apex of developing permanent central incisor is still open. By performing a Cvek pulpotomy, the root of the central incisor will have extra time to complete its root development.

A 7-year-old boy presents to you office after fracturing his left central incisor 3 hours ago. During your examination, you find 2mm o exposed, bleeding pulp tissue. There is no mobility noted. What treatment is indicated? A. One-appointment root canal treatment B. Pulpectomy and apexification C. Pulpotomy with calcium hydroxide (Svek Pulpotomy) D. Direct pulp cap with calcium hydroxide

C. Pulpotomy with calcium hydroxide (Svek Pulpotomy) 1) Concept: Pulpotomy procedures prevent unnecessary extraction of central incisors by removing only the portion of the pulp that was exposed during the trauma 2) Cvek pulpotomy a. a partial pulpotomy for traumatic exposure 3) The root apex of developing central incisor is still open. By performing a Cvek pulpotomy, the root of the central incisor will have extra time to complete its root development. 4) Pulpotomies a. performed as a temporary treatment option in permanent teeth while you wait for the roots of the tooth to fully develop so a root canal treatment can be performed

For an Ideal Class V restoration on a mandibular premolar, which of the following is the MOST IMPORTANT factor when choosing a restorative material? A. Ease of manipulation of material B. Esthetics C. Reaction of gingival tissue D. Compressive strength

C. Reaction of gingival tissue 1) Restorative materials a. do not induce gingival tissue should be considered when selecting the proper restorative material for Class V restorations b. esthetics are not a factor since the Class V restoration is placed at the cervical area of a mandibular premolar and not in the esthetic zone c.

Which of the following factors is MOST important when choosing the optimal restorative martial for a Class V restoration on the buccal surface of a mandibular 2nd premolar? B. Esthetics C. Reaction of gingival tissues

C. Reaction of gingival tissues 1) Esthetics is not a factor because the Class V restoration is placed at the cervical area of a mandibular premolar and is not within the esthetic zone. 2) Restorative materials that do not induce gingival tissue reaction should be considered when selecting the proper restorative material for Class V restorations.

What is the classic clinical sign or symptom of an anteriorly displaced disc with reduction (ADD with Reduction)? A. Decreased range of opening B. Pain C. Reciprocal click D. Closed lock

C. Reciprocal click 1) Anterior Disc Displacement with Reduction a. is when the articular disc, attached anteriorly to the superior head of the lateral pterygoid muscle and posterior to the retrodiscal tissue, moves anteriorly out from between the condyle and the fossa, so that the mandible and temporal bone contact is made on something other than the articular disc and the disc slides back into place. b. sliding of the disc back into place creates the characteristic popping or clicking noise 2) Anterior Disc Displacement without Reduction a. the disc stays anterior so the condylar head upon opening so mouth opening is limited and there is no "pop" or "click" sound on opening.

Which of the following is the reason trituration is completed? A. Activate the copper in the alloy B. All answers are correct C. Remove the oxide coating and wet each particle of alloy with mercury D. Remove the oxide coating and wet each particle of alloy with zinc.

C. Remove the oxide coating and wet each particle of alloy with mercury 1) The primary objective of the process of trituration of amalgam is to remove the oxide coating and wet each particle of alloy with mercury. 2) Properly triturated amalgam is a homogenous mass with slightly reflective surface. It flattens slightly if dropped on a tabletop. 3) A correctly mixed amalgam should have sufficient wetness to aid in achieving a well-adapted restoration and NOT be dry or crumbly.

10 days following an incision and drainage of a submandibular space abscess, the patient returns to your office. The swelling has slightly increased even though the patient was prescribed high dose antibiotics. Which of the following treatments is indicated for this patient? A. Insert a larger drain into the abscess B. Order a complete blood count (CBC) with differential C. Repeat the culture and sensitivity assay D. Use proteolytic enzymes so the antibiotic can move effectively penetrate the abscess

C. Repeat the culture and sensitivity assay 1) Culture and sensitivity assay a. incorrect identification of the causative microorganisms seem to be the reason for increased swelling b. mandatory to repeat the culture and sensitivity tests in order to correctly identify the casative microorganisms so that proper antibiotic can be prescribed

If a permanent maxillary central incisor was avulsed and rigidly splinted, what is the most likely outcome? A. Internal resorption B. Normal healing process C. Replacement resorption D. Root fracture E. Calcific metamorphosis

C. Replacement resorption 1) Avulsed tooth should be splinted with a flexible splint to avoid extensive healing complications like replacement resorption 2) Avulsed teeth should have some degree of freedom to lessen pressure on the bone and allows better healing along the periodical aspect of bone surrounding the traumatized tooth.

An increase in which salivary properties would increase a patient's caries risk? A. mineral content B. protein content C. viscosity D. pH E. Flow

C. viscosity 1) If the saliva becomes more viscous, its ability to flow is altered resulting in diminished function in the oral cavity. 2) Saliva fucntions a. dilute the bacterial acid metabolites that hart teeth b. buffer acid from cardiogenic bacteria c. remove food debris on tooth surface d. remineralized damaged tooth structure

A fracture of the maxillary tuberosity occurred during the extraction of #2. The periosteum of broken piece of bone remains with intact blood vessels. What treatment is indicated? B. Remove the bone fragment C. Reposition the fragment and stabilize with sutures

C. Reposition the fragment and stabilize with sutures Procedures 1) Facture maxillary tuberosity can be repositioned and held in place with a suture provided that there is evidence of an intact blood supply 2) Recovery rates of fracture bone fragments are higher if they are properly repositioned within the fracture site with an adequate blood supply 3) Bone fragments that are completely detached and devoid of blood supply will result in bone necrosis

Which of the following materials demonstrates the greatest coefficient of thermal expansion? A. Compomers B. Glass ionomers C. Resin composites D. Resin-modified glass ionomers

C. Resin composites 1) Resin composites a. greatest value of coefficient of thermal expansion (CTE) among the materials listed b. rate of dimensional change of a material per unit change in temperature c. CTE is approximately 3 times that of the tooth structure d. etching reduces the potential negative effects as well the difference between the CTE of tooth structure and that of the material

A cast gold restoration is retained primarily by... A. All answers are correct B. Cement seal C. Retention and resistance form D. Sharp cavosurface angles

C. Retention and resistance form 1) Concept: Cast gold restoration is retained primarily by retention and resistance form of the cavity preparation. 2) Retention a. form prevents the removal of the restoration along its path of placement by the occlusal forces b. achieved with parallel walls with minimal taper to facilitate a single pathos placement of the restoration 3) Resistance a. resistance form resists the dislodgment and movement of restoration under directed apical, oblique, or occlucsal forces. b. achieved with the use of fish tail and dovetail preparations, presence of grooves, slots and pins.

Which of the following is the usual sequence for the amputation (resection) or a root for periodontal reasons? A. Periodontal therapy followed by observation B. Root canal therapy followed by observation C. Root canal therapy followed by periodontal therapy D. Periodontal therapy followed by root canal therapy

C. Root canal therapy followed by periodontal therapy 1) Amputation (resection) a. root canal system is exposed during root amputation and therefore must immediately undergo root canal therapy to ensure that no bacteria will enter the canal system b. resection of a root with a hopeless or questionable prognosis in its periodontal condition will save the whole tooth from being extracted c. periodontal therapy should follow immediately after root canal treatment is accomplished in order to stop or prevent further periodontal damage d. root amputation typically prolongs the life of the tooth by about 5 years on average

Internal resorption that has not totally perforated the root surface is BEST managed by which of the following? A. Incision and drainage B. Periodic recall and evaluation C. Root canal treatment D. Surgical curettage E. Cvek pulpotomy

C. Root canal treatment 1) Internal root resorption a. root canal therapy is indicated for cases involving internal resorption where the root surface is not perforated by the defect b. most commonly caused by trauma and infection c. infection of trauma results in an inflammatory response within the pulp resulting in abnormal vascularization and production of multinuclear odontoclasts, which initiate the process of resorption d. radiographs are made after 1 year to determine if healing has occurred and the resorptive process is halted.

It has been suggested that daily cleaning of root surfaces by the patient may increase A. Root resorption B. Root sensitivity C. Root surface remineralization D. Epithelial attachment level

C. Root surface remineralization 1) Cleaning root surface on a daily basis with fluoride identifies may result in root surface remineralization. 2) Consistent plaque removal prevents root demineralization by bacteria and enables the root surface to remineralize. 3) Presence of minerals in saliva and fluoride from toothpaste also aid in the remineralization of root surfaces

Cleaning root surfaces daily has been indicated to result in... A. Root resorption B. Root sensitivity C. Root surface remineralization D. Increased epithelial attachment level

C. Root surface remineralization 1) Concept: Cleaning root surfaces on a daily basis with fluoride dentifrices may result in root surface remineralization. 2) Consistent plaque removal prevents root demineralization by bacteria and enables the root surface to remineralize. 3) Presence of minerals in saliva and fluoride from toothpaste also aid in the remineralization of root surfaces

Which of the following describes the structure where the beaks of extraction forceps are designed to transmit extraction pressure to? A. Cementoenamel junction B. Crowns of teeth being extracted C. Roots of the tooth D. Alveolar bone

C. Roots of the tooth 1) Roots of tooth a. optimal place to apply apical force in order to not fracture the crown during the extraction procedure b. enables the apical force to be transmitted effectively along the long axis of the tooth, which results in the expansion of the alveolar bone and the eventual extraction of the tooth

Radiographs of a 22-year old man disclose a sharply outlined radiolucent lesion in the mandibular 1st molar area. This 35 mm lesion scallops between the roots of vital teeth. What is the most likely diagnosis? A. Odontoma B. Radicular cyst C. Traumatic bone cyst D. Dentigerous cyst

C. Traumatic bone cyst 1) Traumatic bone cyst a. well circumscribed radiolucency that usually scallops in between the roots of teeth b. usually occur during the 2nd or 3rd decay of life of active people due to being more prone to trauma from accidental heavy blows to the bone c. affects the jaws of males more than females

The position of the maxilla in relation to the cranial base can be determined from which of the following cephalometric measurements? A. Frankfort Horizontal B. SN-Pog C. SNA D. SNB E. ANB

C. SNA 1) Concept: SNA or Sella-Nasion A a. A point angle is the parameter used to determine the relation of the patients maxilla to their cranial base. b. SNA values below the normal range corresponds to a retruded maxilla and probably a Class III skeletal relationship c. SNA values above the normal range corresponds to a protruded maxilla and Class II skeletal relationship d. ANB value is a useful guide in determining the relation of maxilla to the mandible

Which of the following cephalometric measurements is likely to increase as a 13-year old child with a Class III malocclusion ages? A. SN B. SNA C. SNB D. Y-Axis E. ANB

C. SNB 1) Concept: SNB angle will increase in value when the B point of the mandible extends more anteriorly as demonstrates in a growing child with Class III malocclusion 2) SNB angle is the angle created by the intersection of Sella - Nasion plane and the B point of the mandible. 3) SNB angle value is lower among patins with a Class I or Class II skeletal relationship as compared with skeletal Class III patients.

Clinical signs/symptoms of insulin shock do NOT include... A. Cold sweat B. Nausea C. Seizures D. Weakness E. Confusion

C. Seizures 1) Concept: Insulin shock a. can be induced by insulin overdose, skipped meal, or a strenuous exercise that lowers the blood sugar in a very rapid manner in a diabetic patient. b. clinical signs and symptoms of insulin shock do not include seizures. b. Signs and symptoms 1) Cool moist skin 2) Sweating 3) Weakness 4) Confusion 5) Nausea 6) Dizziness 7) Hunger 8) Tachycardia

Which of the following properties indicates that amalgam was properly triturated? A. Dry and coherent B. Dry and crumbly C. Shiny and coherent D. Sticks to the capsule

C. Shiny and coherent 1) Concept: The shiny and coherent mass of amalgam produced after the process of trituration (mixing) indicates that the material is mixed homogeneously. 2) The purpose of trituration is to remove oxide layers from the alloy particles, to coat each alloy particle with mercury, and to provide a homogenous mass for condensation. 3) The amalgam should NOT be dry and crumbly. 4) No adherence or stickiness to the capsule walls should occur.

Each of the following is characterized by BILATERAL enlargement of the parotid glands EXCEPT ONE. Which is the EXCEPTION? A. Benign lymphoepithelial lesion B. Malnutrition C. Sialolithiasis D. Sjogren's syndrome E. Acute epidemic parotitis

C. Sialolithiasis 1) Concept: Sialolithiasis is a condition characterized by the formation of calcified mass within a salivary duct and is most commonly associated with submandibular glands. 2) Sialolithiasis a. unilateral enlargement and rarely is associated with parotid glands and is also known as salivary calculi or salivary stones. b. All other answer foils mostly describe conditions that are characterized by a bilateral enlargement of parotid salivary glands

Each of the following is characterized by BILATERAL enlargement of the parotid glands EXCEPT ONE. Which is the EXCEPTION? A. Sjogren's syndrome B. Acute epidemic parotitis C. Benign lymphoepithelial lesion D. Malnutrition E. Sialolithiasis

C. Sialolithiasis 1) Concept: Sialolithiasis is a condition characterized by the formation of calcified mass within a salivary duct and is most commonly associated with submandibular glands. 2) Sialolithiasis a. unilateral enlargement and rarely is associated with parotid glands and is also known as salivary calculi or salivary stones. b. All other answer foils mostly describe conditions that are characterized by a bilateral enlargement of parotid salivary glands

Each of the following statements is TRUE concerning resin based composite restorations EXCEPT one. Which is the EXCEPTION? A. Obtaining an adequate contact in the final restoration starts with pre-wedging B. Placing composite ring and matrix band is helpful in avoiding damage to adjacent teeth C. Single wedging is alway more effective than multiple wedging D. You should check the occlusion before restoring if possible

C. Single wedging is alway more effective than multiple wedging 1) Wedging a. Multiple wedging is typically more effective than single wedging concerning resin based composite restorations 2) Occlusion a. It is necessary to carry out occlusal evaluation before restoring if possible 3) Pre wedging a. helps in obtaining an adequate contact in the final restorations 4) Placement of composite ring and matrix band a. damage to inter proximal surfaces of adjacent teeth can be prevented through the placement of composite ring and matrix band

A posterior superior alveolar nerve block performed correctly will not anesthetize... A. 3rd, 2nd, and 1st molars (except mesiobuccal root) B. Buccal plate and overlaying gingival tissue posterior of premolars C. Soft palatal mucosa D. Maxillary sinus mucosa above the maxillary molars

C. Soft palatal mucosa 1) Mucosa of the soft palate is innervated by a. lesser palatine nerve b. pharyngeal branch of vagus nerve c. medial pterygoid merve 2) Posterior superior alveolar nerve block only affects the structures that are innervated by the posterior superior alveolar nerve: a. Maxillary sinus mucosa b. Buccal plate and the overlying gingival tissue posterior to the premolar area c. All the upper molars except the mesiobuccal root of the permanent 1st molar which is innervated by the middle superior alveolar nerve.

Theoretically, the most effective tooth brushing technique is which of the following? A. Side-to-side B. Stillman C. Sulcular D. Charter

C. Sulcular 1) Concept: Sulcular brushing technique a. The sulcular brushing technique effectively eliminates plaque from the sulcular and cervical areas of the teeth thereby preventing the occurrence of periodontal disease. b. Sulcular brushing allows the round ended brush fps o each about 0.5mm subgingivally to effective disrupt plaque buildup in the cervical area c. Sulcular brushing reduces the chance for abfraction that may happen from improper brushing strokes that cause cervical cavities that are prone to plaque retention, tooth sensitivity, and tooth decay.

In the transitional analysis, which of the following dimensions are compared? A. Leeway space to Bolton variable B. Leeway space to freeway space C. Space available to space required and Space available to leeway space D. arch width to arch length

C. Space available to space required and Space available to leeway space 1) Concept: A mixed (transitional) dentition analysis is performed to predict the probability of tooth crowding before the permanent canines and premolars erupt. 2) Predicting the sizes of unerupted permanent canine and premolars provide important diagnostic information so the most appropriate treatment can be provided 3) Mixed dentition analysis is performed by determining the space available for the erupting permanent teeth and comparing it to the appropriate size of the unerupted permanent canine and premolars.

Larger sized condensers and lateral force application are techniques used to properly condense which type of amalgam? A. Admix B. Low copper C. Spherical D. Lathe-cut

C. Spherical 1) Concept: Spherical amalgam alloy has high compressive strength requiring strong condensation forces to ensure proper adaptation to the cavity preparation. 2) Use of strong lateral forces and large condensers should deb used to properly condense spherical amalgam alloy. 3) Spherical alloys have large particle composition necessitating good condensation to minimize the gap between the cavity prep and the amalgam.

A 42 year old patient reports to your office after a car accident where their maxillary central incisor is traumatized, and is slightly tender to percussion, is in good alignment, and responds normally to pulp vitality tests. Radiographic examination reveals a horizontal fracture of the apical 1/3 of the root. Which of the following is the treatment of choice? A. Apexification and subsequent root canal therapy B. Apicoectomy to remove the fracture apical section of the root followed by root canal treatment C. Splint and re-evaluate the tooth for pulp vitality at later time D. Root canal treatment

C. Splint and re-evaluate the tooth for pulp vitality at later time 1) Pulp vitality tests on traumatized teeth are usually inaccurate, necessitating a few weeks later. 2) A tooth that has fracture roots in close proximity with one another have a greater chance of developing calcific metamorphosis, therefore splinting for the immobilization of the traumatized tooth is necessary.

Name the technique where a deep caries are excavated from a tooth, and a small amount of affected dentin is left provided that a caries-free DEJ is present, and a temporary restoration is placed then replaced with a permanent restoration following analysis 6 months later? A. Indirect pulp cap B. Semi-permanent caries excavation C. Step-wise caries removal D. Direct pulp cap

C. Step-wise caries removal 1) Step-wise caries removal a. when decay is excavated around the margins of a preparation and the tooth structure is remineralized until the permanent restoration is placed later. 2) Step-wise caries removal steps a. Totally remove caries from the margins of the preparation, spring the pulpal affected dentin but not the infected dentin b. Place a CaOH liner and restore with glass ionomer. c. 6-12 months later remove glass ionomer and test tooth vitality then placed definitive restoration 3) Indirect Pulp Capping a. different because a permanent restoration is placed at the first appointment and is more aggressive with caries excavation

Which of the following bacterial species is considered a part of normal oral flora and is associated with periodontal health? A. Aggregatibacter Actinomycetemcomitans B. Porphyromonas gingivalis C. Streptococcus gordinii D. Tannerela forsythia

C. Streptococcus gordinii 1) Streptococcus gordinii a. gram positive cocci b. belongs to normal oral flora that adheres to tooth pellicle c. considered one of the initial colonizers of the dental biofilm d. creates a binding site for other bacteria to adhere = congregation e. in normal amounts, helps maintain the balance between gram positive and gram negative bacteria i nthe mouth 2) Periodontitis a. Aggregatibacter Actinomycetemcomitans b. Porphyromonas gingivalis c. Tannerela forsythia

Which of the following is NOT an oral surgery technique/principle that can be used to minimize the likelihood of a buccal plate fracture? A. Making a trough around the tooth B. Preoperative assessment of oral structures C. Strong pressure to make sure that the forceps do not slip off the crown of the tooth D. Use of slow, fluid, careful, and controlled use of instruments E. Using digital palpation to assess the amount of pressure exerted on the alveolus

C. Strong pressure to make sure that the forceps do not slip off the crown of the tooth 1) Buccal plate a. brittle cortical plate that fractures during oral surgical procedure when too much and abrupt force during lunation is applied in a buccal direction 2) Cortical bone a. cannot withstand too much pressure during the luxation of tooth, unlike cancellous bone that expands with pressure 3) Buccal plate fracture can be minimized with a. thorough examination of the oral structures b. slow and fluid control of instruments during tooth extraction c. Using digital palpation to assess the amount of pressure exerted on the alveolus d. Sectioning a tooth can help in reducing the force applied to a molar tooth that is strongly held in its alveolus e. making a trough around the tooth creates a wider space for tooth luxation that does not necessitate forceful extraction of tooth

Each of the following diagnostic aids is helpful in distinguishing between a case of maxillary retrusion and mandibular protrusion in cases presenting with a Class III malocclusion EXCEPT ONE. Which of the following diagnostic aids is the EXCEPTION? A. Clinical evaluation B. Photography C. Study casts D. Cephalometry

C. Study casts 1) Concept: Stone casts are accurate positive copies of a patient's oral structures that enable dentists to study and diagnose the oral condition of the patient when the patient is not present. 2) Stone casts are used in maxillofacial surgical procedures as: a. study models for diagnostic purposes and is used to plan and practice the surgical procedure b. good visual aid to explain the oral condition and proposed procedure to the patient c. models for fabrication of occlusal splints. d. are limited to providing information of the intraoral structures, and can't provide information on skeletal deformities

What is the only structure in the TMJ to exert posterior traction on the articular disc? A. Inferior head of the lateral pterygoid B. Inferior retrodiscal lamina C. Superior retrodiscal lamina D. Superior head of the lateral pterygoid

C. Superior retrodiscal lamina 1) Concept: Superior retrodiscal lamina functions by pulling the articular discs posteriorly against the lateral pterygoid that pulls it forward. 2) Superior retrodiscal lamina provides posterior traction as it connects the articular disc to the tympanic plate. 3) During mouth opening, the retrodiscal lamina is extended preventing overextension and anterior dislocation of the articular disc.

Each of the following procedures is a component of a typical initial periodontal treatment plan EXCPET one. Which procedure is that EXCEPTION? A. Home care instructions B. Root planing C. Surgical pocket reduction D. Extraction of hopeless teeth (if any)

C. Surgical pocket reduction 1) Initial periodontal treatment a. reduces the risk of progression of periodontal disease by removing the predisposing factors, risk factors, and causative factors 2) Surgical pocket reduction a. treatment performed to address deep pockets and is a part of the corrective phase of periodontal treatment

Which of the following treatment modalities should be included to ensure most favorable prognosis for the management of localized aggressive periodontitis? A. High doses of vitamin C B. Peroxide rinses C. Systemic antibiotic therapy D. Xylitol lollipops E. Free gingival grafts

C. Systemic antibiotic therapy 1) Periodontal treatments through nonsurgical and surgical root debridement become more successful when administered with systemic antibiotics. 2) Aggressive periodontitis is a rapidly progressing disease that can be better managed with complete root debridement and antibiotic therapy. 3) Systemic antibiotics ensure that the antibiotic molecule reaches the site as compared with local application. 4) Local antibiotics are not effective since it may not reach the deep pockets of the periodontal disease

Which method of treatment should be utilized to increase the rate of success in case of localized aggressive periodontitis? A. Free gingival grafts B. Peroxide rinses C. Systemic antibiotic therapy D. Xylitol lollipops E. High dose of vitamin C

C. Systemic antibiotic therapy 1) Systemic antibiotics a. periodontal treatments through nonsurgical and surgical root debridement are more successful when administered with systemic antibiotics b. ensure that the antibiotic molecule reaches the site as compared with local application 2) Localized aggressive periodontics a. better managed with complete root debridement and antibiotic therapy 3) Local antibiotics a. sometimes ineffective because it might not reach the deep pockets of the periodontal disease

Each of the following is a characteristic of advanced osteoarthritis of the TMJ EXCEPT one. Which is the EXCEPTION? A. Antegonial notching B. Crepitus C. Tendency toward apertognathia D. Tendency toward prognathism E. Flattening of articular surface

C. Tendency toward apertognathia 1) Concept: Advanced cases of osteoarthritis of the TMJ may exhibit crepitus, flattening of the articular surface of the condyle, limited jaw movement, and pain on mouth opening. 2) Advanced osteoarthritis a. could not cause apertognathia which is the dental malocclusion whee the posterior teeth occlude prematurely resulting in an open bite. b. could result to a pseudoprognathic appearing mandible and could also cause deviation of jaw movement, further aggravating the condition

All 4 mandibular incisors were damaged in an automobile accident 5 years ago. Radiolucencies were noted apically to these 4 incisors. Which of the following is the best treatment for this patient? A. No treatment necessary, evaluate the status of the patient periodically B. Perform root canal therapy on the incisors, raise a flap, and cure out the involved bone C. Test the vitality of the pulps of all 4 incisors and perform root canal therapy in teeth that do not respond D. Extract all 4 mandibular incisors and fabricate a fixed partial denture

C. Test the vitality of the pulps of all 4 incisors and perform root canal therapy in teeth that do not respond 1) Treatment a. root canal therapy is the treatment of choice for teeth with non-vital pulps resulting from trauma b. necrotic tissues (pulp and infected dentin) are removed, canals are cleansed with an irrigating solution like sodium hypochlorite, then filled with an inert material and a coronal restoration is placed to seal the canals from the oral environment and restore masticatory function. 2) Vitality tests a. performed in order to assess the status of the pulpal tissue, especially after a tooth has undergone trauma b. teeth which do not respond to vitality tests are considered non-vital and possess a necrotic pulp

Each of the following characteristics describe acute (active) caries EXCEPT one. Which is the EXCEPTION? A. Dentinal pulp is at risk of irreversible damage B. Rapid rate of progression C. The cavity floor is rough and leathery D. There is little color change

C. The cavity floor is rough and leathery 1) Acute Caries a. really soft to touch and caseous (cheesy) in consistency b. rapid process involving a large number of teeth c. dental pulp is at risk of irreversible damage d. lesions are lighter in door mostly light brown and gray

Which of the following methods describes the MOST CONVENIENT way to conduct an examination of an 18-month-old child? A. A parent sitting in the dental chair with the child sitting on their lap facing forward B. This child sitting in the dental chair with a parent sitting chair side C. The clinician and a parent in a knee-to-knee position with the child's head in the clinician's lap D. The clinician and a parent in a knee-to-knee position with the child's head in the parent's lap

C. The clinician and a parent in a knee-to-knee position with the child's head in the clinician's lap 1) Knee-to-knee position a. helps to build a relation of trust and confidence between the clinician and the patient b. child feels more comfortable and safe while facing his/her parent c. dentist gets maximum visibility to examine the oral cavity and facial structures d. most convenient way for carrying out examination of an infant

Each of the following statements about partial thickness flaps is correct EXCEPT one. Which is the EXCEPTION? A. It is indicated when a bone dehiscence or fenestration is present B. It is indicated when there is <2mm attached gingiva C. The flap include the mucosa and the periosteum D. The flap is raised with an internal bevel incision E. The flap should be uniformly thick

C. The flap include the mucosa and the periosteum 1) Concept: Partial thickness flaps a. include the epithelium and connective tissue, while the periosteum is left attached to the bone. b. used for free gingival grafts and when bone dehiscence or fenestration is present c. an internal bevel is used to reflect the flap about 2mm thick or more. 2) Full thickness flaps a. include the periosteum

After recurrent Class V caries were found on a molar that extends subgingivally, crown lengthening surgery was performed to expose 3+mm of root structure. The best design features would include: A. The cervical line must be re-established in the restoration B. The gingival margin of the restoration must be placed at the new gingival level C. The gingival restorative margin should be moved apically until it is in sound tooth structure D. The restoration should be over contoured to maintain the protection the gingiva provided by the buccal bulge of the crown.

C. The gingival restorative margin should be moved apically until it is in sound tooth structure 1) Margins Concept a. margins of cavity preparation should always be placed in sound tooth structure 2) Prepared teeth should always have supported tooth structure surrounding the restorative material to decrease the chances of tooth fracture from occlusal stresses 3) Preparation margins should ideally be placed supra or equigingivally for ease of instrumentation and ease of cleaning 4) Margins of tooth preparation must not violate the biologic width.

A patient presents with a large facial laceration with irregular/necrotized wound margins. Which of the following surgical principles should you follow when making an elliptical incision to achieve primer closure? A. The excision of the use or irregular/necrotic tissue should be as conservative as possible B. The incision should be 3 times longer than it is wide C. The incision should be parallel to the dermatologic tension lines D. The wound margin tissue should be undermined to achieve primary closure E. Only silk sutures can be used due to their strength

C. The incision should be parallel to the dermatologic tension lines 1) Incision lines should be made parallel to the dermatologic skin tension lines a. to reduce scar visibility after wound healing b. necrotized margin should be removed minimally c. enables the most optimal primary wound closure and will not distort the skin's normal flow, allowing the scar to blend in better d. length of the elliptic incision should be 3 times its width in an ideal guide for making an elliptical incision e. will create a better esthetic appearance after wound healing

Which of the following BEST describes the smear layer? It is... A. Another name for the pellicle B. Produced when phosphoric acid is used when acid etching C. The layer of debris from grinding on the surface of a cavity prep D. The layer of plaque closest to the tooth structure

C. The layer of debris from grinding on the surface of a cavity prep 1) Concept: The smear layer is the layer of debris produced from grinding on the surface of a cavity preparation. 2) When the tooth surface is altered by rotary and manual instrumentation during cavity preparation, the cutting debris is smeared over the enamel and dentin surfaces, forming the smear layer. 3) Smear layer contains a. Calcified debris b. Shattered and crushed hydroxyapatite c. Fragmented and denatured collagen d. Bacteria e. Saliva and f. Dentinal chips

Which of the following BEST describes the smear layer? It is... A. Another name for the pellicle B. Produced when phosphoric acid is used when acid etching C. The layer of debris from grinding on the surface of a cavity prep D. The layer of plaque closest to the tooth structure

C. The layer of debris from grinding on the surface of a cavity prep 1) Smear layer a. layer of debris produced from grinding on the surface of a cavity preparation b. tooth surface is altered by rotary and manual instrumentation during cavity preparation, cutting debris is smeared over the enamel and dentin surfaces, forming the smear layer c. contains 1) Calcified debris 2) Shattered and crushed hydroxyapatite 3) Fragmented and denatured collagen 4) Bacteria 5) Saliva 6) Dentinal chips

The most important factor influencing the solubility of zinc phosphate cement is... A. All answers are correct B. The age of the patient C. The liquid/power ratio D. The brand of the material since phosphate levels vary widely

C. The liquid/power ratio 1) Solubility a. increasing the power to liquid ratio makes the mixture more viscous, stronger, and less soluble b. presence of moisture on a mixing slab also affects the quality of the polyacrylic acid liquid and increases the solubility of the zinc phosphate cement mix c. proper mixing techniques also affects the quality of the mixture d. the zinc phosphate must be uniformly mixed through flat sweeping motion of the spatula while powder is added in increments

Which of the following does a negative response of a tooth to electronic pulp testing (EPT) an hour after a severe lunation indicate? A. The EPT gave a false negative reading B. The negative response may be temporary because the blood supply was interrupted C. The negative response may be temporary because the neural transmission was interrupted D. The pulp is necrotic and should undergo a root canal procedure E. The pulp is inflamed

C. The negative response may be temporary because the neural transmission was interrupted 1) Electric pulp testing a. gives the status of the pulpal nerve fibers not the blood supply of the tooth b. pulpal response to electric current only suggests that some viable nerve fibers are capable of responding to electric current are present 2) Electric pulp testing within a hour of a lunation injury a. is contraindicated because of inconsistent findings b. Positive response from electric pulp testing of retained vitality and DOES NOT differentiate between: 1) Normal pulps 2) Reversible pulp inflammation 3) Irreversible inflammation 2) Negative response a. may not always indicate pulpal necrosis because many teeth take about 2 months to respond to electric pulp testing again

As an amalgam preparation becomes wider buccolingually? A. The integrity of the tooth and restoration is more likely to be maintained B. The occluso-gingival depth should be increased to enhance the resistance of the planned restoration C. The tooth is more subject to fracture

C. The tooth is more subject to fracture 1) Amalgam Restoration a. becomes wider faciolingually, the following drawbacks can result: 1) tooth is more subject to fracture 2) integrity of the restoration is less likely to be maintained 2) Ideal Dimensions a. facilingual width of NO more than 1-1.5mm and a depth of 1.5-2mm are considered ideal taking into consideration the extent of caries progression

Each of the following are TRUE regarding self-curing sealants EXCEPT ONE. Which is the EXCEPTION? A. Benzoyl peroxides or aromatic tertiary amines are components B. Late manipulation may disrupt polymerization C. They cure by an endothermic reaction D. They have 2 components

C. They cure by an endothermic reaction 1) The setting reaction of self cure or chemical cure or auto-curing sealants is exothermic in nature. 2) Significant amount of heat energy is liberated. It is advised to use light curing sealants for safety, good manipulation and ease of handling. 3) Auto curing sealants are composed of benzoyl peroxide or tertiary amines. 4) Late manipulation can disrupt polymerization.

Light-cured composite resin is most affected by which of the following variables? A. Opacity of enamel B. Shade of material C. Thickness of increment D. Length of exposure

C. Thickness of increment 1) Polymerization of light cure composites is dependent upon the penetration of light through the composite resin 2) As the material thickness increases, the penetration of light decreases, which may lead some composite unpolymerized, resulting in the failure of restoration. 3) Place composite resin in small increments is advised because it allows for complete penetration of curing light and complete polymerization. 4) Incremental Placement minimizes shrinkage which a. increases bond strength b. decrease cuspal fractures c. deceases marginal leakage

The most retentive pin design is...? A. Friction-lock system B. No answers are correct C. Threaded D. Cemented

C. Threaded 1) Concept: Threaded pins a. approximately 0.0015 and 0.002 inch bigger than the prepared channels where they would be placed b. considered the provide the most retention. However, their placement also creates stresses and cracks in the dentin. 2) Friction locked pins and threaded pins are both retained by the elasticity of the dentin walls 3) Cemented pins a. least retentive among pin designs used in restorative dentistry but also creates the least stress on dentin

Each of the following is considered a reason for sedating a patient EXCEPT one. Which reason is the EXCEPTION? A. To allay the stress experience by a severely medically compromised patient B. To be able to complete surgeries that would not be able to be normally tolerated by an anxious patient C. To decrease the amount of local anesthesia required D. To assuage the anxiety, apprehension, and fear exhibited by the patient

C. To decrease the amount of local anesthesia required 1) Sedation a. can diminish the stress and fear of the patient but does NOT affect the amount of anesthesia that should be administered to the patient b. used to relax or calm anxious patients through the use of medication or a sedative agent c. is induced for uncooperative patients or those with anxiety or fear of the treatment d. allows the patient to have a higher pain threshold, allowing them to be capable of tolerating painful treatment procedures

Which of the following is the goal of alveoloplasty procedure? C. To make sure that undercuts that interfere with the denture seating are removed D. To make the residual ridge as broad as possible even if undercuts are still present

C. To make sure that undercuts that interfere with the denture seating are removed 1) Alveoloplasty a. promote better denture support and stability during function be creating an ideal alveolar ridge form b. removes undesirable undercuts that impede proper seating of dentures and round-off knife edge alveolar ridges to provide better support during function

Which of the following is the MOST IMPORTANT reason a dentist should visualize the outline form before they begin to prepare a tooth? A. To establish the convenience form B. To establish the resistance and retention form C. To prevent over cutting and overextension D. To make finishing the enamel walls and margins easier

C. To prevent over cutting and overextension 1) Concept: Overcutting and over extension compromises the remaining tooth structure and the success of the restorative procedure 2) Cutting extensively without a thorough knowledge of dental anatomy could result in excessive removal of tooth structure that may extend too close to the pulp, causing accidental pulp exposure or irritation. 3) Undermining cusps and overextending the cavity outline weakens the remaining tooth structure. Large cavities are too difficult to restore and normally result in a greater chance of post-operative sensitivity.

In a clinical exam following a traumatic injury, which of the following is the LEAST reliable finding? A. Displacement of teeth B. Loss of tooth structure C. Tooth vitality D. Tooth mobility

C. Tooth vitality 1) Tooth that recently experience trauma may or may not elicit a response to tooth vitality test because the nerves of the tooth could be damaged 2) Inability of the traumatized tooth to elicit pulpal response during a vitality test can be attributed to the injury within the nerve fibers of the tooth. 3) Tooth with trauma may takes approximately 8 weeks at a minimum before it can display normal pulp response.

Which of the following jaw relations is a pure hinging movement possible? A. Postural position of the mandible (vertical dimension of rest) B. Retruded contact position C. Transverse horizontal axis (Terminal hinge position) D. Centric occlusion

C. Transverse horizontal axis (Terminal hinge position) 1) Transverse horizontal axis a. purely hinging movements of the mandible are possible when the jaws are in transverse horizontal axis or in the terminal hinge position b. once the mandible moves beyond the hinge position, translational movement occurs c. An imaginary line through both temporomandibular joints about which the mandible rotates in the sagittal plane during opening and closing of the mouth. 2) Terminal hinge position a. mandibular hinge position from which further opening of the mandible would produce forward translator movement rather than hinge movement

Which of the following has not been proved to be a causative factor in human pro-facial clefting? A. Anticonvulsant drugs B. Malnourishment C. Tylenol D. Vitamin imbalance

C. Tylenol 1) Concept: Potential causes of orofacial clefting a. maternal nutrition low intake of folic acid b. alcohol and cigarette use c. nitrate compounds and organic solvents d. lead e. medications 2) Cleft lip a. occurs when the frontomaxillary process and maxillary processes that form the upper lip fail to fuse together 3) Cleft palate a. occurs when the palatal shelves that grown medially towards each other fail to fuse 4) During the sixth week in utero, the embryo undergoes these processes ad is very sensitive to environmental pollutants and malnutrition

The restorative material with the highest potential for marginal leakage due to high thermal elasticity properties is... A. Composite resin B. Silicate cement C. Unfilled resin D. Amalgam

C. Unfilled resin 1) Unfilled resin a. exhibit higher coefficient of thermal expansion than tooth structure, leading to increased marginal leakage b. high CTE means that the resin expands and contracts faster than the tooth, resulting in gaps that c. make teeth sensitive to changes in temperature d. increase marginal leakage

The leading reason for the extraction of a primary tooth is... A. Accidental trauma B. Periodontal disease C. Untreated caries D. Treated caries

C. Untreated caries 1) Untreated caries is the leading reason for the extraction of primary teeth 2) Primary teeth have thinner enamel which makes it easier and faster for bacteria to generate cavities

Which of the following types of gloves should be utilized to provide the BEST protection during the cleaning dental operators? B. Latex C. Utility D. Vinyl

C. Utility 1) Utility gloves are thicker than latex and vinyl gloves.

Which of the following characteristics provide the least reliable finding from a clinical examination of teeth subjected to traumatic injury? A. Loss of tooth structure B. Mobility of teeth C. Vitality of teeth D. Displacement of teeth

C. Vitality of teeth 1) Tooth that experienced trauma may or may not elicit a response to tooth vitality test, since the nerves of the tooth could be damaged. 2) Traumatic tooth takes about 8 week minimum before it can display a normal pulp response.

Sjogren's Syndrome is associated with each of the following conditions EXCEPT ONE. Which condition is the EXCEPTION? A. Arthritis B. Cervical caries C. Warthin's tumor D. Xeropthalmia E. Xerostomia

C. Warthin's tumor 1) Warthin's tumor a. also known as papillary cyst adenoma lymphomatous is a benign tumor with unknown etiology that is strongly associated with cigarette smoking and is NOT directly associated with Sjogren's syndrome 2) Sjogren's Syndrome a. systemic autoimmune condition in which the autoimmune system attacks the exocrine glands like the salivary gland and lacrimal gland b. signs and symptoms 1) arthritis 2) Xeropthalmia 3) Xerostomia c. most commonly affects women after menopause

Which of the following facial dimensions remain MOST stable from birth to maturity? A. Depth B. Height C. Width

C. Width 1) Concept: Of all the facial dimension, the width of the face remains constant throughout life. 2) The depth and height of a face undergo remarkable changes over the course of life due to loss of teeth, loss in occlusal/vertical dimensions, and skin wrinkling from age.

Which of the following substances can be used to sedate the pulp of a tooth? A. Calcium Hydroxide B. Fluoride Varnish C. Zinc Oxide Eugenol D. Sodium Hypochorite

C. Zinc Oxide Eugenol 1) Zinc Oxide Eugenol a. type of intermediate restorative material made by mixing zinc oxide powder and eugenol (oil of cloves) b. its anesthetic and antibacterial properties make it an ideal material for sedating pulp and temporarily restoring teeth c. good interim restoration between endodontic treatments and is also use as sealing material in obturation of root canals

Which of the following is the MOST LIKELY diagnosis for a tooth with a cavity floor that is hard and leathery with a very dark appearance? A. acute caries B. aggressive caries C. chronic caries D. localized caries E. severe chronic periodontitis

C. chronic caries 1) Chronic caries a. lesions that are hard, leathery cavity floors b. dark appearance c. affect fewer number of teeth and are smaller in size

Primary occlusal trauma is described by C. mobility caused by excessive forces on a normal periodontium D. mobility caused by excessive forces on a periodontium with bone loss

C. mobility caused by excessive forces on a normal periodontium 1) Primary occlusal trauma a. mobility caused by excessive forces on a normal periodontium b. ove filled dental restorations c. crowns with high occlusion d. heavy biting of hard substances 2) Secondary occlusal trauma a. where a tooth with a compromised periodontium may experience further trauma even under normal occlusal loads

Which of the cell types are phagocytic? A. basophil and eosinophil B. neutrophil and basophil C. neutrophil and histiocyte D. neutrophil and lymphocyte E. plasma cell and lymphocyte

C. neutrophil and histiocyte 1) Leukocytosis a. is usually observed during acute infections b. inflammatory reaction to acute bacterial and parasitic infections c. occurs in the presence of cancer, hemorrhage and may be triggered by the application of steroids

Anterior maxillary incisors in a middle-aged patients can be given a younger appearance by performing an enameloplasty that: A. moves the facial height of curvature gingival B. move the facial line angles proximally C. rounds the incisal point angles

C. rounds the incisal point angles 1) Rounding the incisal point angles of maxillary incisors removes the sharp angles that make the teeth appear worn down and abraded. 2) Rounded incisal edges and pointed cusp tips = younger looking smile 3) Round incisal edges = more feminine 4) Square incisal edges = masculinity

Which of the following personal protective equipment (PPE) would most effectively prevent physical injury to a dentist when using caustic solutions? A. a face mask B. an apron C. safety goggles D. Utility gloves

C. safety goggles 1) Wearing safety goggles would be the most effective way to prevent physical injury to a dentist

Which of the following is a reason recapitulation is indicated between file sizes during stepback enlargement? A. to create a coronal funnel to make filling easier B. to maintain the apical stop C. to remove the dentin fillings from the apical segments that are not removed by irrigation D. to maintain curvature of the canal

C. to remove the dentin fillings from the apical segments that are not removed by irrigation 1) Recapitulation a. serves to maintain canal latency b. it also prevents deposition of dentin chips and fillings that may cause blockage of the canal during step back canal preparation c. performing recapitulation on after each file is used prevents canal preparation errors like apical transportation, incomplete instrumentation, and under-filling during obturation

The major reason for loss of transplanted autogenous 3rd molars is A. immune rejection B. inadequate root length C. traumatic occlusion D. infection

C. traumatic occlusion 1) The failure of autogenous tooth transplants which include chronic root resorption, inflammatory resorption, marginal and apical periodontitis, dental caries and occlusal trauma. 2) 3rd molars used for autogensou tooth transplant procedures most commonly fail due to their inability to withstand the heavy occlusal forces that exist in the posterior during mastication 3) Due to the mechanical advantage of a type III lever like the jaw, occlusal forces increases as you move posteriorly in the mouth (closer to the fulcrum/condyle).

Odontoblasts vs. Odontoclasts

Calcific metamorphosis is carried out by odontoBLASTS not odontoCLASTS. Calcific metamorphosis occurs when trauma induces odontoblasts to rapidly form extensive amount of repartee dentin, causing the pulp to appear smaller and at some point obliterated almost entirely.

Clinical Diagnosis, Case Selection, Treatment Planning, and Patient Management

Clinical Diagnosis, Case Selection, Treatment Planning, and Patient Management Answers

Communication and Interpersonal Skills

Communication and Interpersonal Skills Answers

Which of the following describes an injury where a tooth is damaged but NOT displaced?

Concussion 1) Concussion a. injury within the supporting structures b. loosening and possible displacement among traumatized teeth is not observed 2) Subluxation a. involves displacement of a tooth from its socket and may require additional dental treatment of the affected tooth 3) Luxation a. denotes displacement of tooth to any direction away from its original position in the dental arch

When your nitrous oxide an oxygen tanks have been half emptied, what is the pressure in psi of each cylinder? (The starting gas pressure are O2 [2,000]; N2O [800])? A. 1,000 400 B. 2,000 400 C. 2,000 800 D. 1,000 800

D. 1,000 800 1) N2O tank a. gas pressure in the N2O tank remains constant due to the amount of gas in each cylinder that it still in liquid for, exerting no pressure b. as the tank is emptied, the liquid gas vaporizes into gas as per Boyle's law, keeping the gas pressure constant c. at room temperature, N2O will exist as a liquid in the bottom of the tank whereas oxygen exists as a gas making the pressure decrease directly over emptying d. Because the starting pressure is 2000psi O2 and 800psi N2O, they remain at 1000psi O2 and 800psi N2O.

Which of the following treatments is best when superficial decalcification and staining are evident in the buccal groove of a mandibular molar? B. Full removal of the affected area with good retention and resistance form and restoration D. Enameloplasty limited to the superficial depth of the decalcified enamel

D. Enameloplasty limited to the superficial depth of the decalcified enamel 1) Localized and decalcified superficial stains within the limits of the enamel layer can be addressed by enameloplasty procedures.

In the bonding process, which of the following describes what primer BEST? 1. Remove smear layer and mineral from tooth surface 2. Infiltrate and remove water from enamel prisms, dentin tubules, and exposed collagen network 3. Couple tooth surface to restorative material 4. Made of hydrophobic nonsolvented monomers 5. Made of solvents and bi-functional/amphiphilic penetrating monomers A. 1 B. 1 and 5 C. 2 and 4 D. 2 and 5 E. 3 and 4

D. 2 and 5 1) Concept: Primer use is intended to provide 2 main functions in dentin bonding: a. infiltrate and remove water from enamel prisms, dentin tubules, and exposed collagen network b. form hybrid layer because it is made of solvents and bi-functional/amphiphilic penetrating monomers 2) Hybrid is formed in the following manner: a. etching removes the smear layer and hydroxyapatite within the inter tubular dentin in addition to exposing the collagen fibers b. primers penetrate the collagen network and allow the adhesive resins to form micro tags within the inter tubular dentin.

When placing a light cure resin composite in a deep cavity preparation, the composite should be placed in increments of no greater than... A. 0.5mm B. 1 mm C. 1.5 mm D. 2 mm E. 2.5 mm

D. 2 mm 1) When placing a light cured resin composite in a deep cavity preparation, the composite should be placed in increments of no greater than 2mm. 2) Composite should be placed incrementally to facilitate proper light activation, combat polymerization shrinkage stresses, and the development of correct anatomy. The deepest portions of the tooth preparation are restored first incrementally. 3) An anatomic layering technique is preferred. The operator places and light-activates one increment per cusp at a time and continues to place subsequent increments until the preparation is filled with development of anatomic contours.

Debridement of an intraoral lesion should be performed with which of the following solutions? A. 0.12% Chorhexidine B. 1% Iodine tincture C. 10% Hydrogen peroxide D. 3% Hydrogen peroxide E. Betadine Solution

D. 3% Hydrogen peroxide 1) Concept: Debridement of intraoral lesions are performed to remove dead and infected surfaces through the use of 3% hydrogen peroxide. 2) A mild concentration of hydrogen peroxide provides antimicrobial properties through the release of oxygen as it is applied to the infected wound. 3) Prolonged hydrogen peroxide use is NOT advised due to its ability to cause harm and irritation to oral tissues. 4) Higher concentrations of hydrogen peroxide are used as disinfectants in hospitals, as bleaching agents, and even in water waste management. 5) 1% tincture of Iodine or also known as Lugol's solution is a great antiseptic agent that has a wide spectrum of antimicrobial activity. It is used as a pre and post surgical mouth rinse to reduces chances of bacterial infection over the surgical site. 6) Chlorhexidine is an antibiotic rinse that would NOT help to remove dead tissues.

Although the ADA standards for shrinkage is 20 microns/cm, NO detrimental effects are associated with shrinkage less than which of the following amounts? A. 20 microns/cm B. 30 microns/cm C. 40 microns/cm D. 50 microns/cm

D. 50 microns/cm 1) Concept: The ADA standard for the polymerization shrinkage of composite resins is 20 microns/cm. However, values of polymerization shrinkage less than 50 microns/cm are acceptable haven NO detrimental effects. 2) The presence of inorganic filler particles throughout the composite structure reduces shrinkage of the restorative. Higher the volume of the inorganic fillers, the lower the shrinkage of composites. 3) Microhybrids and nano-hybrids demonstrate the highest percentage of inorganic filler particles and thus, the lowest percentage of polymerization shrinkage.

Which of the following describes when a clinician can reasonable expect a reduction in the size of a periodical lesion to be radiographically evident following endodontic therapy? A. 1-2 weeks B. 1-2 months C. 2-3 weeks D. 6-12 months E. > 1 year

D. 6-12 months 1) Concepts: A clinician can expect bone to be redeposited in the apical area following the removal of the infected tissues and obturation/swelling of the root canals. 2) When the lesion is healed, osteoblasts form a collagen matrix which is then calcified. 3) Calcification of the collagen matrix typically occurs within 6-12 months.

Which of the following describes the blade length on a cutting instrument with a four number formula of : 10 95 7 14 A. 1 mm B. 10 mm C. 14 mm D. 7 mm E. 9.5 mm

D. 7 mm G.V. Black's four number formula for instruments (that have a primary cutting edge or working end that is NOT at right angle to the long axis of the blade). a. First number: width of the blade in tenths of a millimeter b. Second number: cutting edge angle in centigrades c. Third number: length of the blade in millimeter d. Fourth number: blade angle in centigrades

Hospitalization is the MOST RECOMMENDED option to treat which of the following patients? B. A patient being treated with anticoagulants with a prothrombin time of 3.5x the control time requiring a single tooth extraction D. A severe developmentally disable patient requiring a full mouth gingivectomy to treat medication induced gingival hyperplasia

D. A severe developmentally disable patient requiring a full mouth gingivectomy to treat medication induced gingival hyperplasia 1) Full mouth gingivectomy is a serious periodontal surgical procedure requiring complete patient compliance and support. In a disable patient, general anesthesia would be carried out in a hospital set up and emergency equipment on hand.

Which of the following conditions does an anterior crossbite usually indicate is present? A. A Class II-division I malocclusion B. A digit sucking habit C. A self-correcting condition D. A skeletal growth problem E. A Class II-division II malocclusion

D. A skeletal growth problem 1) Anterior crossbite a. typically caused by a skeletal growth problem which affects the positioning of upper and lower front teeth b. cause by insufficient growth of maxilla while the mandibular growth appears to be normal c. could also be due to the more forward position and growth of the mandible compared to the maxilla

Cast gold restorations are indicated to replacing aging amalgams to... A. Achieve better margins B. Remove less tooth structure C. Traumatice the pulp less during preparation D. Achieve more ideal anatomy

D. Achieve more ideal anatomy 1) Concept: Cast gold restorations can achieve more ideal anatomic contours because they are made outside the mouth and under controlled conditions. 2) During the process of carving cast gold restorations, the master cast can be articulated properly and the contours can be corrected immediately prior to its placement in the mouth. 3) Carving and shaping of anatomical contours is done more accurately and efficiently outside the mouth. 4) Since cast restoration are fabricated on the master cast, the margins and the cavity prep will be clearly seen and proper extension of the restoration to the finish lines can be achieved.

Which of the following describes a decalcified and freeze-dried bone graft donated from a human to be graft to another human patient? A. Alloplastic graft B. Autograft C. Xenograft D. Allograft

D. Allograft 1) Concepts: Allograft are grafts taken from one person (usually a cadaver) and given to ANOTHER person. Bone allografts are often freeze-dried. 2) There are three types of bone allograft available: a. Fresh or fresh-frozen bone b. Freeze-dried bone allograft (FDBA) c. Demineralized freeze-dried bone allograft (DFDBA) 3) Autografts a. grafts taken from a part of the body of an individual and is transferred to another area of the body of the SAME person 4) Alloplasts a. an inert or non-reactive foreign material which is transplanted or implanted inside the body 5) Xenografts a. graft taken from a member of a certain species and is transferred to a recipient from a different species

Each of the following is a reason a clinician may section an impacted 3rd molar during the extraction procedure EXCEPT one. Which is the EXCEPTION? A. Minimize exertion of force necessary to extract the tooth B. Minimize quantity of bone removed * C. Minimize the time of the procedure * D. Allow tooth to be delivered from the occlusal aspect

D. Allow tooth to be delivered from the occlusal aspect 1) Sectioning a. Sectioning an impacted molar makes the extraction procedure easier and faster for both the clinic and the patient b. By sectioning the roots of the impacted 3rd molar, each root can be easily luxated and removed in the best manner possible without exerting too much force, thereby preventing trauma to hard and soft tissues c. Too much bone reduction can also be avoided with a planned root resectioning procedure of impacted teeth

Which of the following primarily contributes to the increase in height experiences by the maxillary bones? A. Apposition of the tuberosity B. Apposition of the anterior surface C. Sutural growth D. Alveolar growth

D. Alveolar growth 1) Concept: Increases in maxillary height are due to the growth of alveolar bone which provides enough space for the development and the eruption of teeth 2) Alveolar bone support the tooth as it erupts into the oral cavity. 3) Alveolar bond remodels and grows simultaneously with tooth eruption.

Which of the following is the MOST critical factor determining the prognosis of a periodontally involved tooth? A. Mobility of tooth B. Presence of furcation involvement C. Probing depth D. Amount of attachment loss

D. Amount of attachment loss 1) Attachment Loss a. determines the prognosis of periodontally compromised teeth b. higher the quantity of attachment, the better the prognosis c. teeth with severe attachment loss and vertical mobility presents a hopeless prognosis and must be extracted to avoid infection that may compromise other teeth

Which of the following should a dentist consider primarily when restoring the incisal edge or enameloplasty for a patient with slight chipping of the incisal edge of teeth #8 and #9? A. Height of the smile line B. Location of proximal contacts C. Shape of incisal embrasures D. Amount of translucent enamel present

D. Amount of translucent enamel present 1) Very important to consider morphology and translucency when restoring incisal edges of maxillary incisors 2) Thickness of enamel in the incisal surface area is greater than that of the facial and incisal surfaces 3) Difference in the thickness of enamel in this area make it appear more radiolucent.

A surgical extraction of impacted 3rd molars would be contraindicated if the patient had... A. A fever of undiagnosed etiology B. A history of anemia C. A necrotic pulp D. An oral ulcer of unknown origin E. Hemophilia or was taking blood thinners

D. An oral ulcer of unknown origin 1) Concept: Oral ulcers of unknown origin a. can indicate an underlying systemic condition that needs to be diagnosed prior to any surgical procedure. b. oral ulcers can be associated with viral infections like herpes and HIV, fungal disease, bacterial infections or might be due to mucosal trauma. 2) Surgical extractions can be performed for the following patients with an appropriate medical clearance from the patient's physician. a. Hemophiliacs a. patients can be managed with medications involving transexamic acid and infusion of clotting factor VIII or IX prior to extraction improves clotting time and reduces bleeding time b. Anemic a. Patients can be managed through prophylactic antibiotics prior to extraction with antibiotics post extraction to avoid the possibility of having bacterial endocarditis. c. Fever of undiagnosed etiology a. Patients should be evaluated by their physician first to determine the etiology of the fever. Tooth extraction may be performed only if the patient is already cleared by the physician and is afebrile. d. Necrotic pulps are not a contraindication to dental extraction.

Which of the following describes a major difference between a Class V preparation form amalgam versus a composite restoration A. Convenience form B. Depth of the preparation C. Outline form D. Angulation of the enamel cavosurface margins

D. Angulation of the enamel cavosurface margins 1) Concept: Amalgam restorations always require 90 degree cavosurface margins because amalgam has weak edge strength 2) Enamel cavosurface margins for an amalgam restoration should have no bevel to avoid amalgam fracture due to lack of minimum thickness of material for strength 3) The cavosurface margin preparation for Class V composite restorations require a 45 degree bevel to provide additional surface for adhesion as well as to create an esthetically blended junction between the tooth and the composite material.

A patient reports moderate intermittent pain and popping/clicking associated with their right pre auricular area. The patient has a 45mm maximum incisal opening. What is the most likely reason for the patient's discomfort? A. Anterior disc displacement without reduction B. Coronoid hyperplasia C. Myofascial pain disorder D. Anterior disc displacement with reduction

D. Anterior disc displacement with reduction 1) Anterior disc displacement with reduction a. articular disc is located anterior to the condyle head during opening of the jaw which results in limited mouth opening without any clicking or popping sound 2) Anterior disc displacement without reduction a. observed in patients who experience pain and a popping sound or clicking sound upon opening and closing their mouth b. popping sounds indicate that the condyle head is moving back against the disc, and as the mouth closes, the condyle head slides off to the posterior aspect of the disc creating another clocking sound 3) Myofascial pain syndrome a. condition where the pain is felt along the TMJ area associated with the muscles of mastication

Which of the following valve types should a dentist use on their hand pieces and water spray hoses to prevent cross-contamination to patients from dental operatories? A. Depressurization B. Pressure C. Retraction D. Anti-Retraction

D. Anti-Retraction 1) Anti-retraction valves a. prevent retraction of fluid from a patient into the hand piece and water spray hoses. b. create one way flow of liquids which prevent bodily fluids like saliva from being retracted from the patient into the hand piece and water hose.

Which of the following would most likely be the result of inadvertently sealing a small occlusal carious lesion on a maxillary 1st premolar? C. Progression of the carious lesion D. Arrested caries

D. Arrested caries 1) Sealing a carious lesion a. deprives the bacteria of resources, resulting in arrested dental caries b. sealing occlusal pits and fissures terminates the progression of caries, but also help the tooth remineralize and recover 2) Arrested caries a. may appear darker due to staining from recent dimeralization, and does not necessary require restoration b. teeth arrested occlusal caries should be sealed to protect the tooth from further damage.

Which of the following clinical findings is considered to have the greatest influence on the design of periodontal flap surgeries? D. Attached gingiva

D. Attached gingiva 1) Attached gingiva a. function is to keep the free gingiva in close contact with the tooth while also protecting the soft and free moving alveolar mucosa b. insufficient attached gingiva makes the mucosa more prone to trauma during function and compromises the periodontal support of teeth c. width of the attached gingiva must be preserved

Which of the following describes the MOST COMMON cause of intraoral verruca vulgaris? A. Chronic alcoholism B. Pipe smoking C. Trauma D. Autoinoculation

D. Autoinoculation 1) Concept: Autoinoculation is the most common cause of intraoral warts causing verruca vulgaris 2) The human papilloma virus (HPV) virus self implants inside the mucous epithelium, causing infection. 3) The transmission or auto inoculation of the wart can occur by: a. mechanical touching of one part of the organism to another b. friction removing a part of the infected cells to another surface and then reintroducing these cells upon contact with another body part c. transportation though the blood stream

When finishing the occlusal surface on a posterior composite, the dentist should... A. Develop centric contacts on cavosurface margins B. Eliminate contacts in the fossae C. Make sure to include protrusive contacts into the restoration D. Avoid altering the centric contact on enamel

D. Avoid altering the centric contact on enamel 1) Concept: The centric contact in enamel should be preserved since it establishes the original and proper occlusal contact between maxillary and mandibular teeth. 2) Increasing the intensity of centric contacts during restorative treatments may cause premature contact, occlusal disharmony and eventually dental trauma. 3) Decreasing of removing the centric contact in enamel may cause occlusal disharmony and unstable centric occlusion.

Loss of a tooth letter L in a 3 year old patient requires... A. Distal shoe placement B. Lower lingual holding arch C. No answers are correct D. Band and loop placement

D. Band and loop placement 1) Concept: Placement of a band and loop may help prevent the mesial movement of K into the space created by extracting L. 2) It is important to keep or maintain this space to prevent tooth crowding when the succedaneous counterpart erupts. 3) Keeping the remaining teeth in proper position will also guide the eruption of permanent 1st molar and prevent it from moving medially. 4) The space of tooth letter L can be maintained by placing band/crown loop on tooth K-M. This should be replaced later with lower lingual holding arch as soon as the other lower permanent incisors erupt.

Which of the following is NOT an endogenous opioid? A. Dynorphin B. Endorphin C. Enkephalin D. Bardykinin

D. Bradykinin 1) Endogenous Opioid a. Endorphine b. enkephalin c. dynorphin d. endomorphine e. all the above are endogenous opioid peptides created naturally within the human body f. help increase the pain threshold and also have a potent analgesic effect in the body 2) Bradykinin a. is NOT an endogenous opioid but rather a kinin, or active polypeptide in the body that is created in the blood b. is a mediator of the inflammatory reaction that can also induce vasodilation and increase vascular permeability

Which of the following locations is ideal for the placement of a microfilm composite? A. Class I B. Class III C. Class IV D. Class V

D. Class V 1) Microfill composites are considered an appropriate choice for restoring Class V cervical lesions or defects in which cervical flexures can be significant. 2) This is true for cases of bruxism, clenching and stressful occlusion. Their low modulus of elasticity allows them to flex during tooth flexure, better protecting the bonding interface. 3) Microfilled composites also provide a smooth and polished surface in the finished restoration that is less receptive to plaque.

Which of the following permanent tooth pulp horns is at the most risk for a potential accidental exposure during a preparation for a Class II amalgam? A. Distofacial of a mandibular 1st molar B. Distofacial of a maxillary 1st molar C. Lingual of a mandibular 1st premolar D. Buccal of a mandibular 1st premolar

D. Buccal of a mandibular 1st premolar 1) When making a Class II cavity preparation, the pulpal floor along the buccal cusp of the mandibular 1st premolar should be higher than the lingual to avoid encroaching on its prominent buccal pulp horn. 2) Mandibular premolars have a more prominent buccal cusp and consequently, a prominent buccal pulp horn. 3) The position of the pulp follows the external anatomy of the tooth which must always be considered during preparation of cavities

Which of the following is the primary photo initiator used to initiate curing in resin composites? A. Benzoyl Peroxide B. Bezoyl methyl ether C. Bis-phenol A D. Camphoroquinone

D. Camphoroquinone 1) Most composite resins currently used employ camphoroquinone as the primary photo initiator. 2) Camphoroquinone a. absorbs photons of light energy (predominantly at about 470 nm). 3) Curing lights a. The manufacturer of light curing lights attempt to maximize the light in the absorption range of the photo initiator within the composite being cured. 4) Benzoyl peroxide a. is used as an initiator along with tertiary amine activator in chemical or self cure composites

What structure(s) must be cut in order to enter the superior joint space of the TMJ? A. Capsular and discal ligaments B. Capsular and retrodiscal ligaments C. Discal ligament only D. Capsular ligament only

D. Capsular ligament only 1) Concept: The superior joint space is encompassed by the capsular ligament and must be cut in order to access the superior joint space. 2) The capsular ligament completely encircles the temporomandibular joint and prevents over extensive movements that may cause the TMJ to dislocate.

Geriatric patients are LEAST likely to suffer rampant caries due to which of the following factors? A. Decreased salivary flow B. Medication side effects C. Poor oral hygiene D. Changes in the oral microflora

D. Changes in the oral microflora 1) Oral microflora of younger people is usually more cariogenic than older ones due to the inclusion of sweets in their daily diet. Older people tend to eat less sweets in their diet making them less prone to caries.

Hand instruments classified as "angle formers" are most common used for... A. Anterior Class III composite resin preparations B. Anterior Class III direct filling gold preparations C. Cast onlays D. Class II amalgam preparations

D. Class II amalgam preparations 1) Angle formers a. hand instruments with their cutting edge sharpened at an angle to the axis of the handle b. used to create bevels, refine pulpo-axial line angles, and smooth margins c. also called "marginal trimmers"

Which of the following periodical conditions is associated with vital pulps most often? A. Acute apical abscess B. Apical Cyst C. Chronic apical abscess D. Condensing Osteitis E. Apical Scar

D. Condensing Osteitis 1) Condensing Osteitis a. periapical inflammatory disease resulting from a reaction to an infection of PERIODONTAL origin, leading to bone apposition near root apices b. common in premolars and molars c. lesion appears as a radiopacity in the periodical area due to sclerotic reaction d. sclerotic reaction results from a strong resistance of the patient and a low degree of virulence of the offending bacteria

The alterations in the continuity of the occlusal plane after a tooth is ankylosed is primarily caused by which of the following? A. Ankylosed teeth moving apically as the root becomes incorporated into the alveolar bone B. Differential eruption sequences C. Localized inhibition of alveolar process growth D. Continued eruption of nonankylosed teeth and growth of the alveolar process

D. Continued eruption of nonankylosed teeth and growth of the alveolar process 1) Concept: When teeth are ankylosed, they usually lie below the occlusal plane because the supporting alveolar process does not grow 2) Alveolar process growth occurs simultaneously with the eruption of teeth. Ankylosed teeth do not experience alveolar process growth because of their failure to erupt the same way as the other non-anklyosed teeth due to a lack of periodontal ligament. 3) Ankylosed teeth are teeth with roots completely embedded into the bone without periodontal ligament.

Which of the following cysts is an ameloblastoma most likely to develop from? A. Nasopalatine duct B. Radicular C. Traumatic bone cyst D. Dentigerous

D. Dentigerous 1) Ameloblastoma a. rare benign odontogenic tumor derived from epithelium that occurs more commonly in the mandible than the maxilla b. often associated with unerupted teeth c. rarely malignant or metastatic d. expand the bony cortices at a slow growth rate that allows time for the periosteum to develop a thin shell of bone that cracks when palpated. e. appear multiloculated with "soap bubble" appearance f. resorption of root of involved teeth can be seen g. Symptoms include: 1) painless swelling 2) facial deformity 3) loose teeth 4) ulceration 5) periodontal disease

Which of the following agents is NOT an opioid utilized in outpatient anesthesia? A. Fentanyl B. Meperidine C. Sufentanil D. Diazepam

D. Diazepam 1) Diazepam a. anxiolytic b. anticonvulsant c. hypnotic d. sedative e. skeletal muscle relaxant f. amnestic properties g. has several pharmacologically active metabolites, thus has a long pharmacologic half-life h. has a long duration of action, this drug is available in low doses to prevent drug accumulation and an unnecessarily prolonged effect 2) Opioids a. molecules that are known to bind to the opioid receptors to cause b. anesthesia c. sedation d. respiratory depression e. constipation f. strong sense of euphoria in patients

The Occupational Safety and Health Agency (OSHA) regulates each of the following through the blood borne pathogen standard EXCEPT one. Which is the EXCEPTION? A. Communicating hazards to employees B. Performing housekeeping C. Using barrier techniques D. Using material safety data sheets (MSDS) E. Obtaining hepatitis B vaccinations

D. Using material safety data sheets (MSDS) 1) MSDS is not regulated based on the blood borne pathogen standard.

Which of the following is NOT an advantage of oral sedation? A. Less training and costly equipment B. Lower rate of adverse drug reaction C. Shorter drug half life D. Differences in gastrointestinal drug absorption rate

D. Differences in gastrointestinal drug absorption rate 1) Differences in the rate of gastrointestinal drug absorption among patients account of the greatest disadvantage of oral sedation 2) Drug absorption and metabolism varies widely between individuals, so the duration of effect can only be estimated. 3) Oral sedation is still useful because: a. well regulated drugs b. shorter drug half-life c. less chance of drug toxicity and adverse reactions d. does not require expensive equipment

Which of the following characterizes the primary source of nutrients of a free gingival graft for the first 24 hours? A. Diffusion of nutrients from the vessels of the periodontal ligament B. Primary vascular anastomosis C. Residual nutrients within the graft D. Diffusion of nutrients from the underlying connective tissue

D. Diffusion of nutrients from the underlying connective tissue 1) Concept: Free gingival grafts gain their nutrients from the recipient site through plasmatic diffusion. 2) During the first 24 hours, there is no vascular connection between the free gingival gray and the recipient site. 3) The forming capillary anastomoses provides the source of nutrients for free gingival grafts after 3 days post surgically.

Which of the following treatments is indicated for a patient with a 1mm pulp exposure created when their cusp fracture off 30 minutes ago above the gumline? A. Pulpectomy with apexification when necessary B. Pulpotomy with calcium hydroxide C. Pulpotomy with formocresol D. Direct pulp cap with calcium hydroxide

D. Direct pulp cap with calcium hydroxide 1) Direct pulp cap a. calcium hydroxide should be placed above the exposure site and it should be sealed with immediate restorative material (IRM) b. patient needs to be evaluated clinically and radiographically after 4-6 weeks c. direct placement of calcium hydroxide over the exposure site will stimulate reparative dentin formation in a few weeks and is known as direct pulp capping d. Micro exposures of the pulp during cavity preparation or trauma DO NOT ALWAYS necessitate root canal treatment

Which dental procedure is a rubber dam most indicated... B. Indirect pulp capping D. Direct pulp capping

D. Direct pulp capping 1) Direct pulp capping a. essential to have adequate access, visibility, and isolation of the operating field b. is indicated if a pin-point exposure is made during a cavity preparation c. teeth should be isolated to avoid saliva and bacterial contamination during direct pulp caps

Each of the following can occur during scaling and root planing procedures EXCEPT ONE. Which is the EXCEPTION? A. Calculus removal B. Cementum removal C. Dentin removal D. Enamel removal E. Soft tissue Damage

D. Enamel removal 1) Concept: Enamel removal during scaling and root planing is NOT possible since enamel is highly mineralized structure that is too hard to be removed by such dental procedure. 2) The primary role of scaling and root planing is to remove all plaque and calculus that is attached to the tooth surface and to be able to create a surface and glass-like root surface which is conducive for good gingival health. 3) Creating a smooth and glass like root surface during the root planing procedure allows the formation of new cementum and reattachment of the supporting gingiva through the formation of long junctional epithelium. 4) The scaling and root planing procedure should aways be performed under local anesthesia since it involves removal of calculus and plaque along the cementum. Soft tissue damage, removal of some cementum and dentin are some time inevitable during the scaling and root planing process.

Elevators are instruments that are designed to... A. Engage the tooth coronal to the cementoenamel junction D. Engage the tooth apical to the cementoenamel junction

D. Engage the tooth apical to the cementoenamel junction 1) Elevators a. instruments used to elevate the tooth out of its socket b. designed to engage the root portion of the tooth below the cementoenamel junction c. inserted into the periodontal ligament space, create more room for tooth movement, and help elevate the tooth out of its socket

Tooth #19 is being prepared for an occlusal amalgam restoration. Caries remains in the facial, pulpal, and lingual walls after the ideal outline form and depth were established. Which of the following is the next step in the treatment of this tooth? A. Remove caries and prepare the tooth for a crown B. Remove the caries with a larger round bur C. Remove the caries with a spoon excavator D. Extend the outline form

D. Extend the outline form 1) Concept: The outline form should be extended to completely remove all the infected dentin and enamel. 2) Amalgam restorations require solid parallel walls with no unsupported enamel to avoid failure of the restoration or tooth fracture under function. 3) Proper extension of the outline form when removing carious tooth structure should be done to prevent leakage and recurrence of caries.

If a patient presents to your office with a periodical radiolucency associated with a tooth demonstrating a midroot fracture, where the lower portion of the tooth is nonnegotiable with files, the indicated treatment is... A. Apical surgery with removal of the apical segment root canal treatment of the coronal segment B. Apical surgery with reverse filling C. Calcium hydroxide treatment of the coronal segment only D. Extraction with removal of the apical segment and replantation of the coronal segment E. Extraction of both tooth segments

D. Extraction with removal of the apical segment and replantation of the coronal segment 1) Treatment with mid-root fracture a. to counter-act the possibilities of having failed treatment for teeth with mid root fractures, intentional replantation is conducted 2) Intentional replantation a. indicated when the apical half of the root of the tooth cannot be engaged, cleaned, and shaped b. extract the apical segment and replant the coronal segment that recently received a root end filling

Which type of failure would describe the situation where a diagnostic test failed to identify several cases of true disease? A. False positive B. True negative C. True positive D. False negative

D. False negative 1) True Positive: those who test positive for a condition and are positive (have the condition) 2) False Positive: those who test positive for a condition but are false (do not have the condition). Test indicates presence of the disease when it is absent 3) True Negative: those who test negative and are negative 4) False Negative: those who test negative but are positive. The test indicates absence of the disease when it is present.

In the United State populace, which of the following describes the MOST COMMON location premalignant lesions of squamous epithelium are observed? A. Soft Palate B. Buccal mucosa C. Dorsum of the tongue D. Floor of the mouth E. Gingiva F. Hard palate

D. Floor of the mouth 1) Concept: The floor of the mouth and lateral border of the tongue are the most common locations for the development of squamous cell carcinoma inside the mouth. 2) An explanation for the high potential for premalignant lesions in this area is due to its rich blood supply because rapidly dividing malignant cells require more nutrition than normal cells. 3) If an ulcer or lesion develops on the floor of the mouth, healing is difficult because of the high degree of mobility of the tongue and floor of the mouth.

The formation of which of the following induces sealants to polymerize? A. Bis-GMA B. Camphorquinone C. Filler particles D. Free radicals E. Polymethymethacrylate

D. Free radicals 1) Concept: The formation of free radicals induces the polymerization of pits and fissure sealants. 2) Sealants a. are the most effective clinical technique to proven tpit and fissure caries b. the cost effectiveness of sealants is based upon sealant retention c. classified as self-cure or auto-cure sealants and visible light cure sealants d. Camphorquinone is the most common photo initiator found in resins and is used because its decomposition creates free radicals

Which of the following locations would a perforation present the HIGHEST failure risk? A. Apex B. Apical 1/3 of the root C. Crown D. Furcation E. Middle 1/3 of the root

D. Furcation Perforations a. HIGHEST risk for endodontic failure is the perforation through the furcation because it is more difficult to seal b. Perforations near the root apex can be sealed during obturation. c. Teeth with endodontic perforations may still be successful. d. NOT a guarantee of endodontic failure

Which of the following phases of amalgam is considered the weakest? A. Beta B. Gamma C. Gamma-1 D. Gamma-2 E. Alpha

D. Gamma-2 1) Concept: Gamma-2 phase is the weakest and has the least resistance to corrosion 2) Gamma-1: has the highest strength and corrosion resistance 3) Gamma: is the amalgam phase which is considered unreacted during the chemical reaction between amalgam alloy and mercury.

The duration of action for a single IV bolus dose of diazepam is most dependent on... A. Alpha half-life B. Beta half-life C. Enzymatic degradation D. Hepatic biotransformation E. Renal excretion

D. Hepatic biotransformation 1) Diazepam a. undergoes oxidative metabolism by demethylation, hydroxylation, and glucuronidation in the liver as part of the cytochrome P450 system b. has several pharmacologically active metabolites, the chief one being desmethyldiazepam c. metabolites are conjugated with glucuronide and are excreted in urine d. highly lipid-soluble e. widely distributed throughout the body after administration f. primarily distributes to muscle and adipose tissue

Which of the following portions of a forceps is most likely to accidentally crush the lip of a patient? A. Beak B. Balde C. Handle D. Hinge E. Shank

D. Hinge 1) Hinge a. of the forceps lies nearest to the lips during the extraction procedure and is the most likely portion to cause lip injury b. adaptation of the forces beak and blade into the tooth will have the hinge approximating the lower lip, especially during the extraction of posterior teeth 2) Handle a. is held by the dentist and will less likely cause lip damaged when properly handled.

Which of the following factors of caries susceptibility is modified most by fluoride therapy and occlusal sealants? A. Microflora B. Substrate C. Time D. Host

D. Host 1) Hydroxyapatite crystals are the building blocks of tooth enamel. 2) Fluoride replaces the hydroxyl ions with fluoride ions, resulting in the formation of fluorapatite instead of hydroxyapatite.

Of the following anomalies, the one which normally has a bacterial etiology is: A. Compound odontoma B. Dentinogenesis imperfecta C. Fusion D. Hutchinson's incisors E. Enameloma

D. Hutchinson's incisors 1) Congenital Syphilis a. associated with Hutchinson's incisors occurring in month 4-6 in utero before calcification begins. b. Incisors and 1st molars are most commonly affected because they are the only permanent teeth at this stage of development c. incisors : have a notch in the center of the incisal edge, making them appear like a screwdriver d. 1st molars: typically appear with poorly developed cusps and gnarled enamel, giving arise to the name "mulberry molars" e. primary teeth are topically NOT affected f. esthetics is the primary concern of patients with this condition 2) Enamelomas (Enamel pearls) a. form during tooth development when the cells of the epithelial root sheath remain stuck to predentin b. then they differentiate into ameloblasts and deposit enamel where it normally would not.

Fracture of which of the following would be suggested if a patient experiences paresthesia of their left upper lip, cheek, and left side of the nose after they suffered a midface fracture. A. Maxillary sinus B. Nasal bone C. Zygomatic arch D. Infraorbital rim

D. Infraorbital rim 1) Concept: Paresthesia of the upper lip, cheek, and left nose is due to the damage incurred by the infraorbital nerve. 2) Infraorbital nerve exits the infraorbital foramen and branches out to innervate the upper lips, cheeks and nose up the nasal vestibule. 3) A fracture along the infraorbital rim may result in damaged to the infraorbital nerve which causes paresthesia of the structures it innervates

It is indicated that each of the following types of syncope is treated with the administration of high-flow 100% oxygen EXCEPT ONE. Which is the EXCEPTION? A. Neurogenic B. Orthostatic C. Vasovagal D. Hyperventilation Syndrome

D. Hyperventilation Syndrome 1) Concept: Hyperventilation Syndrom (HVS) is a condition of insufficient CO2, NOT O2, so additional oxygen is CONTRAINDICATED. 2) Hyperventilation Syndrome (HVS) is a psychologically or physiologically based respiratory disorder involving a patient breathing too deeply and/or too rapidly. HVS may present with chest pain and a tingling sensation in the fingertips and around the mouth and may accompany a panic attack. 3) HVS patients a. have roughly the same oxygenation in the arterial blood (98% for hemoglobin saturation) and insufficient CO2 in their blood and other tissues. b. while oxygen is abundant in the bloodstream, HVS reduces effective delivery of that oxygen to vital organs due to low-CO2-induced vasoconstriction and the suppressed Bohr effect.

Each of the following is an advantage of using the acid-etch technique for composite resin restorations with minimally cavity preparations EXCEPT one. Which is the EXCEPTION? A. Improved esthetic outcomes B. Improve retention of the restoration C. Maximum conservation of tooth structure D. Improved access for finishing of the restoration

D. Improved access for finishing of the restoration 1) Acid etching a. creates microporosities that adhesive can infiltrate to create retention for composite resin bonding b. roughens the enamel rods and dentin thus creating micro mechanical abrasions that allows the bonding agent to flow and adhere to the tooth surface c. removes surface debris, allowing for a clean bonding site for improved esthetics d. creates higher bond strength between composite restorations and tooth structure and decreases micro leakage e. DOES NOT IMPROVE ACCESS to the restoration

Each of the following is an advantage indirect composite restorations over direct placement composite restorations EXCEPT one. Which is the EXCEPTION? A. Decreased marginal leakage B. Decreased postoperative sensitivity C. Improved proximal contacts D. Increased polymerization shrinkage E. Increased strength

D. Increased polymerization shrinkage 1) Concept: The primary advantage of indirect resin restorations over direct resin restorations are: a. Improved proximal contacts b. Decreased marginal leakage c. Increased strength d. Decreased postoperative sensitivity 2) The primary disadvantage of indirect resin restorations are: a. time b. patient doesn't leave the office with the definitive restoration

Which of the following is the mechanism by which mercury enters the body that is considered to induce the highest toxicity? A. Absorbed through the skin B. All mechanisms of absorption are equally toxic C. Ingestion D. Inhalation

D. Inhalation 1) Most toxic form of mercury to enter the body is through inhalation of mercury vapors 2) 80% of elemental and inorganic mercury is absorbed in the lungs.

A patient presents with continuous and spontaneous pain associated with the maxillae left central incisor. Tooth #9 has a large Class V composite restoration placed many years ago. Cold testing elicits a lingering pain. There is no sensitivity to percussion or palpation. What is the most probable diagnosis? A. Irreversible pulpits with symptomatic apical periodontitis B. Necrotic pulp with asymptomatic apical periodontitis C. Reversible pulpits with a normal periapex D. Irreversible pulpits with normal periapex

D. Irreversible pulpits with normal periapex Diagnosis 1) Irreversible pulpits a. condition known to elicit spontaneous lingering, and throbbing pain sometimes without a stimulus 2) Continuous pain a. experienced by the patient is an indication that the pulp is already inflamed and irritated even without external stimulus b. large and old Class V restoration could have lost its seal and served as an opening for bacterial invasion.

The condition known as "twinning" normally: A. Is the result of a bacterial etiology B. Occurs only in posterior teeth C. Results in one less than the normal number of dental units in the involved arch D. Is also called gemination E. Is the result of a complete fusion of two teeth

D. Is also called gemination 1) Concept: Gemination (twinning) a. developing tooth splits off into two distinct teeth that are attached to each other by pulp or dentin b. Think of twinning as Siamese twin teeth c. have same number of tooth and two separate root canals 2) Fusion a. where two separate buds join together b. This leaves one tooth less than normal in the dentition c. have one less tooth (in the normal dentition) separate root canals

Which of the following describes why it is recommended that a matrix for a proximal restoration (Class II or III) be taller than the adjacent marginal ridge being restored? A. It allows the wedge to adapt more to the proximal area B. It prevents escape of the amalgam during condensation C. It serves as a template to determine the complete restoration height D. It allows for overfilling of the amalgam so it can be properly condensed and shaped to the proper conformation

D. It allows for overfilling of the amalgam so it can be properly condensed and shaped to the proper conformation 1) Concept: The matrix band should be placed slightly higher than the adjacent marginal ridge during the placement of proximal restorations which allows the clinician to achieve overfilling of the cavity preparation. 2) Overfilling the amalgam in proximal restoration is necessary because it allows for the: a. proper carving of the amalgam restoration b. burnishing and achieving the contact point c. ease in the restoration of tooth morphology

Each of the following characterizes the experimental gingivitis model EXCEPT one. Which is the EXCEPTION? A. It demonstrates that gingivitis is a reversible disease B. It demonstrates that the bacterial ecology changes as plaque accumulates C. It demonstrates the relationship between plaque formation and gingivitis D. It proves that gingivitis progresses to peridontitis E. It support the non-specific plaque hypothesis

D. It proves that gingivitis progresses to peridontitis 1) Gingivitis does not always evolve to periodontitis 2) Experimental gingivitis model a. supports the non-specific plaque hypothesis b. demonstrates the relationship between plaque formation and gingivitis c. demonstrates that the bacterial ecology changes as plaque accumulates d. demonstrates that gingivitis is a reversible disease

Which of the following statements does NOT describe the experimental gingivitis model? A. It demonstrates that gingivitis is a reversible disease B. It demonstrates that the bacterial ecology changes as plaque accumulates C. It demonstrates the relationship between plaque formation and gingivitis D. It proves that gingivitis progresses to periodontitis E. It supports the non-specific plaque hypothesis

D. It proves that gingivitis progresses to periodontitis 1) Gingivitis does not always evolve to periodontitis 2) Experimental gingivitis model a. supports the non-specific plaque hypothesis b. demonstrates the relationship between plaque formation and gingivitis c. demonstrates that the bacterial ecology changes as plaque accumulates d. demonstrates that gingivitis is a reversible disease

Each of the following is an indication for resin composite in posterior teeth EXCEPT one. Which is the EXCEPTION? A. Buccal-lingual width of preparation is less than 1/3 intercuspal distance B. Esthetics are important C. Gingival margin of preparation is on intact enamel D. Large preparations

D. Large preparations 1) Concept: Large cavity preparations are NOT an indication for composite in posterior teeth. 2) Small to moderate sized cavities in posterior teeth should be restored with RBC. 3) Composites in posterior teeth are indicated if: a. Buccal-lingual width of preparation is <1/3 inter-cuspal distance b. Gingival margin of preparation is on intact enamel c. Esthetics are important

Broad knowledge of which of the following pairs is imperative to understand the development of human behavior? A. Dependence and independence B. Ethnicity and Gender C. Facilitation and generalization D. Learning and maturation

D. Learning and maturation 1) Learning and maturation a. importan pre-requisites for understanding the development of human behavior b. growth of the structure of the human body makes it possible to perform several activities and these activities make learning possible c. learning is extremely important for acquiring new knowledge, skills, and attitudes that are necessary for an effective life d. maturation is developmental process within which a person from time to time manifests different traits present in his cells

Barrier or Teflon membranes are utilized by dentists to treat osseous defects through attempting to restrict which of the following? A. Connective tissue attachment B. Coronal migration of cells from the periodontal ligament C. Epithelium from sticking to the bone D. Long junctional epithelium formation E. Osseous regeneration

D. Long junctional epithelium formation 1) The reattachment of gingiva to the cementum through the formation of long junctional epithelium should be restricted in order to allow the newly formed periodontal ligament fibers to interlock with the cementum and create new attachments. 2) Barrier or Teflon membranes are placed to allow cultivation and growth of new attachments without any interference from the long junctional epithelium. 3) Guided tissue regeneration procedure requires the use of barrier membranes in order to guide or direct the growth of new forming bone and soft tissue. 4) Barrier membranes also stops the possible migration of epithelial cell toward the newly formed cementum layer, thus preventing the formation of the long junctional epithelium.

A patient presents with a carious lesion that my result in a pulp exposure upon preparation for an MOD amalgam restoration on a vital and asymptomatic tooth. Which of the following should the clinician do? A. Leave the infected dentin in the deeper areas, temporize the tooth and observe it for 2 weeks then restore or initiate root canal treatment. B. Remove the carious dentin completely then initiate root canal treatment C. Remove the infected dentin laterally first before removing it completely from the deeper parts of the carious lesion D. Leave the affected dentin in the deeper areas, apply a base material, then restore.

D. Leave the affected dentin in the deeper areas, apply a base material, then restore. 1) Indirect Pulp capping a. conservative treatment of choice for teeth presenting with deep carious lesions in close proximity to the dental pulp b. enhances the formation of reparative dentin c. if indirect pulp capping fails, root canal treatment should be performed d. involves the removal of soft carious dentin (infected dentin) without exposing the pulp tissue. Hard dentin (affected dentin) is NOT removed completely. Following excavation, a sedative base is placed and the restoration is completed. e. it is CRITICAL that the margins of the preparation are in CLEAN TOOTH STRUCTURE WITH NO CARIOUS ACTIVITY PRESENT, otherwise the procedure has a significantly higher failure rate.

The closure of which of the following spaces results in the late mesial shift of permanent 1st molars? A. Canine B. Extraction C. Interocclusal D. Leeway E. Primate

D. Leeway 1) Concept: Leeway space is closed as the 1st permanent molars move medially after the exfoliation of the primary 2nd molar and is usually seen among children from 11-12 years of age. 2) Leeway space: a. created from the difference of sizes between the primary canine and molars compared to the erupting premolars and permanent canines b. The discrepancy in size of primary molars and premolars created a free space that allows the late medial migration of permanent 1st molar after the exfoliation of the primary 2nd molar.

Which of the following is the most like fracture site if a patient present to you after receiving a heavy blow to the chin that causes their mandible to deviate to the left upon opening? A. Left body of the mandible B. Right body of the mandible C. Right subconydylar region D. Left subcondylar region

D. Left subcondylar region 1) Concept: Suncondylar fractures are diagnosed in trauma cases by a deviation of the mandible to the side of a fracture because subcondylar fractures leads to improper translation of the mandible, creating deviation. 2) Protrusive, extrusive, and opening movements can be produced by the contraction of the lateral pterygoid muscles. 3) During right excursive movements when the mandible moves to the right, the left lateral pterygoid contacts pushing the mandible downward and forward. The right lateral pterygoid is relaxed and does not antagonize the voce, causing the asymmetrical depression and protrusion of the mandible.

Degeneration of the basal cell layer, a mononuclear inflammatory cell infiltrate in the sub epithelial connective tissue, and "Saw-tooth" tete pegs are indicative of which of the following mucocutaneous disorders? A. Dermatitis herpetiformis B. Erytheme multiforme C. Keratosis follicularis D. Lichen planus E. Benign mucus membrane pemphigoid

D. Lichen planus 1) Concept: Lichen Planus a. Lichen Planus is a chronic mucocutaneous inflammatory disease that affects the skin, tongue, and oral mucosa of unknown origin and presents with 3 variations. b. name refers to the dry and undulating, "lichen-like" appearance of affected skin c. It is sometimes associated with oxidative stress, certain medications and diseases 2) Reticular Lichen Planus a. most common and manifests as white lacy streaks on the mucosa (known as Wickham striae) or as smaller papules.. The lesions tend to be bilateral and are symptomatic. 4) Bullous Lichen Planus a. presents as fluid-filled vesicles which project from the surface 5) Erosive Lichen Planus a. presents with painful erythematous ulcerated lesions which may occur in many areas of the mouth, but mostly on the posterior buccal mucosa and may resemble desquamative gingivitis. Wickham's striae may also be seen near these ulcerated areas 6) Biopys a. is recommended to rule our precancerous (premalignant) change of persistent lesions similar to erosive lichen plans that do not respond to topical corticosteroids

Each of the following is mode of action for a magneto restrictive ultrasonic instrument EXCEPT one. Which is the EXCEPTION? A. Acoustic turbulence B. Cavitation C. Lavage D. Linear movement E. Vibration

D. Linear movement 1) Concept: Magnetostrictive ultrasonic instruments (cavitrons) are used to remove heavy deposits of calculus and stain via an elliptical motion. 2) Cavitrons work by: a. Lavage: flossing of the pocket b. Cavitation: ultrasonic movement creates air bubbles that knock deposits off tooth structure c. Vibration: mechanically removes deposits and debris d. Acoustic turbulence: agitation observed in fluids by mechanical vibrations disrupts bacterial cell walls 3) Piezo instruments a. uses a linear movement to create a back-and-forth motion to crete their ultrasonic movement

Acute suppurative osteomyelitis is commonly associated with each of the following EXCEPT ONE. Which if the following is the one EXCEPTION? A. Deep and intense pain B. High or intermittent fever C. Paresthesia or anesthesia of the inferior alveolar D. Loose teeth with suppurative drainage from the periodontium

D. Loose teeth with suppurative drainage from the periodontium 1) Concept: Loose teeth with suppurative drainage from the periodontium is caused by periodontitis, NOT acute osteomyelitis. 2) Acute osteomyelitis a. a disease that mostly affects children and is usually correlated with a Staphylococcus aureus infection. b. may also occur in adults with compromised immune system, immunosuppressive therapy, or steroid therapy.

Each of the following nerves have branches which innervate the temporomandibular joint (TMJ) except one. Which nerve is the EXCEPTION? A. Auriculotemporal B. Deep temporal C. Mandibular division of the trigeminal (V3) D. Marginal mandibular E. Masseteric

D. Marginal mandibular 1) Concept: The marginal mandibular branch is located within the anterior aspect of the face and neck and supplies the lower lip muscles and chin. 2) The mental is, depressor anguli iris, and depressor labii inferiors are muscles that are innervated by the marginal mandibular branch of the facial nerve. 3) The masseteric nerve, auriculotemporal nerve, and mandibular division of the trigeminal nerve have branches that innervate the TMJ.

Which of the following teeth responds the LEAST to rotational forces during extraction procedures due to its root structure? A. Mandibular 1st premolar B. Mandibular 2nd premolar C. Mandibular central incisor D. Maxillary 1st premolar E. Maxillary central incisor F. Mandibular canine

D. Maxillary 1st premolar 1) Concept: The buccal and lingual placement of the roots and tendency to fracture decreases the usefulness of rotational extraction forces. 2) Maxillary 1st premolars a. are known to fracture during extractions due to the presence of 2 thin roots. b. have variable root forms and normally have two root canals and two roots c. due to the thinness of the tooth root of maxillary premolars, the roots are easily fractured during extraction

When using only standard floss, the mesial root surface of which of the following teeth is typically NOT totally cleaned? A. Mandibular 1st molar B. Mandibular 1st premolar C. Mandibular central incisor D. Maxillary 1st premolar E. Maxillary central incisor

D. Maxillary 1st premolar 1) Concept: The mesial root surface of the maxillary 1st premolar is typically NOT thoroughly cleaned using ONLY the dental floss due to the medial root concavity. 2) Mesial root surface of the maxillary 1st premolar a. because of the lack of cleansibility of the mesial of this tooth b. it is advised to use interdental brushes, end tuft brushes, tooth pick in holders, rubber tips, and powerful floss to clean these inter proximal areas. This is because of the type of gingival embrasure space on the mesial aspect of the maxillary 1st premolar. 3) Gingival Embrasures a. Type I: Having NO inter dental papilla loss b. Type II: Partial loss of inter dental papilla c. Type III: Complete loss of inter dental papilla 4) The mesial surface of the maxillary 1st premolar typically has a Type II or Type III gingival embrasure, resulting in moderate to severe recession with exposure of the root concavity. Interdental brushes clean the root concavity better than dental floss.

Which of the following teeth are measured to predict the size of the unerupted canines and premolars when performing a Moyers mixed dentition analysis? A. Deciduous molars and canines B. Mandibular incisors C. Maxillary incisors D. Maxillary incisors for the maxillary arch and mandibular incisors for the mandibular arch

D. Maxillary incisors for the maxillary arch and mandibular incisors for the mandibular arch 1) Concept: Moyer's Mixed Dentition Analysis a. uses the total mesiodistal width of the central and lateral incisors of a certain arch to predict the size of the unerupted canines and premolars b. Moyer's mixed dentition analysis measures the total mesiodistal width of central and lateral incisors of an arch and matches the measurement with a table of values that corresponds to the predicted size of the premolar and canine c. The prediction chart of Moyer holds possible values or measurements of canines and premolars in proportion wight present permanent central and lateral incisors d. Mixed dentition analysis can be performed once the predicted values are taken by comparing the space available in the arch and the space needed by the erupting canines and premolars.

Which of the following is the predominant etiology of fibrous gingival hyperplasia? C. Leukemia D. Medication E. Puberty

D. Medication 1) Fibrous gingival hyperplasia a. MOST COMMONLY caused by b. phenytoin = anticonvulsant c. cyclosporin = immunosuppressant d. nifedipine = Ca+2 channel blocker

Familiarization is a technique to best manage fearful pediatric patients who A. Are diagnosed with schizophrenia B. Have an aggressive behavior problem C. Have had previous traumatic medical experience D. Mirror the behavior of parents who are fearful of dentists E. Are physically handicapped

D. Mirror the behavior of parents who are fearful of dentists 1) Familiarization a. psychological behavior management technique meant to help children relax while inside the dental clinic b. introducing the dental setting and the common instrument that are used in the clinic in order to remove the fear from pediatric patients c. demonstrates the dental instruments and that there is not anything to fear by their use is the child's mouth

Which of the following is the MOST COMMON sign of occlusal trauma? A. Abrasion B. Attrition C. Migration D. Mobility E. Tipping

D. Mobility 1) Mobility a. describes the amount a tooth moves when pressure is applied to it b. fremitus describes tooth mobility when under function c. another sign of occlusal trauma is a widened periodontal ligament

External splints are better than internal splints primarily because which of the following? A. Increased retention B. Increased rigidity C. Increased strength D. More tooth structure is conserved

D. More tooth structure is conserved 1) External splints a. always recommended for use when conservation of tooth structure is of prime importance b. less retention c. less rigidity compared to the internal splints 2) Internal splints a. require removal of tooth structure in order to be used

Nitrous oxide abuse may result in... A. Acute narrow angle glaucoma B. Deep vein thrombosis C. Systemic myopathy D. Peripheral neuropathy

D. Peripheral neuropathy 1) Nitrous oxide toxicity a. demonstrated to produce neurotoxicity similar to other NMDA antagoniss with prolonged use b. long-term exposure can cause vitamin B12 deficiency, numbness, and reproductive side effects (in pregnant patients)

Addition to tobacco products is caused by which of the following agents? D. Nicotine

D. Nicotine 1) Nicotine a. induces body to release epinephrine from the adrenal glands b. stimulate the body thereby increasing heart rate, respiration, and blood pressure

Which of the following does Occupational Safety and Health Agency (OSHA) "Blood Borne Pathogens Standard: not regulate? A. Exposure incident plan B. Hepatitis B vaccination C. Medical waste disposal D. Nitrous Oxide scavenging systems E. Instrument sterilization and storage

D. Nitrous Oxide scavenging systems 1) Blood Borne Pathogens Standard involves: a. Management of sharps and disposal of regulated wastes b. Communication of hazards to employees c. Training and Hepatitis B vaccination of employees d. Exposure incident plan e. Sterilization and storage of instruments

Which of the following agents is the MOST APPROPRIATE to administer to a 5-year-old child that is shy, fearful, and timid? A. Promethazine only B. Fentanyl C. Hydroxyzine hydrochloride D. Nitrous oxide/oxygen E. Promethazine and meperidine

D. Nitrous oxide/oxygen 1) Nitrous oxide sedation a. generally acceptable to shy and fearful children b. most clinicians find it appropriate to administer nitrous oxide sedation before carrying out treatment in pediatric patients because it helps in the elimination of anxiety and dental treatment phobia c. most children feel more comfortable when being treated under nitrous oxide sedation and are often enthusiastic about its administration

Each of the following describes an ideal ergonomic position for dental work EXCEPT one. Which is the EXCEPTION? A. Back and neck upright B. Elbows bent at 90o or greater C. Elbows kept close to the body D. No more than 10 inches between operator's nose and patient's oral cavity E. Use entire seat/stool surface to support operator's weight

D. No more than 10 inches between operator's nose and patient's oral cavity 1) Ergonomic position a. at least 10 inches or more space between his/her nose and patient's oral cavity in order to achieve better visualization, controlled instrumentation and convenient working environment. b. elbows at 90o or greater c. operator's weight should be supported using the entire seat/stool surface to maintain balance

Most patients suffering from TMJ pain/dysfunction typically exhibit... A. Evidence of destruction of cortical bone of the condylar head on the affected side B. Normal dentitions C. Physical findings of rheumatoid arthritis in other joints D. No obvious TMJ pathology on pantomographs

D. No obvious TMJ pathology on pantomographs 1) Pantomographs are not useful when diagnosing TMJ pain/dysfunction disorders because only hard tissues are visible. 2) Relation of the condylar head to the mandibular fossa and articular tubercle are also often distorted 3) MRI is the preferred method for visualizing the TMJ followed by Cone-beam Computed Tomography.

Each of the following is suggested to cause the deposition of sclerotic dentin EXCEPT one. Which one is the EXCEPTION? A. Abrasion B. Age C. Caries D. Nutrition E. Occlusal stress

D. Nutrition 1) Sclerotic dentin a. results from aging or mild irritation such as b. abrasion c. attrition d. occlusal stress e. is harder, denser, less sensitive and more protective of the dental pulp f. characterized by hyper mineralization or blockage of tubules by the deposition of whitlockite crystals and be denatured collagen 2) Physiologic dentinal sclerosis a. sclerosis occurring with aging is termed physiologic dentinal sclerosis due to mild irritation

Class II amalgam cavity preparations in primary molars DO NOT require a gingival bevel because the enamel rods in that area are tilted which direction? A. Gingivally B. Horizontally C. Lingualy D. Occlusally E. Facially

D. Occlusally 1) Concept: The enamel rods of primary teeth are occlusally directed which makes the gingival bevel unnecessary. 2) Beveling of gingival margins in a Class II amalgam preparation of a permanent molar is performed to remove unsupported enamel rods 3) The enamel rods of permanent molars become apically directed as they approach the cervical aspect of tooth.

Which of the following is regarded as the most toxic form of mercury? A. Each form is considered equally as toxic B. Elemental C. Inorganic D. Organic

D. Organic 1) Concept: Organic mercury is considered to be the most toxic form of mercury. Mercury can form organic compounds such as methyl mercury. 2) Organic mercury compounds can be readily absorbed by many organisms and concentrated as they pass up the food chain. Organically bound mercury in food is the primary source of mercury exposure in most people. 3) Excretion of mercury is less effective from the human body compared to its absorption. Organic mercury has a tendency to concentrate in liver, kidney, and brain after being absorbed. 4) Higher concentration of mercury in the human body is harmful. Mercury toxicity can result in bronchitis, pneumonia, kidney disease, allergic reactions. When inhaled in toxic quantities, it can cause damaged to the central nervous system. 5) Dentists are at an increased risk of mercury toxicity due to the excessive use of amalgam fillings in the operator. The maximum safe dose ingested in a 40 hour work week is 50 microgram per cubic meter. 6) The dental operator should be well-ventilated with seamless floor tiles in order to prevent mercury retention in higher concentrations and decreased mercury inhalation. Use of personal protective equipment is mandatory.

A 19-year-old woman complains of a tingling in her lower lip. An examination reveals an asymptomatic, indurated swelling of her mandibular premolar region that she has been aware of for 3 weeks. Radiographs indicate a loss of cortex with diffuse trabeculation. What is the most likely diagnosis? A. Dentigerous cyst B. Hyperparathyroidism C. Leukemia D. Osteosarcoma E. Ossifying fibroma

D. Osteosarcoma 1) Concept: Osteosarcomas are classified as a form of bone cancer that predominantly occurs in children and teenagers. 2) Osteosarcomas a. The reason for this occurrence of osteosarcoma is still unknown at present but some studies suggest a genetic etiology. b. Usually manifest as a solid, indurated swelling that can be distinguished in radiographs as appearing moth-eaten, or having a sun-burst appearance. The sun-burst appearance is due to the calcified tumor spicules that radiate outward of the bone.

A patient exhibiting a "cotton wool" appearance of the bone in their x-rays and an elevated level of alkaline phosphatase and normal calcium, phosphate, and aminotransferase concentrations in their blood is most probably suffering from which of the following diseases? A. Fibrous dysplasia B. Hyperthyroidism C. Multiple myeloma D. Paget's disease of bone E. Ewing's sarcoma

D. Paget's disease of bone 1) Paget's disease of bone a. chronic disorder that can result in enlarged and misshapen bones, pain, fractures, and arthritis b. diagnosed by the presence of elevated level of alkaline phosphatase and normal calcium, phosphate, and aminotransferase levels in the blood c. "cotton wool" appearance of the bone on x-ray d. Signs 1) enlarged skull and facial bones 2) enlarged maxilla and alveolar ridges 3) displaced teeth 4) hypercementosis

Which of the following describes the placement of the orifice of the 4th canal in permanent maxillary 1st molars? A. Distal to the palatal canal in patients from the pacific islands B. On a line on the pulpal floor from the distobuccal orifice to the mesiobuccal orifice C. On a line on the pulpal floor from the palatal orifice to the distobuccal orifice D. Palatal to the orifice of the mesiobuccal canal

D. Palatal to the orifice of the mesiobuccal canal 1) MB2 canal a. located lingual to the orifice of the mesiobuccal canal (MB1) and is observed in 68-98% of permanent maxillary 1st molars. b. MB2 canal often times joins with the MB1 canal before the apex is reached.

Which of the following is the MOST probable diagnosis for a patient that presents with painful lesions on their buccal mucosa and whose biopsy revealed a suprabasilar vesicle and acantholysis? A. Bullous lichen planus B. Erythema multiforme C. Necrotizing sialometaplasia D. Pemphigus E. Systemic lupus erythematosus

D. Pemphigus 1) Pemphigus Vulgaris a. autoimmune disorder characterized by the production of antibodies against specific proteins in the skin and mucous membranes resulting in formation of skin blisters b. MOST commonly associated with painful lesions on the buccal mucosa. c. Biopsy reveals a suprabasilar vesicle and acantholysis 2) Necrotizing sialometaplasia a. infection condition of salivary glands 3) Erythema multiform a. hypersensitivity reaction occuring in response to medicine, infections, or illness 4) Systemic lupus erythematosus (SLE) a. hypersensitivity reactoin

Which of the following is the instrument grasp MOST commonly used in operative dentistry? A. 2-handed grasp B. Palm grasp C. Palm-thumb grasp D. Pen grasp

D. Pen grasp 1) Pen grasp is the MOST frequently used instrument grasp in operative dentistry procedures. 2) Modified pen grasp a. is similar to the pen grape expect the operator uses the pad of the middle finger o nthe handle of the instrument. It provides more strength and stability in some procedures. 3) Palm grasp a. used for bulky instruments b. commonly used for surgical forceps, rubber damp clamp forceps, straight chisels, and the air/water syringe. 4) Palm-thum grasp a. used by the assistant for holding the oral evacuator. b. operator may use this with instruments that require a more vertical movement

A pantomograph examination reveals a radiopacity in a patient's maxillary sinus. The patient is asymptomatic. The differential diagnosis should contain each of the following EXCEPT one. Which option is this EXCEPTION? A. Antrolith B. Odontoma C. Osteoma D. Periapical Cyst E. Mucus retention pseudocyst

D. Periapical Cyst 1) Periapical cyst a. localized radiolucencies associated with the apex of a tooth that previously suffered chronic pulpal or periodontal infection 2) Odontomas a. radiopaque lessons that may occur along the anterior and posterior teeth 3) Osteomas a. radiopaque benign lesions or bone growth within a membranous bone 4) Antroliths a. calcified lesions or masses which is usually located along the maxillary sinus 5) Mucous retentoin pseudocysts a. collection of mucous deposits within the paranasal sinuses b. appear radiopaque because the air that surrounds the pseudocyst is radiolucent, making the mucous more radiopaque

A pantomograph reveals a radiopaque lesion associated with a patient's maxillary sinus, that the patient does not report as being symptomatic. Which of the following should NOT be included in the differential diagnosis? A. Antrolith B. Odontoma C. Osteoma D. Periapical cyst E. Mucus retention pseudocyst

D. Periapical cyst 1) Concept: Periapical cysts are localized radiolucencies associated with the apex of a tooth that previously suffered chronic pulpal or periodontal infection 2) Odontomas a. radiopaque lesions that may occur along the anterior and posterior teeth 3) Osteoma a. radiopaque benign lesions or bone growth within a membranous bone 4) Antroliths a. calcified lesions or masses which are usually located along the maxillary sinus 5) Mucus retention pseudocysts a. collection of mucous deposits within the paranasal sinuses. They appear radiopaque because the air that surrounds the pseudocyst is radiolucent, making the mucous more radiopaque.

Each of the following is associated with full-thickness flaps EXCEPT one. Which is the EXCEPTION? A. Apical repositioning B. Dissection of the tissue with a blunt instrument C. Osseous recontouring D. Periosteum stays on the bone

D. Periosteum stays on the bone 1) Concept: Full thickness flaps a. includes stripping all of the soft tissue (epithelium, connective tissue, periosteum) from the bone. 2) Following placement of the incisions, the flap is dissected using blunt instruments like a #9 molt to separate the soft tissue from the bone. 3) Full thickness flaps are used when recontouring the osseous structures below of when the gingiva are repositioned apically.

Which of the following is NOT one of the 5 domains of pediatric patient management? A. Aversive (punishments) B. Linguistic C. Pharmacological D. Physical E. Reward oriented F. Authoritative

D. Physical 1) Physical a. physical means to control patient misbehavior and promote positive behavior of pediatric patient b. usually achieved using papoose boards, belts, tape and cloth wraps to keep them restrained 2) Pharmacological a. anesthetics, sedative drugs and inhalers to make a child calm during dental treatment 3) Rewards a. uses rewards to reinforce good behavior from pediatric patients 4) Aversive a. parental spanking and the hand-over-mouth technique 5) Linguistic a. requires the dentist to be a good communicator, a coach and a friend for the pediatric patient to promote good patient behavior b. necessitates that the child be able to comprehend language

Which of the following explains the MOST probable reason for a patient to experience truisms following an inferior alveolar nerve block? A. Accidental injection of the anesthetic near a motor branch of the V3 trigeminal nerve B. Failure to aspirate C. Injection of the anesthetic between the periosteum and the bone D. Piercing the medial pterygoid muscle with the needle

D. Piercing the medial pterygoid muscle with the needle 1) Trismus a. can occur following administration of an inferior alveolar nerve block if the needle is placed too medially b. is given in the space within the pterygomandibular fossa c. limited ability to open one's mouth d. Normal mouth opening ranges from 35 to 45mm e. Males usually have slightly greater mouth opening than females (40-60mm) = average is 35mm f. The normal lateral movement is 8-12mm 2) Trismus Classifications a. Mild Trismus: 20-30mm b. Moderate: 10-20mm c. Severe: > 10mm 3) Placing the needle too far medially can inject the anesthetic solution in the medial pterygoid muscle 4) Impairment of the movement of medial pterygoid muscle results in inability to open the mouth.

Which of the following is considered the primary etiology of gingivitis during pregnancy and puberty? A. Calculus B. Estradiol C. Estrogen D. Plaque E. Progesterone F. Testosterone

D. Plaque 1) Plaque a. primary etiology of gingival inflammation because the bacterial aggregates from plaque initiates the inflammatory reaction. 2) Calculus a. predisposing factor for the formation of plaque b. on its own DOES NOT cause gingivitis 3) Sex hormone level a. induces vasodilation and increases permeability along capillary walls which allow more fluids to extravasate and cause severe inflammation.

A patient presents to your office with an indurated submucosal tumor that when biopsied, suggests a neoplasm of glandular and myoepithelial origin. Which of the following is the most likely diagnosis? A. Fibroma B. Mucocele C. Schwannoma D. Pleomorphic adenoma

D. Pleomorphic adenoma 1) Concept: Pleomorphic adenoma is a common benign salivary gland neoplasm of glandular cells along with myoepithelial components, that can potentially become malignant. 2) Pleomorphic adenoma a. most common variety of salivary gland tumor and also the most common tumor of the parotid gland b. Plemorphic adenoma derives its name from the architectural pleomorphism (variable appearance) observed with light microscopy. c. The tumor usually presents as a slowly proliferating, painless, indurated single nodular mass. 3) Schwannomas and mucoceles are not indurated or firm. 4) Fibromas, mucoceles, and Schwannomas do not present microscopically with glandular or myoepithelial cells.

In which of the following patients is inhalation of 100% oxygen CONTRAINDICATED? A. Acute bronchitis B. Acute viral pneumonia C. Chronic obstructive pulmonary disease (COPD) D. Pneumonthorax

D. Pneumonthorax 1) Concept: Pneumonthorax: Inhalation of pure 100% oxygen or any other gas is contraindicated in a patient of pneumothorax because it can worsen the condition of the patient and can be life-threatening. a. is basically an abnormal collection of air or gas in the pleural space. 2) It is advised to insert a needle with an underwater seal to get rid of the entrapped air. 3) Oxygen interferes with normal breathing and is dealt as an emergency.

Each of the following statements about polishing amalgam are all correct EXCEPT one. Which is the EXCEPTION? A. Can cause an unanesthetic appearance if amalgam becomes too shiny B. Polishing creates a comfortable surface for the patient's tongue C. Polishing should be performed AT LEAST 24 hrs after placement D. Polishing significantly increases the longevity of the restoration

D. Polishing significantly increases the longevity of the restoration 1) Concept: Polishing amalgam restorations has NOT shown any significant increase in the life of restorations. 2) It is advised to polish amalgam restorations AT LEAST 24 hours after the restorative procedure in order to allow complete setting of the alloy. 3) Finishing can create a more comfortable surface for patient's tongue, however, it can produce an unaesthetic and shiny surface in some cases.

Which of the following is the MOST COMMON reason for maxillary incisors to become stained orange and green? A. Diet B. Drugs C. Fluoride ingestion D. Poor oral hygiene E. Trauma

D. Poor oral hygiene 1) Prolonged poor oral hygiene allows the organic compounds from the food or drinks we ingest to adhere to tooth pellicle. Once these colored compounds are oxidized, they remain as tooth stains. 2) Organic compounds that adhere to tooth surfaces through oxidation are known as chromogens. Stains can only be removed through professional dental prophylaxis. 3) Tetracycline stains are permanent bands of brown and gray NOT orange and green. 4) Dental trauma cause pulp necrosis and create a dull darkly colored tooth.

A buccal flap used to cover an oroantral communication associated with tooth #1 would receive nourishment from which of the following arteries? A. Facial B. Greater Palatine C. Nasopalatine D. Posterior superior alveolar

D. Posterior superior alveolar 1) Concept: The buccal flap will receive its nourishment from the blood vessel that supplies tooth #1 which is the posterior superior alveolar artery 2) Posterior Superior Alveolar Artery a. blood vessel that provides blood supply to the lining of the maxillary sinus, the alveolar bone, the molars and premolars of the maxilla and the gingiva 3) Facial Artery a. branches out and supplied blood to several parts of the face which includes the lips, portions of the nose, sub mental areas and some portions of the tonsils. 4) Nasopalatine Artery (Sphenopalatine Artery) a. branches off to the walls of nasal septum. 5) Greater palatine Artery a. gives off blood supply to the nasal septum and the hard palate

Which of the following is the reason why satisfactory gingival seats are NOT POSSIBLE for the proximal box of a deep Class II cavity preparation in PRIMARY teeth? A. The buccal and lingual surfaces converge occlusally for primary molars B. The enamel rods of the gingival 1/3 of primary molars extend occlusally C. The proximal contact of primary molars is flat and broad D. Primary teeth possess a marked cervical constriction

D. Primary teeth possess a marked cervical constriction 1) Concept: Primary teeth possess prominent cervical ridges and are more slender mesiodistally at their cervical portions and narrower at their necks. 2) These features make it extremely difficult to achieve satisfactory gingival seats for proximal boxes of deep class II cavity preparations 3) The enamel rods in the gingival 1/3 of the crown also extend in an occlusal direction compared to permanent dentition that run cervically.

The principal goal of the maintenance phase of periodontal therapy is described by which of the following? A. Complete the remaining procedures on the treatment plan B. Finish off secondary surgical procedures C. Maintain a relationship with the patient D. Precent recurrent disease E. Re-evaluate the results of the initial therapeutic treatment

D. Precent recurrent disease 1) Concept: Maintenance Phase a. The maintenance phase of periodontal therapy focuses on keeping the patient in stable periodontal health and the prevention of disease recurrence. b. Re-evaluation of the results of initial therapeutic treatment is performed during the pasha I or cause-related therapy phase around 6-8 weeks after instrumentation. c. The maintenance phase utilizes a well planned schedule of maintenance visits customized and designed for the needs of each patient on a case-by case basis. d. Surgical procedures are usually performed during the corrective phase of treatment.

The dentist should mentally plan the outline from before beginning tooth preparation to: A. Aid in the finish of a enamel walls and margins B. Establish the convenience form C. Establish the resistance and retention form D. Prevent over cutting and overextension

D. Prevent over cutting and overextension 1) Over cutting and over extension compromises the remaining tooth structure and the success of the restorative procedure. Excess removal of tooth structure may extend close to the pulp, causing accidental pulp exposure or irritation. 2) Outline Form: shape of the boundaries of the completed cavity; cavity is removed 3) Resistance Form: design feature in the cavity preparation which allows tooth and restoration to resist the masticatory stresses without fracture 4) Retention Form: design features in the cavity preparation which allows it to retain restoration securely during function 5) Convenience Form: features in the prepared cavity which improve visibility and accessibility during preparation and restoration

Each of the following can be achieved with a properly trimmed wooden wedge EXCEPT ONE. Which is the EXCEPTION? A. Protect the gingiva B. Provide space for the matrix band C. Reduce leakage of moisture into the cavity preparation D. Prevent overcontouring of the contact

D. Prevent overcontouring of the contact 1) A well-fitted and properly tightened matrix band placed between the filling surface and adjacent proximal tooth surface to prevent over contouring of the contact point. 2) A proper contact point is important for the life of the restoration and gingival health. 3) Food impaction and gingival disease may occur if a contact point is NOT properly established. 4) A properly trimmed wooden wedge placed in the interdental space below the matrix band can help to, a. protect the gingiva b. provide space for the matrix band placement c. reduce leakage of moisture ion the cavity preparation d. prevent material from leaking out during condensation

Amelogensis imperfecta and dentinogenesis imperfecta are MOST STRONGLY differentiated by which of the following features? A. Blue sclera B. Hair loss C. Hereditary background D. Radiographic appearance E. Color of the hair

D. Radiographic appearance 1) Amelogenesis Imperfecta a. teeth are covered with thin, abnormally formed enamel b. enamel is avascular and high calcified and does not contain the proteins ameloblastin, enameling, tuftelin, and amelogenin c. teeth are often brown, gray, or yellow d. crowns of these teeth are often subject to rampant caries and excessive attrition

An oral cytology smear is MOST INDICATED for the diagnosis of which of the following conditions? A. Lichen planus B. Mucous membrane pemphigoid C. Papilloma D. Pseudomembraneous candiodosis E. Squamous cell carcinoma

D. Pseudomembraneous candiodosis 1) Concept: Definitive diagnosis of candidosis requires specialized tests such as a cytologic smear. b. The membranous slough of pseudomembranous candidosis can often times be wiped off with gauze to reveal an erythematous surface underneath. c. The clinical appearance of pseudomembranous candidosis alone is sufficient for the diagnosis of candidosis in most cases. d. Smears are collected by gentle scraping of the lesion with a spatula or tongue blade with the resulting debris being directly applied to a glass slide for a cytopathological exam.

Which clinical sign is consistent with Verrill's sign? A. Nystagmus during lateral glaze B. Ophthalmoplegia after exposure to light C. Paresis due to CNS depression D. Ptosis of eyelid

D. Ptosis of eyelid 1) Concept: Verill's sign is similar to Ptosis, a condition where there is drooping of the upper or lower eyelid. 2) The level of sedation can be evaluated by checking several parameters including the Verill's sign or the drooping of the eyelid which signifies the effect of anesthesia to the muscles that controls the eyelid. 3) Ptosis a. occurs when the eyelid muscles becomes incapable of holding or raising the eyelids properly. It can be due to trauma, aging or degenerative diseases that can be neurogenic or myogenic or may even be genetic.

Which database contains current electronic dental literature? A. Dental Absracts B. index to Dental Literature C. LEXIS-Nexus D. Pubmed-MEDLINE

D. Pubmed-MEDLINE 1) Pubmed-MEDLINE a. US National Library of Medcine's (NLM) bibliographic database 2) LEXIS-Nexis a. legal and public records related information

Which of the following factors account for the most common etiology of dental trauma for children 1-3 years of age? A. Athletic B. Lack fo supervision by parents C. Physical abuse D. Underdeveloped motor coordination skills E. Alcohol related falls

D. Underdeveloped motor coordination skills 1) Dental Trauma for 1-3 year old a. Children under the age of 1-3 year old have very limited and underdeveloped motor coordination which often causes accidents including dental trauma b. Children have prominent anterior teeth and lack of coordination.

In primary teeth, which of the following is least reliable in assessing the pulp vitality? A. Extraoral swelling B. Internal resorption C. Intraoral swelling D. Pulp testing E. Spontaneous pain

D. Pulp testing 1) Concept: Pulp Testing a. can only provide information about the pulp's sensitivity to stimulation, which may result in a tendency for false positive results 2) Presence of spontaneous pain and a possible intramural swelling associated with a primary tooth indicates the presence of irreversible pulp damage and that the infection has probably reached the periodical region. 3) Extraoral swelling and internal resorption suggests that the pulp has become necrotic and that the infection has reached the periodical region.

A patient reports to your clinic in pain for the last two weeks after a well condensed Class V restoration was placed in tooth #20 4 months previous. The patient reports no discomfort up until these last two weeks. What is the most likely cause of the discomfort experience by the patient? A. A fracture in the restoration B. Cervical abrasion caused by factorial in jury during brushing C. Marginal leakage D. Pulpal damage from the preparation leading to irreversible pulpitis

D. Pulpal damage from the preparation leading to irreversible pulpits 1) Cervical region of the tooth has thinner enamel, so caries located there progresses faster if not immediately restored. 2) Restorative treatment procedures performed on the tooth could have caused trauma to the pul pouring caries removal which manifested laster due to the inflammation process. 3) Cervical abrasion would take longer than 4 months to manifest 4) Class V fracture would NOT allow the tooth to flex more causing pain.

A 9 year old presents to your office after he fracture maxillary 1st molar 3 hours ago. After testing vital, you noticed an exposed inflamed coronal pulp in Tooth #14. The recommended treatment of choice is... A. Direct pulp cap D. Pulpotomy

D. Pulpotomy 1) Concept: Pulpotomy a. The pulp exposure was caused by the tooth being fractured and its not due to caries making it a good candidate for pulpotomy b. exposed and inflamed coronal pulp can be removed and the remaining healthy radicular pulp will be left unharmed 2) Direct Pulp Cap a. cannot be performed because the prognosis is POOR if the pulp was exposed for MORE THAN ONE HOUR 3) Pulpectomy a. only indicated for primary teeth having infected coronal and radicular pulp because the apexes of primary molars are still open 4) Indirect Pulp Cap a. not possible since the pulp is already exposed

When designing a flap for periodontal surgery, which of the following findings is most influential? A. Vestibular depth B. Presence of intrabony defect C. Probing depth D. Quantity of attached gingiva E. Frenum attachment level

D. Quantity of attached gingiva 1) Concept: Attached Gingiva a. maintains a tight contact with the periosteum of the bone and certain areas of the tooth root. b. its function is to keep the free gingiva in close contact with the tooth while also protecting the soft and free moving alveolar mucosa. c. insufficient attached gingiva makes the mucosa more prone to trauma during function and compromises the periodontal support of teeth d. in order to avoid periodontal problems after surgery, the width of the attached gingiva must be preserved.

An 8 year old boy presents to your office with a traumatized #8 that no longer responds to electric and thermal vitality tests. Which of the following is the suggested treatment? A. Apexification B. Calcium Hydroxide pulpotomy C. Pulpectomy D. Re-evaluate after 2 weeks E. Svek pulpotomy

D. Re-evaluate after 2 weeks 1) Traumatized immature teeth may give a false reading with thermal and electric vitality tests because their blood supply may still be intact, but their nerve may not respond. 2) As the circulation is restored to the tooth, the responsiveness to pulp testing may return after 2 weeks. 3) Apexification or pulpectomy is NOT indicated at this time because the pulp may still be vital.

Which of the following should be part of an initial periodontal therapy in addition to scaling and root plaint and oral hygiene instruction? A 0.12% Chorhexidine gluconate instruction B. Doxycycline 300mg BID for 10 days C. Enameloplasty to achieve occlusal equilibrium D. Referral to physician if diabetes is suspected

D. Referral to physician if diabetes is suspected 1) Diabetes Mellitus is considered a risk factor for the progression of periodontal disease which must be addressed during the initial periodontal therapy. 2) 1st phase a. remove causative factors, risk factors, and predisposing factors that may worsen the periodontal condition of the patient.

Each of the following are advantages to investigating the biocompatibility of a biomaterial using in vitro testing EXCEPT one. Which is the EXCEPTION? A. An in vitro test can be standardized B. An in vitro test is excellent for examining mechanisms of interactions C. An in vitro test is quick to perform D. Relevance to the use of the material is assured in an in vitro test

D. Relevance to the use of the material is assured in an in vitro test Advantages of an in vitro test a. quick to perform b. can be standardized c. excellent for examining mechanisms of interactions

In which of the following procedures is the dentist least likely to utilize TMJ arthroscopy? A. Diagnosis of disk perforations B. Disk manipulation C. Lysis of adhesions D. Repair of disk perforations E. Therapeutic lavage

D. Repair of disk perforations 1) Concept: Repairing a disk perforations requires that a flap incision is made in order to fully access the temporomandibular disk. 2) Lavage of the temporomanidbular joint space, removing adhesions, and disk manipulations can all be complete using arthroscopy. 3) Diagnosis of disk perforations is best completed using T2 weighted Magnetic Resonance Imaging (MRI)

Which of the following is the suggested protocol if the pulpal floor is perforated during access preparation? A. Perform no treatment at this time and schedule the patient for an extraction B. Repair the perforation and continue the root canal treatment at a subsequent appointment C. Repair the perforation at a subsequent appointment only if symptoms occur D. Repair the perforation immediately with mineral trioxide aggregate (MTA/Portland Cement), inform the patient, and reassess the tooth at the recall appointment E. Continue the root canal treatment as normally planned because tertiary dentin will fill in

D. Repair the perforation immediately with mineral trioxide aggregate (MTA/Portland 1) Root Perforations a. should be addressed immediately through the use of MTA to help stimulate healing of periradicular tissues and to promote cementum deposition b. MTA is ideal material since it create a seal when moisture is present c. MTA is biocompatible and is capable of inducing periradicular tissue repair and cementum deposition

Which of the following describes the most common sequences of treatment for replanting permanent teeth with closed apices? A. Canal debridement, calcium hydroxide therapy, repositioning, splinting, and gutta-percha fill B. Root canal therapy, Respositioning, and splinting C. Repositioning, splinting, and root canal therapy D. Repositioning, splinting, canal debridement, calcium hydroxide therapy, and gutta-percha fill

D. Repositioning, splinting, canal debridement, calcium hydroxide therapy, and gutta-percha fill 1) Replanted Tooth Procedures a. splined with adjacent teeth to ensure table in positioning for about 2 weeks, also serving to keep the tooth properly aligned with the other teeth in the arch. b. once the tooth is stabilized in its socket for about 1-2 weeks, canal debridement is performed to remove necrotic pulp and traces of bacteria that might re-infect the root canal c. After proper canal debridement, calcium hydroxide paste is used as a medicament to fill up the root canal. Gutta percha point is then used with a sealer to fill the canal up to create a hermetic seal.

All of the following are correct about mucous retention phenomena (pseudocysts) EXCEPT one. Which of the following is the EXCEPTION? A. Appears radio-opaque B. Is dome shaped C. Protrudes from the floor of the maxillary sinus D. Require surgical removal E. Lined with respiratory epithelium

D. Require surgical removal 1) Concept: Mucous retention pseudocysts a. are benign inflammatory reactions, there is no need for surgical treatment. b. Mucous retention pseudocysts are dome-shaped radiopaque entities seen on the floor of the maxillary sinus that represent focal accumulation of inflammatory exudate that lifts the epithelial lining of the sinus and the periosteum away from the underlying bone. c. Their histologic appearance is that of normal or inflamed maxillary sinus lining. Because mucous retention pseudocysts are benign inflammatory reactions, there is no need for surgical treatment. d. Mucous retention pseudocysts appear radiopaque because the air surrounding the pseudocyst is radiolucent, making the pseudocyst appear more radiopaque.

Visible light curing units present the MOST danger to which of the following? A. Cornea B. Iris C. Lens D. Retina E. Cilliary muscle

D. Retina 1) Continuous exposure to visible light used for curing the composite can cause serious damage to the RETINA. 2) Care should be taken to look directly at the visible light cure sources. 3) Use a light curing machine with a protective barrier covering the light source. 4) Prolonged visible light exposure can lead to photoreceptor cell damage.

After having a filing placed a week ago that was found to be about 1.5mm from the pulp. The patient returns to your office with acute pain with thermal stimulus. What is the most likely cause of the patient's pain? A. Hyperemic pulp B. Irreversible pulpitis C. Necrotic pulp D. Reversible pulpitis E. Chronic pulpal inflammation

D. Reversible pulpitis 1) Acute Reversible Pulpitis a. transient condition of the pulp where in it experiences unusual sensitivity due to the following causes: b. Thermal Shock 1) pulp is damaged by heat generally created by friction during the preparation process or polishing the restoration c. Trauma 1) induced by a blow to the tooth or improper occlusal relationship d. Desiccation 1) tooth is prepared without the utilization of a high-speed hand piece without a water jet e. Amalgam restoration in contact with gold restoration 1) situation causes reversible pulpits and sensitivity because the tooth will experience galvanic shock having two different metallic restorations in contact with the oral cavity f. Chemical stimulus 1) sweets are easily fermented and bacteria easily forms acid by products that triggers the sensitivity 2) sour foods on their own are acidic by nature and could demineralize the tooth surface and cause sensitivity g. Caries extension 1) continuous bacterial attack on the tooth surface causes pain and sensitivity to tooth 2) pain is lost after stimulus is removed

A 22-year old patient presents your office complaining of mild-to-moderate pain associated with tooth #30 that had an amalgam restoration recently placed. The pain occurs for roughly 2 seconds and is provoked by cold, heat, and sweet foods. Which of the following is the most probable diagnosis? A. Internal resorption B. Irreversible pulpitis C. Pulp necrosis D. Reversible pulpitis E. Externa resorption

D. Reversible pulpitis 1) Reversible Pulpitis a. pain that is provoked by various stimuli including cold, heat, sweet foods, touch, and etc. b. pain that only lasts for just a few seconds after the stimulus is removed 2) Irreversible Pulpitis a. condition of the pulp where it cannot return to a healthy state 3) Internal and External Resorption a. pathoses that are commonly asymptomatic and are usually discovered through routine radiographs 4) Necrotic Pulp a. usually asymptomatic b. characterized by non-vital pulp c. pain associated with a necrotic pulp can be attributed to a periodical infection associated with the necrotic pulp

The angles in the preparation for an amalgam restoration should be... A. Acute angles B. Obtuse angles C. Rounded D. Right angles

D. Right angles 1) Concept: The angles in the preparation for an amalgam restoration should be right angles. 2) Right angles preparation a. Right angles in a preparation direct occlusal forces toward the long axis of the tooth, making the restoration more resistant to occlusal load b. Right angles also help to improve retentive properties of the restoration c. Composite resin retained by adhesives flowing into microporosities exposed during etching.

Which of the following pairs of intracanal instruments and its action is INCORRECT? A. Intracanal instruments are more susceptible to breaking if they are rotated after becoming locked into dentin B. K-files can be used to file and ream, while a reamer can only be used to ream C. Reaming action will enlarge a root canal and make it more round in shape D. Rotating instruments in a clockwise direction forces debris apically, while counterclockwise rotation forces debris coronally

D. Rotating instruments in a clockwise direction forces debris apically, while counterclockwise rotation forces debris coronally 1) Directions a. debris is forced in a coronal direction when rotating instruments are used in a clockwise direction and vice versa 2) K files a. have both reaming and filing actions when used during root canal preparation 3) Reamers a. cause enlargement of root canals producing a round shape b. cannot be used in a filing motion 4) Separated instruments a. to prevent broken or separated instruments, never fro to rotate an instrument that is locked into dentin.

The maxillary incisors of a middle-aged patient can be made to appear younger through enameloplasty that includes A. Flattening the incisal edges B. Moving the facial height of curvature gingivally C. Moving the facial line angles proximally D. Rounding the incisal point angles E. Shortening the incisal edges

D. Rounding the incisal point angles 1) Rounding the incisal point angles of maxillary incisors removes the sharp angles that make the teeth appear worn down and abraded. 2) Rounded incisal angles and pointed cusp tips create the illusion of a younger looking smile. Rounded incisal angles also make a person appear more feminine, while square looking incisal angles suggest masculinity. 3) Shortening and flattening of incisal edges makes a person appear older since it insinuates abrasion of teeth due to old age.

Which of the following effects are commonly observed with drugs like diazepam, meperidine, and pentobarbital? A. Anticonvulsant and hypnotic B. Amnesia and skeletal muscle relaxation C. Analgesia and anxiolysis D. Sedation and dependency risk

D. Sedation and dependency risk 1) Diazepam a. good sedative but a very addictive drug as well b. prolonged use may lead to dependency, tolerance, and physical dependence 2) Pentobarbital a. barbiturates and is known as depressant of the nervous system b. due to its potent effect, it is used in sedation and for people suffering from insomnia and even reported seizures c. its continuous use could cause drug dependency and is therefore prescribed in a well controlled and limited period 3) Meperidine a. opiate sedative drug that is less addictive but still could pose a tendency for dependency when used frequently b. it could also cause euphoria making it more tempting to be abused

Which of the following is the primary characteristics indicating an intra-arterial injection of midazolam (Versed)? A. Rapid decrease in blood pressure B. Rapid decrease in respiration rate C. Severe cardiac arrhythmia D. Severe pain in the extremity E. Exagerrated sedation due to the more direct effect of the drug

D. Severe pain in the extremity 1) Concept: Some accounts of intra-arterial injection of midazolam stated that some of the patients experienced moderate to severe pain within the extremity where it was administered. 2) There are some isolated reports which noted seizure activity occurring after intra-arterial injection of midazolam. However, the reason for such reaction was not established. 3) Limited reports have been documented with regards to the effects of intra-arterial injection of midazolam. Adverse reaction from accidental intra-arterial injection can be localized within the its of injection or systemic.

Each of the following is a recognized disadvantage of amalgam restorations EXCEPT one. Which is the EXCEPTION? A. Brittleness B. Does notnd to tooth structure on its own C. Esthetics D. Short lifespan

D. Short lifespan 1) Amalgam restorations a. short lifespan is NOT recognized as a disadvantage of amalgam restoration b. have documented long life in all areas of the oral cavity c. does NOT bond to tooth structures on its own and require certain retention features like converging walls and adequate cavity depth d. disadvantages 1) poor esthetic 2) brittleness

Admixed amalgam alloy consists of which of the following? A. Copper silver and mercury only B. Lathe-cut particles C. Silver and tin and mercury only D. Spherical and Lathe-cut particles E. Spherical particles

D. Spherical and Lathe-cut particles 1) Concept: Admixed amalgam alloy consists of a mixture of lathe-cut particles and spherical particles 2) Admixed alloys a. have the "body" of lathe-cut alloys when condensing; are easily condensed with goo adaptation but are more easily worked with like the spherical alloys b. can withstand forces of condensation better than the spherical alloys which is why these alloys are preferred for proximal amalgam restorations. c. demonstrate higher levels of creep and corrosion compared to spherical alloys.

Which of the following procedures describes the best method to treat a recurrent ranula? A. Electrosurgery b. Marsupialization C. Parotid gland excision D. Sublingual gland excision E. Cryosurgery

D. Sublingual gland excision 1) Concept: ranuls typically exhibit a recurrence rate of up to 25% after a complete excision of the ranula and up to 2% in cases of complete excision of the ranula and sublingual gland 2) Ranulas a. are mucoceles found on the floor of the mouth that present as soft tissue swellings of collected mucin from a ruptured salivary gland duct b. usually caused by local trauma, and are left alone if they are small and not problematic 3) The salivary level experiences by a patient following an ipsilateral sublingual gland excision is usually not noticed by the patient, so if the first surgery is considered a failure, excision is recommended.

Which of the following is the term for when a tooth is traumatized without displacement of the tooth that increases the mobility of the tooth? A. Concussion B. Extrusion C. Intrusion D. Subluxation E. Luxation

D. Subluxation 1) Concept: Subluxation describes trauma to a tooth, supporting structures resulting in increased mobility, but without displacement of the tooth. Bleeding from the gingival sulcus confirms the diagnosis. This trauma may require immediate dental treatment of the affected tooth. b. increased mobility and pain to percussion c. without displacement of the tooth d. bleeding is evident 2) Concussion a. there is an injury within the supporting structures of a traumatized tooth b. does NOT demonstrate bleeding from the sulcus c. loosening and possible displacement among traumatized teeth is NOT observed in concussion cases d. an injury without increased mobility or displacement of the tooth e. with pain on percussion f. without gingival bleeding 3) Luxation a. used to denote displacement of tooth in any direction away from its original position in the dental arch 4) Extrusion a. partial displacement of the tooth out of its socket and that is characterized by partial or total separation of the periodontal ligament resulting in loosening and displacement of the tooth. b. alveolar socket bone is intact 5) Intrusion a. displacement of the tooth into the alveolar bone and is accompanied by comminution or fracture of the alveolar socket

When communicating with pediatric patients which principle should be employed? A. Both parent and dentist communicate with the child at the time B. IM ketamine 20mg/kg C. Immediately use voice control so that you can establish a power dynamic from the beginning that the child will respect D. Substitute easily understandable words and concepts for dental terminology (e.g. make your tooth fall asleep for anesthesia) E. Only have the parent answer questions since they know what is happening and can accurately report

D. Substitute easily understandable words and concepts for dental terminology (e.g. make your tooth fall asleep for anesthesia)

Which of the following periodontal fibers are putatively responsible for relapse of orthodontic rotation of teeth? A. Diagonal B. Horizontal C. Oblique D. Supracrestal E. Transverse

D. Supracrestal 1) Supracrestal Fibers a. Orthodontic treatment of rotated teeth may experience relapse as the supracrestal periodontal fibers try to revert to its original position. b. possess a "memory" or tendency to recoil back to their normal position, causing the relapse

Which of the following would be the diagnosis for a patient with a tooth that demonstrates spontaneous pain, is not sensitive to hot or cod, but is sensitive to percussion and does NOT have a periodical radiolucency? A. Asymptomatic Apical Periodontitis B. Chronic Apical Periodontitis C. Reversible Pulpitis D. Symptomatic Apical Periodontitis E. Irreversible Pulpitis

D. Symptomatic Apical Periodontitis 1) Symptomatic Apical Periodontitis a. sensitive to percussion and occlusal forces and do NOT necessarily demonstrate periodical radiolucency 2) Asymptomatic Apical Periodontitis a. not sensitive to percussion/palpation and may or may not demonstrate a periodical radiolucency 3) Irreversible Pulpitis a. lingering sensitivity after the cold stimulus is removed. b. may or may not elicit spontaneous pain 4) Reversible Pulpitis a. characterized by a patient that may be sensitive to cold or sweet but the pain quickly diminished once the cold stimulus is removed

Which of the following is the MOST COMMON etiology of severe pain upon percussion of a tooth? A. Chronic apical abscess B. Irreversible pulpitis C. Reversible pulpitis D. Symptomatic apical periodontitis

D. Symptomatic apical periodontitis 1) Concept: Symptomatic apical periodontitis (SAP) presents with pain upon masticatory forces or percussion. 2) Symptomatic Apical Periodontitis a. a periapical diagnosis whereas Irreversible is a pulpal diagnosis b. indicative of inflamed periodical tissues and may present with or without a radiolucency associated with the root apices of the tooth. 3) Chronic Apical Abscess (CAA) a. typically regarded to be NOT AS painful to percussive forces because the sinus tract associated with it is able to relieve some of the built up percussive forces

If a rubber dam is abnormally wrinkled between teeth, the probable reason is... A. Overlapping and/or crowding of the teeth involved B. Teeth with abnormally broad contacts C. The holes were punched too closely D. The holes were punched too far apart E. The holes were not punched large enough

D. The holes were punched too far apart 1) Large areas between holes in the rubber dam sheet cause wrinkling due to excess material between teeth. 2) Excess material between the holes became evident with the folding of the rubber dam septum in-between the holes. 3) Excess material increase the difficulty of passing the rubber dam through the contact area. 4) Prevent incorrect punching of holes a. Assess the distance between each tooth b. Assess the position of teeth in the arch c. Rubber dam holes should be approximately 3-4mm apart from each other on average

Using the split thickness graft technique for vestibuloplasty typically results in... B. Limited vestibular extension in the anterior lingual mandible C. Only a temporary increase in vestibular depth because the skin graft contract during healing D. The maximum ridge that is available for use in any given patient

D. The maximum ridge that is available for use in any given patient 1) Split Thickness Graft Technique a. increases the alveolar ridge length that may be used for denture fabrication b. vestibuloplasty in general helps to create longer residual ridge and a deeper flange area to improve the resistance of fabricated dentures for displacement c. increases the available stable soft tissue that can support a denture while also increases its retention

The most likely cause of interdental papillae bulging out from underneath a rubber dam is... A. Each tooth is not ligated with floss B. Inflamed gingival tissues C. The rubber dam is too thin of material D. The rubber dam holes were punched too close together E. The rubber dam holes were punched too far apart F. Thick periodontal biotype

D. The rubber dam holes were punched too far apart 1) The most likely cause of the interdental papillae bulging out from underneath of a rubber dam is when the rubber dam holes are punched too close together. 2) Holes punching too close together cause the material to stretch too much and prevent the rubber dam from adapting properly. 3) Lack of adaptation creates space of fluid seepage and contamination and rubber dam tearing may occur more easily

Each of the following statements regarding sealants is TRUE EXCEPT one. Which is the EXCEPTION? A. They are considered a non-invasive procedure B. They perform as a physical barrier C. They prevent biofilm from collecting within a pit or fissure D. They should be used on all patient's teeth

D. They should be used on all patient's teeth 1) Sealants a. should NOT be used on all patient's teeth b. indicated for either preventive or therapeutic uses, depending upon patient's caries risk, tooth morphology or presence of incipient enamel caries c. only caries-free pits and fissures or incipient lesions in enamel no extending to the DEJ are recommended for treatment with pits and fissure sealants d. Advantages a. Perform as a physical barrier b. Prevent bio film from collecting within a pit or fissure c. Sealant placement is considered as non-invasive procedures

The first symptom usually felt by patients being administered nitrous oxide is which of the following? A. Euphoria D. Tingling

D. Tingling 1) Extremities and lips may experience tingling sensation as the nitrous oxide gas starts to take effect.

Dental burs are MOST COMMONLY made from which material? A. Galvanized iron B. Iron C. Steel carbide D. Tungsten carbide

D. Tungsten carbide 1) Most dental burs are made up of tungsten carbide material. 2) Tungsten carbide burs a. most efficient in the removal of soft carious lesions by using them with a touching action b. these are less efficient in cutting hard tooth enamel c. Different Types 1) Cutting burs 2) Excavation burs 3) Finishing and polishing burs d. Finishing and polishing tungsten carbide burs have more flutes compared to cutting burs. The number of flutes range commonly ranges from 12-30.

Each of the following is a rationale for placing a bevel on preparations for composite resin restorations EXCEPT one. Which of the following is the EXCEPTION? A. To expose the ends of enamel rods instead of the sides B. To improve the seal of the enamo-resin margin C. To increase the surface area of enamel for etching D. To expose more inorganic tooth structure

D. To expose more inorganic tooth structure 1) Concept: Exposing organic layers of enamel is insignificant for bonding composite and tooth structure and is NOT a reason to place a bevel into the cavosurface margin. 2) Beveling is performed to a. Expose ends of the enamel rods b. Improve the marginal seal of composite restoration with tooth structure c. Increase the surface area of enamel for etching d. Eliminate the ling between filling and tooth

Gingival recession would MOST LIKELY be caused by which of the following? C. Occlusal trauma D. Toothbrush abrasion

D. Toothbrush abrasion 1) Improper tooth brushing is the most common cause of gingival recession

Which of the following is the most UNDESIRABLE property of sodium hypochlorite? A. Corrosiveness to endodontic files B. Reaction with chelating agents C. Tendency to bleach tooth structure D. Toxicity to vital tissues E. Smell

D. Toxicity to vital tissues 1) Sodium Hypochlorite a. undiluted or excessive amounts is toxic to vital tissues and can cause periodical tissue damage and pain b. good disinfectant that is used in dentistry as an endodontic canal irritant at a diluted concentration c. disinfect gutta percha points prior to placement and obturatoin of the canal

Which of the following shapes best describes the outline of an access preparation of a mandibular 1st molar with 3 canals? A. Oval B. Pentagonal C. Triangular D. Trapezoidal

D. Trapezoidal 1) Access Preparation a. design for the mandibular 1st molar is trapezoidal in shape, since it follows the position of the cusp tips 2) Mandibular 1st molar a. has 5 cusp with pulp horns underneath each cusp b. access preparation must expose the whole pulp chamber, so the position of cusps is used as a guide for the access prep outline c. have two roots, a medial and distal root d. mesial root has a mesiobuccal and mesiolingual canals which are both located under the cusp tips that bear the same name. e. distal root has one very large canal or two root canals

The predominant cause for failure of autgenous 3rd molar transplants is... A. Immune rejection B. Inadequate root length C. Infection D. Traumatic occlusion

D. Traumatic occlusion 1) Concept: Autogenous tooth transplantation is the process of utilizing the freshly extracted impacted or erupted teeth from its original position in the arch and transferring them into the recipient site of the same person. 2) There are several reason to explain the failure of autogenous tooth transplants which include chronic root resorption, inflammatory resorption, marginal and apical periodontitis, dental caries, and occlusal trauma. 3) 3rd molars used for autogenous tooth transplant procedures most commonly fail due to their inability to withstand the heavy occlusal forces that exist in the posterior during mastication. 4) Due to the mechanical advantage of a Type III lever like the jaw, occlusal force increases as you move posteriorly in the mouth (closer to the fulcrum/condyle)

Which disorder is a considered a chromosomal abnormality that is a result of either chromosomal nondisjunction or translocation? A. Ehler-Danlos syndrome B. Klinefelter syndrome C. Turner syndrome D. Trisomy 21 (Down syndrome)

D. Trisomy 21 (Down syndrome) 1) Concept: Patients with Down's syndrome possess 3 copies of chromosome 21 instead of 2 copies. This condition is considered as the most commonly occurring chromosomal abnormality in humans. 2) Trisomy 21 occurs before of meiotic nondisjunction, which is a situation where a gamete from a parent contains excess chromosomes. 3) Affected gametes posses 24 chromosomes instead of only 23 which upon combining with a "normal" gamete, will form an embryo that has 47 chromosomes with three set of chromosomes 21.

Signs of occlusal trauma are evidence radiographically by each of the following EXCEPT one. Which is the EXCEPTION? A. Changes of the periodontal space B. Hypercementosis C. Root resorption D. Vascular infiltration of the periodontal ligament E. Changes of the lamina dura

D. Vascular infiltration of the periodontal ligament 1) Vascular infiltration of the periodontal ligament is NOT seen in radiographs. 2) Radiographic images of trauma a. funneling and widening of lamina dura b. root resorption c. hypercementosis

An endodontically treated tooth was restored using a cast post-and-core with a porcelain fused to metal crown. The patient complains of pain on biting 3 months after the restoration was completed. Radiographs and mobility are normal. Which of the following is the most probable reason the patient is feeling pain? A. Loose crown B. Open crown margin C. Psychosomatic D. Vertical root fracture E. Premature contact

D. Vertical root fracture 1) Concept: Endodontically treated teeth with cast post and core restorations that occlude under heavy occlusal stress may end up with a vertical root fracture. 2) Cast post and core restorations are solid restorations that completely adapt into the canal system. 3) Occlusal and shear forces encountered by the cast post-and-core is immediately transmitted to the weakened endodontic tooth, resulting in a vertical root fracture.

Pulpotomy procedures on primary molars depend on primarily on which of the following factors for success? A. Absence of root resorption B. GV black classification of coronal caries C. The age of the patient D. Vitality of tooth pulp

D. Vitality of tooth pulp 1) Pulpotomy a. Success of a pulpotomy procedure lies on the ability of the remaining healthy pulp to remain vital and not irreversibly inflamed after the procedure b. procedure that removes only the portion of he coronal pulp that is infected and inflamed, which keeps the radicular portion vital c. cover the pulptomized tooth with an SSC crown may increase the chance for success of treatment

Each of the following statements about restorative dentistry is TRUE EXCEPT one. Which is the EXCEPTION? A. Attrition wear occurs in occlusal contact areas B. Polymerization shrinkage occurs toward the walls of cavity preparations to which it is bonded most strongly C. The release of unbound monomers and/or additives is hypothesized to be the cause of the cytotoxicity observed in in-vitro studies of resin-based composites D. Water absorption is greater in composites with smaller particles than in composites than with larger particles

D. Water absorption is greater in composites with smaller particles than in composites than with larger particles 1) Water absorption is NOT greater in composites with micro fine particles than in composites with fine particles 2) Materials with HIGHER filler contents exhibit LOWER water absorption than materials with lower filler content 3) Polymerization shrinkage occurs toward the walls of cavity preparations to which it is bonded most strongly. 4) Attrition wear occurs in occlusal contact areas. 5) Release of unbound monomers and/or additives is hypothesized to be the cause of the cytotoxicity observed in in-vitro studies of resin-based composites.

Which of the following statements is TRUE concerning the carving of amalgam? A. Carving strokes should always start on amalgam and proceed to enamel B. It is best to use dull instruments when carving amalgam C. Marginal angles should be made acute to help in resisting fracture of occlusal amalgams D. While the matrix is in place, the marginal ridge should be carved very nearly to the height of the adjacent marginal ridge

D. While the matrix is in place, the marginal ridge should be carved very nearly to the height of the adjacent marginal ridge 1) Concept: While the matrix is in place, the marginal ridge should be carved very nearly to the height of the adjacent marginal ridge. 2) Initial carving of the occlusal surface should be rapid, concentrating primarily on the marginal ridge height and occlusal embrasure areas. 3) The explorer tip is pulled along the inside of the matrix band mimicking the adjacent tooth, creating the occlusal embrasure form.

Which of the following describes how the surface of the enamel should appear after acid etching of the enamel is complete? A. Blue and opaque B. Brown and opaque C. Brown and shiny D. White and opaque E. White and shiny

D. White and opaque 1) Concept: After acid etching of the enamel tooth surface, it appears white and opaque in appearance commonly described as possessing a frosty appearance. 2) The frosty appearance is due to the action of acids causing selective demineralization of the area, creating porosities in the superficial enamel surface. 3) Resin tags are formed in the porosities following application of adhesive and resin composite materials, creating micro mechanical interlocking and a strong resin bond is formed.

Which of the following is the most likely etiology of a sudden increase in temperature one week after surgery? A. Atelectasis B. Subacute Bacterial Endocarditis C. Urinary tract infection D. Wound infection

D. Wound infection 1) Concept: Sudden increases in body temperature is an indication of fever which are typically initiated when infection is present in the body 2) A patient experiencing fever one week after a surgical procedure indicates a presence of infection. 3) After one week, the surgical wound should have reduction or total absence of swelling and should demonstrate a decrease in the redness of the wound. 4) Patients who experiences swelling and fever that does not subside after the 3rd or 4th day after the surgical procedure should seek care as it may be due to infection.

Leakage of the margins of an amalgam restoration... A. is high compared to silica cements B. is least at the time of restoration placement C. is related to the ratio of gamma versus beta phases of amalgam D. decreases as the restoration gets older

D. decreases as the restoration gets older 1) Amalgam restorations a. release corrosion products that fill in and seal the micro-gap in between the tooth and restorative material with age b. improves the seal within the margins, decreasing marginal leakage and percolation of fluids in the micro-gap c. considered to be the only dental restorative material that improves its marginal sealing with age

Precarve burnishing after condensation amalgam is important because which of the following reasons? A. It eliminates the need for retentive grooves B. It enhances the final shine of the restorations C. It ensures that any residual mercury is incorporated into the restoration D. it produces denser amalgam at the margin of restorations

D. it produces denser amalgam at the margin of restorations 1) Concept: Precarve burnishing should take place soon after condensation of the amalgam is completed in order to produce denser amalgam at the margin of restorations. 2) When the amalgam is placed or inserted to slight excess with condensers, it should be pre carved burnished with a burnisher. 3) It is done to achieve: a. Finalization of condensation b. Removal of excess mercury rich amalgam c. Initiation of carving process

The primary symptoms to demonstrate when midazolam (Versed) has been injected intra-arterially rather than intravenously is: A. rapid decrease in blood pressure B. rapid decrease in respiration rate C. severe cardiac arrhythmia D. severe pain in the extremity E. exaggerated sedation due to the more direct effect of the drug

D. severe pain in the extremity 1) Intra-arterial injection of midazolam stated that some of the patients experiences moderate to severe pain within the extremity where it was administered. 2) Adverse reactions from accidental intra-arterial injection can be localized within the site of injection or systemic.

Dental Caries

Dental Caries Answers

Dentofacial Variations

Dentofacial Variations Answers

Disabled and Medically Compromised

Disabled and Medically Compromised Answers

Which of the following age groups is most likely to incur primary herpetic gingivostomatitis? A. 11-15 years B. 16-20 years C. 21-25 years D. 6-10 years E. 1-5 years

E. 1-5 years 1) Concept: Vesicles commonly appear on the gingiva, oral mucosa, and vermilion border and usually affects children 5 years old or less. 2) Herpetic gingivostomatitis a. an acute viral disease cause by the herpes simplex virus 3) Primary herpetic gingivostomatitis a. is an infection transmitted through direct contact or through aerosol droplet that clinically apparent as yellow fluid-filled vesicles that leave ulcers when the vesicles rupture. 4) Secondary herpetic gingivostomatitis a. a reactivation of the latent virus due to trauma, stress or immunosuppression. b. The vesicles maybe found intramurally or extra orally on keratinized surfaces.

A patient has a 5mm probing depth on the medial of their maxillary first molar. 2mm of recession was also charted in that area. Which of the following describes the attachment loss in that area? B. 3mm E. 7mm

E. 7mm 1) CAL = PD - (FGM-CEJ) = 5 - (-2) = 7mm 2) Health periodontium has a sulcular depth of 1-3mm 3) Health junctional epithelium is typically associated with an attachment of the junctional epithelium at the cementoenamel junction (CEJ).

Which of the following should be the proximal cavosurface angle be on for a MO preparation? A. 120o B. 30o C. 45o D. 60o E. 90o

E. 90o 1) Concept: The proximal cavo-surface angle for a MO preparation should be approximately 90o. Enamel cavosurface angles of 90-100o are considered ideal. 2) It helps to conserve the dentinal support and the strength of the restoration and enables the restoration to withstand the forces of mastication adequately without undergoing fracture.

Which of the following conditions indicates excisional biopsy MOST? A. A 10mm dome shaped fluctuant lesion in the labial mucosa B. A 20mm asymptomatic ulcer present for 9 weeks C. A 30mm submucosal indurated lump D. A white surface lesion covering most of the buccal mucosa E. A 6mm exophytic mass on the buccal mucosa

E. A 6mm exophytic mass on the buccal mucosa 1) Concept: Exophytic masses a. are proliferating tissues that enlarge in its size as they grow outward from its origin. b. may become tumors and sometimes may turn malignant 2) Excisional Biopsy a. involve the complete removal of lesions and is most appropriate for small and accessible lesions amenable to surgery b. should be performed to rule out malignancies and avoid involvement of other structures in the mouth c. should be performed instead of incisional biopsies if: 1) The lesion is < 1cm (10mm) 2) The lesion is pedunculated

Two weeks following the obturation of a maxillary central incisor due to trauma, a radio-opaque horizontal line was noted in the midroot area, extending both medially and distally and into the periodontal ligament (PDL) space. This finding is most likely indicative of... A. Accessory canals E. A midroot fracture

E. A midroot fracture 1) Midroot fracture a. become more visible after root canal treatment as the sealer flows out of the space in between the fractures b. prognosis of a midroot fracture is relatively poor c.

If there is insufficient oral mucosa to achieve closure when repairing an alveolar cleft, the most common method to achieve closure is... A. Dermal transplant B. Freeze-dried dura C. Stem cell pack D. Teflon-proplast E. A tongue/pedicle flap

E. A tongue/pedicle flap 1) Pedicle Flap a. indicated in cases where mucosa quantity is insufficient to securely cover the wound opening or cleft b. created with a portion of its base remaining attached from the original site so that a blood supply is still present within the flap even after repositioning it to cover the wound opening c. best option for wound closures and carry a high rate of success

Root surface caries is MOST COMMONLY associated with which of the following microbial species? A. Porphromonas gingivalis B. Streptococcus mutans C. Streptococcus salivarius E. Actionmyces viscosus

E. Actionmyces viscosus 1) Actinomyces viscosus = commonly associated with acid production and root caries 2) Porphyromonas gingivalis = pathogen of periodontitis 3) Streptococcus gordinii = early colonizers involved in plaque formation 4) Streptococcus mutans = coronal caries 5) Streptococcus salivarius = common oral bacterium and resides on the tongue

A 20-year-old female presents to you with diffuse gingival erythema and scattered isolated ulcers on the tongue and oropharynx. The marginal gingiva appears inflamed and edematous. The patient states that she has been in pain for three days duration and demonstrates a fever and malaise. Which of the following is the most probable diagnosis? A. Acute necrotizing ulcerative gingivitis (ANUG) B. Apthous stomatitis C. Burning mouth syndrome D. Erosive lichen planus E. Acute herpetic gingivostomatitis

E. Acute herpetic gingivostomatitis 1) Concept: Gingivostomatitis a. is an inflammation of the oral mucosa and gingiva caused predominantly by HSV-1 b. Primary herpetic gingivostomatitis is an infection transmitted through direct contact or through aerosol droplet that clinically apparent as yellow fluid-filled vesicles that leave ulcers when the vesicles rupture. c. Secondary herpetic gingivostomatitis is a reactivation of the latent virus due to trauma, stress or immunsuppression. The vesicles and ulcerations may be found intramurally and extra orally on keratinized surfaces. d. Herpes presents as numerous pin-head vesicles, which rupture rapidly to form painful irregular ulcerations covered by yellow-grey membranes. e. Prodromal symptoms include 1) fever 2) anorexia 3) irritability 4) malaise 5) headache

Which of the following bacterial species is the most associated with localized aggressive periodontitis? C. Streptococcus mutans D. Treponema denticola E. Aggregatibacter Actinomycetemcomitans

E. Aggregatibacter Actinomycetemcomitans 1) Aggregatibacter Actinomycetemcomitans is the primary etiologic pathogen of localized aggressive periodontitis 2) Streptococcus gordinii is one of the early colonizers involved in plaque formation. 3) Streptococcus mutans is the most common oral bacteria and is highly associated with caries 4) Lactobacillus casei is commonly founding the intestines and oral cavity and produces acid critical in enamel demineralization.

Which of the following treatment is indicated for a patient with irreversible pulpits on tooth #8 with an open apex? A. Calcium hydroxide pulpotomy B. Conventional root canal treatment C. Direct pulp cap D. Indirect pulp E. Apexification

E. Apexification 1) Apexification a. procedure where calcium hydroxide or MTA is placed in the apical end of a tooth to close the apex of teeth with a necrotic pulp so that the tooth can be filled with a biocompatible material such as gutta percha so the tooth will be restored permanently 2) Direct pulp capping a. procedure where vital pulp that is mechanically exposed is capped with calcium hydroxide and a base to promote secondary dentin formation 3) Indirect pulp capping a. procedure where a pulp capping material is placed over an area of in which the clinician left affected dentin during caries excavation that is approaching the pulp so that it can be reminderalized so that a root canal can be possibly avoided 4) Pulpotomy a. procedure commonly performed on carious primary tooth where the pulpal inflammation extends only in the crown

Each of the following conditions is characterized by an ulcerations that are preceded by a vesicular lesion EXCEPT ONE. Which condition is the EXCEPTION? A. Coxackie virus infection (Hand-foot-and-mouth-disease) B. Herpangina C. Herpes zoster D. Herpetic stomatitis E. Apthous stomatitis

E. Apthous stomatitis 1) Concept: Apthous stomatitis is characterized by the repeated formation of ulcers in the mouth, in otherwise healthy individuals. b. Apthous stomatis ulcers occur periodically and heal completely between attacks with pain symptoms ranging from a minor nuisance to interfering with eating and drinking. c. Ulcerations present in apthous stomatitis cases are NOT preceded by vesicular lesions. d. All other conditions listed are characterized by the presence of ulcers with vesicular lesions.

A 14-year-old patient presents with an indurated and asymptomatic swelling in the mandibular premolar area. All teeth tested vital. Radiographs reveal a discrete 41mm radiolucency in the premolar area. Which of the following diagnostic procedures should be done to discern the proper diagnosis? A. Alkaline phosphatease determination B. Complete blood cell count C. Computed tomography scan D. Excisional biopsy E. Aspiration/needle biopsy

E. Aspiration/needle biopsy 1) After evaluating the diagnostic data, the next step is to determine the type of cells within the lesion through needle biopsy. 2) Histopathologic examination of cells within a lesion can determine if the lesions neoplastic or not. 3) Combining the radiographic, visual, physical and histopathologic assessment of the lesion improves the changes of arriving at a correct diagnosis and the proper treatment.

Which of the following teeth is most like to NOT erupt into the arch due to overcrowding? A. 1st premolar B. Central incisor C. Lateral incisor D. 2nd premolar E. Canine

E. Canine 1) Concept: Canines are the anterior teeth that are most likely to be unable to erupt within a crowded dentition because the canine erupts after the incisors and premolars have already erupted. 2) Tooth crowding may occur in patients with teeth that are too wide mesiodistally or if their dental arch is too small to accommodate all the erupted teeth. 3) Tooth crowding carries a genetic predisposition because it is often a matter of tooth size vs. arch size.

Which permanent teeth must erupt to prevent the mandibular intercanine width from increasing? A. 1st premolar B. 2nd premolar C. Central Incisor D. Lateral Incisor E. Canine

E. Canine 1) Concept: When permanent mandibular canine erupts, it moves mesially and maintains a stable position within the corner of the mandibular arch. 2) Mandibular canines a. are the corner tooth which, by definition, establishes the inter canine width of the mandibular arch b. As the mandibular canine push the lateral incisors in a mesial direction, they also prevent any further increase in the intercanine width.

Which of the following ocular conditions necessitate that extra precautions be taken when curing composite resins with visible-light curing devices? A. Myopia B. Narrow angle glaucoma C. Presbyopia D. Wide angle glaucoma E. Cataract removal

E. Cataract removal 1) Following cataract removal, eyes are very sensitive to light.

A 60-year old female "Connie" presents to your office with a 8mm x 8mm blue-black soft fluctuant lesion of her lower lip. She states that the lesion is asymptomatic and has exhibited persistent growth over the last 10 years, and bleeds profusely when she bites her lip. Which of the following is the most likely diagnosis? A. Lipoma B. Neurofibroma C. Nodular melanoma D. Verrucous carcinoma E. Cavernous hemangioma

E. Cavernous hemangioma 1) Concept: Cavernous hemangiomas a. class of blood vessel malformation characterized by a large blood-filled space and contain blood, not tissue b. Hemangiomas can arise virtually anywhere in the body and are considered to be benign neoplasms. c. Unlike capillary hemangiomas, cavernous hemangiomas can be disfiguring and do not tend to regress. d. Cavernous hemangiomas may lead to spontaneous or traumatic bleeding.

Which of the following anatomical features of a tooth is mistaken MOST OFTEN for a carious lesion in radiographs? A. Cingulum B. Cusp C. Marginal ridge D. Pulp Horn E. Cemento-enamel junction

E. Cemento-enamel junction 1) Apparent radiolucency is often found just below the cemento-enamel junction on the root surface called "cervical burn out" and is often misdiagnosed as root caries in many cases. 2) Cervical burn-out is due to a. gap between the enamel and cementum or bone covering the root anteriorly b. concave root formation posteriorly

Delayed eruption of permanent teeth can be cause by which of the following conditions? A. Peutz-Jeghers syndrome B. Hyperparathyroidism C. Hyperthyroidism D. Paget's disease of bone E. Cherubism

E. Cherubism 1) Cherubism a. bony prominence in the lower portion in the face and uneruption of permanent teeth b. associated with premature deciduous tooth loss due to the sponge-like quality of bone c. associated with the uneruption of permanent teeth due to displacement of cysts and lesions d. loss of bone in the mandible replaced with excessive amounts of fibrous tissue e. condition should fade as child grows but may continue to disform the patient's face

Which of the following describes the condition where teeth are joined only by cementum? A. Dilaceration B. Fusion C. Gemination D. Hypercementosis E. Concrescence

E. Concrescence 1) Concrescence a. condition where the cementum overlying the roots of at least 2 teeth connects to join the teeth together b. surgical separation of the teeth may be necessary if one is to be extracted, which may result in damage to the adjacent tooth root c. incidence of concrescence can sometime be attributed to trauma or crowding of teeth 1) most common in the permanent molars 2) different from fusion because it occurs following eruption and involves ONLY CEMENTUM

Most vertical root fractures of endodontically treated teeth result from which of the following? A. Cementation of a cast post and core B. Permanent cementing of crowns after treatment C. Removal of temporary restorations on prepared teeth D. Traumatic occlusion E. Condensation forces during gutta percha filling

E. Condensation forces during gutta percha filling 1) Concept: Endodontically treated test are weaker and more prone to fracture when excessive condensation forces is used during obturation. 2) The width of the prepared root canal compared to the remaining tooth structure also affects the ability of the endodontically treated to withstand condensation forces. 3) During the condensation of the gutta percha, the force exerted during the compaction of the gutta percha is transmitted back to the walls of the tooth.

A patient presents to your office with a tooth #19 that tests indicate pulp necrosis, pain on percussion; and no periodical lesion present. Which of the following should the emergency treatment consist of? A. Open the tooth to achieve drainage and leave the tooth open to drain B. Open the tooth, perform apical trephination, temporize, and check occlusion C. Perform a pulpotomy, temporize, and check the occlusion D. Prescribe antibiotics and analgesics and reschedule the patient for later E. Debride the canals, temporize, and check the occlusion

E. Debride the canals, temporize, and check the occlusion 1) Emergency endodontic treatment a. removal of necrotic pulp through debridement of the root canals b. placement of temporary dressing material within the root canals c. affected tooth needs to be evaluated for high occlusal spots d. occlusion of the affected tooth should be thoroughly checked and sometimes needs to be removed from occlusion as a preventive measure to relieve pain on occlusion

The primary goal of the initial patient visit is..... C. Diagnosis and treatment plan formulation E. Determining the patient's chief complaint

E. Determining the patient's chief complaint 1) build rapport and trust with the patient 2) understand the patient's needs and expectations

Seizures induced by a lidocaine overdose can be treated by which of the following options? A. Epinephrine (EpiPen) B. Flumazenil (Roamzicon) C. Meperidine (Demerol) D. Naloxone (Narcan) E. Diazepam (Valium)

E. Diazepam (Valium) 1) Diazepam a. rapid acting and highly efficacious anticonvulsant drug indicated to treat anxiety, panic, and acute seizure attacks such as those induced by lidocaine b. injecting lidocaine which may reach a concentration beyond 8 to 9 ug/mL is said to pose greater risk for acute convulsions. c. intravascular Diazepam administration can stop a seizure attack triggered by lidocaine d. intramuscular injection of diazepam one hour prior to lidocaine injection can help prevent lidocaine induced-seizure attacks

A traumatic injury is possibly etiologic agent in: A. All answers apply B. Dwarfed roots C. Gemination D. No answers apply E. Dilaceration

E. Dilaceration 1) Concept: Dilaceration describes a severe angular or curve in the root or crown of a tooth that is thought to result from trauma during development. 2) Dilaceration a. trauma during tooth development may result in the poison of calcified portion of the tooth being changed so the remainder of the tooth is formed at an angle. b. curve or bend occur anywhere along the length of the tooth and depends on the amount of root that is already formed when the injury is sustained. c. An injury resulting in dilaceration may often occur when a deciduous predecessor is driven aplically into the jaw and applies force to the permanent tooth below d. may make future endodontic procedures more difficult

Severe dysplasia or carcinoma in situ is MOST OFTEN associated histologically with which of the following clinical lesions? A. Hairy leukoplakia B. Leukoplakia C. Lichen planus D. White sponge nevus E. Erythroplakia

E. Erythroplakia 1) Concept: Erythroplakia a. Erythroplakia are red lesions that considered to be one of the most common premalignant lesions in the oral cavity b. Histopathological examination of erythroplakias exhibit severe dysplasia, carcinoma in situ and invasive squamous cell carcinoma 2) Erythroplakias are MOST COMMONLY found: 1) Floor of the mouth 2) Tongue 3) Soft Palate 3) Erythroplakia appears red due to the following premalignant changes: a. Absence of keratin production b. Reduced number of epithelial cells c. Increased vascularity due to inflammation d. Thin and neoplastic epithelium

Each of the following aids in the diagnosis or evaluation of Sjogren's syndrome EXCEPT ONE. Which is the EXCEPTION? A. Labial salivary gland biopsy B. Salivary flow rate C. Serology D. Sialograms E. Exfoliative cytology

E. Exfoliative cytology 1) Exfoliative cytology a. uses the scraped off or exfoliated tissues to evaluate and examine lesions through cytopathologic analysis and CANNOT be used to diagnose Sjogren's syndrome. 2) Labial salivary gland biopsy a. important diagnostic tool in identifying affected salivary glands and is performed by taking a small sample of the salivary gland for analysis 3) Salivary flow rate a. any abnormality in saliva flow rate may hint for the presence of Sjogren's Syndrome 4) Sialograms a. used to determine any blockage or clogged structure at the salivary ducts or glands. It is a helpful tool to diagnose or rule out Sjogren's syndrome.

An anomaly in which there is a distortion of only the root of a tooth, which may be in the form of a right angle bend, is: A. Consecrence B. Dilaceration C. Dwarded root D. Segmented root E. Flexion

E. Flexion 1) Concept: Flexion is distortion of the ROOT portion ONLY 2) Dilaceration: a. distortion of the root and crown from their normally linear relationship 3) Consecrence a. union of the root structure of two or more teeth through CEMENTUM ONLY. 4) Dwarfed root a. exists when normal sized crowns have abnormally short roots 5) Segmented root a. two separated root segments due to break in Hertwig's sheath

Which of the following statements is TRUE? A. 2-step etch-and-rinse adhesives save time and improve bond performance B. Acid-etching should be done on enamel, not on dentin C. Adhesive resin removes the smear layer and creates a microporous layer D. Enamel has a variable structure with a higher percentage of organic material compared to dentin E. Formation of resin tags in enamel is important for bond strength

E. Formation of resin tags in enamel is important for bond strength 1) Bond Strength a. relies upon the formation of resin tags in enamel 2) Acid Etching a. thought to remove the smear layer and creates a micro porous enamel layer on which resin tags are formed by adhesive resins b. 2 step etch and rinse adhesives consume longer time periods as compared to the self-etch adhesives c. can be done on both enamel as well as dentin

Ramifications due to a lack of inter proximal contacts between amalgam restorations usually include which of the following? A. Poor marginal adaptation B. Sensitivity to cold C. Less interproximal debris D. Fremitis E. Gingival inflammation

E. Gingival inflammation 1) Lack of inter proximal contacts a. causes food debris to be trapped in the inter proximal space b. presence of food debris within the interproximal space promotes caries formation and gingival inflammation

During operative procedures, which of the following is the best method for prevention pulpal damage? A. Keep the dentinal surface without frequent cleaning B. Prepare dentin structure with slow-speed hand piece C. Use anesthetics without vasoconstrictors D. Use light, short duration bursts of tooth-bur contact using air cooling E. Minimize dehydration of the dentinal surface

E. Minmize dehydration of the dentinal surface 1) Pulpal damage a. when too much heat is transmitted to dentin causing dentin dehydration b. dry cutting of the tooth structure causes heat build-up along the tooth surface, eliciting pain and causes damaged due to trauma to the pulp c. use of water spray during tooth preparation decreases heat build-up and minimizes dehydration of dentin

Which of the following changes in lingual papillae results in the manifestation of a black hairy tongue? A. Atrophy of the circumvallate papillae B. Atrophy of the filiform papillae C. Atrophy of the fungiform papillae D. Hypertrophy of the circumvallate papillae E. Hypertrophy of the filiform papillae F. Hypertrophy of the fungiform papillae

E. Hypertrophy of the filiform papillae 1) Concept: Black hairy tongue a. Hypertrophy of the filiform papillae manifests itself as black hairy tongue after the bacteria or yeast attach the papillae b. Filiform papillae become elongated and give hair like presentation on the dorsum side of the tongue. c. Black hairy tongue is most commonly caused by too much bacteria or yeast growth on the tongue d. Black hairy tongue is more common in men, people who use intravenous drugs, and those who are HIV-positive. e. The dorsum of the tongue may become blackish in color due to the use of 1) poor oral hygiene 2) smoking tobacco 3) drinking a lot of coffee or tea 4) using antibiotics (which may disrupt the normal balance of bacteria in the mouth) 5) being dehydrated 6) taking medications that contain the chemical bismuth (such as Pepto-Bismol for upset stomach) 7) xerostomia 8) regularly using mouthwash that contains peroxide, witch hazel, or menthol 9) getting radiation therapy to the head and neck

Freeze dried bone is characterized by which of the following? C. It has osteogenic potential E. It will be replaced by the host bone eventually

E. It will be replaced by the host bone eventually 1) Freeze dried bone allografts a. demonstrates osteoconductivity, which it means that it provides a scaffold or matrix for the repairing host bone to replace it b. help support new bony defects by forming new bone and periodontal attachment through providing a scaffold as it is being replaced by the newly formed host bone c. does not have osteogenic potential but DOES help to repair bone through osteoconduction

Acute pyogenic bacterial infections are known to result in a state of... A. Leukopenia B. Lymphocytosis C. Lymphopenia D. Neutropenia E. Leukocytosis

E. Leukocytosis 1) Leukocytosis a. white blood cell count above the normal range b. sign of inflammatory response that is most commonly the result of infection c. observed with certain parasitic infections, cancer, after strenuous exercise, emotional stress, pregnancy, anesthesia steroid, and epinephrine administration

Permanent maxillary 1st molars may show evidence of congenital syphilis by exhibiting a mulberry shape. Which of the following teeth are most likely to exhibit anomalous form due to congenital syphilis? A. Mandibular 2nd premolar B. Mandibular canine C. Mandibular lateral incisor D. Maxillary 1st premolar E. Maxillary central incisor

E. Maxillary central incisor 1) Concept: Hutchinson's Teeth and Mulberry Molars a. caused by a prenatal Treponema Pallidum (Syphilis) infection that disturbs the calcification of the teeth during the morphodifferentiation of ameloblasts. b. incisors and 1st molars are most commonly affected because they are the only permanent teeth at this stage of development c. incisors typically exhibit a notch in the center of the incisal edge, making them appear like a screwdriver d. 1st molars typically appear with poorly developed cusps and gnarled enamel, giving arise to the name "mulberry molars" 1) primary teeth are typically NOT affected 2) esthetics is the primary concern of patients with this condition

Which of the following has been NOT a common reason why dentists retire early? D. Rotator cuff injury E. Mercury toxicity syndrome

E. Mercury toxicity syndrome 1) Dentists use suction to remove mercury vapor coming from the removal of amalgam. Cutting of amalgam without water spray should be avoided.

Which of the following bacterial species is considered an early colonizer of supra gingival plaque? B. Lacrobacillus casei C. Porphyromonas gingivalis D. Streptococcus salivarious E. Streptococcus sanguinis

E. Streptococcus sanguinis 1) Plaque formation Stages a. formation of pellicle on the tooth surface b. initial colonization by bacteria c. secondary colonization with plaque maturation 2) Supragingival plaque consists of gram positive facultative microorganisms a. Actionmyces viscosus b. Streptococcus sanguinis

Each of the following is associated with acute herpetic gingivostomatitis EXCEPT one. Which is the EXCEPTION? A. A self-limiting disease lasting 10 days B. A viral infection cause by the herpes simplex virus C. Fever D. Initial oral lesions start as vesicles E. Mucosal scaring after lesions are healed

E. Mucosal scaring after lesions are healed 1) Acute Herpetic Gingivostomatitis a. MOST COMMON oral infection viral oral infection seen in children b. lymphadenopathy and fever c. vesicles appear 2 days later, which go on to pop and form ulcerations c. preference for the hard palate d. clear preference for keratinized tissues e. treated with valcyclovir

A 35 year-old man presents with multiple macular pigmented spots on the skin and multiple soft nodules. What is the most likely diagnosis? A. Lipomatosis B. Metastatic malignant melanoma C. Nevoid basal cell carcinoma syndrome D. Polyostotic fibrous dysplasia E. Neurofibromatosis

E. Neurofibromatosis 1) Concept: Neurofibromatosis a. genetic condition that manifests as cafe au lair spots and multiple soft nodules which may occur anywhere else in the body. b. Neurofibromatosis occurs with many varieties with the most commonly occurring variety being the NF1 or von Recklinghausens disease. c. No cure currently exists for neurofibromatosis and is only managed by treating the symptoms as they arise.

Which of the following is the MOST numerous cell type observed in inflammatory infiltrate of an acute periodontal abscess? A. B-lymphocytes B. CD-4 T-lymphocytes C. CD-8 T-lymphocytes D. Macrophages E. Neutrophils F. Plasma cells

E. Neutrophils 1) Neutrophils (PMNs) a. first to arrive at the site of inflammation b. first line of defense against any foreign invader of the body c. comprise about 50-60% of the total number of leukocytes circulating in the bloodstream

Which of the following agents does NOT produce vasoconstriction? A. Levonordefrine B. Norepinephrine C. Phenylephrine D. Epinephrine E. Nitrous oxide

E. Nitrous oxide 1) Concept: Nitrous oxide is converted into nitric oxide, which is known to induce vasoDILATION not vasoCONSTRICTION. 2) Epinephrine, levonodefrine, norepinephrine, and phenylephrine all induce vasoconstriction when administered parenterally.

Which of the following lists the restorative materials in the order of greatest to least fluoride release? I. Compomer II. Resin-Modified Glass-Ionomer III. Glass-Ionomer IV. Resin Composite A. I, II, III, and IV B. III, I, II, IV C. III, II, IV, I D. IV, III, II, I E. No Answers apply

E. No Answers apply 1) Concept: The fluoride releasing ability of various dental restorative material decreases in the order of: glass ionomer cements > resin modified glass ionomers > compomers > composites 2) This order is the same for fluoride recharging ability of materials from outside sources. 3) The greatest release occurs on the 1st day and subsequently decreases as the time passes. 4) Fluoride releasing materials can be recharged with external fluoride sources in order to increase their fluoride release and re-mineralizing potential.

Xerostomia (drymouth) is LEAST LIKELY to be induced with the administration of which of the following agents? D. Benzodiazepines E. Opioids

E. Opioids

When a dentist thinks they know what is best for the patient and tries to control a patients behavior, it is referred to as... A. Autonomy B. Benevolence C. Competence D. Justice E. Paternalism F. Nonmaleficence

E. Paternalism 1) Paternalism a. is displayed when we decide to follow what we think is best for out patient regardless of the choice of treatment that the patient has made for themselves, thereby infringing on the patient's right to autonomy.

Which of the following does a clinician place a mucosal graft onto in order to directly increase the anterior mandibular vestibular depth? A. Bone B. Epithelium C. Mucosa D. Muscle E. Periosteum

E. Periosteum 1) Mucosal Grafts a. clinicians place a mucosal graft directly onto the periosteum in order to increase the anterior mandibular vestibular depth b. have good prognosis with minimal chances of graft rejection c. periosteum has a rich blood supply that promotes healing and a quick connection to the graft d. less successful when placed directly over the bone, mucosa, muscle, or epithelium

Which of the following periodontal diagnoses is MOST COMMON in adults? A. Generalized chronic periodontitis E. Plaque associated gingivitis

E. Plaque associated gingivitis 1) Plaque associated gingivitis (marginal gingivitis) is the MOST Common periodontal diagnosis.

Which of the following patient health histories would make the use of nitrous oxide sedation CONTRAINDICATED? A. Controlled hypertension B. Dental anxiety C. Diabetes Type I D. Diabetes Type II E. Psychotic care

E. Psychotic care 1) Nitrous oxide sedation a. contraindicated in patients who are undergoing psychotic care b. can be administered to patients with 1) controlled hypertension 2) dental anxiety 3) diabetes type I 4) diabetes type II 2) Various CONTRAINDICATIONS of nitrous oxide sedation are a. severe emotional disturbance b. severe drug-related dependence c. COPD-bronchitis, emphysema, or Upper Respiratory infection (URI) d. Otitis Media e. Claustrophobia or irrational fear of "gas" f. Maxillofacial deformities or nasal obstructinos g. Pregnant patients - especially in first trimester h. Sickle cell disease i. Patients being treated with bleomysin sulfate j. Methylenetetrahydrofolate reductase deficiency

Each of the following statements about resin based composites (RBCs) as a posterior restorative material are correct EXCEPT one. Which is the EXCEPTION? A. RBC preps conserve more tooth structure than amalgam preps B. RBCs adhere to tooth structure C. RBCs demonstrate low thermal conductivity D. RBCs provide superior esthetics E. RBCs should always be used instead of amalgam

E. RBCs should always be used instead of amalgam 1) Concept: Amalgam has been demonstrated to be a better restoration for rests of removable partial dentures than that of resin based composites. 2) Amalgam core buildups have also demonstrates better longevity than that of composites. 3) It is preferable to use amalgam in areas experiencing high masticatory and functional loads, areas where isolation cannot be maintained and patients with parafunctional habits like bruxism and clenching. 4) Posterior composite restorations are recommended in the given conditions: a. Where esthetics is the primary concern b. In cases requiring conservation of tooth structure c. Where low thermal conductivity is required d. Where bonded restorations are feasible and isolation can be obtained

The most likely outcome of waiting for intruded permanent central incisors to erupt in an 8 year old is... A. Ankylosis B. External resorption C. Hypocalcification spots on the tooth upon eruption D. Internal resorption E. Re-eruption

E. Re-eruption 1) Concept: Re-eruption a. An intruded permanent central incisor will eventually start to re-erupt within a period of approximately 2-3 weeks b. Deciding for the mode of treatment for an intruded tooth may vary depending upon the severity of tooth intrusion and the maturity of its roots c. The modalities of treatment for an intruded tooth include: 1) Passive repositioning (PR): provides enough time for the re-eruption of intruded tooth 2) Active repositioning is performed through surgical repositioning (SR) 3) Orthodontic repositioning is achieved with the use or removable or fixed appliances (OR)

During an ostectomy procedure, the surgeon always... A. Performs a block graft B. Performs an allograft C. Performs an autograft D. Removes non-tooth supporting bone E. Removes tooth-supporting bone

E. Removes tooth-supporting bone 1) Concept: Ostectomy a. is a surgical procedure that removes supporting bone in order to help reduce or eliminate periodontal pockets 2) Osteoplasty a. a surgical procedure that removes non-supported bone b. completed before the ostectomy in order to allow for the most conservative removal of bundle bone (supporting bone) around the teeth 3) When pocket depth is decreased, the effectiveness of oral hygiene measures increases.

Which of the following methods should NOT be used in the extraction of a mandibular 1st molar with severe coronal decay? A. Elevating the root sections with an east-west elevator B. Reflecting a modest buccal flap and removing the buccal bone C. Splitting the tooth into medial and distal root sections D. Using a cow horn forceps E. Removing the thin lingual cortical plate of bone

E. Removing the thin lingual cortical plate of bone 1) Lingual cortical plate a. is very thin and close to the lingual nerve and should not be removed 2) Mandibular 1st molar extraction a. removed easier with the use of cowhorn forceps that engages the tooth in a furcation area or by sectioning the root b. buccal plate can be reduced with a buccal trough to provide adequate access and purchase area.

Which of the following characteristics is generally the LEAST favorable in a sealant material? A. Filled B. Fluoride release C. Light cured D. Opaque/tinted E. Self-cured

E. Self-cured 1) Self-curing nature of a resin a. least favorable characteristic in a sealant b. DO NOT provide sufficient working time to the operator 2) Favorable properties a. light cured polymerization b. fluoride release c. opaque/tinted composites resins in order to differentiate from tooth structure d. filled composite resin

The beaks of a needle holder compared to the beaks of a hemostat are... A. Longer and thicker B. Longer and thinner C. More curved D. Short and thinner E. Shorter and thicker

E. Shorter and thicker 1) Needle holder a. beaks are shorter and thicker b. designed to provide grip and stability while holding the needle in place during suturing 2) Hemostats a. beaks are a bit longer and thinner b. designed to hold the tissues together to control bleeding

The filler particles within resin composites are coated with which of the following substances to promote adhesion to the matrix? A. Bis-EMA B. Big-GMA C. Comphorquinone D. Polymethylmethacrylate E. Silane

E. Silane 1) Silane a. filler particles of the resin composites are coated with silane coupling agents to promote adhesion to the matrix b. coupling agent that enhances the bonding between the filler particles and resin matrix c. synthetic hybrid inorganic-organic compounds which are also used to promote adhesion between dissimilar materials. Good in promoting adhesion in silica based materials such as porcelain. d.

A clinician correctly classifies a group of patient as being free of disease based on a diagnostic test results. These results demonstrate high... A. Generality B. Precision C. Repeatability D. Sensitivity E. Specificity

E. Specificity 1) Specificity = being free of disease based on results 2) Sensitivity = show the group of patients which are suffering from a particular disease

Each of the following about spherical alloys compared to admixed alloys is true EXCEPT one. Which is the EXCEPTION? A. Spherical alloys are less sensitive to variations in condensation pressure B. Spherical alloys demonstrate a lower mercury: alloy ratio C. Spherical alloys demonstrate a shorter working time and faster set comparatively D. Spherical alloys demonstrate poorer adaptation to cavity walls E. Spherical alloys typically create a better inter proximal contact in Class II restorations

E. Spherical alloys typically create a better inter proximal contact in Class II restorations 1) Admixed alloys a. yield better inter proximal contacts for Class II restorations b. tends to resist condensation better than sperical alloys 2) Spherical a. very plastic and cannot rely on pressure of condensation to establish proximal contour b. require lower mercury:alloy ratio due to decreased surface area per unit volume c. have shorter working time, faster setting reaction and poorer adaptation to the cavity walls

Which of the following is the most likely complication of bleaching a tooth that was not adequately obturated? A. Asymptomatic apical periodontitis B. Discoloration C. External cervical root resorption D. Fracture E. Symptomatic apical periodontitis

E. Symptomatic apical periodontitis 1) Bleaching a non-obturated tooth a. Bleaching agents may leach ion the root canal treated tooth and beyond, into the apical portion causing periodical swelling or apical pain b. goal of canal obturation is to produce a hermetic seal of the apical terminus and prevent re-infection of the periodical structures c. without adequate seal at the apex of the root canal treated tooth, infection and chemicals may penetrate the apical tissues and cause the failure of the root canal treatment

The primary concern in planning a patient's treatment is... A. All answers are correct B. Arresting of all carious activity C. Establishment of good home care and dental home D. No answers are correct E. The general comfort and overall health of the patient

E. The general comfort and overall health of the patient 1) Beneficence a. dentist has a duty to promote your patient's comfort and welfare b. primary objective of the clinician is to respect the patient's right to be treated in a way that puts their rights to autonomy, beneficence, justice, nonmaleficence, and veracity above all else

A patient who places aspirin directly into the oral vestibule, often sees a well-circumscribed white patch on the mucosa where the aspirin was placed. What is the most likely description for disease process in the affected tissue? A. Atrophy B. Focal hyperkeratosis C. Focal hyperplasia D. Hypertrophy E. Tissue Necrosis

E. Tissue Necrosis 1) Aspirin a. contains acetyl salicylic acid which can cause acid burns within the mucosa of the oral cavity b. acid within the aspirin binds to the lining epithelium and causes protein destruction that results in acid burns of the mucosa c. affected mucosa experiences burning pain as a result of coagulation necrosis

Which of the following does not damage soft tissue? A. Home bleaching treatment B. Placement of a matrix band in a Tofflemire retainer C. Placement of gingival retraction cord D. Use of an inter proximal carver E. Topical fluoride treatment

E. Topical fluoride treatment 1) Fluoride a. in small controlled doses helps to strengthen teeth nd does not cause harm to the gums 2) Home bleaching treatment a. has high percentage of peroxide components which may cause harm or damage to gingiva and soft tissues 3) Gingival retractors a. placement of gingival retractors into the sulcus also causes certain damage to the marginal gingiva as the retraction cord displaces them from their original location. 4) Placement of matrix band in a Tofflemire retainer a. damages the gums surrounding the tooth because it keeps on sliding down into the gingival sulcus as the retainer is tightened 5) Interproximal carvers a. damage the gums while carving the restoration that approximates or extends into the marginal gingiva b. sometime such gingival injury is inevitable and is sacrificed for the sake of attaining proper anatomic contours and preventing overhangs in the restorations.

Which of the following bacteria are commonly associated with acute necrotizing ulcerative gingivitis (ANUG)? A. Actinomyces viscosus B. Aggregatibacter Actinomycetemcomitans C. Bacteroids forsythus D. Porphyromonas gingivalis E. Treponema denticola

E. Treponema denticola 1) Acute Necrotizing Gingivitis/Periodontitis (ANUG/ANUP) a. Treponema Denticola b. Risk factors 1) poor oral hygiene 2) smoking 3) malnutrition 4) fatigue 5) stress 6) immunocompromised patients 2) Aggregatibacter Actinomycetemcomitans a. localized aggressive periodontitis 3) Porphyromonas gingivalis a. chronic periodontitis 4) Actinomyces viscosus a. healthy gingiva 5) Bacteroids forsythus a. chronic periodontitis

Which bone makes up all or part of the inferior concha? A. Ethmoid B. Maxilla C. Palatine D. Sphenoid E. Turbinate

E. Turbinate 1) Concept: Tubrinates are narrow and sea-shell appearing bone that extends into the nasal air passage. 2) Turbinates a. have four grooved air channels which divides and forces the inspired air to flow steadily into the nasal passages. b. responsible for warming, humidifying, and filtering the inhaled air.

Which of the following correctly describes the position of the mandibular condyle during a procedure to correct mandibular prognathism? A. Advanced B. Intruded C. Protruded D. Retruded E. Unaltered

E. Unaltered 1) Condyle Position a. Recording of the position of the mandibular condyle during correction of mandibular prognathism should be done without any alteration b. In normal practice, the mandibular condyle position is recorded in protrusion. c. Patients of mandibular prognathism have condyles already in a protruded state.

Which of the following cancer diagnoses has the best 5-year survival rate? A. Stage 1 well-differentiated squamous cell carcinoma involving the floor of the mouth B. Stage 1 well-differentiated squamous cell carcinoma involving the right lateral tongue C. Stage 1 well-differentiated squamous cell carcinoma involving the soft palate D. Verrucous carcinoma of the floor of the mouth E. Verrucous carcinoma of the right lower posterior vestibule

E. Verrucous carcinoma of the right lower posterior vestibule 1) Concept: Verrucous carcinoma a. Verrucous carcinoma is a slow forming tumor that is usually seen along the mandibular vestibule, buccal mucosa, floor of the mouth, and mandibular gingiva. b. usually seen among people who had a long history of chewing tobacco c. usually appears as a white, papillary exophytic mass which is non-ulcerated d. prognosis of vestibular verrucous carcinoma is better than the ones in the floor of the mouth because there are many other important intraoral structures underneath the floor of the mouth to which the carcinoma may extend. e. With complete surgical removal of the carcinoma, patients become disease free over a five year period at about 90% of the time.

Each of the following is indicative of an intra-arterial injection EXCEPT one. Which is the EXCEPTION? A. Aspirated blood is bright red in color B. Injection of the test dose elicits acute pain C. Penetration of the vessel is met with resistance and elicits pain D. The needles moves with the rhythm of the heartbeat E. Vessels tend to collapse and obstruct

E. Vessels tend to collapse and obstruct 1) No obstruction or collapse of arteries is experienced during administration of an intra-arterial injection 2) Resistance is met during administration due to blood flow. 3) Aspirated blood is bright red in color due to the high concentration of oxygen in blood passing through the arteries 4) Intra-arterial injections are painful in nature.

Which of the following patient situations is associated with the highest caries risk? A. All answers are correct B. Arthritic C. Lactating D. Pregnant E. Xerostomic

E. Xerostomic

Bleeding under the conductive may indicate... A. Frontal sinus fracture B. Le Forte II fracture C. Le Forte III fracture D. Nasal bone fracture E. Zygomaticomaxillary complex fracture F. Le Fort I fracture

E. Zygomaticomaxillary complex fracture 1) Concept: Zygomaticomaxillary complex fracture or tripod/malar fracture involves fracture of a. Maxillary sinus and its walls and orbital floor b. Zygomatic arch c. Lateral orbital rim and the lateral orbital wall d. caused by a direct blow to malar eminence during a fight, a sport or vehicular accident, or due to a fall. 2) Clinical features a. flattened malar eminence b. facial bruising/swelling c. bleeding under the conjuctiva d. facial sensation loss below the orbit in an infraorbital nerve involvement and trismus

Which of the following is the principal reason dressings are used to treat localized alveolar osteitis (dry socket)? A. to enhance blood clot formation B. to prevent salivary contamination of the surgical site C. to promote epithelial growth D. to provide a vehicle for the obtundent medication E. to stimulated osteoblastic reconstruction of the surgical site

E. to stimulated osteoblastic reconstruction of the surgical site 1) Dressings a. placed inside the dry socket to stimulate the osteoblastic reconstruction of the surgical site b. promote clot formation in the socket and enhance healing c. changed on alternate days to speed up the healing process 2) Localized alveolar osteitis (dry socket) a. occurs when the blood clot fails to form or is lost from the socket b. bone lining the empty alveolar socket gets exposed c. condition associated with increased pain and delayed healing time

Epidemiology

Epidemiology Answers

Etiology

Etiology Answers

Evaluation of Dental Literature

Evaluation of Dental Literature Answers

Examination, Diagnosis, & Treatment Planning

Examination, Diagnosis, & Treatment Planning Answers

Which of the following diagnoses is the MOST PROBABLE for a patient with spontaneous necrotizing ulcers of the oral cavity and a CBC with differential that reveals 68% lymphocytes, 28% monocytes, 2% polymorphonuclear neutrophils, 1% eosinophils, and 1% basophils and WBC count of 1,986? A. Infectious mononucleosis B. Leukemia C. Primary Herpes D. Recurrent apthae E. Systemic Lupus Erythematosis F. Agranulocytosis

F. Agranulocytosis 1) Agranulocytosis a. health condition manifested with spontaneous mucous membrane ulcerations in the mouth, vagina or rectum b. occurs when the body fails to create mature and normal white blood cells making the body more susceptible to bacterial infection c. other manifestations 1) sore throat 2) rigor 3) chills 4) fever

The purpose of a reservoir bag in an anesthesia machine is to A. All answers are correct B. No answers are correct C. Provide a means of mixing N2O and O2 in measured amounts D. Provide for urinary incontinence in the sedated patient E. Provide resistance when the patient breathes out so more absorption of N2O is achieved in the lung F. Make up for the difference between the constant gas flow from the machine and the cyclic breathing pattern of the patient

F. Make up for the difference between the constant gas flow from the machine and the cyclic breathing pattern of the patient 1) Reservoir bag a. part of the anesthesia machine that allows accumulation of gas and serves as a gas reservoir that can be used to supply gas when controlling the ventilation manually. b. act as a visual guide to evaluate the breathing depth and the rate of depression of the patient c. balances the difference between the patient's breathing pattern and the available gas that is constantly flowing from the machine. d. manages to balance off the difference between the patient's breathing pattern and the available gas that is constantly flowing from the machine.

Which of the following acts as the BEST conditioner to clean the tooth's surface when applying glass ionomer adhesives? A. Acetone B. Ethanol C. Isopropyl alcohol D. No answers apply E. Phosphoric Acid F. Polyalkenoic Acid

F. Polyalkenoic Acid 1) Concept: Polyalkenoic Acid is used for glass ionomers because it is a weaker acid that does NOT penetrate into the dentin as much as phosphoric acid. 2) This is because the glass ionomers have a much higher molecular weight polymer that cannot penetrate into the smaller tubules created further gingival by the phosphoric acid. 3) Ethanol is a solvent primer used to remove excess water after rinsing acid.

Periapical cemental dysplasia in its early stages is best differentiated clinically from apical abscess by which of the following methods> A. Palpation B. Patient symptoms C. Percussion D. Periodontal examination E. Radiographic appearance F. Pulp Testing

F. Pulp Testing 1) Pulp Testing a. tooth should be tested for vitality to rule-out apical abscess b. a vital pulp designation suggest that the radiolucency is probably due to other causes 2) Periapical cemental dysplasia a. during the first phase of periodical cemental dysplasia or the osteopathic phase, there is usually bone loss predominantly along the apices of the mandibular central incisors. This abscess is usually mistaken as an apical abscess. b. presence of an intact lamina dura along the apical portion of the involved tooth also shows that the radiolucency did not come from an infected tooth.

Waiting for intruded permanent central incisors to re-erupt in an 8-year-old child will likely result in A. Ankylosis B. Calcific metamorphosis C. External resorption D. Hypocalcificaton spots on the tooth upon eruption E. Internal resorption F. Re-eruption

F. Re-eruption 1) An intruded permanent central incisor will eventually start to re-erupt within a period of approximately 2-3 weeks.

Which of the following material is typically used for the walking bleach technique? A. 10% Phosphoric acid B. 14% Carbamide peroxide C. 2.6% Sodium Hypochlorite D. 30% Carbamide peroxide E. 30% Hydrogen peroxide F. Sodium Perborate

F. Sodium Perborate 1) Walking bleach technique a. intracoronal bleaching technique which utilizes a mix of sodium perborate and anesthetic or sterile water to create a paste to be sealed into the pulp chamber for 14-21 days b. should be repeated as necessary until the desired tooth color is attained c. used on teeth that have already undergone endodontic therapy and dose not lighten staining due to previous amalgam placement d. external root resorption may occur if the canal is NO obturated properly or if 30% hydrogen peroxide (superoxol) is used. 2) Superoxol (30% hydrogen peroxide) a. used for many years for internal bleaching b. discontinued due to high incidence of external root resorption 3) Carbamide peroxide a. used for vital tooth bleaching (external bleaching)

General Operative Procedures

General Operative Procedures Answers

Hard Tissue

Hard Tissue

Immune System

Immune System Answers

Infection Control

Infection Control Answers

Pulpal necrosis and damage

Intrusive luxation 1) Intrusive luxation a. of primary teeth among children alway result to pulpal necrosis b. occurs when tooth is pushed into the socket, directly damaging and severing the vascular supply of the tooth, immediately results to pulpal necrosis 2) Avulsed primary tooth a. better chance of revascularization when implanted immediately and will not always result in pulp necrosis 3) If tooth is replanted after a long period of time, the changes of ankylosis will increase.

Materials and Equipment Safety

Materials and Equipment Safety Answers

Pathogenesis

Pathogenesis Answers

Periodontics Quiz

Periodontis Quiz Answers

Post Treatment Evaluation

Post Treatment Evaluation Answers

Preparation of Cavities

Preparation of Cavities Answers

Professional Responsibility/Liability

Professional Responsibility/Liability Answers

Prevention and Maintenance

Prevention and Maintenance Answers

Prevention of Oral Disease

Prevention of Oral Disease Answers

Procedural Complications

Procedural Complications Answers

Prognosis

Prognosis Answers

Ankylosis

Replacement resorption = ankylosis 1) splinting should last for about 2 weeks. Freedom for slight movement lessens the pressure on bone and allows better healing along the periodical aspect of bone.

Restoration of Prepared Cavities

Restoration of Prepared Cavities Answers

Soft Tissue

Soft Tissue Answers

Occupational Safety and Health Agency (OSHA) regulates the waste in a dental office OSHA regulates the transport of waste from a dental office.

Statement I is a true statement and Statement II is false.

True/False: The dental community believes that rubber dams must be used to complete all root canal treatments. In certain cases, a clinician can choose not to use a rubber dam with proper informed consent to complete endodontic procedures.

Statement I is true and Statement II is false. 1) Rubber Dams a. using rubber dams during root canal procedures is a standard operating procedure and is accepted and practiced worldwide b. root canal treatment requires an operating field free of contamination to prevent saliva and bacteria from entering the canal orifice c. Proper isolation techniques decrease the changes of canal infection

Supporting Tooth Pathology

Supporting Tooth Pathology Answers

Surgery

Surgery Answers

Systemic Pathology

Systemic Pathology Answers

TMJ

TMJ Answers

Which describes the correct order for the progression of carious lesions? i. Subsurface enamel lesions ii. Cavitation iii. Infection of demineralized dentin iv. Dentin demineralization v. Dentin-matri dissolution vi. Pulpal necrosis

The correct order of progression of a carious lesion is 1) Subsurface enamel lesions 2) Cavitation 3) Dentin demineralization 4) Infection of demineralized dentin 5) Dentin-matrix dissolution 6) Pulpal necrosis

True/False: A minor traumatic injury can result in enamel breaking-off of the underlying dentin. Traumatization of odontoblasts will eventually lead to pulpal degeneration.

The first statement is FALSE, the second is TRUE 1) Enamel fractures are defined as damage to the enamel layer of the tooth causing partial or total removal of the enamel structure which may expose the dentin layer underneath. 2) Small enamel fractures can be recontoured and smoothened with enameloplasty. Extensive enamel fractures are better restored with composites, crowns, inlays/onlays, and veneers 3) Trauma to odontoblasts over a very short period of time will not degenerate the pulp. However, continuous trauma to odontoblasts may result in pulpal irritation and eventually the initiation of tertiary dentin formation.

True/False: A minor trauma to a tooth can lead to enamel fracturing and separating from the dentin beneath it The trauma to the odontoblasts will eventually result in pulpal degeneration.

The first statement is TRUE, the second is FALSE. 1) Concept: Enamel fractures a. defined as damage to the enamel layer of the tooth causing partial or total removal of the enamel structure which may expose the dentin layer underneath. b. commonly due to accidental dental trauma, abnormal eating habits, and biting of substances that are harder and tougher than enamel itself c. are relatively small that can be recontoured and smoothened with enameloplasty. d. too extensive are better restored with composites, crowns, inlays/onlays and veneers 2) Trauma to odontoblasts over a short period of time a. will not degenerate the pulp b. continuous trauma to odontoblasts may result in pulpal irritation and eventually the initiation of tertiary dentin formation.

True/False: Polymorphonuclear neutrophils (PMN) cause tissue damage while protecting host tissues. Overall, the role of PMNs is described as mostly destructive.

The first statement is true and the second is false 1) Polymorphonuclear leukocytes or PMNs a. white blood cells of the immune system that actively engulf foreign bodies that enter the body b. first defense cells to migrate to the site of injury and fight against pathogens during an acute inflammatory reaction c. engulf and eliminate most of the pathogen are able to evade phagocytosis, causing an accumulation of PMNs in the affected area d. excess PMN's that are unable to engulf the pathogens die and degranulate which results in the release of host proteases which also may damage the hosts' cell matrix e. uncontrolled and destructive potential of host proteases release by the PMNs can kill pathogens and at the same time damage the host's tissue f. severe uncontrolled PMN functions may lead to endotoxic shock and disorders mediated by the immune complex .

Therapy

Therapy Answers

Traumatic Injuries

Traumatic Injuries Answers

Treatment Plan

Treatment Plan Answers

Treatment Planning

Treatment Planning Answers

Which of the following intervals does the United States Centers for Disease Control (CDC) recommend sterilizing units be monitored and tested?

weekly


Related study sets

#1: Information and Communication Technology

View Set

Turning Points of the Revolution

View Set